You are on page 1of 248

https://upscpdf.com << Download From >> https://upscpdf.

com

VISIONIAS
130/-

भारतीय संविधान एिं शासन


भाग-1
क्रम ऄध्याय पृष्ठ संख्या
संख्या
1. भारतीय संिध
ै ावनक योजना की ऄन्य देशों के साथ तुलना 1-62
2. ऐवतहावसक अधार, विकास, विशेषताएं, संशोधन, महत्िपूणण प्रािधान 63-107

और बुवनयादी संरचना
3. संविधान की ईद्देवशका 108-121
4. संघ और ईसका राज्य क्षेत्र 122-143
5. नागररकता 144-169
6. मूल ऄवधकार 170-236
7. मूल कर्त्णव्य 237-247

Copyright © by Vision IAS


All rights are reserved. No part of this document may be reproduced, stored in a retrieval system or
transmitted in any form or by any means, electronic, mechanical, photocopying, recording or otherwise,
without prior permission of Vision IAS.

Google it:- https://upscpdf.com


https://upscpdf.com << Download From >> https://upscpdf.com

भारतीय संिध
ै ावनक योजना की ऄन्य देशों के साथ तुलना
विषय सूची
1. पररचय_________________________________________________________________________________ 7

2. विरिश संविधान (British Constitution) _________________________________________________________ 7

2.1. मुख्य विशेषताएँ _______________________________________________________________________ 7


2.1.1. ऄवलवखत (Unwritten) _________________________________________________________________ 7
2.1.2. क्रवमक विकास (Evolutionary) ___________________________________________________________ 7
2.1.3. लचीलापन (Flexibility) ________________________________________________________________ 7
2.1.4. एकात्मक बनाम संघीय विशेषताएँ (Unitary Vs Federal Features) _______________________________ 8
2.1.5. संसदीय कायणकारी (Parliamentary Executive) ______________________________________________ 8
2.1.6. संसद की संप्रभुता (Sovereignty of Parliament) _____________________________________________ 9
2.1.7. पररपारियों की भूवमका (Role of Conventions) ______________________________________________ 9
2.1.8. विवध का शासन (Rule of Law) __________________________________________________________ 9
2.1.9. न्यायपावलका की स्ितंत्रता (Independence Of Judiciary) _____________________________________ 10

2.2. राज्य के ऄंग (Organs of the State) _______________________________________________________ 10


2.2.1. कायणपावलका (Executive) ______________________________________________________________ 10
2.2.1.1. क्राईन: राजा मृत है। राजा ऄमर रहे। (King is dead. Long live the King.) _____________________ 10
2.2.1.2. राजतंत्र की प्रकृ वत (Nature of Monarchy) _____________________________________________ 11
2.2.1.3. विरिश प्रधानमंत्री और मंवत्रपररषद् _____________________________________________________ 12
2.2.1.3.1. वििेन का प्रधानमंत्री____________________________________________________________ 12
2.2.1.3.2. प्रधानमंत्री समकक्षों में प्रथम होता है ________________________________________________ 12
2.2.1.3.3. प्रधानमंत्री विवभन्न तारों के बीच एक चन्रमा की भांवत होता है (P.M. As Moon Among Stars) ____ 13
2.2.1.4. विरिश और भारतीय प्रधानमंत्री के मध्य ऄंतर _____________________________________________ 13
2.2.1.5. वप्रिी काईं वसल ___________________________________________________________________ 13
2.2.1.6. स्थायी वसविल सेिक/विरिश नौकरशाह __________________________________________________ 13
2.2.2. विधावयका (Legislature) ______________________________________________________________ 14
2.2.2.1. दो प्रणावलयों के मध्य मौवलक ऄंतर _____________________________________________________ 14
2.2.2.2. हाईस ऑफ लॉर्डसण_________________________________________________________________ 14
2.2.2.2.1. हाईस ऑफ लॉर्डसण और संयुक्त राज्य ऄमेररका के सीनेि के मध्य तुलना _________________________ 15
2.2.2.2.2. हाईस ऑफ लॉर्डसण में सुधार ______________________________________________________ 16
2.2.2.2.3. लंवबत सुधार _________________________________________________________________ 16
2.2.2.3. हाईस ऑफ़ कॉमंस ________________________________________________________________ 16
2.2.2.4. भारतीय और ऄमेररकी स्पीकर (ऄध्यक्ष) के साथ हाईस ऑफ कॉमंस के स्पीकर की वस्थवत की तुलना ________ 17
2.2.2.4.1. विरिश स्पीकर की विशेषताएँ _____________________________________________________ 17
2.2.2.4.2. ऄमेररका में स्पीकर (हाईस ऑफ़ ररप्रेजन्े िेरिव्स का ऄध्यक्ष) _________________________________ 17
2.2.2.4.3. भारत में लोकसभा का ऄध्यक्ष _____________________________________________________ 17
2.2.3. न्यायपावलका________________________________________________________________________ 17

Google it:- https://upscpdf.com


https://upscpdf.com << Download From >> https://upscpdf.com

2.2.3.1. भारतीय और विरिश न्यायपावलका के मध्य तुलना __________________________________________ 18

2.4. ईपयुक्त
ण तुलना का एक संवक्षप्त सारांश _________________________________________________________ 18
2.4.1. विरिश संविधान _____________________________________________________________________ 18
2.4.2. विरिश सम्राि और भारत के राष्ट्रपवत के मध्य तुलना _____________________________________________ 19
2.4.3. विरिश सम्राि और ऄमेररकी राष्ट्रपवत के मध्य तुलना _____________________________________________ 19

3. संयक्त
ु राज्य ऄमेररका का संविधान _____________________________________________________________ 19

3.1. प्रमुख विशेषताएँ ______________________________________________________________________ 19


3.1.1. संविधान की प्रकृ वत____________________________________________________________________ 20
3.1.2. संघिाद की प्रकृ वत ____________________________________________________________________ 20
3.1.3. सरकार का स्िरूप ____________________________________________________________________ 21

3.2. राष्ट्रपवत ____________________________________________________________________________ 21


3.2.1. ऄमेररकी राष्ट्रपवत पद के वलए ऄहणता _______________________________________________________ 21
3.2.2. राष्ट्रपवत का वनिाणचन __________________________________________________________________ 22
3.2.3. संयुक्त राज्य ऄमेररका के राष्ट्रपवत का प्रकायण __________________________________________________ 22
3.2.4. विधायी प्रस्ताि ______________________________________________________________________ 23
3.2.5. शपथ और सेिावनिृवर्त् की वतवथ___________________________________________________________ 23
3.2.6. प्राआमरीज (Primaries) ________________________________________________________________ 24
3.2.7. संयुक्त राज्य ऄमेररका के राष्ट्रपवत के विरूद्ध महावभयोग __________________________________________ 24
3.2.8. संयुक्त राज्य ऄमेररका के संविधान की कु छ शब्दािवलयां __________________________________________ 24

3.3. ईप-राष्ट्रपवत _________________________________________________________________________ 25


3.3.1. ईप-राष्ट्रपवत का चुनाि _________________________________________________________________ 25
3.3.2. राष्ट्रपवत के रूप में ईप-राष्ट्रपवत का कायणकाल __________________________________________________ 25
3.3.3. संयुक्त राज्य ऄमेररका के ईप-राष्ट्रपवत के प्रकायण ________________________________________________ 25

3.4. ऄमेररकी विधावयका/ऄमेररकी कांग्रेस _________________________________________________________ 25


3.4.1. हाईस ऑफ़ ररप्रजेन्िेरिव्स _______________________________________________________________ 25
3.4.2. सीनेि _____________________________________________________________________________ 26

3.5. संयक्त
ु राज्य ऄमेररका में सवमवत प्रणाली (Committee System In USA)_______________________________ 26

3.6. कें रीय स्तर पर आन प्रवतवनवध वनकायों की ऄिवध _________________________________________________ 26


3.6.1. संयुक्त राज्य ऄमेररका __________________________________________________________________ 26
3.6.2. भारत _____________________________________________________________________________ 26

3.7. शवक्त के पृथक्करण का वसद्धांत और वनयंत्रण एिं संतल


ु न की प्रणाली ______________________________________ 26
3.7.1. संयुक्त राज्य ऄमेररका __________________________________________________________________ 27
3.7.2. भारत _____________________________________________________________________________ 27
3.7.3. वनयंत्रण एिं संतुलन ___________________________________________________________________ 27

3.8. मूल ऄवधकार (Fundamental Rights) ______________________________________________________ 28

3.9. विधायी शवक्त का वितरण (Distribution Of Legislative Power) ___________________________________ 29

Google it:- https://upscpdf.com


https://upscpdf.com << Download From >> https://upscpdf.com

3.9.1. भारत _____________________________________________________________________________ 29


3.9.2. संयुक्त राज्य ऄमेररका __________________________________________________________________ 30

3.10. अपात-काल और ररि का वनलंबन __________________________________________________________ 30

3.11. न्यायपावलका _______________________________________________________________________ 30

3.12. संविधान का संशोधन __________________________________________________________________ 31


3.12.1. ऄमेररकी संविधान में संशोधन ___________________________________________________________ 31
3.12.2. भारतीय संविधान में संशोधन ___________________________________________________________ 31

4. चीन का संविधान _________________________________________________________________________ 31

4.1. चीन के संविधान की प्रमुख विशेषताएं (Salient Features Of The Constitution) ________________________ 32
4.1.1. प्रस्तािना __________________________________________________________________________ 32
4.1.2. संविधान की प्रकृ वत____________________________________________________________________ 32
4.1.3. अधारभूत वसद्धांत ____________________________________________________________________ 32
4.1.4. एकात्मक प्रणाली _____________________________________________________________________ 32
4.1.5. लोकतांवत्रक कें रिाद (Democratic Centralism) _____________________________________________ 33
4.1.6. एक दलीय प्रणाली (One Party System) __________________________________________________ 33
4.1.7. विधावयका (Legislature) ______________________________________________________________ 33
4.1.8. कायणपावलका (Executive) ______________________________________________________________ 35
4.1.8.1. राज्य पररषद (स्िेि काईं वसल)_________________________________________________________ 35
4.1.8.2. प्रधानमंत्री (Premier) _____________________________________________________________ 36
4.1.8.3. राष्ट्रपवत ________________________________________________________________________ 36
4.1.9. न्यायपावलका________________________________________________________________________ 36
4.1.10. कें रीय सैन्य अयोग (Central Military Commission) ________________________________________ 37
4.1.11. ऄवधकार और कतणव्य _________________________________________________________________ 37
4.1.12. चीनी कम्युवनस्ि पािी (Communist Party of China) ________________________________________ 37
4.1.12.1. विचारधारात्मक अधार (Ideological Foundations) _____________________________________ 37
4.1.12.2. पािी संगठन ____________________________________________________________________ 38
4.1.12.3. पोवलत ब्यूरो ___________________________________________________________________ 38
4.1.12.4. नेशनल कांग्रेस ऑफ़ कम्युवनस्ि पािी ऑफ़ चाआना (NCCPC) ________________________________ 38
4.1.12.5. कें रीय सवमवत ___________________________________________________________________ 38
4.1.12.6. ऄन्य दल और समूह _______________________________________________________________ 38

5. फ्ांसीसी संविधान (French Constitution) ______________________________________________________ 39

5.1. भूवमका ____________________________________________________________________________ 39

5.2. राष्ट्रपवत ____________________________________________________________________________ 39


5.2.1. राष्ट्रपवत का हिाया जाना _______________________________________________________________ 40
5.2.2. राष्ट्रपवत की अपातकालीन शवक्तयां ________________________________________________________ 40
5.2.3. ऄमेररका और फ्ांस के राष्ट्रपवतयों का तुलनात्मक विश्लेषण _________________________________________ 40
5.2.4. फ्ांस के राष्ट्रपवत और विरिश प्रधानमंत्री का तुलनात्मक विश्लेषण ____________________________________ 41

Google it:- https://upscpdf.com


https://upscpdf.com << Download From >> https://upscpdf.com

5.3. विधवयका (Legislature) ________________________________________________________________ 41


5.3.1. नेशनल ऄसेंबली (National Assembly) ___________________________________________________ 41
5.3.2. सीनेि (The Senate) _________________________________________________________________ 41

5.4. फ्ांसीसी संविधान की प्रमुख विशेषताएँ _______________________________________________________ 42

5.5. संविधान का संशोधन ___________________________________________________________________ 42

6. जमणनी का संविधान (Germany’s Constitution) __________________________________________________ 42

6.1. प्रमुख विशेषताएँ ______________________________________________________________________ 42


6.1.1. चांसलर डेमोक्रेसी (Chancellor’s Democracy) _____________________________________________ 42
6.1.2. कै वबनेि वसद्धांत (Cabinet Principle) _____________________________________________________ 43
6.1.3. रचनात्मक ऄविश्वास प्रस्ताि (Constructive Vote of No-Confidence) ___________________________ 43
6.1.4. संसद _____________________________________________________________________________ 43
6.1.4.1. बुंदस्े िैग (The Bundestag) _________________________________________________________ 43
6.1.4.2. बुंदस्र
े त (The Bundesrat) __________________________________________________________ 44

7. जापान का संविधान _______________________________________________________________________ 45

8. कनाडा का संविधान _______________________________________________________________________ 45

8.1. प्रमुख विशेषताएं ______________________________________________________________________ 45


8.1.1. संिैधावनक राजतंत्र____________________________________________________________________ 45
8.1.2. संसदीय सरकार ______________________________________________________________________ 46
8.1.3. संघिाद (Federalism) ________________________________________________________________ 47
8.1.4. न्यायपावलका________________________________________________________________________ 47
8.1.5. ऄवधकार ___________________________________________________________________________ 47

9. ऑस्रेवलया का संविधान (Constitution of Australia) _______________________________________________ 48

9.1. प्रमुख विशेषताएं ______________________________________________________________________ 48


9.1.1. शासन के प्रकार (Form of Government) __________________________________________________ 48
9.1.2. संविधान की प्रकृ वत____________________________________________________________________ 49
9.1.2.1 संशोधन की प्रक्रक्रया ________________________________________________________________ 49
9.1.3. संसद _____________________________________________________________________________ 49
9.1.4. वनिाणचन की प्रकृ वत____________________________________________________________________ 50
9.1.5. मतदान ____________________________________________________________________________ 50
9.1.6. सरकार के स्तरों के मध्य संबंध ____________________________________________________________ 50

10. वस्िट्ज़रलैंड का संविधान ___________________________________________________________________ 51

10.1. भारतीय संविधान से तुलना ______________________________________________________________ 51

10.2 प्रत्यक्ष लोकतंत्र की व्यिस्था ______________________________________________________________ 51

11. भारतीय संविधान की प्रमुख विशेषताएँ एिं तुलनात्मक ऄध्ययन _________________________________________ 51

11.1. ईद्देवशका __________________________________________________________________________ 51

Google it:- https://upscpdf.com


https://upscpdf.com << Download From >> https://upscpdf.com

11.2. वलवखत संविधान _____________________________________________________________________ 52

11.3. नाममात्र का राज्य प्रमुख ________________________________________________________________ 52

11.4. मंवत्रमंडलीय प्रणाली (Cabinet System) ___________________________________________________ 52

11.5. विसदनात्मक संसदीय व्यिस्था ____________________________________________________________ 52

11.6. वनचले सदन के ऄवधक शवक्तशाली होने की ऄिधारणा _____________________________________________ 52

11.7. वनम्न सदन का ऄध्यक्ष (Speaker in the Lower House) ________________________________________ 53

11.8. न्यायपावलका _______________________________________________________________________ 53


11.8.1. ईच्चतम न्यायालय की संकल्पना __________________________________________________________ 53
11.8.2. ईच्चतम न्यायालय की कायणप्रणाली ________________________________________________________ 54
11.8.3. न्यायपावलका की स्ितंत्रता और न्यावयक पुनर्विलोकन___________________________________________ 54
11.8.4. ईच्चतम/ईच्च न्यायालय के न्यायाधीशों को पद से हिाने की विवध ___________________________________ 54

11.9. मूल ऄवधकार _______________________________________________________________________ 55


11.9.1. अपातकाल के दौरान मूल ऄवधकारों का वनलंबन ______________________________________________ 55

11.10. मूल कर्त्णव्य ________________________________________________________________________ 55

11.11. संघीय व्यिस्था _____________________________________________________________________ 56


11.11.1. सुदढ़ृ कें र के साथ संघीय व्यिस्था ________________________________________________________ 56
11.11.2. ऄमेररकी संघिाद के साथ भारतीय संघिाद की तुलना _________________________________________ 56

11.12. व्यापार और िावणज्य की स्ितंत्रता ________________________________________________________ 57

11.13. राज्य की नीवत के वनदेशक तत्ि (DPSP) ____________________________________________________ 58

11.14. राष्ट्रपवत िारा संसद सदस्यों को मनोनीत करना ________________________________________________ 58

11.15. संविधान के प्रमुख ईपबंधों के स्रोतों की सूची __________________________________________________ 58

12. विगत िषों में Vision IAS GS मेंस िेस्ि सीरीज में पूछे गए प्रश्न (Previous Year Vision IAS GS Mains Test
Series Questions) ________________________________________________________________________ 59

Google it:- https://upscpdf.com


https://upscpdf.com << Download From >> https://upscpdf.com

1. पररचय
 भारत के संविधान के विवभन्न देशों के संविधान से तुलनात्मक ऄध्ययन के दौरान आस ऄध्याय में
वनम्नवलवखत दो प्रमुख पहलुओं पर ध्यान कें क्ररत क्रकया गया है:
o विवभन्न देशों के संविधान के संवक्षप्त ज्ञान, ईनकी ितणमान वस्थवत और महत्ि या आस तथ्य पर
विचार करना क्रक भारतीय संविधान में ईनसे ऄव्यक्त रूप से या स्पष्टतया क्या ग्रहण क्रकया
गया है; तथा
o संविधान की विवभन्न विशेषताओं के साथ तुलनात्मक ऄध्ययन {जैसे- मूल ऄवधकार, राज्य की
नीवत के वनदेशक तत्ि (DPSPs), संघिाद अक्रद}।

2. विरिश संविधान (British Constitution)


2.1. मु ख्य विशे ष ताएँ

2.1.1. ऄवलवखत (Unwritten)

 विरिश संविधान की सबसे महत्िपूणण विशेषताओं में से एक आसका ऄवलवखत होना है। यह कहना
सम्यक् रूप से ईवचत है क्रक ऐसा कोइ वलवखत, वनयमवनष्ठ और सुगरठत दस्तािे़ नहीं है वजसे
विरिश संविधान जैसा कु छ कहा जा सकता है।
 आसका प्रमुख कारण यह है क्रक यह एक हजार िषों से ऄवधक की ऄिवध के दौरान वनरंतर हुए
विवभन्न समझौतों (Conventions) और राजनीवतक परंपराओं (Political traditions) पर
अधाररत है, जो क्रकसी दस्तािे़ में वलवखत रूप में नहीं हैं। आस प्रकार यह ईन वलवखत संविधानों
से वभन्न है, जो सामान्यतः एक संविधान सभा िारा वनर्वमत होते हैं।
 विरिश संविधान की तुलना में भारतीय संविधान विश्व का सबसे लंबा वलवखत संविधान है।

2.1.2. क्रवमक विकास (Evolutionary)

 विरिश संविधान, क्रवमक विकास का एक ईदाहरण है। आसे क्रकसी संविधान सभा िारा कभी तैयार
नहीं क्रकया गया। यह एक हजार िषण से ऄवधक की ऄिवध के दौरान वनरंतर विकवसत हुअ है।
ऐसा कहा जाता है क्रक विरिश संविधान समझ एिं पररवस्थवत की ईपज है।
 आस विशेष पहलू के संदभण में भारतीय संविधान की आससे कु छ समानताएं एिं ऄसमानताएं हैं। यह
विरिश संविधान से ईस सीमा तक वभन्न है क्रक यह एक वलवखत दस्तािे़ है और
आसमें सभी प्रािधानों को बेहतर तरीके से पररभावषत क्रकया गया है। परंतु, यह भी क्रवमक विकास
के वलए खुला है। आसमें संशोधन के प्रािधान आस तरह शावमल हैं क्रक समय की मांग और समझ के
ऄनुसार आसे विकवसत क्रकया जा सके ।

2.1.3. लचीलापन (Flexibility)

 विरिश संविधान, लचीले संविधान का एक ईत्कृ ष्ट ईदाहरण है। चूंक्रक यहाँ संिैधावनक कानून और
साधारण कानून दोनों के साथ एक समान व्यिहार क्रकया जाता है तथा ईनके मध्य कोइ विभेद
नहीं क्रकया जाता है, ऄतः आसके विवभन्न प्रािधानों को संसद के साधारण बहुमत (ईपवस्थत और
मतदान करने िाले सदस्यों का 50%) के िारा पाररत, संशोवधत और वनरवसत क्रकया जा सकता
है। लचीलेपन के तत्ि ने विरिश संविधान को ऄनुकूलन एिं सामंजस्य की विशेषता प्रदान की है।
आस विलक्षणता ने आसे समय की मांग के ऄनुरूप विकवसत होने में सक्षम बनाया है।

7 www.visionias.in ©Vision IAS

Google it:- https://upscpdf.com


https://upscpdf.com << Download From >> https://upscpdf.com

 आसके विपरीत, भारतीय संविधान में लचीलापन एिं कठोरता, दोनों का वमश्रण है। यह भारतीय

संविधान की अधारभूत विचारधारा को बेहतर तरीके से प्रवतबबवबत करता है, जहाँ संप्रभुता,

पंथवनरपेक्षता और गणतंत्र जैसी कु छ विशेषताओं को ऄनुल्लंघनीय माना गया है, लेक्रकन ऄन्य

सभी मामलों में संविधान संशोधन करने के वलए ऄनुमवत देता है।

2.1.4. एकात्मक बनाम सं घीय विशे ष ताएँ ( Unitary Vs Federal Features )

 विरिश संविधान, संघीय विशेषता के विपरीत एकात्मक विशेषता धारण करता है। सरकार की

सभी शवक्तयां विरिश संसद में वनवहत हैं, जो क्रक एक संप्रभु संस्था है। राज्य के कायणकारी ऄंग संसद

के ऄधीनस्थ हैं, प्रत्यायोवजत शवक्तयों का ईपयोग करते हैं तथा आसके प्रवत जिाबदेह हैं। िहाँ वसफण

एक ही विधान-मंडल है। आंग्लैंड, स्कॉिलैंड, िेल्स अक्रद प्रशासवनक आकाआयां हैं और राजनीवतक

रूप से स्िायर्त् आकाआयां नहीं हैं।

 दूसरी तरफ, भारतीय संविधान संघीय विशेषताओं से युक्त है।

एकात्मक (Unitary) संघात्मक (Federal) ऄवधसंघ (Confederation)

सभी शवक्तयां कें र में वनवहत प्रांतीय सरकारें , संविधान कइ आकाआयां एक साथ वमलकर राज्य
होती हैं।
से शवक्तयां प्राप्त करती हैं। का वनमाणण करती हैं।

कें र सरकार, प्रांतीय सरकारों िास्तविक शवक्तयां, आकाआयों में

को शवक्तयां प्रत्यायोवजत वनवहत होती हैं (ऄथाणत् एकात्मक के

करती है। विपरीत)।

ईदाहरण: वििेन ईदाहरण: भारत ईदाहरण: यूरोपीय संघ, संयक्त


ु राज्य

ऄमेररका

2.1.5. सं स दीय कायण कारी (Parliamentary Executive)

 यह विरिश और भारतीय संविधान के मध्य एक महत्िपूणण समानता है। (संसद की संप्रभुता के


ऄवतररक्त)

 वििेन में शासन की संसदीय प्रणाली (Parliamentary form of government) है। सम्राि, जो

क्रक संप्रभु है, ईसे सभी शवक्तयों और ऄवधकारों से िंवचत कर क्रदया गया है। िास्तविक प्रावधकार

मंवत्रयों में वनवहत होते हैं, जो संसद में बहुमत प्राप्त दल से संबंवधत होते हैं और जब तक ईन्हें

(संसद के प्रवत) बहुमत प्राप्त होता है तब तक िे पद पर बने रहते हैं।


 प्रधानमंत्री और मंत्री ऄपने कृ त्यों और नीवतयों के वलए विधावयका के प्रवत ईर्त्रदायी होते हैं। आस
प्रणाली में, शासन की राष्ट्रपतीय प्रणाली (Presidential form of government) की भांवत

कायणपावलका और विधावयका को पृथक नहीं क्रकया जाता है।

8 www.visionias.in ©Vision IAS

Google it:- https://upscpdf.com


https://upscpdf.com << Download From >> https://upscpdf.com

2.1.6. सं स द की सं प्र भु ता (Sovereignty Of Parliament)

 संप्रभुता का ऄथण सिोच्च शवक्त है। विरिश संविधान की एक ऄवत महत्िपूणण विशेषता विरिश संसद
की संप्रभुता है (एक वलवखत संविधान की ऄनुपवस्थवत के बािजूद)।
 विरिश संसद, बंधन-मुक्त विधायन शवक्त से पररपूणण देश का एकमात्र विधायी वनकाय है। यह
क्रकसी भी कानून का वनमाणण, संशोधन या ईसे वनरवसत कर सकती है।
 हालांक्रक, भारत के मामले में राज्य स्तर पर भी विधावयका ईपवस्थत हैं, क्रफर भी भारतीय संसद
की कानून वनमाणण की शवक्त मुख्यतः विरिश संसद के समान है।
 न्यायालयों को विरिश संसद िारा पाररत कानूनों की िैधता पर प्रश्न करने (न्यावयक पुनर्विलोकन)
की शवक्त नहीं है। विरिश संसद देश के साधारण कानून के समान ऄपने प्रावधकार
का ईपयोग संविधान में संशोधन के वलए कर सकती है। यह ऄिैध को िैध बना सकती है तथा
िैध को ऄिैध बना सकती है।
 आस संदभण में भारतीय पररप्रेक्ष्य में स्पष्ट ऄंतर विद्यमान है। भारतीय न्यायपावलका की शवक्त
वनर्वमत कानून की िैधता की जांच करने की है। आसके ऄवतररक्त, 'मूल ढांचे' के वसद्धांत, कानून की
िैधता के संबंध में प्रश्न करने के वलए भारतीय न्यायपावलका को शवक्त प्रदान करता है। आस तथ्य
के अलोक में भारत का ईच्चतम न्यायालय भारत के संविधान का सबसे बड़ा व्याख्याकार है।

2.1.7. पररपारियों की भू वमका (Role Of Conventions)

 पररपािी को संविधान के ऄवलवखत वसद्धांतों (वनयम) के रूप में जाना जाता है। ये संविधान को
नम्यता प्रदान कर आसे संशोवधत होने से संरक्षण प्रदान करती हैं।
 विश्व के ऄवधकांश संविधान कु छ पररपारियों से युक्त हैं। विरिश संविधान के ऄवलवखत चररत्र के
वलए एक अिश्यक ईपप्रमेय यह है क्रक ये पररपारियाँ , विरिश राजनीवतक प्रणाली में बहुत ही
महत्िपूणण भूवमका वनभाती हैं। ईदाहरण के वलए, यद्यवप साम्राज्ञी के पास विरिश संसद िारा
पाररत कानून को ऄस्िीकृ त करने का विशेषावधकार है, लेक्रकन पररपािी यह है क्रक िह ऐसा नहीं
करती हैं और यह ऄपने अप में ईस संविधान का एक वसद्धांत बन गया है।
 हालांक्रक, पररपारियों की कानूनी वस्थवत वलवखत कानून के ऄधीनस्थ है।

2.1.8. विवध का शासन (Rule Of Law)

 विरिश संविधान की एक ऄन्य महत्िपूणण विशेषता विवध का शासन है। संविधानिाद या सीवमत
सरकार, विवध के शासन का सार है। यह कायणकाररणी की मनमानी कायणिाही को वनयंवत्रत करता
है। डायसी के ऄनुसार, वििेन में विवध के शासन के तीन वसद्धांत विद्यमान हैं:
o मनमाने ढंग से वगरफ्तारी के विरुद्ध संरक्षण और स्ियं प्रवतिाद करने का ऄिसर।
o विवध के समक्ष समता (Equality before Law): सभी व्यवक्त, ऄपनी वस्थवत या पद से
पृथक विवध के समक्ष समान हैं। प्रशासवनक कानून की ऄिधारणा विवध के समक्ष समता से
वभन्न है, जो सरकारी कमणचाररयों के वलए विवभन्न प्रकार की ईन्मुवक्त प्रदान करता है। वििेन
में संविधान और मूल ऄवधकारों की ऄनुपवस्थवत में न्यायपावलका आस विवध की रक्षा करती
है। आसवलए आस प्रणाली को “सामान्य विवध का वसद्धांत” (Principle of Common Laws)
कहा जाता है {संयक्त
ु राज्य ऄमेररका और भारत (मेनका गांधी िाद के पश्चात्) में “नैसर्वगक
विवध/कानून का वसद्धांत” (Principle of Natural Law)}।

9 www.visionias.in ©Vision IAS

Google it:- https://upscpdf.com


https://upscpdf.com << Download From >> https://upscpdf.com

o वििेन में लोगों के ऄवधकारों की गारंिी न्यायपावलका िारा प्रदान की गइ है। न्यायपावलका
सामान्य विवध को मान्यता प्रदान करती है। आस प्रकार, वििेन में लोगों को वबल ऑफ़ राआट्जस
या मौवलक ऄवधकारों के ऄभाि में भी ऄवधकार प्राप्त हैं।
 हालांक्रक यह देखा गया है क्रक कइ बार िास्तविक ऄथों में विवध के शासन का प्रचलन नहीं है।
आसके वलए वनम्नवलवखत कारण ईर्त्रदायी हैं:
o प्रशासवनक विवध का विकास;
o प्रत्यायोवजत विधान की िृवद्ध;
o अंतररक एिं बाह्य अपातकाल; अक्रद।
 आन विकासक्रमों को 'निीन स्िेछाचाररता' (New Despotism) का नाम क्रदया गया है।
 निीन स्िेछाचाररता (New Despotism): आसे ईस प्रचवलत वस्थवत के रूप में पररभावषत क्रकया
जाता है जहां लोकतांवत्रक व्यिस्था के ऄवस्तत्ि में होने के बािजूद, नौकरशाही को ऄसाधारण
शवक्तयां प्राप्त हैं। यहीं कारण है क्रक नौकरशाहों के वलए प्राय: ‘निीन स्िेछाचारी’ पद का प्रयोग
क्रकया जाता है, जो लोकतांवत्रक देश में भी ऄसाधारण शवक्त का ईपभोग करते हैं।

2.1.9. न्यायपावलका की स्ितं त्र ता (Independence Of Judiciary)

 वििेन में विवध का शासन आस प्रािधान से संरवक्षत है क्रक न्यायाधीशों को के िल गंभीर दुव्यि
ण हार
के मामले में हीं पद से हिाया जा सकता है और आस हेतु संसद के दोनों सदनों की सहमवत की
अिश्यकता होती है। आसवलए, न्यायाधीश वबना क्रकसी भय या पक्षपात के वनणणय देने में सक्षम हैं।
 भारत में भी आसे ऄपनाया गया है। यहाँ न्यायपावलका की स्ितंत्रता को संविधान में समाविष्ट
क्रकया गया है। (यह ‘मूल ढांच'े के वसद्धांत की विशेषताओं में से एक है)

2.2. राज्य के ऄं ग (Organs Of The State )

2.2.1. कायण पावलका (Executive)

 वििेन में कायणपावलका को “क्राईन” कहा जाता है। आससे पूिण क्राईन, राजा का प्रतीक माना जाता
था। ऄब राजा क्राईन का एक ऄंग मात्र है।
 एक संस्था के रूप में, क्राईन में वनम्नवलवखत शावमल होते हैं:
o राजा,
o प्रधानमंत्री,
o मंवत्रपररषद् (CoM),
o स्थायी कायणपावलका (वसविल सेिक), और
o वप्रिी काईं वसल।

2.2.1.1. क्राईन: राजा मृ त है। राजा ऄमर रहे। ( King is dead. Long live the King.)

“वििेन में, प्रारंभ में सभी शवक्तयां राजा में वनवहत थीं। बाद में, राजा से शवक्तयों का हस्तांतरण
प्रधानमंत्री की ऄध्यक्षता िाले मंवत्रपररषद्, स्थायी कायणपावलका, वप्रिी काईं वसल अक्रद संस्थाओं को कर
क्रदया गया। ितणमान में, क्राईन में ये सभी संस्थान शावमल हैं। आसवलए, कथन के पहले भाग में राजा का
िणणन एक व्यवक्त के रूप में क्रकया गया है, जबक्रक दूसरे भाग में राजा या क्राईन का िणणन एक संस्था के
रूप में है।”

10 www.visionias.in ©Vision IAS

Google it:- https://upscpdf.com


https://upscpdf.com << Download From >> https://upscpdf.com

2.2.1.2. राजतं त्र की प्रकृ वत (Nature of Monarchy)

 वििेन में संिैधावनक राजतंत्र है और संिैधावनक राजतंत्र , लोकतंत्र से ऄसंगत नहीं होता है। आसका
कारण यह है क्रक संिैधावनक राजतंत्र में ऄवनिायण रूप से राज्य प्रमुख के रूप में राजा की शवक्तयों
(ितणमान में महारानी एवलजाबेथ-II) की प्रकृ वत औपचाररक प्रकार की होती हैं। आसमें िास्तविक

शवक्तयाँ संसद सदस्यों में वनवहत होती हैं जो सरकार का गठन करते हैं, लेक्रकन वनरपिाद रूप से
राजा िारा आस पररपािी का पालन क्रकया जाता है। ऄतः ऄब िास्तविक कायणपावलका शवक्त हाईस
ऑफ कॉमंस में बहुमत प्राप्त राजनीवतक दल या गठबंधन के नेता के पास होती है।
 िास्तविक शवक्त की कमी के बािजूद, समकालीन वििेन में राजशाही की ऄभी भी कइ महत्िपूणण
भूवमकाएँ हैं। आसमें वनम्नवलवखत सवम्मवलत हैं:
o देश और विदेश में UK का प्रवतवनवधत्ि करना;

o नागररकता और पाररिाररक जीिन के मानकों को वनधाणररत करना;

o मतभेदों के बािजूद लोगों को एकजुि रखना;

o सशस्त्र बलों की राजवनष्ठा;

o विरिश परंपराओं की वनरंतरता को बनाए रखना;

o इसाइ धमण संबंधी नैवतकता का संरक्षण; अक्रद।

आसके ऄवतररक्त, वनम्नवलवखत पर भी विचार करना अिश्यक है:


 संसदीय प्रणाली में दो प्रमुखों की अिश्यकता होती है:
o प्रथम प्रमुख (फस्िण हेड) िस्तुतः राज्य प्रमुख होता है। िह राष्ट्र का प्रवतवनवधत्ि करता है और
प्रशासन को वनरंतरता प्रदान करता है; तथा
o वितीय प्रमुख (सेकंड हेड) िस्तुतः सरकार का प्रमुख होता है। ईसके पास िास्तविक शवक्तयाँ
होती हैं क्योंक्रक सदन को प्रधानमंत्री में विश्वास होता है। प्रधानमंत्री सदन का नेता होता है।
िह सदन के बहुमत का प्रवतवनवधत्ि करता है।
 राजपद (kingship) की व्यिस्था िस्तुतः मनोिैज्ञावनक संतुवष्ट का एक स्रोत है। यह कहा जाता है

क्रक, "बककघम पैलेस में राजा की ईपवस्थवत से, वििेनिासी ऄपने घरों में चैन की नींद से सोते हैं"।
 राजा िारा संकिकालीन पररवस्थवतयों में ऄत्यवधक सहायता की जाती है। ईन्हें सामान्यतः
ऄत्यवधक ऄनुभि प्राप्त होता है और िे देश वहत में बहुमूल्य सलाह दे सकते हैं।
 बजहॉि के ऄनुसार, राजा के पास वनम्नवलवखत तीन ऄवधकार हैं:

o चेतािनी देने का ऄवधकार;

o प्रोत्सावहत करने का ऄवधकार; और


o सूवचत क्रकए जाने का ऄवधकार।
 राजपद को समाप्त करने के वलए एक वनिाणवचत प्रधान की अिश्यकता होगी। वनिाणवचत प्रमुख,

वजसके पास कोइ िास्तविक शवक्तयां नहीं होती हैं, स्ियं कु छ नयी समस्याएं ईत्पन्न करेगा।

 आसके विपरीत, राजशाही का कोइ प्रािधान भारतीय संविधान में मौजूद नहीं है। िास्ति में, राजा

अक्रद जैसी पदिी को संविधान के ऄनुच्छेद 18 (मूल ऄवधकार) के िारा समाप्त कर क्रदया गया है।

आस प्रकार, यहाँ सभी भारतीय नागररकों की समानता पर बल क्रदया गया है।

11 www.visionias.in ©Vision IAS

Google it:- https://upscpdf.com


https://upscpdf.com << Download From >> https://upscpdf.com

2.2.1.3. विरिश प्रधानमं त्री और मं वत्रपररषद्


 वििेन में मंवत्रमंडल (कै वबनेि) स्िरूप िाले सरकार का प्रािधान है। कै वबनेि िस्तुतः सरकार का
एक बहुल या कॉलेवजएि रूप है। आसके ऄंतगणत शवक्त क्रकसी एक व्यवक्त में वनवहत न होकर संपूणण
मंवत्रपररषद् में वनवहत होती है। आसके पीछे वसद्धांत यह है क्रक “सभी मंत्री या तो सामूवहक रूप से
सदन में बने रहते है या बहुमत न रहने पर ईनके िारा सामूवहक रूप से त्यागपत्र क्रदया जाता है ”
(“All Ministers sink and swim together.”)। यह वनचले सदन के प्रवत सामूवहक
ईर्त्रदावयत्ि पर अधाररत है।
 कै वबनेि की ईत्पवर्त् राजा को सलाह देने के वलए गरठत वप्रिी काईं वसल से हुइ है। कै वबनेि की
भूवमका में वनम्नवलवखत सवम्मवलत हैं:
o नीवत को स्िीकृ वत प्रदान करना (प्रमुख नीवत-वनमाणण वनकाय),
o वििादों का समाधान,
o प्रधानमंत्री को वििश करना,
o एकीकृ त सरकार,
o संसदीय दल को एकीकृ त करना, अक्रद।
 आसके ऄवतररक्त, कै वबनेि, संसदीय प्रणाली में कानून वनमाणण हेतु सिणश्रेष्ठ वनकाय के रूप में मौजूद
होता है। यह ईस दल/समूह से गरठत होता है, वजसे सदन में बहुमत प्राप्त होता है। कै वबनेि की
बैठक का अयोजन गुप्त रूप से क्रकया जाता है।

2.2.1.3.1. वििे न का प्रधानमं त्री


प्रधानमंत्री की वस्थवत
 प्रधानमंत्री, राज्य प्रमुख होता है (राज्य रूपी जहाज का कप्तान)।
 प्रधानमंत्री, मंवत्रमंडल का प्रमुख होता है।
 प्रधानमंत्री, सदन में बहुमत प्राप्त दल से संबंवधत व्यवक्त होता है।
 िह राजा और कै वबनेि तथा राजा और संसद के बीच कड़ी का कायण करता है।
 सदन का कायणकाल प्रधानमंत्री पर वनभणर करता है। िह सदन को भंग करने की सलाह भी दे सकता
है।
 ऄन्य मंत्री, प्रधानमंत्री की सलाह पर वनयुक्त क्रकए जाते हैं।
 मंवत्रयों का कायणकाल भी प्रधानमंत्री पर वनभणर करता है।

2.2.1.3.2. प्रधानमं त्री समकक्षों में प्रथम होता है


 आसे Primus Inter Pares ऄथिा Inter Stella Luna Minores भी कहा जाता है। यह ऄन्य
मंवत्रयों के संबंध में प्रधानमंत्री की वस्थवत को व्यक्त करता है। कै वबनेि प्रणाली में सामूवहक
ईर्त्रदावयत्ि का वसद्धांत लागू होता है; आसवलए प्रधानमंत्री के साथ-साथ ऄन्य मंत्री भी महत्िपूणण
होते हैं।
 संसदीय प्रणाली में प्रधानमंत्री और ऄन्य मंवत्रयों की सापेवक्षक वस्थवत की तुलना राष्ट्रपतीय
प्रणाली में राष्ट्रपवत और ईसके सवचि की सापेवक्षक वस्थवत से की जा सकती है।
 राष्ट्रपतीय प्रणाली (Presidential system) में, मंवत्रमंडल के सदस्यों को राष्ट्रपवत िारा चुना
जाता है। संयुक्त राज्य ऄमेररका में, स्पॉआल्स वसस्िम (spoils system) मौजूद है। ऄमेररका में
सवचि, कांग्रेस के सदस्य नहीं होते हैं।

12 www.visionias.in ©Vision IAS

Google it:- https://upscpdf.com


https://upscpdf.com << Download From >> https://upscpdf.com

 संसदीय प्रणाली में मंत्री क्रकसी भी सदन के सदस्य हो सकते हैं। प्रधानमंत्री ईनके साथ ऄपने
ऄधीनस्थ के रूप में व्यिहार नहीं कर सकता है। सैद्धांवतक रूप से, प्रधानमंत्री को स्ियं को ऄपने
समकक्षों में प्रथम मानना चावहए तथा मंवत्रमंडल के ऄन्य सदस्यों को सम्मान देना चावहए और
ईनके साथ विचार-विमशण कर वनणणय लेने चावहए।
 हालांक्रक, व्यिहार में प्रधानमंत्री की वस्थवत सिोच्च होती है क्योंक्रक:
o ईसे ही सिणप्रथम वनयुक्त क्रकया जाता है तथा िह हाईस ऑफ कॉमंस का नेता होता है।
o ऄन्य मंवत्रयों को ईसकी सलाह पर वनयुक्त क्रकया जाता है।
o ऄन्य मंवत्रयों को ईसकी सलाह पर हिाया जा सकता है।

2.2.1.3.3. प्रधानमं त्री विवभन्न तारों के बीच एक चन्रमा की भां वत होता है ( P.M. As Moon
Among Stars)
 यह कथन प्रधानमंत्री की ऄवधक िास्तविक वस्थवत को प्रस्तुत करता है। व्यिहार में, प्रधानमंत्री को
विवशष्टता प्राप्त होती है और िह के िल समकक्षों के मध्य प्रथम नहीं होता है। औपचाररक और
ऄनौपचाररक दोनों कारक आसके वलए ईर्त्रदायी हैं।
o औपचाररक कारक: िह संसद और राजा के बीच की कड़ी है और ईसकी सलाह पर मंवत्रयों की
वनयुवक्त की जाती है ऄथिा ईन्हें हिाया जाता है।
o ऄनौपचाररक कारक: व्यवक्तत्ि कारक, ईसके दल की वस्थवत, बाह्य/अंतररक अपातकाल जैसी
वस्थवत।

2.2.1.4. विरिश और भारतीय प्रधानमं त्री के मध्य ऄं त र


 भारतीय प्रधानमंत्री की संिैधावनक वस्थवत एक ऄंतर के साथ विरिश प्रधानमंत्री के समान है।
भारत में, प्रधानमंत्री संसद के क्रकसी भी सदन (ऄथाणत् लोकसभा या राज्यसभा) का सदस्य हो
सकता है। हालांक्रक, वििेन में ऐसा नहीं है। वििेन में एक परंपरा है क्रक प्रधानमंत्री सदैि के िल
वनचले सदन (हाईस ऑफ कॉमंस) का सदस्य होगा।

2.2.1.5. वप्रिी काईं वसल


 यह राजा के सलाहकारी वनकायों में से एक होता था, परंतु ितणमान में यह कै वबनेि के ईद्भि के
कारण ऄपनी प्रासंवगकता खो चुका है। ितणमान में कै वबनेि के वनणणय ही वप्रिी काईं वसल के वनणणय
होते हैं। आनकी ऑक्सफोडण, कै वम्िज विश्वविद्यालय अक्रद के संबंध में कु छ पयणिेक्षी भूवमका है। यह
एडवमरल्िी मामलों में ऄपीलीय ऄदालत के साथ-साथ चचण से संबंवधत वििादों के समाधान में भी
कु छ भूवमका वनभाती है।

2.2.1.6. स्थायी वसविल से ि क/विरिश नौकरशाह


 भारतीय नौकरशाही, विरिश नौकरशाही प्रणाली के काफी समरूप है।
 प्रमुख विशेषताएँ:
o वििेन में नौकरशाही सामान्यज्ञ (generalist) है।
o नौकरशाहों से राजनीवतक रूप से तिस्थ रहने की ऄपेक्षा की जाती है।
o प्रवतयोगी परीक्षाओं के माध्यम से भती।
o कइ ईन्मुवक्तयां प्राप्त।
o यह माना जाता है क्रक विरिश नौकरशाही प्रवतवनवधिादी नहीं है। यह ऄभी भी संभ्ांतिादी है।
o नौकरशाहों को निीन स्िेच्छाचारी (New Despots) के रूप में जाना जाता है।
o यह कहा जाता है क्रक नौकरशाही, मंवत्रमंडलीय ईर्त्रदावयत्िों के तहत कायण करती है।
o आसकी तुलना फ्ें कस्िीन के राक्षस से की गइ है (मंवत्रयों की ऄत्यवधक शवक्तशाली वस्थवत)।

13 www.visionias.in ©Vision IAS

Google it:- https://upscpdf.com


https://upscpdf.com << Download From >> https://upscpdf.com

2.2.2. विधावयका (Legislature)

2.2.2.1. दो प्रणावलयों के मध्य मौवलक ऄं त र


 स्िाभाविक रूप से विरिश संसद के साथ भारतीय संसद की तुलना की जाती रही है। लेक्रकन
हमारी संसद तथा संसदीय संस्थाएं और प्रक्रक्रयाएं , िेस्िबमस्िर प्रणाली का के िल एक प्रवतरूप नहीं
हैं। ईनकी प्रणाली और हमारी प्रणाली के मध्य मौवलक ऄंतर हैं।
 विरिश संसद का विकास लगभग तीन सौ िषों के दौरान हुअ है। वििेन में संसद संप्रभु शवक्तयों का
ईपयोग करने िाली एकमात्र संस्था है क्योंक्रक िहाँ कोइ वलवखत संविधान नहीं है।
 दूसरी तरफ, भारत में वलवखत संविधान है। यहाँ सरकार के सभी ऄंग और प्रत्येक प्रावधकारी की
शवक्तयां ि प्रावधकार संिैधावनक दस्तािेजों िारा पररभावषत और सीमांक्रकत हैं।
 संसद की शवक्तयां भी स्ियं संविधान िारा स्पष्ट रूप से पररभावषत और सीमांक्रकत की गयी हैं।
हालांक्रक, संसद ऄपने ही क्षेत्र के भीतर सिोच्च है। आसके ऄवतररक्त, संसद लोगों की प्रवतवनवध
संस्था है। लेक्रकन यह विरिश संसद के समान संप्रभु नहीं है जो कु छ भी कर सकती है या पूिणित
वस्थवत को स्थावपत कर सकती है। ईल्लेखनीय है क्रक संिैधावनक संप्रभुता के ऄथण में विरिश संसद
की शवक्तयां क्रकसी संिैधावनक दस्तािेज के िारा सीवमत नहीं हैं।
 िहीं हमारा संविधान व्यवक्त को मूल ऄवधकार प्रदान करता है। आन्हें वनषेधात्मक ऄवधकार भी कहा
जाता है और ये न्यायालय िारा प्रितणनीय (न्यायोवचत) हैं। संसद िारा पाररत कोइ भी कानून जो
क्रकसी भी मूल ऄवधकार को न्यून करते हैं, ईसे न्यायालयों िारा ऄवधकारातीत घोवषत क्रकया जा
सकता है।
 न्यायालय िारा वििादों पर वनणणय क्रदए जाते हैं और आस क्रम में िे संविधान और कानूनों की
व्याख्या कर सकते हैं। आसके ऄवतररक्त, संसद को संिैधावनक ऄवधकार प्राप्त हैं और कु छ सीमाओं के
भीतर यह संविधान में ईपयुक्त संशोधन कर सकती है।
 विरिश संसद विसदनीय है, यथा- हाईस ऑफ लॉर्डसण (संख्या वनधाणररत नहीं) और हाईस ऑफ
कॉमंस (650 सदस्य)। हाईस ऑफ लॉर्डसण में िंशानुगत सदस्य होते हैं। आसके ऄवतररक्त, हाईस

ऑफ लॉर्डसण में सिाणवधक संख्या में लाआफ वपयसण, चचण/ररलीवजयस वपयसण (चचण संबंधी वपयसण) और
लॉ लॉर्डसण हैं।

2.2.2.2. हाईस ऑफ लॉर्डसण

 हाईस ऑफ लॉर्डसण िस्तुतः यूनाआिेड ककगडम के विसदनात्मक संसद का दूसरा चैम्बर या ईच्च सदन
है। ितणमान समय में आसमें लगभग 800 सदस्य हैं।

 यूनाआिेड ककगडम की संसद का गठन हाईस ऑफ कॉमंस, हाईस ऑफ़ लॉर्डसण और क्राईन से

वमलकर हुअ है। हाईस ऑफ लॉर्डसण में वनम्नवलवखत प्रकार के सदस्य होते हैं:
o लाआफ वपयसण (Life peers): आनकी संख्या (लगभग 700) सदन में सिाणवधक है। आन्हें वनयुक्त

करने की शवक्त औपचाररक रूप से क्राईन के पास है, लेक्रकन सदस्यों को ऄवनिायण रूप से

प्रधानमंत्री की सलाह पर क्वीन (queen) िारा वनयुक्त क्रकया जाता है। लाआफ वपयसण की
ईपावध मृत्यु के साथ ही समाप्त हो जाती है।

o वबशप (Bishops): आंग्लैंड के चचण के 26 िररष्ठ वबशपों को सदन की सदस्यता प्रदान की गयी
है। आसका कारण यह है क्रक आं ग्लैंड के चचण राज्य िारा 'स्थावपत' चचण हैं। जब ये वबशप चचण
से सेिावनिृत होते हैं, तो सदन से आनकी सदस्यता भी समाप्त हो जाती है।

14 www.visionias.in ©Vision IAS

Google it:- https://upscpdf.com


https://upscpdf.com << Download From >> https://upscpdf.com

o वनिाणवचत िंशानुगत वपयसण (Elected Hereditary peers): हाईस ऑफ लॉर्डसण

ऄवधवनयम, 1999 िारा िंशानुगत वपयसण के हाईस ऑफ लॉर्डसण में ईपवस्थत होने और

मतदान करने का ऄवधकार समाप्त कर क्रदया गया था। तब तक िहाँ लगभग 700 िंशानुगत
वपयसण थे। परंतु आस विधेयक पर विचार की प्रक्रक्रया के दौरान ही एक संशोधन कर (लॉडण िेदरइल
िारा प्रस्तावित िेदरइल संशोधन के रूप में प्रवसद्ध) मौजूदा िंशानुगत वपयसण में से 92 वपयसण को
सदस्य बने रहने की ऄनुमवत दी गयी।
 हाईस ऑफ लॉर्डसण के सदस्य क्रकसी संशोधन को प्रस्तावित कर सकते हैं या कोइ संशोधन
कर सकते हैं। हालांक्रक, हाईस ऑफ लॉर्डसण की शवक्तयां सीवमत हैं; यक्रद यह क्रकसी कानून

के भाग का ऄनुमोदन नहीं करता है, तो आसके िारा कानून के पाररत होने की ऄिवध को के िल एक

िषण (धन विधेयक को ऄवधकतम एक माह) तक के वलए विलंवबत क्रकया जा सकता है। साथ ही,
यह सुवनवश्चत करने के वलए कठोर वनयम विद्यमान हैं क्रक हाईस ऑफ कॉमंस की आच्छाओं और
ईस समय की सरकार के वनणणय का ऄनुपालन क्रकया जाए।

 िास्ति में, हाईस ऑफ लॉर्डसण को विश्व के सबसे कमजोर उपरी सदनों में से एक के रूप में

वचवननत क्रकया जा सकता है। 1919 और 1949 के ऄवधवनयम के पाररत होने के ईपरांत हाईस
ऑफ लॉर्डसण ने सभी िास्तविक विधायी शवक्तयों को खो क्रदया है। ितणमान में यह के िल एक
विलंबकारी सदन बन गया है। यह साधारण विधेयक के मामले में ऄवधकतम एक िषण की ऄिवध के
वलए और धन विधेयक के मामले में ऄवधकतम एक महीने की ऄिवध के वलए विलंब कर सकता है।
 राज्यसभा की तुलना में, हाईस ऑफ लॉर्डसण एक कमजोर सदन है। राज्यसभा के पास साधारण

विधेयक के मामले में (हालांक्रक, संयुक्त ऄवधिेशन का प्रािधान है लेक्रकन यह एक ऄसाधारण

ईपकरण है) लोकसभा के समान ऄवधकार हैं।

 जहाँ तक संविधान संशोधन का प्रश्न है,राज्यसभा को लोकसभा के समान शवक्तयां प्राप्त हैं। धन
विधेयक के संबंध में राज्यसभा भी हाईस ऑफ़ लॉर्डसण के समान विलंब करने िाला सदन है।
राज्यसभा ऄवधकतम चौदह क्रदनों के वलए धन विधेयक को विलंवबत कर सकता है। राज्यसभा के
पास कु छ विशेष शवक्तयां हैं, जो लोकसभा के पास नहीं हैं (ईदाहरण के वलए: ऄनुच्छेद 249 और

ऄनुच्छेद 312)।

2.2.2.2.1. हाईस ऑफ लॉर्डसण और सं यु क्त राज्य ऄमे ररका के सीने ि के मध्य तुल ना

 सीनेि को शवक्तशाली उपरी सदन कहा जाता है। आसे साधारण विधेयक, संिैधावनक विधेयक और
यहां तक क्रक धन विधेयक को पाररत करने के संदभण में हाईस ऑफ़ ररप्रेजेन्िेरिव्स के समान शवक्तयाँ
प्राप्त हैं। वनचले सदन में धन विधेयक को प्रस्तुत करना प्रथागत है।
 सीनेि को कु छ विशेष शवक्तयां भी प्राप्त हैं, जो हाईस ऑफ़ ररप्रेजेन्िेरिव्स के वलए ईपलब्ध नहीं है।

ईदाहरण के वलए, ऄंतराणष्ट्रीय संवधयों का ऄनुसमथणन, ईच्चतर वनयुवक्तयों का ऄनुसमथणन आत्याक्रद।


 हाईस ऑफ लॉर्डसण को पूिण में एक विशेषावधकार प्राप्त था क्रक यह वििेन में ऄपील का सिोच्च
न्यायालय था। परंतु संिैधावनक सुधार ऄवधवनयम, 2005 िारा सुप्रीम कोिण के सृजन के साथ

(ईच्चतम न्यायालय िषण 2009 में स्थावपत) ही ऄब आसका ऄवस्तत्ि समाप्त हो गया।

15 www.visionias.in ©Vision IAS

Google it:- https://upscpdf.com


https://upscpdf.com << Download From >> https://upscpdf.com

2.2.2.2.2. हाईस ऑफ लॉर्डसण में सु धार


 हाईस ऑफ लॉर्डसण में लंबी ऄिवध से सुधार क्रकए जा रहे हैं। कु छ प्रमुख क्रकए गए सुधार
वनम्नवलवखत हैं:
o लाआफ वपयसण का समािेशन।
o िंशानुगत वपयसण की संख्या को सीवमत करना।
o 1919 और 1949 के ऄवधवनयम ने आसे एक विलंबकारी संस्था बना क्रदया है।
o संिैधावनक सुधार ऄवधवनयम, 2005 िारा ऄपील की सिोच्च न्यायालय के रूप में आसकी
भूवमका को समाप्त कर क्रदया गया।
o लॉडण चांसलर के स्थान पर ऄब यहाँ लाडण स्पीकर िारा ऄध्यक्षता का प्रािधान क्रकया गया है।

2.2.2.2.3. लं वबत सु धार

 आसके नाम को पररिर्वतत करना, क्योंक्रक यह ऄलोकतांवत्रक है।


 आसकी सदस्य संख्या को कम करना क्योंक्रक आसमें बड़ी संख्या में मनोनीत सदस्य हैं। प्रस्ताि है क्रक
सदस्यों को वनिाणवचत क्रकया जाए।
 िंशानुगत वपयसण का दजाण समाप्त करना।

2.2.2.3. हाईस ऑफ़ कॉमं स


 यह वनचला सदन है, लेक्रकन सिाणवधक ऄवधकार आसी के पास हैं। आसकी ऄध्यक्षता स्पीकर (ऄध्यक्ष)
िारा की जाती है। ऄमेररकी हाईस ऑफ़ ररप्रजेन्िेरिव्स के स्पीकर के विपरीत, यह पद गैर-
राजनीवतक है और िास्ति में, परंपरा के ऄनुसार, विवभन्न राजनीवतक दल ऄध्यक्ष के संसदीय क्षेत्र
में ऄपने ईम्मीदिार खड़े नहीं करते।
 सदस्यों की संख्या समय-समय पर जनसंख्या पररितणन को प्रवतबबवबत करने हेतु पररिर्वतत होती
जाती है।
 ितणमान पररिेश में, प्रधानमंत्री सरकार का प्रमुख होता है और सदैि बहुमत प्राप्त दल या हाईस
ऑफ कॉमंस में गठबंधन िाले दल का सदस्य होता है। कै वबनेि में मुख्य रूप से हाईस ऑफ़ कॉमंस
में बहुमत प्राप्त दल के सदस्य सवम्मवलत होते हैं, हालांक्रक हाईस ऑफ लॉर्डसण के सदस्यों ने भी
कै वबनेि मंत्री के रूप में कायण क्रकया है। िास्ति में विगत कु छ िषों में यह देखने को वमला है क्रक,
संसद से बाहर के क्रकसी व्यवक्त को “लाआफ वपयसण” के रूप में नावमत करना, वनजी जीिन से क्रकसी
व्यवक्त को सरकार में सवम्मवलत करने का एक माध्यम बन गया है।
 यद्यवप, प्रधानमंत्री सरकार का प्रमुख और एक सांसद होता है, तथावप िह सामान्यतः हाईस ऑफ
कॉमंस का नेता नहीं होता है। हाईस ऑफ कॉमंस का नेता सरकार का एक सदस्य होता है तथा
हाईस ऑफ कॉमंस के अंतररक संचालन के मामलों में बहुमत दल का मुख्य प्रिक्ता होता है। सदन
(हाईस ऑफ कॉमंस) के नेता का कायाणलय हाईस ऑफ़ कॉमंस के अगामी कायणक्रम की घोषणा
करता है।
 हाईस ऑफ कॉमंस में पािी संगठन (ररपवब्लकन कांफ्ेंस या डेमोक्रेरिक कॉकस के सदृश) वनयवमत
रूप से नीवतयों पर चचाण करने के वलए बैठक करते हैं और एक वनजी मंच पर मंवत्रयों या छाया
मंवत्रमंडल (shadow cabinet) के सदस्यों के समक्ष ऄपने विचार प्रस्तुत करने हेतु पािी के बैक
बेंचसण सदस्यों (वजन्हें सरकार में पद प्राप्त नहीं होता है) को एक ऄिसर प्रदान करते हैं।

16 www.visionias.in ©Vision IAS

Google it:- https://upscpdf.com


https://upscpdf.com << Download From >> https://upscpdf.com

2.2.2.4. भारतीय और ऄमे ररकी स्पीकर (ऄध्यक्ष) के साथ हाईस ऑफ कॉमं स के स्पीकर की
वस्थवत की तु ल ना

2.2.2.4.1. विरिश स्पीकर की विशे ष ताएँ


 यहाँ स्पीकर के पद को ऄत्यवधक प्रवतष्ठा प्राप्त है। वििेन में एक परंपरा है क्रक एक बार ऄध्यक्ष बनने
के पश्चात् िह सदैि के वलए ऄध्यक्ष बन जाता है। ईल्लेखनीय है क्रक ऄध्यक्ष का वनिाणचन क्षेत्र
वनर्विरोध होता है। एक बार जब क्रकसी व्यवक्त को ऄध्यक्ष के रूप में वनयुक्त क्रकया जाता है तो िह
ऄपने राजनीवतक दल से औपचाररक रूप से त्यागपत्र दे देता है। ईसके पास वनणाणयक मत देने और
सदन के संचालन तथा सांसदों के अचरण के संबंध में ऄंवतम ऄनुशासनात्मक कायणिाही करने का
ऄवधकार होता है।

2.2.2.4.2. ऄमे ररका में स्पीकर (हाईस ऑफ़ ररप्रे जे न्िे रिव्स का ऄध्यक्ष)
 सामान्यतः िह क्रकसी पािी का सदस्य होता है तथा ईससे तिस्थ रहने की ऄपेक्षा नहीं की जाती
है। िह ऄपनी पािी का पक्ष-समथणन करता है। ईनके पास ऄंवतम ऄनुशासनात्मक कायणिाही करने
की शवक्तयां नहीं होती हैं, जो स्ियं सदन में वनवहत होती हैं। संयुक्त राज्य ऄमेररका में ऄध्यक्ष
प्रारंवभक मतदान कर सकता है।

2.2.2.4.3. भारत में लोकसभा का ऄध्यक्ष

 हालांक्रक, भारत की वस्थवत विरिश और ऄमेररकी मॉडल के मध्य की है। यह सैद्धांवतक रूप से
विरिश मॉडल के करीब है। लेक्रकन यहाँ समान परंपरा मौजूद नहीं है। ईदाहरणाथण:
o यहाँ ऄध्यक्ष के वलए यह अिश्यक नहीं होता क्रक िह ऄपनी पािी से त्यागपत्र दे।
o यक्रद ईसने त्यागपत्र देने का वनणणय क्रकया है तो ईसे दल-बदल विरोधी कानून के तहत वनरहण
घोवषत नहीं क्रकया जाएगा।
o भारत में ऄध्यक्ष के वनर्विरोध वनिाणवचत क्रकए जाने की कोइ परंपरा नहीं है।

2.2.3. न्यायपावलका

 संसदीय संप्रभुता के वसद्धांत के तहत, न्यायपावलका के पास संसद के ऄवधवनयम को समाप्त करने
के वलए स्िाभाविक शवक्त का ऄभाि है। हालांक्रक, सांविवधक कानून के सामान्य कानून के
ऄधीनस्थ होने का तात्पयण यह नहीं है क्रक न्यायपावलका कायणपावलका के ऄधीनस्थ है। वििेन में
न्यायालयों को कु छ शवक्तयाँ प्राप्त हैं, यथा:
o सांविवधक कानून के ऄथण की सिीक व्याख्या।
o ऄवधकारातीत (शवक्तयों से परे) के वसद्धांत को लागू कर मंवत्रयों और ऄन्य सरकारी
ऄवधकाररयों के कायों की समीक्षा।
o मंवत्रयों और दूसरों के कायों पर प्राकृ वतक न्याय की ऄिधारणा को लागू करना।
 चूंक्रक यहाँ संसद संप्रभु है, ऄतः यहाँ सरकार संशोधन कानून पाररत करके न्यायालयों के वनणणयों
को ईलिने का प्रयास कर सकती है। न्यावयक पुनर्विलोकन की शवक्त, नीवत प्रक्रक्रया में
न्यायपावलका को संभावित महत्िपूणण भूवमका प्रदान करती है।
 हाल के दशकों में, िहाँ कइ कारणों से न्यावयक सक्रक्रयता (judicial activism) में िृवद्ध देखी गइ है:
o न्यायाधीश, मंत्री की कायणिाही की समीक्षा करने और ईन्हें रद्द करने के वलए ऄवधक आच्छु क हैं।
o यूरोवपयन कन्िेंशन ऑन नयूमन राआि (ECHR) को घरेलू कानून में समािेश करना।

17 www.visionias.in ©Vision IAS

Google it:- https://upscpdf.com


https://upscpdf.com << Download From >> https://upscpdf.com

o स्कॉिलैंड, िेल्स और ईर्त्री अयरलैंड में वनिाणवचत विधानसभाओं के वलए शवक्तयों का


हस्तांतरण।
o िषण 2009 में सुप्रीम कोिण का गठन।

2.2.3.1. भारतीय और विरिश न्यायपावलका के मध्य तु ल ना


ऄंतर
 विरिश प्रणाली के मामले में, 'मूल ढांचे (Basic Structure)’ की ऄिधारणा के ऄभाि के कारण
संशोधन करने की संसद की शवक्त क्रकसी भी न्यावयक वनणणय का ऄवतक्रमण कर सकती है। जबक्रक,
भारतीय न्यायपावलका प्रणाली के मामले में, 'मूल ढांचे’ की ऄिधारणा ने न्यायपावलका को एक
शवक्तशाली ईपकरण प्रदान क्रकया है, वजसके िारा यह क्रकसी भी ऐसी कायणपावलका या विधायी
कायणिाही को वनरस्त कर सकती है जो संविधान की मूल भािना के विरुद्ध प्रतीत होती है।
 विरिश विवधक प्रणाली पूरी तरह से 'सामान्य विवध प्रणाली' (Common Law System) पर
अधाररत है। सामान्य विवध प्रणाली का तात्पयण यह है क्रक कानून , न्यायाधीशों के फ़ै सलों, अदेशों
या वनणणयों (पूिण वनणणयों से भी संबंवधत) के माध्यम से विकवसत हुए हैं। हालांक्रक, विरिश प्रणाली
जो पूरी तरह से सामान्य विवध प्रणाली (वििेन में ईद्धि) पर अधाररत है, के विपरीत, भारतीय
प्रणाली में सामान्य विवध प्रणाली के साथ िैधावनक और वनयामक कानूनों को भी सवम्मवलत क्रकया
गया है।
समानताएं
 दोनों प्रणावलयों में कायणपावलका की कायणिाही को ऄवधकारातीत घोवषत क्रकया जा सकता है।
 न्यायपावलका को संविधान का सबसे बड़ा व्याख्याकार माना जाता है।
 हाल के समय में, वििेन में न्यावयक सक्रक्रयता में िृवद्ध हुइ है और न्यायपावलका ऄत्यवधक सक्रक्रय
होती जा रही है। भारतीय संदभण में भी न्यावयक सक्रक्रयता में िृवद्ध हुइ है।
नोि: वििेन में संिैधावनक सुधार ऄवधवनयम, 2005 के माध्यम से सिोच्च न्यायालय को ऄपील के
सिोच्च न्यायालय के रूप में स्थावपत क्रकया गया। एक राष्ट्रीय न्यावयक वनयुवक्त अयोग को प्रस्तावित
क्रकया गया।

2.4. ईपयुण क्त तु ल ना का एक सं वक्षप्त सारां श

2.4.1. विरिश सं विधान

 ऐवतहावसक अधार पर ईद्धि और विकास का पररणाम।


 वसद्धांत और व्यिहार के मध्य ऄंतर।
 लचीला और एकात्मक संविधान।
 संसदीय सरकार।
 विवध का शासन और नागररक स्ितंत्रता लागू।
भारतीय संविधान विरिश संविधान

वलवखत ऄवलवखत

संघीय एकात्मक

कें र और राज्यों के मध्य शवक्तयों का विभाजन शवक्त कें र में वनवहत

राजतंत्र नहीं/गणतंत्र राजा/रानी की वस्थवत/संिैधावनक राजतंत्र

18 www.visionias.in ©Vision IAS

Google it:- https://upscpdf.com


https://upscpdf.com << Download From >> https://upscpdf.com

2.4.2. विरिश सम्राि और भारत के राष्ट्रपवत के मध्य तु ल ना

विरिश सम्राि भारत के राष्ट्रपवत

राजा की वस्थवत िंशानुगत वनिाणवचत

राजा को पूणण ईन्मुवक्त प्राप्त है; भारत में राष्ट्रपवत पर संविधान के ईल्लंघन के अधार पर
यह कहा जाता है क्रक राजा कोइ महावभयोग चलाया जा सकता है।
भी गलती नहीं कर सकता।

राजा के पास कोइ वििेकाधीन भारत में भारतीय राष्ट्रपवत के संबंध में स्पष्टता का ऄभाि है। आस
शवक्तयां नहीं है। ईसे 'गोल्डन संबंध में भी भ्ांवत बनी हुइ है क्रक क्या ईसके पास कोइ
जीरो' के रूप में जाना जाता है। वििेकाधीन शवक्त है या िह के िल एक रबर स्िाम्प है।
 24िें संविधान संशोधन ऄवधवनयम िारा स्पष्ट क्रकया गया क्रक
ईसके पास कोइ वििेकाधीन शवक्तयां नहीं हैं। िास्तविक
शवक्त, प्रधानमंत्री के पास वनवहत है, जबक्रक राष्ट्रपवत के िल
एक 'रबर स्िाम्प' है।
 44िें संविधान संशोधन ऄवधवनयम के माध्यम से पुनः आस
वस्थवत को ईलि क्रदया गया तथा राष्ट्रपवत को कु छ
वििेकाधीन शवक्तयां प्रदान की गइ। ितणमान में िह क्रकसी
कानून को मंवत्रपररषद (CoM) के पास पुनर्विचार हेतु भेज
(के िल एक बार) सकता है।

2.4.3. विरिश सम्राि और ऄमे ररकी राष्ट्रपवत के मध्य तु ल ना

विरिश सम्राि ऄमेररकी राष्ट्रपवत

राजा नाममात्र का प्रमुख राष्ट्रपवत िास्तविक प्रमुख और नाममात्र का प्रमुख दोनों होता है।

िंशानुगत वनिाणवचत; और महावभयोग चलाया जा सकता है।

कोइ वििेकाधीन प्राप्त शवक्तयां िास्तविक कायणकारी शवक्तयां, हालांक्रक वनयंत्रण एिं संतुलन के
नहीं ऄधीन।

3. संयुक्त राज्य ऄमेररका का सं विधान


3.1. प्रमु ख विशे ष ताएँ

 विश्व के प्रमुख देशों के संविधान की तुलना में ऄमेररकी संविधान सबसे छोिा और प्रथम वलवखत
संविधान है, जबक्रक भारतीय संविधान विश्व का सबसे लंबा वलवखत संविधान हैI
 संयुक्त राज्य ऄमेररका के संविधान को ऄत्यंत कठोर (संशोधन प्रक्रक्रया करठन) संविधान माना
जाता है वजसमें के िल 7 ऄनुच्छेद ही हैं और ऄभी तक आसमें के िल 27 बार संशोधन क्रकया गया हैI
मूल रूप से, भारतीय संविधान में 8 ऄनुसूवचयां 22 भाग और 395 ऄनुच्छेद शावमल थे। ितणमान
समय में आसमें 12 ऄनुसूवचयां 25 भाग और 448 ऄनुच्छेद सवम्मवलत हैं।

19 www.visionias.in ©Vision IAS

Google it:- https://upscpdf.com


https://upscpdf.com << Download From >> https://upscpdf.com

 संयुक्त राज्य ऄमेररका के संविधान को 17 वसतम्बर 1787 को अयोवजत एक कं िेंशन में ऄंवतम रूप
क्रदया गया था, वजसे लागू करने के वलए कम से कम 9 राज्यों के ऄनुसमथणन की अिश्यकता थी।
जुलाइ 1788 तक 11 राज्यों िारा आसका ऄनुमोदन कर क्रदया गया था और 13 वसतम्बर 1788 को
ऄमेररकी संविधान को लागू क्रकया गया। दूसरी तरफ भारतीय संविधान को 26 निंबर 1949 को
संविधान सभा िारा ऄंगीकृ त क्रकया गया और यह 26 जनिरी 1950 को ऄवस्तत्ि में अया।
 संयुक्त राज्य ऄमेररका में नागररकता और संविधान के मामले में दोहरी नीवत के वसद्धांत को
ऄपनाया गया है, वजसके तहत दो संविधान हैं, पहला, संपूणण देश के वलए और दूसरा प्रत्येक राज्य
का ऄपना संविधान। आसके साथ ही ऄमेररकी नागररकों के वलए दोहरी नागररकता का प्रािधान
क्रकया गया है। पहला, संयुक्त राज्य ऄमेररका की नागररकता और दूसरी, प्रत्येक संबंवधत राज्य की
नागररकताI दूसरी तरफ, भारत में सभी नागररकों के वलए एक संविधान और एकल नागररकता
की संकल्पना को ऄपनाया गया है।

3.1.1. सं विधान की प्रकृ वत

 ऄमेररकी संविधान को मूलतः संघीय संविधान के रूप में िर्वणत क्रकया जाता है , वजसे 50 स्ितंत्र
राज्यों िारा ऄनुमोक्रदत क्रकया गया है। आसके ऄवतररक्त संघ सरकार और राज्य सरकारों का ऄपना
संविधान है और ये दोनों एक दूसरे के कायों में हस्तक्षेप नहीं करते हैं।
 दूसरी तरफ, भारत में एक संविधान है तथा राज्य सरकार के कायणकलापों में संघ सरकार
वनम्नवलवखत रूप में हस्तक्षेप करती है:
o राज्यपालों की वनयुवक्तयां।
o राज्यपाल के पास राज्य विधान-मंडल िारा पाररत विधेयक को राष्ट्रपवत की स्िीकृ वत के वलए
अरवक्षत रखने की शवक्त।
o राज्यों में राष्ट्रपवत शासन लागू करने की संघ सरकार की शवक्त।

3.1.2. सं घ िाद की प्रकृ वत

 ऄमेररका एक िैध संघ (Dual Federation) का जबक्रक भारत एक सहकारी संघ (Cooperative
Federation) का ईदाहरण है।

िैध संघ (संयक्त


ु राज्य ऄमेररका) सहकारी संघ (भारत)

 ऐसे संघ में कें र और राज्य दोनों पूणण  ऐसे संघ में कें र और राज्य दोनों एक दूसरे से स्ितंत्र
रूप से स्ितंत्र होते हैं और िे ऄपने नहीं होते हैं, बवल्क ऄपने कायणवनष्पादन के वलए एक
अप में पूणण सरकार होते हैं।
दूसरे पर वनभणर रहते हैं। सामान्यतः कें र के पास
 ऄपकें रीय संघिाद (Centrifugal
ऄवधक वनयंत्रणकारी शवक्तयाँ होती हैं।
federalism)  ऄवभकें रीय संघिाद (Centripetal federalism)

 समवमत संघिाद (Symmetrical  ऄसमवमत संघिाद (Asymmetrical federalism):


federalism): सीनेि में सभी o राज्यों को ईनकी जनसंख्या के अधार पर राज्य
राज्यों को समान प्रवतवनवधत्ि क्रदया सभा में प्रवतवनवधत्ि प्रदान क्रकया गया है।

गया है। o संविधान के ऄनुच्छेद 370 और 371 में कु छ


राज्यों के वलए विशेष ईपबंध क्रकए गए हैं।
(ईल्लेखनीय है क्रक ऄनुच्छेद 370 के ऄवधकांश
प्रािधानों को वनरवसत कर क्रदया गया है।)

20 www.visionias.in ©Vision IAS

Google it:- https://upscpdf.com


https://upscpdf.com << Download From >> https://upscpdf.com

 संयुक्त राज्य ऄमेररका एक विधायी  भारत एक कायणकारी संघ है, वजसका तात्पयण यह है
संघ है, वजसका तात्पयण यह है क्रक क्रक राज्य की वस्थवत के िल कायणकारी स्तर पर ही
विवध वनमाणण की प्रक्रक्रया में राज्यों महत्िपूणण होती है।
का प्रभुत्ि होता है।

 संयुक्त राज्य ऄमेररका ऄविनाशी  भारत विनाशी राज्यों का एक ऄविनाशी संघ है।
राज्यों का एक ऄविनाशी संघ है।

 संयुक्त राज्य ऄमेररका का संविधान  भारतीय संविधान में राज्यों के वलए ऐसा कोइ
राज्यों को सीनेि के माध्यम से प्रािधान (ऄंतराणष्ट्रीय संवधयों के ऄनुमोदन से
ऄंतराणष्ट्रीय संवधयों को ऄनुमोक्रदत संबंवधत) नहीं क्रकया गया है।
करने की शवक्त प्रदान करता है।

3.1.3. सरकार का स्िरूप

संयक्त
ु राज्य ऄमेररका
 संयुक्त राज्य ऄमेररका में शासन की राष्ट्रपतीय प्रणाली (Presidential system) को ऄपनाया
गया है, वजसमें जनता सीधे कायणकारी राष्ट्रपवत का चुनाि करती है।
 राष्ट्रपवत शवक्तशाली होता है और िह कांग्रेस (ऄमेररकी संसद) के प्रवत जिाबदेह नहीं होता है।
 ऄमेररकी राष्ट्रपवत का कायणकाल 4 िषों (वनयत काल) का होता है।
 कोइ भी व्यवक्त के िल दो कायणकाल के वलए ही राष्ट्रपवत का पद धारण कर सकता है।
 सरकार के प्रशासवनक कायों में सहयोग करने हेतु राष्ट्रपवत ऄपने कमणचाररयों की वनयुवक्त स्ियं
करता है। आसके वलए यह अिश्यक नहीं है क्रक िे हाईस ऑफ़ ररप्रेजेन्िेरिव्स या सीनेि के सदस्य
हों। कमणचारी ऄमेररकी संसद (कांग्रेस) के सदनों के प्रवत ईर्त्रदायी नहीं होते हैं।
आसका ऄथण यह है क्रक सरकार के प्रशासन में ऄमेररकी राष्ट्रपवत पूणणत: स्ितंत्र है तथा प्रत्यक्ष रूप से
ऄमेररकी जनता के प्रवत ईर्त्रदायी होता है।
भारत
 भारत में शासन की संसदीय प्रणाली को ऄपनाया गया है।
 भारत का राष्ट्रपवत सरकार का कायणकारी प्रमुख होता है। ईसका वनिाणचन परोक्ष रीवत से संसद
सदस्यों और राज्य विधान-मंडल के सदस्यों (नामवनर्ददष्ट सदस्य नहीं) िारा क्रकया जाता है।
राष्ट्रपवत संसद के प्रवत ईर्त्रदायी नहीं होता है।
 राष्ट्रपवत प्रधानमंत्री और ईसके मंवत्रमंडल की सहायता और परामशण से देश की सरकार का
संचालन करता है।
 ऄमेररकी राष्ट्रपवत के विपरीत भारत का राष्ट्रपवत ऄपने पद पर 5 िषों तक बना रहता है।
 एक व्यवक्त ऄनेक बार के वलए राष्ट्रपवत पद पर वनिाणवचत हो सकता है।
राष्ट्रपवत पर महावभयोग चलाने की प्रक्रक्रया के संदभण में भारत और संयुक्त राज्य ऄमेररका के संविधान
में समानता है।

3.2. राष्ट्रपवत

 ऄमेररका में राष्ट्रपवत की वस्थवत सरकार के प्रमुख के साथ-साथ राज्य प्रमुख की भी है।

3.2.1. ऄमे ररकी राष्ट्रपवत पद के वलए ऄहण ता

 ऄमेररकी राष्ट्रपवत पद के वलए, प्राकृ वतक रूप से ऄमेररका में जन्मे ऄमेररकी नागररक ही ऄहण होते
हैं, न क्रक क्रकसी ऄन्य देश का व्यवक्त वजसने ऄमेररकी नागररकता प्राप्त की हो। आसके ऄवतररक्त, िह
35 िषण की अयु पूरी कर चुका हो तथा साथ ही कम से कम 14 िषों से ऄमेररका में ऄवधिावसत हो।

21 www.visionias.in ©Vision IAS

Google it:- https://upscpdf.com


https://upscpdf.com << Download From >> https://upscpdf.com

 दूसरी तरफ, भारत के राष्ट्रपवत पद हेतु व्यवक्त को भारत का नागररक होना चावहए, चाहे िह
जन्म-जात नागररक हो या ऄर्वजत नागररकता धारक।

3.2.2. राष्ट्रपवत का वनिाण च न

 ऄमेररकी राष्ट्रपवत का चुनाि एक वनिाणचक मंडल िारा ऄप्रत्यक्ष रीवत से क्रकया जाता है।
वनिाणचक मंडल (Electoral College)
 वनिाणचक मंडल की सदस्य संख्या = हाईस ऑफ़ ररप्रेजेन्िेरिव्स में कु ल सदस्यों की संख्या + सीनेि
के सदस्य + िाबशगिन डी.सी. से 3 सदस्य (ऄथाणत् 435+100+3 = 538)
 विजयी सदस्य को वनिाणचक मंडल के कु ल सदस्यों का पूणण बहुमत (50%+1) प्राप्त होना चावहए
(ऄथाणत् 270)।
नोि: हाईस ऑफ़ ररप्रेजेन्िेरिव्स में विवभन्न राज्यों का प्रवतवनवधत्ि (सदस्यों की संख्या) समान नहीं है,
जबक्रक सीनेि में राज्यों का प्रवतवनवधत्ि समान है।
वनिाणचकों का चुनाि (Election of Electors)
 सिणप्रथम मतदाता ऄपने मत के िारा वनिाणचक मंडल के सदस्यों का चुनाि करते हैं।
 चुनाि सूची प्रणाली (List System) िारा होता है।
 प्रत्येक राज्य का यह ईर्त्रदावयत्ि होता है क्रक िह आन चुनािों का संचालन करे।
 बहुमत प्राप्त करने िाली पािी संपूणण राज्य का प्रवतवनवधत्ि करती है।
 वनिाणचाकगण (Electors) ऄपने-ऄपने राज्य की राजधानी में एकवत्रत होकर राष्ट्रपवत पद के
ईम्मीदिार के वलए िोि करते हैं।

3.2.3. सं यु क्त राज्य ऄमे ररका के राष्ट्रपवत का प्रकायण

कायणकारी प्रकायण
 वनयुवक्तयां,
 राष्ट्र का प्रवतवनवधत्ि करना, तथा
 बजि की प्रस्तुवत।
विधायी कायण
 विधवयका में ईपवस्थवत नहीं।
 ऄमेररकी राष्ट्रपवत विधवयका को संबोवधत नहीं करता है।
 राष्ट्रपवत विधवयका को भंग नहीं कर सकता।
 राष्ट्रपवत ऄपना संदश े विधवयका को भेज सकता है। (संयुक्त राज्य ऄमेररका में संदश े भेजने की प्रथा
मौजूद है क्योंक्रक िहां शवक्तयों का पृथक्करण क्रकया गया है। आसवलए राष्ट्रपवत को ऄपनी सहभावगता
दजण कराने का यही एक साधन हैI आस प्रकार का संदश
े भेजने का प्रािधान भारत में भी है, लेक्रकन
आस प्रािधान के पीछे का तकण स्पष्ट नहीं है क्योंक्रक राष्ट्रपवत को आस मामले के संदभण में वििेकाधीन
शवक्तयां प्राप्त नहीं हैं और िह प्रधानमंत्री की सलाह पर कायण करने के वलए बाध्य होता है।)
o िीिो शवक्त: संविधान के तहत, राष्ट्रपवत, कांग्रेस िारा पाररत विधेयक पर तीन तरीकों में से
एक के माध्यम से प्रवतक्रक्रया व्यक्त कर सकता है। िह आस पर हस्ताक्षर कर सकता है , िीिो का
प्रयोग कर आसे कांग्रेस को िापस लौिा सकता है या िह आस पर कोइ प्रवतक्रक्रया व्यक्त न करे।
यक्रद राष्ट्रपवत िारा विधेयक पर कोइ प्रवतक्रक्रया व्यक्त नहीं की जाती है तो, आसके 10 क्रदन
पश्चात् (रवििार को छोड़कर) विधेयक स्ित: कानून बन जाता है। हालांक्रक, यक्रद 10 क्रदन की
ऄिवध के भीतर कांग्रेस का स्थगन हो जाता तो विधेयक, “पॉके ि िीिो” के प्रािधान के तहत
समाप्त हो जाता है। यक्रद राष्ट्रपवत विधेयक पर िीिो करता है तो आसके बािजूद भी कांग्रेस के
दोनों सदनों में 2/3 बहुमत िारा विधेयक को पास कर कानून बनाया जा सकता है।

22 www.visionias.in ©Vision IAS

Google it:- https://upscpdf.com


https://upscpdf.com << Download From >> https://upscpdf.com

3.2.4. विधायी प्रस्ताि

 संविधान, राष्ट्रपवत को ऄवधकृ त करता है क्रक िह “कांग्रेस को विधेयक के संदभण में अिश्यक और

समीचीन विचार करने हेतु ऄनुशंसा करे ”। िीिो (जो विवध-वनमाणण को रोकने का एक सीवमत और

कु छ सीमा तक एक नकारात्मक साधन है) के विपरीत, विधान-वनमाणण के संबंध में वसफाररश करने
संबंधी दावयत्ि समय के साथ प्राथवमक व्यिस्था बन गइ है वजसके िारा देश के राजनीवतक एजेंडे
को प्रभावित क्रकया जाता है।
भारतीय राष्ट्रपवत
 भारत में राष्ट्रपवत को यह शवक्त प्राप्त है क्रक िह विधेयक को संसद को पुनर्विचार हेतु लौिा सकता
हैI लेक्रकन, यक्रद संसद िारा विधेयक पर पुनर्विचार क्रकया जाता है और अिश्यक बहुमत से पाररत

कर पुनः राष्ट्रपवत को भेजा जाता है, तो राष्ट्रपवत को आस पर हस्ताक्षर (ऄन्य कोइ विकल्प ईपलब्ध
नहीं है) करना ऄवनिायण होगा।
 व्यािहाररक रूप से, गठबंधन सरकार को छोड़कर, प्रधानमंत्री और ईसकी मंवत्रपररषद को संसद में

सदैि बहुमत प्राप्त होता है, आसवलए, प्रधानमंत्री और ईसके मंवत्रमंडल को विधेयक पर राष्ट्रपवत की
सहमवत प्राप्त करने में कोइ करठनाइ नहीं होती है।
 हालांक्रक, यहाँ ऄमेररकी राष्ट्रपतीय प्रणाली की विषय-िस्तु से महत्िपूणण ऄंतर यह है क्रक ऄमेररका
के विपरीत, भारतीय संविधान में राष्ट्रपवत के वलए क्रकसी भी विधेयक पर हस्ताक्षर करने की कोइ

वनवश्चत समय-सीमा वनधाणररत नहीं की गइ है, आसवलए िह विधेयक को वबना हस्ताक्षर क्रकए
ऄवनवश्चत काल तक ऄपने पास रख सकता है जो प्रधानमंत्री और ईसके मंवत्रमंडल की विवध-
वनमाणण प्रक्रक्रया को बावधत कर सकता है। स्पष्ट रूप में, यह व्यिस्था हमें यह प्रश्न करने के वलए
प्रोत्सावहत करती है क्रक क्या भारतीय राष्ट्रपवत का पॉके ि िीिो ऄमेररकी राष्ट्रपवत के पॉके ि िीिो
से ऄवधक शवक्तशाली है।

3.2.5. शपथ और से िावनिृ वर्त् की वतवथ

संयक्त
ु राज्य ऄमेररका
 संयुक्त राज्य ऄमेररका के संविधान की एक ऄनूठी विशेषता यह है क्रक वनितणमान राष्ट्रपवत और
कांग्रेस के सदस्यों की सेिावनिृवर्त् का समय एिं वतवथ वनधाणररत है।
 राष्ट्रपवत और ईप-राष्ट्रपवत का कायणकाल वनयत ऄिवध के ईपरांत 20 जनिरी के क्रदन दोपहर में
समाप्त हो जाता है।
 आसका ऄथण है क्रक, नए राष्ट्रपवत और ईप-राष्ट्रपवत िारा शपथ ऄपने कायणकाल के प्रथम िषण की 20

जनिरी (यक्रद 20 जनिरी को रवििार है तो 21 जनिरी को) की दोपहर के समय ली जाएगी।


 राष्ट्रपवत और ईप-राष्ट्रपवत का चुनाि निंबर महीने में होता है और आसी माह में चुनाि पररणाम
भी घोवषत कर क्रदया जाता है।
आस प्रकार, ऄमेररकी जनता को ऄपने नए राष्ट्रपवत के बारे में पूिण में ही सूवचत कर क्रदया जाता है।
स्िाभाविक तौर पर, यहाँ यह प्रश्न ईठता है क्रक आस समय सारणी का पालन कै से क्रकया जाता होगा।

ईल्लेखनीय है क्रक ऄमेररकी राष्ट्रपवत की मृत्यु , पदत्याग या महावभयोग के मामले में , ईप-राष्ट्रपवत ही

शेष ऄिवध के वलए राष्ट्रपवत बन जाता है। आस प्रकार, राष्ट्रपवत का पद ररक्त नहीं रहता है और ऄगला
वनिाणवचत राष्ट्रपवत वनयत समय पर ही शपथ लेता है।

23 www.visionias.in ©Vision IAS

Google it:- https://upscpdf.com


https://upscpdf.com << Download From >> https://upscpdf.com

भारत
 भारत में, यक्रद राष्ट्रपवत की मृत्यु हो जाए ऄथिा महावभयोग के िारा या ऄपने पद से िह त्यागपत्र
दे देता है, तो ईपराष्ट्रपवत, तब तक के वलए राष्ट्रपवत बन जाता है जब तक नया चुनाि न हो जाए।
नया वनिाणवचत राष्ट्रपवत ऄपने पद पर 5 िषण की ऄिवध तक बना रहता है।
 ऄमेररकी समय-सीमाबद्ध प्रणाली के विपरीत, भारत में चुनाि की आस प्रकार की प्रणाली लागू
नहीं है।

3.2.6. प्राआमरीज (Primaries)

 प्राआमरीज एक प्रकार का चुनाि है वजसका अयोजन ईम्मीदिारों का चयन करने के वलए क्रकया
जाता है।
 आस चुनाि का अयोजन राजनीवतक दलों िारा क्रकया जाता है।

3.2.7. सं यु क्त राज्य ऄमे ररका के राष्ट्रपवत के विरूद्ध महावभयोग

 कारण: देशरोह, ररश्वत, दुराचरण संबंधी गंभीर ऄपराध। भारतीय संविधान के विपरीत, ऄमेररका
में संविधान के ईल्लंघन के अधार पर महावभयोग का कोइ प्रािधान नहीं है।
 प्रक्रक्रया
o हाईस ऑफ़ ररप्रजेंिेरिि में राष्ट्रपवत के विरूद्ध अरोप लगाए जाएंगे।
o आसे 2/3 बहुमत से पाररत क्रकया जाएगा।
o सीनेि िारा आन अरोपों की जाँच की जाएगी।
o आस प्रक्रक्रया में, संयुक्त राज्य ऄमेररका के ईच्चतम न्यायालय का मुख्य न्यायाधीश पीठासीन
ऄवधकारी होगा।
o यक्रद अरोप वसद्ध हो जाता है, तो ईसे पद से तभी हिाया जाएगा जब सीनेि िारा आस अशय
के प्रस्ताि को 2/3 बहुमत से पाररत कर क्रदया जाएगा।

3.2.8. सं यु क्त राज्य ऄमे ररका के सं विधान की कु छ शब्दािवलयां

Filibustering (विधायी कायणिाही में बाधा ईत्पन्न करना): सीनेिरों का यह एक विशेषावधकार है क्रक
िे ऄसीवमत ऄिवध तक के वलए ऄपने बोलने के ऄवधकार का प्रयोग कर सकते हैं। यह क्रकसी विधेयक
को रोकने के वलए एक ऄंवतम ईपाय है। िस्तुतः, ऄब एक नया वनयम बनाया गया है, वजसके ऄनुसार
2/3 सदस्य एक प्रस्ताि लाकर सीनेिरों के आस विशेषावधकार का वनषेध कर सकते हैं।

Senatorial Courtesy (विधायी वशष्टाचार): आसके तहत राष्ट्रपवत, औपचाररक रूप से ईच्च पदों पर

वनयुवक्त के वलए नाम वनर्ददष्ट करने से पूिण, वनयुवक्त के वलए संभावित ईम्मीदिारों के संबंध में सीनेि को
सूवचत करता है ताक्रक ऐसी वस्थवत ईत्पन्न न हो क्रक राष्ट्रपवत िारा भेजे गए नामों की पुवष्ट सीनेि न करे।

Gerrymandering (वनिाणचक क्षेत्रों का सीमांकन या गेरीमैन्डररग): यह एक ऐसी प्रक्रक्रया है वजसके


तहत वनिाणचक वजलों का सीमांकन क्रकया जाता है, वजससे राज्यों में सर्त्ारुढ़ दल को लाभ प्राप्त हो। िे
वनिाणचक वजलों को ऄपने चुनािी लाभ के ईद्देश्य से आस तरीके से सीमांक्रकत करते है क्रक ईनके समथणक
एक स्थान पर के वन्रत हो जाएं और विरोधी दल के समथणक एकजुि न हो पाएं।

24 www.visionias.in ©Vision IAS

Google it:- https://upscpdf.com


https://upscpdf.com << Download From >> https://upscpdf.com

Log Rolling (लॉग रॉबलग): एक दल के सदस्य दूसरे पक्ष के विधेयक या दृवष्टकोण का समथणन कर
सकते हैं।

Pork Barrel (िोि प्रावप्त हेतु सरकारी धन का व्यय स्थानीय कायों में करना): यह हाईस ऑफ़
ररप्रेजेन्िेरिव्स की राजनीवत को प्रदर्वशत करता है जहाँ स्थानीय वहत हािी होते हैं और प्रवतवनवध ऄपने
वनिाणचन क्षेत्रों के वलए ऄवधक से ऄवधक लाभ ईठाना चाहते हैं।

3.3. ईप-राष्ट्रपवत

3.3.1. ईप-राष्ट्रपवत का चु नाि

 संयुक्त राज्य ऄमेररका में राष्ट्रपवत और ईपराष्ट्रपवत पद हेतु ऄहणता एक समान ही है। चूंक्रक दोनों पदों
के वलए एक साथ चुनाि अयोवजत क्रकए जाते हैं, आसवलए चुनाि की प्रक्रक्रया भी एक समान है।
 पूिण की पद्धवत: जो ईम्मीदिार चुनाि में प्रथम रहता था, ईसे राष्ट्रपवत और दूसरे को ईपराष्ट्रपवत
चुना जाता था।
 ितणमान पद्धवत: दोनों के वलए चुनाि पृथक-पृथक होते हैं, लेक्रकन आसका अयोजन एक ही
समयािवध के दौरान और एक ही रीवत से क्रकया जाता है।

3.3.2. राष्ट्रपवत के रूप में ईप-राष्ट्रपवत का कायण काल

 राष्ट्रपवत का पद ररक्त होने के कारण ईपराष्ट्रपवत, राष्ट्रपवत का पद वनम्नवलवखत दो पररवस्थवतयों में


ग्रहण कर सकता है:
o जब राष्ट्रपवत ऄपने पद पर दो िषण से ऄवधक का कायणकाल पूणण कर चुका होता है और तब
ईपराष्ट्रपवत यक्रद ईसका पद ग्रहण करता है तो शेष ऄिवध के वलए िह राष्ट्रपवत बन सकता हैI
आसके ऄवतररक्त, िह दो और कायणकाल हेतु राष्ट्रपवत के रूप में कायण कर सकता है।
o जब वनितणमान राष्ट्रपवत का कायणकाल दो िषों से ऄवधक की ऄिवध के वलए शेष हो और यक्रद
ईपराष्ट्रपवत राष्ट्रपवत का कायणभार ग्रहण करता है तो शेष ऄिवध के ऄवतररक्त िह एक और
कायणकाल (राष्ट्रपवत) हेतु ऄहण होगा।

3.3.3. सं यु क्त राज्य ऄमे ररका के ईप-राष्ट्रपवत के प्रकायण

 िह सीनेि का पदेन चेयरपसणन होता है और वनणाणयक मत डालता है।


 भारतीय ईपराष्ट्रपवत का पद संयुक्त राज्य ऄमेररका के ईपराष्ट्रपवत के पद के समान ही बनाया गया
था (कु छ वभन्नताओं के साथ)।
 ईपराष्ट्रपवत के पद को “ वहज सुपरफ्लुऄस हाआनेस (His Superfluous Highness)” कहा जाता है।

3.4. ऄमे ररकी विधावयका/ऄमे ररकी कां ग्रे स


 संयुक्त राज्य ऄमेररका की संसद दो सदनों से वमलकर बनी है: हाईस ऑफ़ ररप्रजेन्िेरिव्स और
सीनेि।

3.4.1. हाईस ऑफ़ ररप्रजे न्िे रिव्स

 यह विश्व के सबसे शवक्तहीन वनम्न/वनचले सदनों में से एक है।


 आसकी कु ल सदस्य संख्या 435 है।
 आसके सदस्यों का चुनाि प्रत्यक्ष वनिाणचन प्रणाली िारा होता है।
 प्रत्येक राज्य से सदस्यों की संख्या ऄलग-ऄलग हो सकती है।

25 www.visionias.in ©Vision IAS

Google it:- https://upscpdf.com


https://upscpdf.com << Download From >> https://upscpdf.com

3.4.2. सीने ि

 यह स्थायी सदन है।


 विश्व का सबसे शवक्तशाली ईच्च सदन है।
 सीनेि के पास साधारण विधेयक, संविधान संशोधन विधेयक और धन विधेयक से संबंवधत समान
ऄवधकार हैं।
 एक सदस्य का कायणकाल 6 िषण का होता है और 1/3 सदस्य प्रत्येक 2 िषण पश्चात् सेिावनिृर्त् हो
जाते हैं।

3.5. सं यु क्त राज्य ऄमे ररका में सवमवत प्रणाली (Committee System In USA)

 संयुक्त राज्य ऄमेररका विश्व में सबसे सुदढ़ृ सवमवत प्रणाली िाला देश है। यह कहा जाता है क्रक यहाँ
(USA) की कांग्रेस सवमवतयों में कायण करती है।
विरिश और भारतीय प्रणाली से ऄंतर
 वििेन और भारत में, सिणप्रथम सदन में विधेयक को प्रस्तुत क्रकया जाता है, तदुपरान्त
प्रथम िाचन होता है और क्रफर आसके बाद सवमवत को आसे संदर्वभत क्रकया जाता है।
 संयुक्त राज्य ऄमेररका में, सदन में विधेयक को प्रस्तुत क्रकया जाता है और आसे कइ बार वबना
िाचन के ही सवमवत को संदर्वभत कर क्रदया जाता है।

वपजन होल (Pigeon Hole): संयुक्त राज्य ऄमेररका में विधेयक को सवमवत के स्तर पर ही समाप्त
क्रकया जा सकता है। यह विधेयक की Pigeon Holing के नाम से जाना जाता है।

3.6. कें रीय स्तर पर आन प्रवतवनवध वनकायों की ऄिवध

3.6.1. सं यु क्त राज्य ऄमे ररका

 संयुक्त राज्य ऄमेररका का हाईस ऑफ़ ररप्रजेन्िेरिव्स और सीनेि स्थायी वनकाय हैं।


 हाईस ऑफ़ ररप्रेजेन्िेरिव्स और सीनेि के सदस्यों का कायणकाल 3 जनिरी को समाप्त होता है।

3.6.2. भारत

 भारत में अपात वस्थवत में लोकसभा का कायणकाल एक िषण तक बढ़ाया जा सकता है या समय से
पूिण चुनाि कराकर आसे कम क्रकया जा सकता है।
 सर्त्ारुढ़ दल ऄगले चुनाि में ऄपनी जीत की संभािनाओं को देखते हुए राष्ट्रपवत को लोक सभा भंग
करने और ईपयुक्त समय (वजसमें पािी को लाभ हो) पर चुनाि कराने की सलाह देता है।

3.7. शवक्त के पृ थ क्करण का वसद्धां त और वनयं त्र ण एिं सं तु ल न की प्रणाली

 शवक्तयों के पृथक्करण के वसद्धांत की शुरुअत जॉन लॉक ने की थी और मॉन्िेस्क्यू ने आसे लोकवप्रय


बनाया (दोनों 18िीं सदी के दाशणवनक)।
शवक्तयों/प्रकायों का पृथक्करण क्यों?
 मॉन्िेस्क्यू के ऄनुसार- स्ितंत्रता तब तक सुरवक्षत नहीं होती जब तक क्रक शवक्तयों का पृथक्करण न
क्रकया गया हो।
 सभी लोकतांवत्रक देशों में न्यायपावलका सवहत शवक्तयों का पृथक्करण एक सािणभौवमक विशेषता है।
 शासन की राष्ट्रपतीय प्रणाली में सरकार के तीनों ऄंगों के मध्य शवक्तयों का पृथक्करण होता है
जबक्रक संसदीय शासन प्रणाली में विधायी और कायणकारी शवक्तयों का समेकन पाया जाता है।

26 www.visionias.in ©Vision IAS

Google it:- https://upscpdf.com


https://upscpdf.com << Download From >> https://upscpdf.com

3.7.1. सं यु क्त राज्य ऄमे ररका

 संयुक्त राज्य ऄमेररका का संविधान मॉन्िेस्क्यू और जॉन लॉक िारा प्रस्तावित शवक्त के पृथक्करण
वसद्धांत का दृढ़ता से पालन करता है। संयुक्त राज्य ऄमेररका में शवक्त के पृथक्करण का वसद्धांत पूरी
तरह से लागू है।
 सरकार के सभी तीनों ऄंगों के कायण पृथक हैं।
 विधावयका और कायणपावलका का कायणकाल वनवश्चत होता है और ये परस्पर वनभणर नहीं हैं।
 विधावयका का कोइ भी सदस्य, कायणपावलका का सदस्य नहीं हो सकता है।
 ऄमेररकी कांग्रेस के सदन िारा विवध-वनमाणण क्रकया जाता है, राष्ट्रपवत ईस विवध को कायाणवन्ित
करता है और ईच्चतम न्यायालय ईस विवध की व्याख्या करता है।
 ऄमेररकी राष्ट्रपवत को कानून बनाने का विशेषावधकार प्राप्त नहीं है क्योंक्रक िह न तो हाईस ऑफ़
ररप्रेजेन्िेरिव्स और न ही सीनेि का सदस्य होता है।
 राष्ट्रपवत को िीिो की शवक्त प्राप्त है लेक्रकन कानून बनाने की शवक्त नहीं, ऄतः कांग्रेस आस मामले में
राष्ट्रपवत को वनयंवत्रत करती है और आस प्रकार ‘वनयंत्रण एिं संतुलन’ बनाए रखा जाता है।

3.7.2. भारत

 सैद्धांवतक तौर पर, हम कह सकते हैं क्रक हमारे संविधान में भी शवक्त के पृथक्करण का वसद्धांत
दृवष्टगत होता है, लेक्रकन यह वसद्धांत मुख्यतः के िल कायणपावलका और न्यायपावलका के मध्य पाया
जाता है।
 राष्ट्रपवत संघीय कायणपावलका का वहस्सा होता है, तथावप प्रधानमंत्री और ईसकी मंवत्रपररषद ही
िास्तविक कायणपावलका के वहस्से होते हैं, क्योंक्रक राष्ट्रपवत मंवत्रमंडल की सलाह पर कायण करने के
वलए बाध्य है।
 आनके पास दोहरी क्षमता होती है:
o प्रथम, कायणपावलका के रूप में; और
o वितीय, विवध वनमाणता के रूप में।
 सर्त्ारुढ़ दल के नेता के रूप में प्रधानमंत्री विवध वनर्वमत कर सकता है, वजसे ईसके प्रशासन िारा
कायाणवन्ित क्रकया जाता है। आस प्रकार, प्रधानमंत्री और ईसकी मंवत्रपररषद विवध-वनमाणण करती है
और प्रशासन आसे कायाणवन्ित करता है, वजससे स्ियं में यहाँ शवक्त के पृथक्करण का वसद्धांत
विरोधाभासी प्रतीत होता है।

3.7.3. वनयं त्र ण एिं सं तु ल न

 सरकार के क्रकसी भी ऄंग को पूणण स्ितंत्रता प्रदान नहीं की गइ है। आसवलए यहां वनयंत्रण एिं
संतुलन का होना अिश्यक है।
संयक्त
ु राज्य ऄमेररका के संविधान में वनयंत्रण एिं संतल
ु न कै से स्थावपत क्रकया गया है?
 न्यायपावलका सरकार के ऄन्य ऄंगों ऄथाणत् कायणपावलका और विधवयका के कायों पर न्यावयक
पुनर्विलोकन के िारा वनयंत्रण रखती है।
 कांग्रेस, राष्ट्रपवत और ईसके प्रकायों पर क्रकस प्रकार वनयंत्रण रखती है?
o ऄंतराणष्ट्रीय समझौतों और ईच्च पदों पर वनयुवक्तयों की कांग्रेस में ऄवभपुवष्ट करना ऄवनिायण हैI
o प्रवतवनवधत्ि नहीं तो कर नहीं का वसद्धांत।
o राष्ट्रपवत के विरुद्ध महावभयोग।
 राष्ट्रपवत, कांग्रेस पर क्रकस प्रकार से वनयंत्रण रखता है?

27 www.visionias.in ©Vision IAS

Google it:- https://upscpdf.com


https://upscpdf.com << Download From >> https://upscpdf.com

o िीिो शवक्त का प्रयोग करके {हालांक्रक, कांग्रेस 2/3 बहुमत के साथ राष्ट्रपवत के िीिो का
ऄध्यारोहन कर विधेयक पाररत कर सकती है, आसवलए राष्ट्रपवत के पास अत्यंवतक िीिो
(absolute veto) की शवक्त ईपलब्ध नहीं है}।
o पॉके ि िीिो: आस संदभण में दो पररवस्थवतयां हैं-
 जब कांग्रेस दस क्रदनों के वलए सत्र में होती है तो राष्ट्रपवत की स्िीकृ वत के वबना भी
विधेयक पाररत कर क्रदया जाता है।
 जब कांग्रेस दस क्रदनों से कम समय के वलए सत्र में होती है तो विधेयक व्यपगत हो जाता
है।
 राष्ट्रपवत और कांग्रेस िारा न्यायपावलका पर वनयंत्रण
o न्यायाधीशों की वनयुवक्त: राष्ट्रपवत िारा वनयुवक्त और सीनेि िारा ऄवभपुवष्ट की जाती है।
o न्यायाधीशों को पदच्युत करना: कांग्रेस महावभयोग के माध्यम से हिाने का प्रस्ताि पाररत
करती है और आसे राष्ट्रपवत िारा स्िीकृ वत प्रदान की जाती है।
o िेतन और पररलवब्धयाँ राष्ट्रपवत िारा वनयंवत्रत की जाती हैं।

3.8. मू ल ऄवधकार (Fundamental Rights)

 जहाँ संयुक्त राज्य ऄमेररका के संविधान में “वबल ऑफ़ राआट्जस” (ऄवधकार पत्र) का समािेश है,
िहीं भारत के संविधान में ‘मूल ऄवधकार‘ को शावमल क्रकया गया है।
 हालांक्रक, ऄमेररकी संविधान में ऄवतररक्त मानि ऄवधकार प्रदान क्रकए गए हैं, जो भारतीय
संविधान में स्पष्ट रूप से दृवष्टगत नहीं होते हैं।
o संयुक्त राज्य ऄमेररका के संविधान के प्रथम संशोधन के तहत प्रेस की स्ितंत्रता स्पष्ट रूप से
प्रदान की गयी है, जबक्रक भारतीय संविधान के ऄनुच्छेद 19[1](A) के तहत िाक् एिं
ऄवभव्यवक्त की स्ितंत्रता प्रत्याभूत है।
o भारत में, ईच्चतम न्यायालय में आससे संबंवधत यावचका दायर करना भी एक मूल ऄवधकार है,
िहीं, संयुक्त राज्य ऄमेररका में “सरकार” के विरुद्ध यावचका दायर की जाती है (संयुक्त राज्य
ऄमेररका के मामले में “सरकार” शब्द का व्यापक ऄथण है वजसके दायरे में न के िल
कायणपावलका बवल्क ईच्च न्यायपावलका भी शावमल है)।
o ऄमेररकी नागररकों को संविधान के दूसरे संशोधन के ईपरांत ऄपने जीिन ि संपवर्त् की
सुरक्षा करने हेतु हवथयार और बंदक
ू रखने का ऄवधकार प्रदान क्रकया गयाI आसवलए ऄमेररका
में बंदक
ू और हवथयार वबना क्रकसी कानूनी ऄिरोध के ऄन्य िस्तुओं के समान बेचे जाते हैं,
जबक्रक भारत में वस्थवत एकदम विपरीत है, क्योंक्रक यह ऄवधकार मूल ऄवधकार नहीं है तथा
यह ऄत्यवधक विवनयवमत विवधक ऄवधकार है।
o संयुक्त राज्य ऄमेररका के संविधान का पाँचिां संशोधन आसकी गारंिी प्रदान करता है क्रक
फौजदारी ऄपराध के वलए ऄवभयुक्त को ‘ग्रैंड ज्यूरी‘ प्रणाली से गुजरना पड़ेगा। ग्रैंड ज्यूरी का
ऄथण यह है क्रक सरकार िारा अम लोगों में से समुदाय का नेतृत्ि करने िालों को यादृवच्छक
रूप से चुनना। िे ऄवभयुक्त पर अरोवपत दोषों का वनणणयन करते हैं। ग्रैंड ज्यूरी में चुने गए
लोगों की संख्या 6 से 12 के मध्य होती है और यक्रद मामला वििादास्पद है तो यह संख्या 12
से भी ऄवधक हो सकती हैI दूसरी तरफ, भारत में अपरावधक मामलों को के िल न्यायाधीशों
िारा ही वनपिाया जाता है।
 आसके ऄवतररक्त, संयुक्त राज्य ऄमेररका में क्रकसी भी व्यवक्त को ईसके जीिन एिं स्ितंत्रता से
“विवध की सम्यक् प्रक्रक्रया (Due process of law)” के वबना िंवचत नहीं क्रकया जा सकता।

28 www.visionias.in ©Vision IAS

Google it:- https://upscpdf.com


https://upscpdf.com << Download From >> https://upscpdf.com

o सम्यक् प्रक्रक्रया से अशय यह है क्रक विवध के ऄियि एिं प्रक्रक्रया ऄिश्य हीं ईवचत, वनष्पक्ष
और न्यायसंगत होने चावहए, वजन्हें न्यायपावलका िारा वनधाणररत क्रकया जाएगा।
o एक व्यवक्त को ईसकी स्ितंत्रता से िंवचत करने िाली विधायी शवक्त को प्रवतबंवधत क्रकया गया
है तथा न्यायपावलका िारा आसका परीक्षण और मूल्याकं न क्रकया जाता है।
 भारत में एक व्यवक्त को ईसके जीिन और स्ितंत्रता से “विवध िारा स्थावपत प्रक्रक्रया”
(Procedure established by law) के िारा ही िंवचत क्रकया जा सकता है।
o “विवध िारा स्थावपत प्रक्रक्रया” पद िस्तुतः स्ितंत्रता को सीवमत करने हेतु विधावयका को
व्यापक शवक्त प्रदान करती है।
o क्रफर भी, मेनका गाँधी िाद में (भले ही न्यायालय ने “सम्यक् प्रक्रक्रया” पद का प्रयोग नहीं
क्रकया हो) ईच्चतम न्यायालय ने वनणणय क्रदया क्रक विवध िारा स्थावपत प्रक्रक्रया ऄिश्य हीं
ईवचत, वनष्पक्ष और न्यायसंगत होनी चावहए।
 िषण 1978 में भारतीय संसद ने संपवर्त् के ऄवधकार को मूल ऄवधकारों की सूची से हिा क्रदया,
जबक्रक, संयुक्त राज्य ऄमेररका में ऄभी भी संपवर्त् का ऄवधकार मूल ऄवधकारों की सूची में शावमल
है और वबना ईवचत क्षवतपूर्वत के क्रकसी की भी संपवर्त् का ऄवधग्रहण नहीं क्रकया जा सकता है।
 संयुक्त राज्य ऄमेररका के छठे संविधान संशोधन के तहत क्रकसी ऄपराध के अरोपी व्यवक्त को कु छ
वनवश्चत ऄवधकार प्राप्त हैं, जैसे- त्िररत एिं सािणजवनक जांच, ऄवभयोग के वलए नोरिस, ऄपने पक्ष
में गिाह प्राप्त करने की ऄवनिायण प्रक्रक्रया और ऄपने पसंद के िकील की सहायता प्राप्त करना।
o यद्यवप भारत के संविधान में आन सभी ऄवधकारों का स्पष्ट ईल्लेख नहीं है, क्रफर भी ऄनुच्छेद
21 के ऄंतगणत प्राण एिं दैवहक स्ितंत्रता के संरक्षण की व्यापक व्याख्या कर ईच्चतम न्यायालय
ने आन ऄवधकारों को प्रदान क्रकया है।
 आसके ऄवतररक्त, संयुक्त राज्य ऄमेररका के संविधान के अठिें संशोधन के ऄनुसार एक ऄवभयुक्त
को जमानत से िंवचत नहीं क्रकया जाएगा, अरोवपत जुमाणना ऄवधक नहीं होना चावहए और प्रदर्त्
दंड ऄमानिीय नहीं होने चावहए। ये ऄवधकार भारतीयों को भी प्रदर्त् हैं, क्योंक्रक ऄनुच्छेद 21 के
तहत ईच्चतम न्यायालय ने एक व्यवक्त के ऄवधकार के रूप में आनकी ऄवभपुवष्ट की है।
 संयुक्त राज्य ऄमेररका के संविधान का 9िां संशोधन ऄत्यवधक महत्िपूणण है। आसके ऄनुसार
संविधान में िार्वणत कु छ विवशष्ट ऄवधकारों की व्याख्या, ऄमेररकी लोगों के ऄन्य ऄवधकारों को
ऄस्िीकार नहीं करेगी। संिैधावनक ऄवधकारों के बािजूद लोगों को कु छ प्राकृ वतक ऄवधकार भी
प्रदान क्रकए गए हैं। संयुक्त राज्य ऄमेररका का संविधान लॉक के िारा प्रस्तुत मानि के ऄपररहायण
प्राकृ वतक ऄवधकारों (Inalienable Natural Rights of Human Being) के दशणन से प्रभावित
है। दूसरी तरफ, भारतीय संविधान में आस तरह का कोइ ऄनुच्छेद नहीं हैं। आसवलए, भारतीयों को
िे ही ऄवधकार प्राप्त हैं जो संविधान में स्िीकृ त हैं। ये ऑवस्िन एिं बेन्थम के विवधक वसद्धांतों के
दशणन पर अधाररत हैं।

3.9. विधायी शवक्त का वितरण (Distribution Of Legislative Power)

3.9.1. भारत

 भारतीय संविधान की सातिीं ऄनुसूची विधायी शवक्तयों का कें र और राज्य सरकारों के मध्य
विभाजन करती है। कें र और राज्य सरकार को संघ एिं राज्य सूची के ऄंतगणत सूचीबद्ध क्रमशः 97
और 66 विषयों पर ऄनन्य रूप से कानून बनाने का ऄवधकार है। समिती सूची के ऄंतगणत 47
विषयों पर कानून बनाने का ऄवधकार कें र और राज्य सरकार दोनों के पास हैं तथा क्रकसी वििाद
के मामले में कें र सरकार िारा वनर्वमत कानून ऄवभभािी होगा।

29 www.visionias.in ©Vision IAS

Google it:- https://upscpdf.com


https://upscpdf.com << Download From >> https://upscpdf.com

 संघ सूची की 97िीं प्रविवष्ट में यह ईल्लेख है क्रक ऐसा कोइ भी विषय जो क्रकसी सूची में सूवचबद्ध
नहीं है, ईसपर कानून बनाने का स्ितः ऄवधकार संसद को प्राप्त होगा। आस प्रकार, हमारे संविधान
वनमाणताओं िारा एक सुदढ़ृ संघ और कमजोर राज्य सरकार (वजन्हें अर्वथक सहायता हेतु कें र
सरकार पर वनभणर रहना पड़ता है) की संकल्पना की गइ थी।

3.9.2. सं यु क्त राज्य ऄमे ररका

 संयुक्त राज्य ऄमेररका के संविधान में आस तरह की कोइ व्यापक व्यिस्था नहीं है। कु छ स्पष्ट रूप से
ईवल्लवखत विषय संघ के पास और ऄन्य विषय राज्य सरकारों के पास होते हैं।

3.10. अपात-काल और ररि का वनलं ब न


 भारत में युद्ध या बाह्य अक्रमण और सशस्त्र विरोह के दौरान अपात-काल की घोषणा की जा
सकती हैI ऐसी अपात वस्थवत के दौरान जीिन के ऄवधकार के ऄवतररक्त ऄन्य सभी मूल ऄवधकारों
को वनलंवबत क्रकया जा सकता है।
 संयुक्त राज्य ऄमेररका के संविधान में अपात-काल जैसे क्रकसी िाक्यांश का प्रयोग नहीं
क्रकया गया है, लेक्रकन विरोह और सािणजावनक सुरक्षा के ईल्लंघन के मामले में बंदी प्रत्यक्षीकरण
ररि को वनलंवबत क्रकया जा सकता है।

3.11. न्यायपावलका
 ऄमेररका में ईच्चतम न्यायालय के न्यायाधीशों की वनयुवक्त हेतु क्रकसी योग्यता का ईल्लेख नहीं हैं।
ईच्चतम न्यायालय के न्यायाधीशों की वनयुवक्त में राष्ट्रपवत का वनणणय ऄंवतम होता है। िह सीनेि को
न्यायाधीशों के नाम वनर्ददष्ट करता है और सीनेि की सलाह एिं सहमवत पर न्यायाधीशों की
वनयुवक्त करता है। ईच्चतम न्यायालय के वलए प्रस्तावित न्यायाधीशों की ऄहणताओं का मूल्यांकन
करने में सीनेि की न्यावयक सवमवत महत्िपूणण भूवमका वनभाती है। िह न्यायाधीशों की पृष्ठभूवम की
जांच करती है, न्यायाधीशों के साथ बैठक कर प्रश्नों के माध्यम से िाताण का अयोजन कर ऄन्य
तथ्यों की जांच करती हैI संपूणण प्रक्रक्रया सािणजवनक और पारदशी तरीके से अयोवजत की जाती है।
यक्रद संयुक्त राज्य ऄमेररका के क्रकसी नागररक को न्यायाधीशों की सत्यवनष्ठा के बारे में कु छ भी
ज्ञात हो तो िह आस संबंध में सूचना को प्रमाण के साथ अगे की जांच हेतु सीनेि की न्यावयक
सवमवत के पास भेज सकता है ताक्रक क्रकसी ऄयोग्य ईम्मीदिार को ईच्चतम न्यायालय के न्यायाधीश
के रूप में वनयुक्त न क्रकया जा सके । न्यायाधीशों की वनयुवक्त में ऄमेररका के लोग भी भाग लेते हैं
और आसमें न्यायपावलका की कोइ भूवमका नहीं होती हैं। न्यायाधीशों की वनयुवक्त की संपूणण प्रक्रक्रया
को पारदशी तरीके से संपाक्रदत क्रकया जाता है।
 न्यायाधीशों का कोइ वनवश्चत कायणकाल नहीं होता है। क्रफर भी, यक्रद िे 70 िषण की अयु में
सेिावनिृर्त् होते हैं, तो ईन्हें एक सेिारत न्यायाधीश के समान िेतन और भर्त्ा प्राप्त होता है।
 दूसरी तरफ, भारत में न्यायाधीशों की वनयुवक्त की संपूणण प्रक्रक्रया न्यायपावलका और कायणपावलका
के मध्य गुप्त रूप से संचावलत की जाती है। जन सामान्य को आस संबंध में जानकारी वनयुवक्त के बाद
ही प्राप्त होती हैI ज्ञातव्य है क्रक न तो जनता को ऄवग्रम तौर पर सूवचत क्रकया जाता है और न ही
कायणपावलका न्यायाधीशों के बारे में खुली जांच करती है। ईच्चतम न्यायालय के न्यायाधीशों की
वनयुवक्त की प्रक्रक्रया में भारत के मुख्य न्यायाधीश और ईच्चतम न्यायालय के 4 िररष्ठ न्यायाधीशों
(कॉलेवजयम) की प्रभािी और वनणाणयक भूवमका होती हैI न्यायाधीशों की वनयुवक्त की संपूणण
प्रक्रक्रया गुप्त रूप से संचावलत होती है वजसमें जन सामान्य की कोइ भागीदारी नहीं होती है, वजसे
कइ लोगों िारा भारतीय कानून प्रणाली में व्याप्त गंभीर दोष माना जाता है। न्यायाधीश ऄपने पद
पर 65 िषण की अयु तक बने रहते हैं।

30 www.visionias.in ©Vision IAS

Google it:- https://upscpdf.com


https://upscpdf.com << Download From >> https://upscpdf.com

3.12. सं विधान का सं शोधन

3.12.1. ऄमे ररकी सं विधान में सं शोधन

ऄमेररकी संविधान में संशोधन करने के दो तरीके हैं:


 कांग्रेस िारा प्रस्तावित और राज्यों िारा ऄवभपुवष्ट (मंजूरी);
o दोनों सदनों िारा 2/3 बहुमत से संशोधन का पाररत होना, तथा
o कम से कम ¾ राज्यों के विधान-मंडल िारा आसे ऄनुमोक्रदत करना।
 राज्यों िारा प्रस्तावित और राज्यों िारा ही ऄवभपुवष्ट;
o आस हेतु 2/3 राज्यों िारा ऐसे प्रस्ताि पाररत होने चावहए,
o िे कांग्रेस से िाताण करेंगे और कांग्रेस िारा ऄवधिेशन (convention) का अयोजन क्रकया
जाएगा, तथा
o ऄवधिेशन के दौरान, आसे ¾ (तीन चौथाइ) राज्यों िारा ऄनुमोक्रदत क्रकया जाना अिश्यक है।

3.12.2. भारतीय सं विधान में सं शोधन

 संयुक्त राज्य ऄमेररका के संविधान की तुलना भारत के संविधान में संशोधन प्रक्रक्रया असान और
लचीली है। भारत में, के िल संसद ही संविधान संशोधन का प्रस्ताि पाररत कर सकती है और
राज्य आस संदभण में कोइ विशेष भूवमका नहीं वनभाते हैं।
 कु छ ऄनुच्छेद साधारण बहुमत से संशोवधत हो सकते हैं, कु छ विशेष बहुमत िारा, जबक्रक, कु छ
सीवमत ऄनुच्छेद विशेष बहुमत और 50% से ऄवधक राज्यों के ऄनुसमथणन के पश्चात् ही संशोवधत
हो सकते हैं।
 यहाँ साधारण बहुमत का ऄथण , वजस क्रदन संशोधन क्रकया जाना है, ईस क्रदन संसद में ईपवस्थत
सांसदों के बहुमत से हैं, न क्रक संसद की कु ल सदस्य संख्या से।
िास्तविकता यह है क्रक विगत 225 िषों में संयुक्त राज्य ऄमेररका का संविधान के िल 27 बार
संशोवधत हुअ है, जो यह प्रदर्वशत है क्रक भारत की तुलना में संयुक्त राज्य ऄमेररका के संविधान के
संशोधन की प्रक्रक्रया क्रकतनी कठोर है।

4. चीन का संविधान
 चीन एक समाजिादी देश है, जहाँ समाजिादी विचारधारा की सिोच्चता विद्यमान है। चीन का
संविधान चीनी कम्युवनस्ि पािी (Communist Party of China: CPC) के नेतृत्ि को स्िीकार
करता है।
 CPC विश्व की सबसे बड़ी राजनीवतक पािी में से एक है (ितणमान में BJP विश्व की सबसे बड़ी
पािी बन गइ है), वजसमें स्थानीय स्तर पर सदस्यों की संख्या लाखों में हैं। यह लोकतांवत्रक
के न्रिाद (Democratic Centralism) के वसद्धांत पर कायण करती है। पािी की पूणण बैठक, वजसे
नेशनल पािी कांग्रेस (NPC) कहा जाता है, पांच िषो में एक बार अयोवजत होती है। यद्यवप,
सैद्धांवतक रूप से, सभी शवक्तयां जनता में वनवहत होती हैं, लेक्रकन व्यिहार में यह शीषण नेताओं के
पास ही रहती हैं।
 नेशनल पािी कांग्रेस के सदस्य के न्रीय सवमवत (सेंरल कवमिी) के सदस्यों का चयन करते हैं।
के न्रीय सवमवत, पोवलत ब्यूरो का चयन करती है (वजसमें लगभग 200 सदस्य होते हैं)। पोवलत
ब्यूरो िारा ऄपनी स्थायी सवमवत (स्िैंबडग कवमिी) का चयन क्रकया जाता है (ितणमान में आसके
सदस्यों की संख्या 24 है जो पािी के सबसे शवक्तशाली सदस्य होते हैं)।

31 www.visionias.in ©Vision IAS

Google it:- https://upscpdf.com


https://upscpdf.com << Download From >> https://upscpdf.com

4.1. चीन के सं विधान की प्रमु ख विशे ष ताएं (Salient Features Of The

Constitution)

4.1.1. प्रस्तािना

 यहाँ राजनैवतक प्रणाली के िैचाररक लक्ष्यों के संबंध में माक्सणिाद, लेवननिाद और माओ की
वशक्षाओं को सिोपरर स्थान क्रदया गया है। संिैधावनक ढांचे के ऄंतगणत पारंपररक रूप से
लोकतांवत्रक के न्रिाद के वसद्धांत को भी विशेष स्थान क्रदया गया है। चीन की पुरानी पररभाषा
ऄथाणत् “सिणहारा की एकसर्त्ािाद” (Dictatorship of the Proletariat) को “जनिादी
लोकतांवत्रक एकसर्त्ािाद” (People’s Democratic Dictatorship) से प्रवतस्थावपत कर क्रदया
गया है।
 प्रस्तािना में स्पष्ट रूप से ताआिान को चीन के एक ऄवभन्न ऄंग के रूप में स्िीकार क्रकया है और
आसकी मुवक्त (liberation) को चीनी लोगों के दावयत्ि के रूप में घोवषत क्रकया गया है। विदेशी
संबंधों के क्षेत्र में पांच बबदुओं को ऄन्तर्वनवहत वसद्धांत के रूप में ऄपनाया है। आसमें शावमल है:
o सभी राष्ट्रों की क्षेत्रीय ऄखंडता का सम्मान और संरक्षण करना;
o अक्रामक नीवत का त्याग करना;
o दूसरे राष्ट्रों के अंतररक मामलों में हस्तक्षेप न करना;
o ऄंतराणष्ट्रीय सहयोग का संिधणन ; और
o शांवतपूणण सह-ऄवस्तत्ि।

4.1.2. सं विधान की प्रकृ वत

 चीन के संविधान की विषय-िस्तु और विचारधारा भूतपूिण सोवियत संघ के संविधान के साथ


समानता दशाणती है। यह न तो ऄत्यवधक कठोर है और न ही ऄत्यवधक लचीली।

4.1.3. अधारभू त वसद्धां त

 संविधान के तहत, पीपुल्स ररपवब्लक ऑफ़ चाआना (जनिादी चीनी गणराज्य) एक समाजिादी


राज्य (Socialist State) है, वजसकी स्थापना जनिादी लोकतांवत्रक एकसर्त्ािाद (People’s
Democratic dictatorship) के नाम पर हुइ है, वजसमें जनता का मागणदशणन करने का
ईर्त्रदावयत्ि कम्युवनस्ि पािी का है। जनता को शवक्त और सर्त्ा के स्रोत के रूप में घोवषत क्रकया
गया है और िे आस शवक्त का प्रयोग नेशनल पीपुल्स कांग्रेस के माध्यम से करेंगे।

4.1.4. एकात्मक प्रणाली

 जनिादी चीनी गणराज्य एक एकात्मक बहु-राष्ट्रीय राज्य (मल्िी-नेशनल स्िेि) है वजसका वनमाणण
आसकी राष्ट्रीयता धारण करने िाले सभी लोगों िारा संयुक्त रूप से क्रकया गया है।
 चीन में, एक सुदढ़ृ के न्रीय सरकार मौजूद है जबक्रक क्षेत्रीय सरकारें , पृथक सर्त्ा के रूप में विद्यमान
हैं, वजन्हें संविधान के तहत वनर्वमत नहीं क्रकया गया है। आसवलए नीवत-वनमाणण में लोगों की
भागीदारी को प्रोत्सावहत करने और सािणजावनक मामलों में ईनके वहतों की रक्षा करने के वलए,
सरकारी मामलों में विकें रीकरण की प्रक्रक्रया को अरंभ क्रकया गया है। के न्रीय सरकार ने क्षेत्रीय
और स्थानीय प्रशासवनक आकाइयों को ऄत्यवधक ऄवधकार और शवक्तयाँ प्रत्यायोवजत की हैं।

32 www.visionias.in ©Vision IAS

Google it:- https://upscpdf.com


https://upscpdf.com << Download From >> https://upscpdf.com

4.1.5. लोकतां वत्रक कें रिाद (Democratic Centralism)

 भूतपूिण सोवियत संघ की राजनैवतक प्रणाली के समान, जनिादी चीनी गणराज्य में भी
“लोकतांवत्रक कें रीय ऄवधकारिाद का वसद्धांत” पूणण रूप से लागू है। लोकतांवत्रक मापदंडों को ध्यान
में रखते हुए, चुनािी प्रक्रक्रया का वसद्धांत के िल सरकारी संस्थानों के भीतर ही नहीं बवल्क पािी
संगठन के स्तर पर भी शुरू क्रकया गया है। देश के सभी नागररकों को ियस्क मतावधकार के अधार
पर मतदान करने का ऄवधकार प्राप्त है।

4.1.6. एक दलीय प्रणाली (One Party System)

 कम्युवनस्ि पािी देश के संिैधावनक ढांचे के भीतर लगभग एकसर्त्ािादी शवक्तयों का ईपयोग
करती है और आसे सभी व्यािहाररक प्रयोजनों के वलए राजनैवतक प्रावधकार के एकमात्र स्रोत के
रूप में माना गया है।
 पािी संगठन सरकारी संस्थाओं के समानांतर संचावलत है। सरकार में सभी ईच्च पदों पर पािी के
ईच्च-िगण (Party elite) के लोगों का अवधपत्य है।
 व्यिहार में, क्रकसी ऄन्य राजनीवतक दल को कायण करने की िास्तविक स्ितंत्रता नहीं है।
o कु छ विशेष युिा संगठनों को, जो पािी और पािी के साथ संबद्ध कायण समूह के प्रवत वनष्ठािान
होते हैं, ईन्हें वनणणय-प्रक्रक्रया में भाग लेने का ऄवधकार है।

4.1.7. विधावयका (Legislature)

 राष्ट्रीय जनिादी कांग्रेस (National People’s Congress: NPC) विधायी शाखा को समाविष्ट
करता है।
 यह 3,000 से ऄवधक सदस्यों िाली एक सदनीय विधावयका है।
 सैद्धांवतक रूप से, यह चीन की वनणणय-वनमाणण करने िाली शीषण वनकाय है। यह नीवतयों, संशोधनों
और सरकार में मंवत्रयों की वनयुवक्त पर ऄंवतम वनणणय देती है।
 आसे एक ऐसे ऄंग के रूप में घोवषत क्रकया गया है वजसके माध्यम से लोग राज्य शवक्त का ईपयोग
करते हैं।
 आसके सदस्यों का चुनाि, ऄपने-ऄपने कोिे के ऄनुसार, क्षेत्रीय कांग्रेस, स्िायर्त् क्षेत्रों, के न्रीय
सरकार के ऄधीन कायणरत नगरपावलकाओं और पीपुल्स वलबरेशन अमी िारा क्रकया जाता है।
 चुनाि का प्रकार गुप्त मतदान पर अधाररत है, जबक्रक संविधान स्ितंत्र और वनष्पक्ष चुनाि की
गारंिी देता है।
 NPC का िास्तविक कायण आसके एक छोिे से वनकाय के िारा क्रकया जाता है, जो NPC की स्थायी
सवमवत (स्िैंबडग कवमिी) कहलाती है। यह लगभग 150 सदस्यों से वमलकर बनी है।
ऄिवध
 कांग्रेस का चुनाि 5 िषों के वलए होता है लेक्रकन यह ऄपने कायणकाल समाप्त होने से पूिण भी
विघरित हो सकती है या आसके कायणकाल को बढ़ाया भी जा सकता है। कांग्रेस की स्थायी सवमवत,
कायणकाल समाप्त होने से पूिण नया चुनाि अयोवजत करने की संपूणण तैयारी करने के वलए
ईर्त्रदायी है।
सत्र
 कांग्रेस का सत्र िषण में एक बार बीबजग में अयोवजत होता है। कांग्रेस की स्थायी सवमवत सामान्यतः
सत्र को अहूत करती है। आसके ऄवतररक्त, कांग्रेस के 1/5 सदस्यों के ऄनुरोध पर आसका ऄध्यक्ष भी
सत्र अहूत कर सकता है।

33 www.visionias.in ©Vision IAS

Google it:- https://upscpdf.com


https://upscpdf.com << Download From >> https://upscpdf.com

शवक्तयां
 NPC विवध-वनमाणण की सिोच्च वनकाय है, जो कानून बनाने, मौजूदा कानूनों में पररितणन करने या
वनरसन करने हेतु पूणण रूप से ऄवधकृ त है। यह राज्य के वलए प्रशासकीय नीवतयों को भी मंजूरी
प्रदान करता है।
(i) कानून का ऄवधवनयमन
 ऄपने सत्र के दौरान कांग्रेस नए कानूनों को बनाती है और पररवस्थवत के ऄनुसार मौजूदा नीवतयों
में अिश्यक पररितणन भी करती है। संविधान का संशोधन कांग्रेस के 2/3 सदस्यों के बहुमत से ही
क्रकया जा सकता है जबक्रक, ऄन्य कानून साधारण बहुमत से ही ऄवधवनयवमत हो जाते हैं। यह
ध्यान देने योग्य है क्रक कांग्रेस के कायों को ईच्चतम न्यायालय में चुनौती नहीं दी जा सकती है।
(ii) कायणकारी शवक्तयां
 NPC को संविधान के तहत संिैधावनक कानून और विवधयों के कायाणन्ियन की वनगरानी का भी
ऄवधकार प्राप्त है। यह ऄपनी पसंद के ऄनुसार सिोच्च सािणजवनक ऄवधकाररयों की वनयुवक्त के
माध्यम से प्रशासकीय नीवतयों को वनयंवत्रत और प्रभावित कर सकती है। ऄपने विभागीय कायों के
वनष्पादन के संदभण में सभी प्रशासवनक विभाग ऄपने मंवत्रयों के साथ कांग्रेस के प्रवत जिाबदेह होते
हैं। कांग्रेस ऄपनी शवक्तयों का प्रयोग राष्ट्रीय अर्वथक नीवत और िार्वषक बजि के ऄनुमोदन में भी
करती है। संविधान के तहत कांग्रेस ऄपने कायण के दायरे में अने िाली ऐसी सभी शवक्तयों का
प्रयोग करने के वलए ऄवधकृ त है वजसे िह पूरी तरह से ईवचत और अिश्यक समझे।
(iii) वनिाणचक शवक्तयां
 NPC सरकारी प्रावधकरणों के ईच्च पदों पर असीन होने िाले व्यवक्तयों के चुनाि करने की शवक्त
के अधार पर सरकारी ढांचे के भीतर एक वनणाणयक स्थान रखती है। संविधान के तहत, यह
गणराज्य के राष्ट्रपवत और ईपराष्ट्रपवत का भी चुनाि करती है तथा राष्ट्रपवत की वसफाररश पर
राज्य पररषद के प्रमुख/प्रधानमंत्री (Premier of the State Council) की वनयुवक्त करती है। यह
प्रधानमंत्री के सलाह पर ऄन्य मंवत्रयों की वनयुवक्त करती है। कांग्रेस को मंवत्रयों को हिाने की भी
शवक्त प्राप्त होती है। साथ ही, आसे सुप्रीम कोिण के प्रेवसडेंि और सुप्रीम प्रोक्यूरेि के मुख्य
प्रोक्यूरेिर (Chief Procurator of the Supreme Procurate) की वनयुवक्त ऄथिा ईन्हें पद से
हिाने की शवक्त प्राप्त है।
 हालांक्रक, संविधान िारा NPC को पूणण रूप से ईपयुणक्त शवक्तयों का प्रयोग करने के वलए ऄवधकृ त
क्रकया गया है, परंतु व्यिहार में यह एक सक्रक्रय वनकाय नहीं है। कानून बनाने िाले स्ितंत्र वनकाय
के रूप में आसकी वस्थवत के िल सैद्धांवतक स्तर तक ही सीवमत है। आसके प्रमुख कारण वनम्नवलवखत हैं:
o आसका सत्र यदाकदा ही वनयवमत तौर रूप से अयोवजत होता है।
 आसकी बैठक िषण में के िल एक बार ही अयोवजत होती है और िो भी के िल कु छ क्रदनों के
वलए।
o कांग्रेस की शवक्तयों का िास्तविक प्रयोग आसकी स्थायी सवमवत िारा ही क्रकया जाता है।
स्थायी सवमवत (Standing Committee)
 कांग्रेस की स्थायी सवमवत एक प्रभािी और सक्रक्रय वनकाय है तथा यह व्यिहार में कांग्रेस की
ऄवधकावधक शवक्तयों का प्रयोग करती है। परन्तु, सैद्धांवतक रूप में यह कांग्रेस का एक ऄधीनस्थ
वनकाय है। यह ऄपने मुख्य वनकाय के प्रवत ईर्त्रदायी भी है और यह ऄपने कायणकलापों के बारे में
कांग्रेस को वनयवमत रूप से सूवचत करने के वलए बाध्य है। सवमवत के सभी सदस्य कांग्रेस िारा
वनिाणवचत होते हैं और आसी के वििेकावधकार पर हिाए भी जाते हैं।

34 www.visionias.in ©Vision IAS

Google it:- https://upscpdf.com


https://upscpdf.com << Download From >> https://upscpdf.com

शवक्तयां
 यह सवमवत, कांग्रेस के सत्र को अहूत करती है और साथ ही नए चुनाि कराने का अदेश भी जारी
करती है।
 यह संविधान के वनयमों के साथ-साथ विवभन्न विधानों की व्याख्या का कायण करती है। आस तरह के
न्यावयक प्रकृ वत के कायों का वनष्पादन आसके महत्ि और शवक्त के दायरे में िृवद्ध करता है।
 यह स्िेि काईं वसल, ईच्चतम न्यायालय और प्रोक्यूरेिर की कायण-प्रणाली का वनरीक्षण करती है। ये
कायण संविधान के िारा स्थायी सवमवत को प्रदान क्रकए गए हैं।
 सवमवत को सरकारी विभागों, स्िायर्त् क्षेत्रों, प्रांतों और यहाँ तक क्रक कें र सरकार के ऄधीन कायण
करने िाली नगरपावलकाओं के िारा वलए गए ऄनुवचत वनणणयों को पररिर्वतत या वनरस्त करने
का ऄवधकार है।
 कांग्रेस के सत्र में न रहने के दौरान यह िास्ति में मूल शवक्तयों का स्रोत है। आस ऄिवध के दौरान,
प्रीवमयर (प्रधानमंत्री) की सलाह पर यह नए मंवत्रयों की वनयुवक्त और पुराने मंवत्रयों को हिाने
संबंधी अदेश जारी करती है। यह ईपराष्ट्रपवत और ईप मुख्य प्रोक्यूरेिर (Deputy Chief
Procurator) की वनयुवक्त या ईन्हें हिाने का अदेश जारी कर सकती है।
ऄध्यक्ष
 सवमवत के ऄध्यक्ष को राजनीवतक व्यिस्था में सबसे शवक्तशाली व्यवक्त के रूप में माना गया है। िह
स्थायी सवमवत की बैठकों की ऄध्यक्षता करता है। ईसे वडक्री और ऄध्यादेश जारी करने की शवक्त
प्राप्त है। आसके कतणव्यों की सूची में वनम्नवलवखत शावमल हैं:
o ऄन्य देशों के राजनवयकों का स्िागत करना;
o ऄन्य देशों के साथ क्रकए गए समझौता की पुवष्ट करना; और
o ऄन्य देशों के वलए राजनवयकों के समूह को वनयुक्त करना।
ऄन्य सवमवतयां
 जनिादी कांग्रेस ऄपने कायणकाल के दौरान कइ सवमवतयों का गठन करती है, जैसे - विर्त्ीय और
अर्वथक मामलों की राष्ट्रीय सवमवत; वशक्षा, विज्ञान, संस्कृ वत और स्िास्थ्य के मुद्दों की सवमवत;
विदेशी मामलों की सवमवत; ऄप्रिासी चीनी लोगों से संबंवधत मामलों की सवमवत आत्याक्रद। ये सभी
सवमवतयां कांग्रेस के सत्र में न रहने पर आस ऄिवध के दौरान NPC की स्थायी सवमवत के पयणिेक्षण
में कायण करती हैं।
 स्थायी सवमवत की शवक्तयों और कायों को देखते हुए यह स्पष्ट है क्रक यह एक शवक्तशाली और
प्रभािी ऄंग है। जैसाक्रक कांग्रेस का िार्वषक सत्र के िल कु छ ही क्रदनों में समाप्त हो जाता है, तो शेष
ऄिवध के वलए जब कांग्रेस सत्र में नहीं होती है तो आसकी शवक्तयों का मुख्य रूप से स्थायी सवमवत
िारा प्रयोग क्रकया जाता है। सवमवत के सदस्य, चीनी कम्युवनस्ि पािी के सदस्य होने के कारण
प्रशासवनक मामलों में भी महत्िपूणण भूवमका वनभाते हैं।

4.1.8. कायण पावलका (Executive)

4.1.8.1. राज्य पररषद (स्िे ि काईं वसल)


 स्िेि काईं वसल ही चीन की कायणपावलका या मंवत्रमंडल है। आसका गठन एक प्रीवमयर (प्रधानमंत्री),
चार ईप प्रीवमयर और राज्य काईन्सलरों से वमलकर हुअ है। संविधान के ऄंतगणत , स्िेि काईं वसल
सरकार का मुख्य कायणकारी ऄंग है। आसके सभी सदस्यों का चुनाि कांग्रेस िारा क्रकया जाता है और
ये ईसी के प्रवत जिाबदेह होते हैं। स्िेि काईं वसल का मुख्य कायण , कानून का प्रितणन करना तथा
प्रशासवनक नीवतयों का वनमाणण और क्रक्रयान्ियन करना है। स्िेि काईं वसल के सदस्य विधेयक को
प्रस्ताि के रूप में कांग्रेस के पिल पर रखते हैं वजसे बाद में संसदीय प्रक्रक्रया
के िारा कानून के रूप में पररिर्वतत क्रकया जाता है।

35 www.visionias.in ©Vision IAS

Google it:- https://upscpdf.com


https://upscpdf.com << Download From >> https://upscpdf.com

4.1.8.2. प्रधानमं त्री (Premier)


 प्रधानमंत्री प्रशासन के प्रमुख के रूप में ऄत्यवधक महत्िपूणण भूवमका वनभाता है तथा प्रशासवनक
तंत्र में एक वनणाणयक वस्थवत रखता है।

4.1.8.3. राष्ट्रपवत

 चीनी गणराज्य का राष्ट्रपवत, राज्य का प्रमुख होता है।


 िह कांग्रेस के िारा 5 िषों के वलए चुना जाता है।
 राष्ट्रपवत को प्रशासवनक तंत्र में सिाणवधक प्रवतवष्ठत स्थान प्राप्त होता है।
 देश के संविधान के तहत, राष्ट्रपवत का पद सीवमत शवक्तयों िाला मुख्यतः एक औपचाररक पद है।
हालाँक्रक, 1993 के पश्चात् से, यह पररपािी रही है क्रक राष्ट्रपवत िारा कम्युवनस्ि पािी ऑफ चाआना
के महासवचि के साथ-साथ सेना प्रमुख का भी पद धारण क्रकया जाता है। राष्ट्रपवत के पद को
अवधकाररक रूप से प्रशासवनक पद के बजाय राज्य की संस्थान के रूप में माना जाता है। सैद्धांवतक
रूप से, राष्ट्रपवत नेशनल पीपुल्स कांग्रेस के ऄधीन कायण करता है और िह ऄपने विशेषावधकार के
अधार पर कायणकारी कारणिाइ करने हेतु कानूनी रूप से ऄवधकृ त नहीं है।
 राष्ट्रपवत के पास कानूनों को लागू करने, प्रधानमंत्री का चयन करने और ईसे पदच्युत करने के
साथ-साथ राज्य पररषद के मंवत्रयों, राष्ट्रपतीय क्षमादान प्रदान करने, अपातकाल की ईद्घोषणा
करने, िृहत स्तर पर लामबंदी का अदेश जारी करने और राजकीय सम्मान जारी करने की शवक्त
है। आसके ऄवतररक्त, राष्ट्रपवत ऄन्य देशों में चीनी राजदूतों को वनयुक्त कर सकता है और ईन्हें हिा
सकता है तथा विदेशी संस्थाओं के साथ संवध कर सकता है या मौजूदा संवधयों को वनरस्त कर
सकता है। संविधान के प्रािधानों के ऄनुसार, ईपयुणक्त सभी शवक्तयों को नेशनल पीपुल्स कांग्रेस के
ऄनुमोदन या पुवष्ट की अिश्यकता है।
 राष्ट्रपवत के पद के वलए व्यवक्त को संपूणण वनिाणचन ऄवधकार प्राप्त होने के साथ-साथ चीनी नागररक
होना चावहए। राष्ट्रपवत के पद के वलए 45 िषण की अयु होनी चावहए। िषण 2018 तक, यह
प्रािधान था क्रक राष्ट्रपवत और ईपराष्ट्रपवत पद पर कोइ व्यवक्त दो ऄिवध तक ही वनयुक्त हो सकता
था। माचण 2018 में नेशनल पीपुल्स कांग्रेस िारा राष्ट्रपवत पद से संबंवधत आस सीमा को समाप्त
करने हेतु संिैधावनक संशोधन पाररत क्रकया गया, वजसने ितणमान राष्ट्रपवत और ईपराष्ट्रपवत को
ऄवनवश्चत काल तक पद पर बने रहने की ऄनुमवत प्रदान की गइ। िषण 1976 में माओ-त्से-तुंग की
मृत्यु के ईपरांत चीन के संविधान में राष्ट्रपवत पद के वलए पाँच िषीय दो ऄिवधयों का प्रािधान
क्रकया गया था। आस व्यिस्था को डेंग वजयाओबपग िारा लागू क्रकया गया था, वजन्होंने एक-व्यवक्त
के शासन और व्यवक्तत्ि अधाररत राजनीवत (cult of personality) के खतरों को पहचाना और
आनके स्थान पर सामूवहक नेतृत्ि का समथणन क्रकया। यह संिैधावनक संशोधन ितणमान राष्ट्रपवत शी
वजनबपग की विचारधारा "शी वजनबपग विचार" (Xi Jinping thought) के साथ-साथ ईनके
पूिणिती हू बजताओ के "िैज्ञावनक विकास" वसद्धांत को भी प्रवतष्ठावपत करता है।

4.1.9. न्यायपावलका

 चीन में एक प्रवतबद्ध न्यायपावलका है ऄथाणत जो समाजिाद के लक्ष्य के प्रवत प्रवतबद्ध है। आसका
ईच्चतर वनकाय सिोच्च जनिादी न्यायालय (Supreme People’s Court) है। चीन में एक कोिण
ऑफ़ प्रोक्यूरेिरेट्जस (Court of Procuratorates) भी है जो ऄवधकाररयों के भ्ष्टाचार से संबंवधत
मामलों को देखता है।

36 www.visionias.in ©Vision IAS

Google it:- https://upscpdf.com


https://upscpdf.com << Download From >> https://upscpdf.com

 चीनी कानूनों को कभी भी व्यिवस्थत रूप से संवहताबद्ध नहीं क्रकया गया है। ऄवधकांश वििादों
और झगड़ों का वनपिारा ऄधण-न्यावयक संस्थाओं िारा ही कर क्रदया जाता है। चीनी न्यावयक
प्रणाली कानून के बजाय परम्पराओं िारा ऄवधक संचावलत है।

4.1.10. कें रीय सै न्य अयोग (Central Military Commission)

 पािी और सरकार, कें रीय सैन्य अयोग के माध्यम से सेना पर वनयंत्रण बनाए रखती है।
 सेना को कम्युवनस्ि पािी के रक्षक के रूप में भी िर्वणत क्रकया गया है।

4.1.11. ऄवधकार और कतण व्य

ऄवधकार
 चीनी संविधान ऄपने नागररकों को मौवलक ऄवधकार प्रदान करता है और कु छ कतणव्यों का भी
प्रािधान करता है।
 18 िषण तथा ईससे ऄवधक अयु के सभी नागररकों को मतदान करने का ऄवधकार प्राप्त है। ईन्हें
चुनाि लड़ने का भी ऄवधकार प्राप्त है। सभी तरह के पत्राचार की गोपनीयता, िाक् एिं ऄवभव्यवक्त
की स्ितंत्रता, संघ में शावमल होने या संघ बनाने की स्ितंत्रता तथा सािणजवनक बैठकों का
अयोजन करना और साथ ही धरना प्रदशणन करने ऄथिा ऄपने ऄवधकारों की मांग के वलए हड़ताल
करने के ऄवधकार को संविधान के तहत संरवक्षत क्रकया गया है।
 संविधान के ऄनुसार, सरकार पूरी तरह से एक व्यवक्त की सत्यवनष्ठा के ऄवतररक्त ईसके
पाररिाररक जीिन के संरक्षण के वलए बाध्य है। सभी नागररकों को गैर-कानूनी वगरफ्तारी के
विरुद्ध व्यवक्तगत सुरक्षा का ऄवधकार प्राप्त है। संविधान िारा सभी नागररकों को वशक्षा का
ऄवधकार और सांस्कृ वतक स्ितंत्रता का समान ऄवधकार प्रदान क्रकया गया है। जीिन के सभी क्षेत्रों
में पुरुषों और मवहलाओं को समानता का दजाण प्रदान क्रकया गया है।
कर्त्णव्य
 चीनी संविधान ऄपने नागररकों के वलए स्पष्ट रूप से कु छ कर्त्णव्यों का वनधाणरण करता है जो
प्रितणनीय (justifiable) हैं। नागररकों का यह प्रथम और सबसे महत्िपूणण कर्त्णव्य है क्रक िे प्रत्येक
प्रकार से साम्यिादी नेतृत्ि को सहयोग प्रदान करें तथा संविधान और ऄन्य राज्य कानूनों का
पालन करें।
 साथ ही िे सािणजवनक संपवर्त् की रक्षा करें और कानून व्यिस्था को बनाए रखने में ऄपना सहयोग
दें। नागररकों का एक ऄन्य कर्त्णव्य यह भी है क्रक िे विदेशी अक्रमणों के विरुद्ध देश की रक्षा करें।

4.1.12. चीनी कम्यु वनस्ि पािी (Communist Party of China)

 चीनी कम्युवनस्ि पािी िषण 1921 में ऄवस्तत्ि में अयी। लेवनन ने ऄपने एक प्रवतवनवध को नि-
स्थावपत पािी को व्यिवस्थत करने में सहायता करने के वलए चीन भेजा। चेंग तू -वहसू (Cheng
Tu-hisu) को चीनी कम्युवनस्ि पािी का प्रथम महासवचि वनयुक्त क्रकया गया था। ऄत्यंत कम
ऄिवध में ही ऄनेक कस्बों और शहरों में पािी की कइ शाखाओं की स्थापना की गयी।

4.1.12.1. विचारधारात्मक अधार (Ideological Foundations)


 चीनी कम्युवनस्ि पािी की विचारधारा ऄपनी स्थापना के समय से ही लेवनन और माक्सण के
वशक्षाओं से प्रेररत थी। आसने ऄपने प्रारंवभक दौर में , िैवश्वक कम्युवनस्ि अन्दोलन के साथ गहरा
संबंध विकवसत क्रकया था। चीनी लोगों के साम्यिादी संघषण में माओ ने भी वनणाणयक भूवमका
वनभाइ थी।

37 www.visionias.in ©Vision IAS

Google it:- https://upscpdf.com


https://upscpdf.com << Download From >> https://upscpdf.com

4.1.12.2. पािी सं ग ठन
 यह पािी लोकतांवत्रक के न्रिाद के वसद्धांत पर अधाररत है। तदनुसार पािी के सभी पदावधकाररयों
का चयन क्रकया जाता है। पािी की प्राथवमक आकाइ (Primary unit) िारा वजला कांग्रेस का चयन
क्रकया जाता है जबक्रक वजला कांग्रेस िारा उपरी स्तर के कांग्रेस के प्रवतवनवधयों का चयन क्रकया
जाता है।
 पािी के सदस्यों को पािी नेतृत्ि की अलोचना करने का भी ऄवधकार प्राप्त है और आनके िारा पािी
की नीवतयों को तैयार करने संबंधी प्रस्ताि को प्रस्तुत क्रकया जा सकता है। आसी प्रकार, पािी की
प्राथवमक शाखा ऄपनी वशकायत ईच्च स्तरीय नेतृत्ि के समक्ष प्रस्तुत कर सकती है।
 दूसरी तरफ, पािी िारा कठोर ऄनुशासन स्थावपत क्रकया जाता है और वनणणय-वनमाणण प्रक्रक्रया में

के न्रीयता के वसद्धांत का पालन क्रकया जाता है। पािी में वनचले स्तर के सदस्य, पािी के ईच्च नेतृत्ि

के वनणणयों को मानने के वलए बाध्य होते हैं।

4.1.12.3. पोवलत ब्यू रो

 आसे वनणणय लेने की प्रक्रक्रया में सिाणवधक शवक्तशाली वनकाय के रूप में माना जाता है क्योंक्रक यह न
के िल सभी महत्िपूणण वनणणय लेता है बवल्क पािी की कें रीय सवमवत के सत्र को भी अहूत करता है।
आसके पास 7 सदस्यों की एक स्थायी सवमवत होती है। जब कें रीय सवमवत सत्र में नहीं रहती है तो

पोवलत ब्यूरो की स्थायी सवमवत, सरकार के भीतर ऄपने समकक्ष की भांवत, कें रीय सवमवत की
सभी शवक्तयों का प्रयोग करती है।

4.1.12.4. ने श नल कां ग्रे स ऑफ़ कम्यु वनस्ि पािी ऑफ़ चाआना (NCCPC)

 पािी की नीवत-वनमाणण में NCCPC को वनणाणयक वस्थवत प्राप्त हैI कांग्रेस के सदस्य {जो हजारों की

संख्या में होते हैं (संख्या वनवश्चत नहीं है)} 5 िषों के वलए संबंवधत क्षेत्रीय और स्थानीय

पािी कांग्रेस िारा चुने जाते हैं।

4.1.12.5. कें रीय सवमवत

 NCCPC का सत्र 5 िषों में एक बार (कु छ क्रदनों के वलए) ही अयोवजत होता है। कें रीय कायणकारी

सवमवत, जो सीवमत सदस्यों से वमलकर बनी होती है, कांग्रेस के सत्र में न रहने पर ईसकी शवक्तयों

का प्रयोग करती है। व्यिहार में, कें रीय कायणकारी सवमवत की शवक्तयों का प्रयोग आसके पोवलत

ब्यूरो िारा क्रकया जाता है, क्योंक्रक कें रीय कायणकारी सवमवत की बैठक यदाकदा ही अयोवजत होती
है। कें रीय सवमवत पोवलत ब्यूरो के सदस्यों के साथ-साथ आसके ऄध्यक्ष और ईपाध्यक्ष का भी चुनाि
करती है।

4.1.12.6. ऄन्य दल और समू ह

 चीनी जनिादी गणराज्य में , पूणण रूप से सोवियत संघ के समान एकल पािी प्रणाली नहीं ऄपनायी

गयी है, बवल्क छोिी पार्टियों, जैसे- कु ओबमतांग रेिोल्यूशनरी कवमिी, डेमोक्रेरिक लीग, नेशनल
कं स्रक्शन एसोवसएशन और विवभन्न युिा संगठनों को कायण करने की ऄनुमवत प्रदान की गयी है।
आसवलए चीन एक बहु-राष्ट्रीय और बहु-दलीय देश है। चीन में, लोकतांवत्रक दलों से अशय चीनी

साम्यिादी दल के ऄवतररक्त 8 ऄन्य दलों से है। नइ व्यिस्था की स्थापना के पश्चात् से

आन दलों ने विवभन्न स्तरों पर चीनी कम्युवनस्ि पािी के साथ सहयोग विकवसत क्रकया है।

38 www.visionias.in ©Vision IAS

Google it:- https://upscpdf.com


https://upscpdf.com << Download From >> https://upscpdf.com

 लेक्रकन, चीन में कम्युवनस्ि पािी को राजनैवतक एकावधकार प्राप्त है, जबक्रक ऄन्य पार्टियों का के िल
कानूनी रूप से ही ऄवस्तत्ि है। पािी संगठन का संचालन सरकार के समानांतर होता है। एक व्यवक्त
के सरकारी ऄवधकारी के रूप में महत्िपूणण पद पर रहते हुए ईसे पािी के भीतर भी वजम्मेदारी

प्रदान की जाती है। पािी का कें रीय नेतृत्ि मुख्य रूप से सरकारी नीवतयों की रूपरेखा तैयार करने
के वलए ईर्त्रदायी है। क्रकसी भी सरकारी विभाग का महत्ि के िल कानूनी वस्थवत के अधार पर
नहीं अंका जा सकता है क्योंक्रक पािी के ऄंदर आसकी भूवमका भी महत्ि रखती है।

5. फ्ांसीसी संविधान (French Constitution)


5.1. भू वमका

 फ्ांस को ‘राजनीवतक परीक्षण की प्रयोगशाला' (Laboratory of Political experiment) के


रूप में जाना जाता है।
 यहाँ सरकार का एकात्मक रूप है और सरकार की प्रकृ वत सेमी-प्रेवसडेंवशयल (ऄद्धण-ऄध्यक्षीय) है।

 आसमें कु छ विशेषताएँ संसदीय प्रणाली की हैं और ऄन्य विशेषताएँ प्रेवसडेंवशयल (ऄध्यक्षीय)

प्रणाली की हैं।

 फ्ांसीसी संसद को विवध-वनमाणण की सिोच्च वस्थवत प्राप्त नहीं है। विधावयका के िल एक वनधाणररत

सूची के तहत सवम्मवलत विषयों पर ही कानून बना सकती है, जबक्रक शेष विषय राष्ट्रपवत के
ऄधीन होते हैं (ऄथाणत् शेष विषयों पर िह कानून बनाता है)।
 संभित: यह एकमात्र लोकतांवत्रक संविधान है जो कायणकाररणी की सिोच्चता (Supremacy of

Executive) के वसद्धांत पर अधाररत है।

 ऐवतहावसक रूप से फ्ांस को राजनीवतक ऄवस्थरता का सामना करना पड़ा। आसवलए, 5िें गणतंत्र

(ररपवब्लक) के संविधान में 5 िषण की एक वनवश्चत ऄिवध के साथ, एक मजबूत राष्ट्रपवत

का प्रािधान क्रकया गया है और ईसे ऄत्यवधक शवक्तयां प्रदान की गइ हैं।

5.2. राष्ट्रपवत

 फ्ांस का राष्ट्रपवत फ्ें च प्रणाली के भीतर और विश्व के ऄन्य सभी लोकतांवत्रक देशों की

कायणकाररणी की तुलना में सिाणवधक शवक्तशाली है।

 आसमें ऄमेररका के राष्ट्रपवत पद का विशेषावधकार, ऄथाणत् कायणकाल की सुरक्षा और सरकार के

प्रमुख के साथ राज्य प्रमुख होने तथा विरिश प्रधानमंत्री के कायाणलय के विशेषावधकार, ऄथाणत्

विधान सभा को भंग करने की शवक्त (जो ऄमेररकी राष्ट्रपवत को प्राप्त नहीं है) सवम्मवलत हैं।

 फ्ांस में प्रधानमंत्री और राष्ट्रपवत दोनों के पदों का प्रािधान है।

o फ्ांसीसी प्रधानमंत्री, भारत और वििेन के विपरीत, राष्ट्रपवत का सहायक होता है।

o दोनों पदों में शवक्त के विभाजन के बजाय, कायों का विभाजन हैं।

 फ्ांस का राष्ट्रपवत विदेश नीवत और राष्ट्रीय मुद्दों को देखता है।

 दूसरी ओर, प्रधानमंत्री के ऄधीन सरकार के प्रवतक्रदन के वनयवमत कायण और स्थानीय

घरेलू मुद्दे होते हैं।

39 www.visionias.in ©Vision IAS

Google it:- https://upscpdf.com


https://upscpdf.com << Download From >> https://upscpdf.com

 प्रधानमंत्री, राष्ट्रपवत िारा वनयुक्त क्रकया जाता है-


o राष्ट्रपवत को प्रधानमंत्री के चुनाि में पूणण खुली छु ि प्राप्त नहीं है;
o प्रधानमंत्री के रूप में वनयुक्त व्यवक्त को सदन का विश्वास प्राप्त होना चावहए।
 'सहजीिन' (Cohabitation) की ऄिधारणा’
o एक ऐसी वस्थवत, जहां राष्ट्रपवत और प्रधानमंत्री वभन्न राजनीवतक दलों के सदस्य होते हैं।
 प्रधानमंत्री, ऄपने कै वबनेि के सहयोवगयों को चुन सकता है।
 सरकार का कोइ भी सदस्य विधावयका का भाग नहीं हो सकता है।
 कै वबनेि का संचालन राष्ट्रपवत िारा क्रकया जाता है।
 वनम्न सदन, प्रधानमंत्री और ईसकी मंत्रीपररषद के विरुद्ध 'बनदा प्रस्ताि' पाररत कर सकता है।
प्रस्ताि पाररत होने पर ईन्हें वनवश्चत रूप से त्यागपत्र देना होता है।
 राष्ट्रपवत एक वनवश्चत ऄिवध के वलए चुना जाता है। प्रारंभ में पदािवध 9 िषण थी, तत्पश्चात आसे
घिाकर 7 िषण की गयी और ितणमान में यह ऄिवध 5 िषण है।
 राष्ट्रपवत चुनाि में वितीय मतदान (Second Ballot) प्रणाली का ऄनुसरण क्रकया जाता है।
(ऄथाणत् मतदान में शावमल कु ल मतों के पूणण बहुमत की अिश्यकता होती है।)
o फ्ांस गणराज्य के राष्ट्रपवत को मतों के पूणण बहुमत से वनिाणवचत क्रकया जाता है; यक्रद चुनाि के
पहले दौर में, क्रकसी भी व्यवक्त को पूणण बहुमत प्राप्त नहीं होता है तब के िल शीषण दो
ईम्मीदिारों को छोड़कर बाकी सभी ईम्मीदिारों को हिा क्रदया जाता है। तदुपरान्त चुनाि
का दूसरा दौर संपन्न होता है, वजसमें एक व्यवक्त पूणण बहुमत प्राप्त करने में सक्षम होता है।

5.2.1. राष्ट्रपवत का हिाया जाना

 फ़्ांस के राष्ट्रपवत को ऄमेररकी राष्ट्रपवत के समान अधार पर ही महावभयोग िारा हिाया जा


सकता है। हालांक्रक, आसकी प्रक्रक्रया ऄस्पष्ट है।
 संविधान के ऄनुच्छेद 67 में कहा गया है क्रक दोनों सदनों को समान प्रस्ताि पाररत करना चावहए।
 आसके बाद, राष्ट्रपवत के मामले का एक विशेष वनकाय िारा समाधान क्रकया
जाता है वजसे हाइ कोिण ऑफ जवस्िस कहा जाता है।
 यह वनकाय सरकार के मंत्रालयों िारा राज्य के विरुद्ध भ्ष्टाचार और षड्यंत्र के मामलों की भी
जांच करता है।

5.2.2. राष्ट्रपवत की अपातकालीन शवक्तयां

 संविधान का ऄनुच्छेद 16, राष्ट्रपवत को िास्तविक अपात शवक्तयां प्रदान करता है। आस वस्थवत में
ईसे ऄसीवमत शवक्तयां प्राप्त होती हैं और यह लोकतांवत्रक तानाशाही या लोकतांवत्रक तख्तापलि
(democratic coup-d’etat) के समान है।

5.2.3. ऄमे ररका और फ्ां स के राष्ट्रपवतयों का तु ल नात्मक विश्ले ष ण

 ऄमेररकी राष्ट्रपवत विधानसभा को भंग नहीं कर सकता, जबक्रक फ्ांस का राष्ट्रपवत ऐसा कर सकता
है। आस पर सीमा के िल यह है क्रक िह ऐसा िषण में दो बार से ऄवधक नहीं कर सकता है।
 ऄमेररकी राष्ट्रपवत के विपरीत, फ्ांस का राष्ट्रपवत ऄनुच्छेद 16 के तहत तानाशाही
शवक्तयां ग्रहण कर सकता है।

40 www.visionias.in ©Vision IAS

Google it:- https://upscpdf.com


https://upscpdf.com << Download From >> https://upscpdf.com

5.2.4. फ्ां स के राष्ट्रपवत और विरिश प्रधानमं त्री का तु ल नात्मक विश्ले ष ण

 विरिश प्रधानमंत्री के िल तब तक ऄपने पद पर बना रह सकता है जब तक ईसे वनचले सदन में


बहुमत प्राप्त होता है। दूसरी तरफ, फ्ांस का राष्ट्रपवत, एक वनवश्चत ऄिवध के वलए चुना जाता है।

5.3. विधावयका (Legislature)

 फ्ें च प्रणाली में विधावयका स्पष्ट रूप से कायणपावलका के ऄधीनस्थ होती है। संविधान का ऄनुच्छेद
37, फ्ांसीसी संसद की विधायी शवक्तयों का स्पष्ट सीमांकन करता है। आसमें कहा गया है क्रक संसद
के िल संविधान में ईवल्लवखत मामलों पर ही कानून बना सकती है। ऄन्य सभी मामलों में सरकार
साधारण अदेश या वडक्री िारा कानून बना सकती है।
 राष्ट्रपवत, प्रधानमंत्री के माध्यम से सीधे ऄसेंबली (विधानसभा) के विधायी कायों को प्रभावित कर
सकता है। यक्रद ऄसेंबली क्रकसी विशेष विधेयक से सहमत नहीं है तो राष्ट्रपवत िारा ईसे जनमत
संग्रह के वलए प्रस्तुत क्रकया जा सकता है।
 फ्ांसीसी संसद, दो सदनों से वमलकर बनी है: नेशनल ऄसेंबली और सीनेि।

5.3.1. ने श नल ऄसें ब ली (National Assembly)

 ऄन्य विसदनीय संसदों के मामलों के समान, फ्ें च विसदनीय प्रणाली भी एक ऄसमान प्रणाली है,
क्योंक्रक यहां नेशनल ऄसेंबली को सीनेि की तुलना में ऄवधक व्यापक ऄवधकार प्राप्त हैं:
o यह ऄके ले सरकार को 'बहुमत' प्रदान करने से मना करके या ‘बनदा प्रस्ताि’ पाररत करके
जिाबदेह बना सकती है (आस विचार के ऄनुसार, फ्ें च गणराज्य के राष्ट्रपवत िारा के िल
नेशनल ऄसेंबली को ही भंग क्रकया जा सकता है)।
o विधायी प्रक्रक्रया में सीनेि के साथ ऄसहमवत होने पर, सरकार नेशनल ऄसेंबली को
“ऄंवतम वनणणय” (final say) का ऄवधकार प्रदान कर सकती है (संिैधावनक कानूनों और
सीनेि से संबद्ध संस्थागत कानूनों को छोड़कर);
o संविधान नेशनल ऄसेंबली को विर्त् विधेयक और सामावजक सुरक्षा विर्त्पोषण विधेयक की
जांच के मामले में और ऄवधक महत्िपूणण भूवमका प्रदान करता है। आस तरह के विधेयक
के प्रथम िाचन के पूिण आन्हें नेशनल ऄसेंबली के समक्ष प्रस्तुत क्रकया जाना चावहए। आनकी
जांच के वलए नेशनल ऄसेंबली के वलए वनधाणररत समय-सीमा भी ऄत्यवधक होती है।

5.3.2. सीने ि (The Senate)

 नेशनल ऄसेंबली के विपरीत, सीनेि को भंग नहीं क्रकया जा सकता है। सीनेि का एक स्थायी
वनकाय होना सरकार की वस्थरता के वलए तब महत्िपूणण भूवमका वनभाता है जब फ्ांस गणराज्य के
राष्ट्रपवत का पद ररक्त हो जाता है। राष्ट्रपवत िारा ऄपने पद से त्यागपत्र देने या बीमारी के कारण
ऄपने कतणव्यों का वनिणहन करने में विफल होने की वस्थवत में सीनेि के ऄध्यक्ष को फ्ांस गणराज्य के
राष्ट्रपवत के रूप में वनयुक्त क्रकया जाता है। आस प्रकार, राष्ट्रपवत का पद ररक्त होने पर, शवक्त शून्यता
(power vacuum) की वस्थवत से बचा जा सकता है।
 यह ऄंतररम व्यिस्था राष्ट्रपवत पद का चुनाि संपन्न कराने की समयािवध तक सीवमत है (व्यिहार
में, यह ऄिवध लगभग 50 क्रदनों तक की होती है)।

41 www.visionias.in ©Vision IAS

Google it:- https://upscpdf.com


https://upscpdf.com << Download From >> https://upscpdf.com

5.4. फ्ां सीसी सं विधान की प्रमु ख विशे ष ताएँ

 ऑगेवनक लॉ (Organic Law): ऑगेवनक या फं डामेंिल लॉ िस्तुतः क्रकसी सरकार या संगठन के


अधार (फाईं डेशन) का सृजन करते हैं। ‘संविधान’ िस्तुतः एक संप्रभु राष्ट्र के वलए ऑगेवनक लॉ का
एक विवशष्ट रूप होता है। फ्ांसीसी संविधान में ऑगेवनक लॉ के रूप में कु छ कानूनों का ईल्लेख
क्रकया गया है। संसद और कायणपावलका के अदेश िारा वनर्वमत विवधयों को ऄिश्य हीं ऑगेवनक लॉ
से पुष्ट होना चावहए। ऄतः आन कानूनों की समीक्षा एक वनकाय के िारा की जाती है वजसे
संिध
ै ावनक पररषद (Constitutional Council) के रूप में जाना जाता है। आसमें 9 सदस्य होते हैं,
यथा- तीन राष्ट्रपवत के प्रवतवनवध, तीन फ्ें च नेशनल ऄसेंबली के प्रवतवनवध और शेष तीन सीनेि के
प्रवतवनवध।
 हाइ काईं वसल ऑफ़ जवस्िस (ईच्च न्यावयक पररषद): यह पररषद् न्यायाधीशों की वनयुवक्त करती है।
आसका नेतृत्ि राष्ट्रपवत और न्यायपावलका के सदस्य करते हैं। राष्ट्रपवत को 'न्यायपावलका के
संरक्षक' के रूप में भी जाना जाता है।
 आकॉनवमक एंड सोशल काईं वसल (अर्वथक और सामावजक पररषद): यह सामावजक और अर्वथक
मुद्दों से संबंवधत एक संिैधावनक सलाहकार वनकाय है।

5.5. सं विधान का सं शोधन

 कठोर प्रक्रक्रया;
 संसद के दोनों सदनों िारा 3/5 बहुमत से प्रस्ताि पाररत करना होता है;
 राष्ट्रपवत, जनमत संग्रह के िारा संशोधन प्रस्तुत करने का विकल्प चुन सकता है।

6. जमणनी का सं विधान (Germany’s Constitution)


 जमणनी एक संघीय राष्ट्र है और ऄिवशष्ट शवक्तयां राज्यों में वनवहत हैंI
 राज्यों को ‘लैंडसण (Landers)’ के रूप में संदर्वभत क्रकया जाता हैI
 यहां विरिश संसदीय प्रणाली के समान ही सरकार की संसदीय प्रणाली विद्यमान है, ककतु यह
पूणणतः ईसकी प्रवतकृ वत नहीं हैI
 जमणनी को ‘चांसलर डेमोक्रेसी’ (Chancellor’s Democracy) की संज्ञा दी जाती हैI
 चांसलर, प्रधानमंत्री होता हैI
 राष्ट्रपवत, संिैधावनक प्रमुख होता हैI

6.1. प्रमु ख विशे ष ताएँ

6.1.1. चां स लर डे मोक्रे सी (Chancellor’s Democracy)

 चांसलर ऄन्य मंवत्रयों से श्रेष्ठ होता हैI


 चांसलर वसद्धांत: चांसलर को एक व्यापक नीवत वनधाणररत करने का विशेषावधकार प्राप्त होता है
और ऄन्य मंवत्रयों से आन नीवतयों के ऄनुसार कायण करने की ऄपेक्षा की जाती हैI आन नीवतयों के
ऄनुसार कायण करने की वस्थवत में मंवत्रयों को ऄपने विभाग के संबंध में व्यापक स्िायर्त्ता प्राप्त
होती हैI
यह प्रणाली गठबंधन सरकार की वस्थरता सुवनवश्चत करती हैI

42 www.visionias.in ©Vision IAS

Google it:- https://upscpdf.com


https://upscpdf.com << Download From >> https://upscpdf.com

6.1.2. कै वबने ि वसद्धां त (Cabinet Principle)

 यह के िल विवभन्न विभागों के मध्य परस्पर वििाद ईत्पन्न होने की वस्थवत में ही प्रभािी होता हैI
ऐसी वस्थवत में वनणणय सामूवहक सहमवत के अधार पर वलए जाते हैंI

6.1.3. रचनात्मक ऄविश्वास प्रस्ताि (Constructive Vote of No-Confidence)

 चांसलर के विरुद्ध ऄविश्वास प्रस्ताि लाने की ऄनुमवत के िल तभी दी जाती है जब ऄविश्वास


प्रस्ताि पेश करने िाला यह वसद्ध करे क्रक िह िैकवल्पक सरकार का गठन करने की वस्थवत में हैI
 यह व्यिस्था वत्रशंकु विधानसभा (hung assembly) या गठबंधन सरकार की समस्याओं से
वनपिने के वलए की गयी हैI

6.1.4. सं स द

 जमणनी की संसद में दो सदन (विसदनीय व्यिस्था) विद्यमान हैं, यथा- बुद
ं स्े िैग (Bundestag)
और बुद
ं स्र
े त (Bundesrat)I

6.1.4.1. बुं देस्िै ग (The Bundestag)


 जमणनी की राजनैवतक व्यिस्था में वनचले सदन को बुंदस्े िैग कहा जाता हैI आसके सदस्यों का चुनाि
4 िषण के कायणकाल के वलए होता हैI आनका वनिाणचन वमवश्रत सदस्य अनुपावतक प्रवतवनवधत्ि
(Mixed Member Proportional Representation: MMPR) िारा होता है। ज्ञातव्य है क्रक
यह फस्िण-पास्ि-द-पोस्ि (FPTP) से ऄवधक जरिल प्रणाली है, लेक्रकन यह ऄवधक अनुपावतक
पररणाम प्रदान करती हैI {‘ऄवतररक्त सदस्य प्रणाली’ (additional member system) िस्तुतः आसी
प्रणाली का एक प्रकार है, वजसका ईपयोग स्कॉरिश संसद और िेल्श ऄसेंबली में क्रकया जाता है। }
वनिाणचन पद्धवत
 बुंदस्े िैग के अधे सदस्यों का वनिाणचन 299 वनिाणचन क्षेत्रों से प्रत्यक्ष रीवत के माध्यम से FPTP
प्रणाली के अधार पर होता हैI
 शेष अधे (ऄन्य 299) सदस्यों का चुनाि पार्टियों की सूची से प्रत्येक लैंड ऄथिा लैंडर के अधार
पर क्रकया जाता हैI {जमणनी को 16 संघीय राज्यों (लैंडर) में विभावजत क्रकया गया हैI)}
आसका ऄथण यह है क्रक बुंदस्े िैग के चुनाि में प्रत्येक मतदाता के पास दो मत होते हैंI
 मतदाता ऄपने पहले मत का प्रयोग संसद के वलए ऄपने स्थानीय प्रवतवनवध को वनिाणवचत करने के
वलए करते हैंI यह तय करता है क्रक एक वनिाणचन क्षेत्र से कौन-सा ईम्मीदिार संसद के वलए
वनिाणवचत होगाI
 मतदाता ऄपने दूसरे मत का प्रयोग पािी वलस्ि (सूची) के वलए करते हैं और यही दूसरा िोि
बुंदस्े िैग में प्रवतवनवधत्ि करने िाली पार्टियों की अनुपावतक संख्या को वनधाणररत करता हैI
संसद की कु ल 598 सीिों का वितरण ईन्हीं पार्टियों के मध्य में क्रकया जाता है, वजन्होंने वितीय मतदान
(second votes) के 5% से ऄवधक मत या कम से कम 3 प्रत्यक्ष जनादेश प्राप्त क्रकए होंI बुंदस्े िैग में
एक पािी को सीिों का अबंिन ईनके िारा प्राप्त क्रकए गए मतों की संख्या के ऄनुपात में क्रकया जाता हैI
ईपयुक्त
ण वनिाणचन पद्धवत ऄपनाने के कारण
 यह प्रणाली छोिी ि ईग्रपंथी पार्टियों को बुंदस्े िैग के सदस्य के रूप में वनिाणवचत होने से रोकने के
वलए वनर्वमत की गइ हैI पररणामस्िरूप, बुंदस्े िैग में प्रवतवनवधत्ि प्राप्त पार्टियों की संख्या सदैि कम
होती हैI

43 www.visionias.in ©Vision IAS

Google it:- https://upscpdf.com


https://upscpdf.com << Download From >> https://upscpdf.com

बुद
ं स्े िैग का तुलनात्मक विश्लेषण
 जब बुंदस्े िैग की तुलना ऄमेररकी कांग्रेस या विरिश हाईस ऑफ़ कॉमंस से की जाती है तो एक
विशेष ऄंतर यह पाया जाता है क्रक जमणनी में सदस्य ऄपने वनिाणचन क्षेत्रों में कम समय दे पाते हैं।
 ऐसा आसवलए है क्योंक्रक:
o बुंदस्े िैग के के िल 50% सदस्य ही एक विवशष्ट भौगोवलक क्षेत्र का प्रत्यक्ष प्रवतवनवधत्ि करने
के वलए चुने जाते हैंI
o मतदाताओं तथा प्रवतवनवधयों िारा ऄपने वनिाणचन क्षेत्र को विवध वनमाणता के महत्िपूणण
कायणक्षेत्र के रूप में नहीं समझा जाता हैI
o आसका एक ऄन्य कारण, बुंदस्े िैग के सदस्यों के वनजी स्िाफ की संख्या में कमी का होना है
(विशेष रूप से ऄमेररकी कांग्रेस की तुलना में)I

6.1.4.2. बुं देस्र त (The Bundesrat)


 जमणनी की राजनैवतक व्यिस्था में उपरी सदन को बुंदस्र
े त कहा जाता हैI
 बुंदस्र
े त की संरचना ऄन्य राष्ट्रों के उपरी सदनों, जैसे- ऄमेररकी सीनेि के लगभग समान ही है,
क्योंक्रक यह (बुंदस्र
े त) सभी जमणन लैंडर का प्रवतवनवधत्ि करने िाला सदन हैI हालांक्रक जमणन
प्रणाली में दो मौवलक वभन्नताएं विद्यमान हैं:
o आसके सदस्य वनिाणवचत नहीं होते हैं (न तो जनता िारा और न ही संसद सदस्यों िारा)I आन्हें
राज्य मंवत्रमंडल के सदस्यों िारा वनयुक्त क्रकया जाता है और क्रकसी भी समय हिाया जा
सकता हैI सामान्यतः, लैंडर के प्रवतवनवध मंडल का नेतृत्ि, ईसकी सरकार के प्रमुख िारा
क्रकया जाता है वजसे जमणनी में मंत्री-ऄध्यक्ष (Minister-President) के रूप में जाना जाता हैI
o बुंदस्र
े त में प्रत्येक लैंडर को समान प्रवतवनवधत्ि प्रदान नहीं क्रकया गया है ऄथाणत् प्रत्येक लैंडर
के प्रवतवनवधयों की संख्या समान नहीं होती है। लैंडर में मतों का अबंिन ईनकी जनसंख्या
(प्रवतवनवधयों के बजाय) के अधार पर क्रकया जाता हैI मतों का अबंिन 2.01 + ऄवतररक्त
वनधाणररत 6 िोि के साथ देश की जनसंख्या (लाखो में) के िगणमूल के रूप में क्रकया जाता है,
आसीवलए आसे गेम थ्योरी (game theory) पर अधाररत पेनरोज विवध (Penrose
method) कहा जाता हैI आसका ऄथण यह है क्रक 16 लैंडर में से प्रत्येक के प्रवतवनवधयों की
संख्या तीन से छह होगीI
 यह ऄसामान्य विवध, बुंदस्र
े त की संरचना में कु ल 69 मत (सीि नहीं) प्रदान करती है I लैंडर का
मंवत्रमंडल आसे (लैंडर) प्राप्त मतों के बराबर ही प्रवतवनवधयों को वनयुक्त कर सकता है , हालांक्रक यह
एक बाध्यकारी प्रािधान नहीं है; आस प्रकार यह प्रवतवनवध-मंडल को एक प्रवतवनवध तक भी
सीवमत करने की ऄनुमवत प्रदान करता हैI औपचाररक रूप से सदस्यों की संख्या या क्रकसी विशेष
लैंडर से प्रवतवनवधत्ि करने िाले प्रवतवनवधयों की संख्या का विशेष महत्ि नहीं होता हैI ऄन्य
विधायी वनकायों के विपरीत, यहाँ क्रकसी लैंडर से चुने गए बुंदस्र
े त के प्रवतवनवधयों को एक ब्लॉक
के रूप में िोि करना होता है (ऄथाणत् ईक्त िोि संबंवधत प्रवतवनवध का नहीं होता है)I आस
प्रकार व्यािहाररक रूप से (पूणणतः प्रथागत) यह संभि है क्रक के िल एक ही प्रवतवनवध
(Stimmfuhrer या “िोि के नेता” – सामान्यतः मंत्री-ऄध्यक्ष) ऄपने संबंवधत लैंड के सभी मतों को
डालता है, चाहे ऄन्य सदस्य सदन (chamber) में ईपवस्थत होंI
 सभी 69 प्रवतवनवधयों की वनयुवक्त के पश्चात् भी, बुंदस्र
े त, बुंदस्े िैग के 598 सदस्यों की तुलना में
एक ऄवत लघु वनकाय हैI यह व्यिस्था (ऄसमान सीि) एक विसदनात्मक प्रणाली के दोनों सदनों के
वलए पूणणतः ऄसामान्य हैI हालांक्रक, बुंदस्र
े त को क्रकसी विधेयक के संदभण में िीिो की शवक्त प्राप्त हैI

44 www.visionias.in ©Vision IAS

Google it:- https://upscpdf.com


https://upscpdf.com << Download From >> https://upscpdf.com

7. जापान का संविधान
 जापान में सरकार की संसदीय प्रणाली विद्यमान हैI
 यहाँ संिैधावनक राजतंत्र की प्रणाली हैI
o हालांक्रक, राजा एक रबर स्िाम्प की भांवत कायण करता है जबक्रक प्रधानमंत्री मंवत्रमंडल का
प्रमुख होता हैI
 प्रधानमंत्री का वनिाणचन
o प्रधानमंत्री, जापान की संसद (वजसे डायि कहा जाता है) के दोनों सदनों िारा चुना जाता हैI
o डायि के दो सदन हैं:
 हाईस ऑफ़ ररप्रजेंिेरिि (House of Representatives), तथा
 हाईस ऑफ़ काईं वसलसण (House of Councillors)
o प्रधानमंत्री का पद प्राप्त करने के वलए एक व्यवक्त के वलए के िल बहुमत िाले दल का नेता
होना ही पयाणप्त नहीं हैI ईसे संसद के दोनों सदनों िारा वनिाणवचत होना चावहएI
 यक्रद क्रकसी ईम्मीदिार के संबंध में संसद के दोनों सदनों में कोइ सहमवत नहीं बनती है
तब आस मामले को दोनों सदनों की संयुक्त सवमवत को सौंप क्रदया जाता हैI सवमवत को
वनणणय लेने के वलए 10 क्रदन का समय क्रदया जाता हैI
 10 क्रदन के ईपरांत भी यक्रद समझौता नहीं हो पाता है तो ईसके पश्चात् वनचले सदन का
मत मान्य होगाI
 जापान के संविधान की एक महत्िपूणण विशेषता ऄनुच्छेद 9 में वनवहत हैI
o जापान औपचाररक रूप से ऄंतराणष्ट्रीय वििादों के वनपिारे के वलए युद्ध की नीवत का पररत्याग
करता है, हालांक्रक, यह अत्मरक्षा के वलए सेना रख सकता हैI

8. कनाडा का संविधान
 कनाडा के संविधान में वसद्धांत और मूल्यों के विस्तृत समुच्चय को शावमल क्रकया गया है, जो कनाडा
में प्रमुख राजनैवतक संबंधों को संचावलत करता हैI

8.1. प्रमु ख विशे ष ताएं

8.1.1. सं िै धावनक राजतं त्र

 यह कनाडा के संिैधावनक ढांचे का कें रीय घिक हैI


 संविधान ऄवधवनयम, 1867 में ईल्लेख क्रकया गया है क्रक कनाडा में कायणकारी शासन और
प्रावधकार, कनाडा की राजशाही (वजसे कनाडा, ग्रेि वििेन और कु छ ऄन्य पूिणिती विरिश
ईपवनिेशों से साझा करता है) में वनवहत हैI विरिश साम्राज्ञी राज्य की औपचाररक प्रमुख हैंI
o यह ऄवधवनयम, कनाडा के गिनणर जनरल (संघीय स्तर पर) और लेवफ्िनेंि गिनणर (प्रांतीय
स्तर पर) के पद का प्रािधान करता हैI आन्हें कनाडा में राजा के प्रवतवनवध के रूप में स्िीकार
क्रकया गया हैI
 ईल्लेखनीय है क्रक, यद्यवप वलवखत संविधान िारा स्पष्ट रूप से सम्राि और ईसके प्रवतवनवधयों को
कायणकारी प्रावधकार प्रदान क्रकए गए हैं, क्रफर भी ऄवलवखत संिैधावनक परंपराओं के ऄनुसार
िास्तविक तौर पर आन ऄवधकारों का प्रयोग प्रधानमंत्री और ईसके मंवत्रमंडल िारा क्रकया जाता हैI

45 www.visionias.in ©Vision IAS

Google it:- https://upscpdf.com


https://upscpdf.com << Download From >> https://upscpdf.com

8.1.2. सं स दीय सरकार

 कनाडा के संविधान में भी सरकार की संसदीय प्रणाली को ऄपनाया गया हैI


संविधान ऄवधवनयम, 1867 में ईवल्लवखत संसदीय सरकार की विशेषताएँ
 यह ऄवधवनयम एक संघीय संसद की स्थापना का प्रािधान करता है, वजसमें सम्राि और दो
विधान-मंडल {हाईस ऑफ़ कॉमंस (वनचला सदन) एिं सीनेि (उपरी सदन)} शावमल हैंI
 आस ऄवधवनयम में ईवल्लवखत है क्रक आन विधानमंडलों के प्रावधकार और शवक्तयों का प्रारूप विरिश
संसद के समान हैंI
 आसके ऄवतररक्त, यह ऄवधवनयम प्रांतीय स्तर पर भी विधान-मंडलों की स्थापना का प्रािधान
करता हैI
आस ऄवधवनयम के वलवखत प्रािधानों के ऄवतररक्त, यहाँ कइ ऄवलवखत संिैधावनक परंपराएं
(unwritten constitution conventions) भी प्रचवलत हैं जो कनाडा की संसदीय प्रणाली के
संचालन का मुख्य अधार हैंI आसमें प्रधानमंत्री और मंवत्रमंडल (संघीय स्तर पर) एिं प्रीवमयर और
मंवत्रमंडल (प्रांतीय स्तर पर) के कायणकारी प्रभुत्ि तथा ईर्त्रदायी सरकार की व्यिस्था शावमल हैI
हाईस ऑफ़ कॉमंस
 कनाडा की राजनैवतक प्रणाली में वनचले सदन को हाईस ऑफ़ कॉमंस कहते हैं, वजसका नाम
विरिश राजनैवतक प्रणाली से ऄपनाया गया हैI हाईस ऑफ़ कॉमंस में 308 सदस्य शावमल होते हैं
वजन्हें वििेन की भांवत संसद सदस्य (MPs) कहा जाता हैI
वनिाणचन पद्धवत
 सदस्यों का चुनाि देश के प्रत्येक वनिाणचन वजलों में फस्िण-पास्ि-द-पोस्ि प्रणाली िारा क्रकया जाता
हैI आन वनिाणचन वजलों को सामान्यतः रीबडग्स (ridings) (वििेन में आन्हें वनिाणचन क्षेत्र कहा जाता
है) के रूप में जाना जाता हैI वनचले सदन (हाईस ऑफ़ कॉमंस) में सीिों का विभाजन सामान्यतः
प्रांतों और क्षेत्रों की जनसंख्या के ऄनुपात में क्रकया जाता हैI कु छ रीबडग्स की जनसंख्या ऄन्य से
ऄवधक है तथा कनाडा के संविधान में प्रांतीय प्रवतवनवधत्ि के संबंध में कु छ विशेष प्रािधानों को
सवम्मवलत क्रकया गया है I
ऄिवध और कायणकाल
 सदस्यों का ऄवधकतम कायणकाल 4 िषण होता है, लेक्रकन आससे पूिण भी अम चुनाि हो सकते हैंI
शवक्तयां
 विरिश राजनैवतक प्रारूप की भांवत, दोनों सदनों में हाईस ऑफ़ कॉमंस ऄवधक शवक्तशाली है, क्रफर
भी सभी विधेयक दोनों सदनों िारा ऄनुमोक्रदत क्रकए जाते हैI हालांक्रक, व्यिहार में वनिाणवचत सदन
की आच्छा सीनेि पर प्रभािी होती हैI हाईस ऑफ़ कॉमंस की प्रक्रक्रयाएं एिं अयोजन संभितः
विरिश पररपारियों को प्रवतबबवबत करते हैंI
सीनेि
 कनाडा की राजनैवतक प्रणाली में सीनेि (उपरी सदन) का नामकरण, ऄमेररकी राजनैवतक प्रणाली
के उपरी सदन के अधार पर क्रकया गया हैI
 सीनेि में 105 सदस्य शावमल होते हैंI सीनेि के सदस्य प्रधानमंत्री की सलाह पर गिनणर जनरल
िारा वनयुक्त क्रकए जाते हैंI सीिें क्षेत्रीय अधार पर वनर्ददष्ट की जाती हैंI चार प्रमुख क्षेत्रों में से
प्रत्येक के वलए 24 सीिें और छोिे क्षेत्रों के वलए शेष 9 सीिें वनर्ददष्ट की गइ हैंI

46 www.visionias.in ©Vision IAS

Google it:- https://upscpdf.com


https://upscpdf.com << Download From >> https://upscpdf.com

8.1.3. सं घ िाद (Federalism)

 कनाडा के संविधान िारा देश के वलए संघीय प्रणाली का प्रािधान क्रकया गया है वजसमें सरकार के
दो मुख्य स्तर होते हैं: संघीय सरकार (राष्ट्रीय) और प्रांतीय सरकारें (क्षेत्रीय)I कनाडा एक सुदढ़ृ
कें र िाला संघ है, वजसमें ऄिवशष्ट शवक्तयां कें र में वनवहत हैंI
 संविधान ऄवधवनयम, 1867 िारा सरकार के प्रत्येक स्तरों के वलए विवशष्ट शवक्तयों और
क्षेत्रावधकार को वनधाणररत गया है, जैसे- सािणजवनक नीवत के वनमाणण, सरकार के प्रत्येक स्तर हेतु
राजस्ि संबंधी क्षेत्रावधकार, अक्रदI हाल के िषों में आन संिैधावनक प्रािधानों को न्यावयक वनणणयों
िारा (पहली बार विरिश वप्रिी कौंवसल की न्यावयक सवमवत िारा और बाद में कनाडा के सुप्रीम
कोिण िारा) और ऄवधक स्पष्ट रूप से वनधाणररत एिं विकवसत क्रकया गया हैI
कनाडा के संघिाद की प्रकृ वत में पररितणन
 कनाडा के संविधान में कइ बार संशोधन क्रकए गए हैं वजनका कनाडा की संघीय प्रणाली पर
महत्िपूणण प्रभाि पड़ा हैI विगत कु छ िषों में राज्यों को और ऄवधक ऄवधकार प्रदान क्रकए गए हैंI
ईदाहरण के वलए, संविधान ऄवधवनयम, 1930 िारा पवश्चम कनाडा के प्राकृ वतक संसाधनों के
स्िावमत्ि को संघीय सरकार से पवश्चमी प्रांतों को हस्तांतररत क्रकया गयाI कु छ ऄन्य महत्िपूणण
संशोधन, संविधान ऄवधवनयम, 1982 िारा क्रकए गए हैंI यह ऄवधवनयम संघीय सरकार तथा
प्रांतीय सरकारों के मध्य अर्वथक और सामावजक समानता के वनवश्चत स्तर को सुवनवश्चत करने के
वलए प्रवतबद्ध थाI
 आस प्रकार से आन ऄवधवनयमों ने समकक्ष कायणक्रमों के विकास और सािणजवनक वनवध को सरकारों
के मध्य साझा करने का मागण प्रशस्त क्रकयाI

8.1.4. न्यायपावलका

 कनाडा में सुप्रीम कोिण वसविल, अपरावधक और संिैधावनक मामलों पर वनणणयन के वलए ऄंवतम
एिं ईच्चतम वनकाय हैI प्रधानमंत्री और कानून मंत्री के परामशण पर गिनणर जनरल के िारा आसके 9
सदस्यों की वनयुवक्त की जाती हैI िे 75 िषण की अयु तक पद पर बने रहते हैंI
 प्रत्येक प्रांत की ऄपनी पृथक-पृथक न्यावयक प्रणाली विद्यमान हैI देश की कानून-प्रणाली आंवग्लश
कॉमन लॉ पर अधाररत है, लेक्रकन क्यूबेक प्रान्त में यह फ़्ांसीसी वसविल लॉ पर अधाररत हैI

8.1.5. ऄवधकार

 कै नेवडयन चािणर ऑफ़ राआट्जस एंड फ़्ीडम िस्तुतः कनाडा के संविधान में समावहत वबल ऑफ़
राआट्जस (ऄवधकार-पत्र) की भांवत है तथा यह संविधान ऄवधवनयम, 1982 के प्रथम भाग का
वनमाणण करता हैI
 यह चािणर कनाडा के नागररकों को कु छ विशेष राजनैवतक ऄवधकारों की और कनाडा में प्रत्येक
व्यवक्त के वलए नागररक ऄवधकारों की गारंिी प्रदान करता हैI
 यह चािणर, सरकारी कानून और कायणिावहयों (संघीय, प्रांतीय और नगरपावलका सरकारों के कानून
और कायणिावहयां तथा पवब्लक स्कू ल बोडों की कायणिावहयां) तथा कभी-कभी सामान्य कानून के
वलए लागू होता है, लेक्रकन वनजी गवतविवधयों पर लागू नहीं होता हैI
 चािणर में ईवल्लवखत ऄवधकारों के ईल्लंघन की वस्थवत में , ऄदालतों ने कइ बार संघीय और प्रांतीय
कानूनों एिं विवनयमों को पूणणतया या ईसके क्रकसी भी भाग को ऄसंिैधावनक घोवषत कर वनरस्त
क्रकया हैI

47 www.visionias.in ©Vision IAS

Google it:- https://upscpdf.com


https://upscpdf.com << Download From >> https://upscpdf.com

9. ऑस्रे वलया का संविधान (Constitution Of


Australia)
 ऑस्रेवलया की शासन प्रणाली एक ईदार लोकतांवत्रक परंपरा है। धार्वमक सवहष्णुता, ऄवभव्यवक्त
एिं संगम की स्ितंत्रता और विवध के शासन के मूल्यों पर अधाररत ऑस्रेवलया के संस्थान तथा
सरकार की कायणप्रणाली, विरिश और ईर्त्री ऄमेररकी मॉडल को प्रदर्वशत करती है। हालांक्रक, आस
प्रणाली में कु छ ऑस्रेवलयाइ विवशष्टताएं भी शावमल हैं।
o ऑस्रेवलयाइ संघ, ऄमेररकी संघ के मॉडल पर अधाररत है। ईदाहरण के वलए, ऄिवशष्ट

शवक्तयां राज्यों में वनवहत हैं, राज्यों के गिनणर जनता िारा वनिाणवचत और औपचाररक रूप से

विरिश साम्राज्ञी िारा वनयुक्त क्रकए जाते हैं।

o हाल के िषों में, ऑस्रेवलया में सहकारी संघिाद में िृवद्ध हुइ है।

9.1. प्रमु ख विशे ष ताएं

9.1.1. शासन के प्रकार (Form of Government)

 विश्व में सबसे प्राचीन लोकतांवत्रक देशों में से एक कॉमनिेल्थ ऑफ़ ऑस्रेवलया का िषण 1901 में

सृजन क्रकया गया था, जब यहाँ के पूिणिती विरिश ईपवनिेशों (ितणमान में छह राज्य) ने एक

संघ के रूप में ऄपने अप को संगरठत करने का वनणणय वलया था। िे लोकतांवत्रक व्यिहार एिं

वसद्धांत (जैसे- 'एक व्यवक्त एक िोि' और ‘मवहलाओं को मतावधकार’), वजन्हें ितणमान संघ के

ऄवस्तत्ि में अने से पूिण यहाँ के औपवनिेवशक संसदों ने स्िीकार क्रकया था, ईन्हें ऑस्रेवलया की
पहली संघीय सरकार िारा भी ऄपनाया गया।
 ऑस्रेवलयाइ संविधान, सरकार की शवक्तयों को तीन पृथक भागों, यथा- विधावयका, कायणपावलका

और न्यायपावलका में विभावजत करता है। हालांक्रक आसमें स्पष्ट ईल्लेख है क्रक विधावयका के सदस्यों

को कायणकाररणी का भी सदस्य होना चावहए। व्यिहार में, संसद, कायणपावलका को विस्तृत

विवनयामकीय शवक्तयां प्रत्यायोवजत करती है।

 ऑस्रेवलया की संसद (सामान्यतः वनिाणवचत) का गठन दो सदनों से वमलकर हुअ है: हाईस ऑफ

ररप्रजेन्िेरिव्स और सीनेि। आन सदनों से वनयुक्त मंत्री कायणकारी सरकार का संचालन करते हैं और
नीवतगत वनणणय मंवत्रमंडल (कै वबनेि) की बैठकों में वलए जाते हैं। वनणणयों की घोषणा के ऄवतररक्त
कै वबनेि के विचार विमशण को सािणजवनक नहीं क्रकया जाता है। मंत्री, मंवत्रमंडल की एकजुिता के

वसद्धांत (Principle of Cabinet Solidarity) से बंधे होते हैं, जो संसद के प्रवत ईर्त्रदायी

कै वबनेि सरकार के विरिश मॉडल को प्रवतबबवबत करता है।

 यद्यवप, ऑस्रेवलया एक स्ितंत्र राष्ट्र है, तथावप ग्रेि-वििेन की महारानी एवल़ाबेथ-II ऑस्रेवलया
की भी औपचाररक साम्राज्ञी हैं। महारानी िारा ऄपने प्रवतवनवधत्ि के वलए (वनिाणवचत ऑस्रेवलयाइ
सरकार की सलाह पर) एक गिनणर-जनरल वनयुवक्त की जाती है। गिनणर-जनरल को व्यापक
ऄवधकार प्राप्त होते हैं, लेक्रकन व्यिहारतः िह लगभग सभी मामलों पर के िल मंवत्रयों की सलाह
पर कायण करता है।

48 www.visionias.in ©Vision IAS

Google it:- https://upscpdf.com


https://upscpdf.com << Download From >> https://upscpdf.com

9.1.2. सं विधान की प्रकृ वत

 संयुक्त राज्य ऄमेररका के समान ही ऑस्रेवलया का भी ऄपना एक वलवखत संविधान है।


ऑस्रेवलयाइ संविधान, संघीय सरकार के दावयत्िों को पररभावषत करता है, वजसमें विदेशी संबंध,
व्यापार, रक्षा और अव्रजन सवम्मवलत हैं।
 संघीय सरकार को सौंपे गए विषयों से वभन्न ऄन्य सभी विषयों के वलए राज्य और
कें र शावसत प्रदेशों की सरकारें ईर्त्रदायी होती हैं और िे भी ईर्त्रदायी सरकार के वसद्धांतों का
ऄनुपालन करती हैं। राज्यों में, गिनणर महारानी का प्रवतवनवधत्ि करते हैं।
 ऑस्रेवलया का ईच्च न्यायालय संघ और राज्यों के मध्य वििादों की मध्यस्थता करता है। न्यायालय
के ऄनेक वनणणयों िारा संघीय सरकार की संिैधावनक शवक्तयों और ईर्त्रदावयत्िों का विस्तार
क्रकया गया है।

9.1.2.1 सं शोधन की प्रक्रक्रया


 ऑस्रेवलयाइ संविधान में संशोधन के िल राष्ट्रीय स्तर पर जनमत संग्रह (referendum) के माध्यम
से वनिाणचकों के ऄनुमोदन िारा क्रकया जा सकता है, वजसमें वनिाणचक सूची में सवम्मवलत सभी
ियस्क ऄवनिायणतः भाग लेते हैं। एक संशोधन विधेयक को सिणप्रथम संसद के दोनों सदनों से
पाररत कराना अिश्यक होता है, या क्रफर कु छ सीवमत पररवस्थवतयों में संसद के के िल एक सदन
िारा आसे पाररत क्रकया जा सकता है।
 कोइ भी संिैधावनक संशोधन दोहरे बहुमत िारा ऄनुमोक्रदत क्रकया जाना चावहए - ऄथाणत् राष्ट्रीय
वनिाणचकों के बहुमत के साथ-साथ राज्यों के बहुमत (छह में से कम से कम चार) िारा। जहाँ कोइ
एक या ऄवधक राज्य विशेष रूप से जनमत संग्रह के विषय से प्रभावित हो, ईस वस्थवत में आस
विषय पर ईन राज्यों के मतदाताओं के बहुमत की स्िीकृ वत ऄवनिायण है।
आसे प्रायः ’ररपल मेजोररिी’ (वतहरा बहुमत) का वनयम कहा जाता है।
 दोहरे बहुमत का प्रािधान संविधान में पररितणन को कठोर स्िरूप प्रदान करता है। िषण 1901 में
संघ के गठन से ितणमान समय तक संविधान संशोधन हेतु प्रस्तावित 44 में से के िल 8 प्रस्तािों को
ही ऄनुमोक्रदत क्रकया गया है। सामान्यतः मतदाता संघीय सरकार की शवक्त में िृवद्ध करने िाले
प्रस्तािों का समथणन करने के वलए ऄवनच्छु क रहे हैं। राज्य और कें र शावसत प्रदेश भी जनमत संग्रह
का अयोजन कर सकते हैं।

9.1.3. सं स द

 हाईस ऑफ़ ररप्रजेन्िेरिव्स में बहुमत प्राप्त पािी िारा सरकार का गठन क्रकया जाता है।
 ऄल्पसंख्यक दल, प्रायः सीनेि में शवक्त संतुलन बनाए रखते हैं, जो सरकार के वनणणयों की समीक्षा
करने िाले सदन के रूप में कायण करता है। सीनेिरों को छह िषण के कायणकाल के वलए वनिाणवचत
क्रकया जाता है तथा अम चुनाि में के िल अधे सीनेिरों का चुनाि ही मतदाताओं िारा जाता है।
 ऑस्रेवलयाइ संसद में प्रश्न वबना क्रकसी सूचना के पूछे जा सकते हैं और िहाँ प्रश्नकाल के दौरान
मंवत्रयों से पूछे गए सरकारी और विपक्ष के प्रश्नों के मध्य स्पष्ट भेद होता है। विपक्ष ऄपने प्रश्नों का
ईपयोग सरकार को लवक्षत करने के वलए करता है। हालांक्रक, सरकारी सदस्य ऄपने प्रश्नों का प्रयोग
मंवत्रयों को सरकार की नीवतयों और गवतविवधयों को ईवचत स्िरूप प्रदान करने या विपक्ष को
लवक्षत करने के वलए करते हैं।
 संसद में क्रदए गए क्रकसी भी िक्तव्य की ररपोर्टिग मानहावन संबंधी मुकदमे के भय से रवहत होकर
वनष्पक्ष और सिीक रूप से की जा सकती है। संसदीय प्रश्न काल और बहस का प्रसारण क्रकया जा
सकता है। यह कायणपावलका शवक्त पर एक ऄनौपचाररक वनयंत्रण के रूप में कायण करता है।

49 www.visionias.in ©Vision IAS

Google it:- https://upscpdf.com


https://upscpdf.com << Download From >> https://upscpdf.com

9.1.4. वनिाण च न की प्रकृ वत

 राष्ट्रीय अम चुनािों का अयोजन नयी संघीय संसद की प्रथम बैठक के तीन िषण के भीतर क्रकया
जाना चावहए। संसद की औसत ऄिवध लगभग ढाइ िषण की होती है। व्यिहार में, अम चुनािों का
अयोजन तब क्रकया जाता है, जब गिनणर-जनरल प्रधानमंत्री के ऄनुरोध से सहमत होता है।
प्रधानमंत्री चुनाि की वतवथ का वनधाणरण करता है।
 िषण 1901 में संघ के गठन के बाद से सर्त्ाधारी दल को लगभग प्रत्येक पांच िषण पर पररिर्वतत कर
क्रदया जाता है। वलबरल पािी ने िषण 1949 से िषण 1972 तक 23 िषण के सबसे लंबे समय तक
गठबंधन का नेतृत्ि क्रकया। वितीय विश्व युद्ध से पूिण, कइ सरकारें एक िषण से भी कम समय तक
सर्त्ा में रही, लेक्रकन िषण 1945 के बाद से सरकार में के िल सात बार पररितणन हुअ है।

9.1.5. मतदान

 18 िषण से ऄवधक अयु के सभी नागररकों के वलए संघीय और राज्य सरकारों दोनों के चुनािों में
मतदान करना ऄवनिायण है और ऐसा करने में ऄसफल होने का पररणाम अर्वथक दंड या
ऄवभयोजन हो सकता है।

9.1.6. सरकार के स्तरों के मध्य सं बं ध

 राज्य की विधावयका (State parliaments), राष्ट्रीय संविधान के साथ-साथ संबंवधत राज्य के


संविधानों के ऄधीन होती हैं। संघीय कानून और राज्य के क्रकसी कानून के मध्य गवतरोध होने की
वस्थवत में संघीय कानून प्रभािी होता है।
 व्यिहार में, सरकार के दोनों स्तर ऐसे ऄनेक क्षेत्रों में सहयोग कर रहे हैं वजनके वलए राज्य और
संघ शावसत प्रदेश औपचाररक रूप से ईर्त्रदायी होते हैं, जैसे- वशक्षा, पररिहन, स्िास्थ्य, कानून
प्रितणन अक्रद। अयकर, संघ िारा अरोवपत क्रकया जाता है, हालांक्रक राजस्ि के ईपयोग एिं व्यय
कायों के दोहराि (डु प्लीके शन) के बारे में सरकारों के स्तरों के मध्य बहस ऑस्रेवलयाइ राजनीवत
की एक विवशष्ट विशेषता है। स्थानीय सरकारी वनकायों का गठन राज्य और संघ शावसत प्रदेश के
कानून िारा क्रकया जाता है।
 काईं वसल ऑफ़ ऑस्रेवलयन गिनणमेंट्जस (COAG) िस्तुतः सरकार के तीनों स्तरों, यथा- राष्ट्रीय,
राज्य या संघ शावसत प्रदेश और स्थानीय स्तरों के मध्य सहकारी कायणिाही की अिश्यकता िाले
राष्ट्रीय नीवतगत सुधारों की पहल करने , ईन्हें तैयार करने और लागू करने के वलए एक मंच
है। आसके लक्ष्यों में ऄग्रवलवखत प्रमुख मुद्दों का समाधान करना शावमल हैं: सरकार के स्तर पर
संरचनात्मक सुधार हेतु सहयोग तथा एक एकीकृ त, प्रभािी राष्ट्रीय ऄथणव्यिस्था और एकल राष्ट्रीय
बाजार के लक्ष्य को प्राप्त करने के वलए अिश्यक सुधार की क्रदशा में सहयोग करना।
 COAG के ऄंतगणत प्रधानमंत्री, राज्य प्रमुख (State Premiers), प्रदेशों के मुख्यमंत्री और
ऑस्रेवलयन लोकल गिनणमेंि एसोवसएशन (Australian Local Government Association)
के ऄध्यक्ष शावमल होते हैं।
 आसके ऄवतररक्त, मंवत्रस्तरीय पररषदें (राष्ट्रीय, राज्य और प्रदेश के मंत्री तथा जहां प्रासंवगक हो,
स्थानीय सरकार एिं न्यूजीलैंड और पापुअ न्यू वगनी की सरकारों के प्रवतवनवध) विकास और
विवशष्ट नीवतगत क्षेत्रों में ऄंतर-सरकारी कायणिाही को ऄपनाने के वलए वनयवमत रूप से बैठकें
करती हैं।

50 www.visionias.in ©Vision IAS

Google it:- https://upscpdf.com


https://upscpdf.com << Download From >> https://upscpdf.com

10. वस्िट्ज़रलैंड का संविधान


 गणतंत्रात्मक व्यिस्था और संघीय स्िरूप वस्िस संविधान की प्रमुख विशेषताएँ हैं।
 वस्िट्ज़रलैंड प्रत्यक्ष लोकतंत्र (direct democracy) प्रणाली के वलए भी प्रवसद्ध है।
 आसे गवतशील संविधान के रूप में जाना जाता है (जन संरक्षण, कल्याणकारी राज्य संबंधी
विशेषताओं सवहत)।

10.1. भारतीय सं विधान से तु ल ना

भारतीय संविधान वस्िस संविधान

कायणकारी शवक्तयां राष्ट्रपवत में वनवहत। कायणकारी शवक्तयां संघीय पररषद में वनवहत।

राष्ट्रपवत, वनिाणचक मंडल िारा वनिाणवचत। संघीय पररषद, संघीय सभा िारा वनिाणवचत।

दलीय सरकार। दलीय सरकार नहीं।

राज्यों को ऄंतराणष्ट्रीय संवध करने की ऄनुमवत कैं िन (Cantons) को ऄंतराणष्ट्रीय संवध करने की
प्राप्त नहीं। ऄनुमवत प्राप्त है।

न्यायपावलका की सिोच्चता। न्यायपावलका क्रकसी संघीय कानून को ऄमान्य


घोवषत नहीं कर सकती।

जनमत संग्रह का प्रािधान नहीं। जनमत संग्रह संभि।

10.2 प्रत्यक्ष लोकतं त्र की व्यिस्था

 जनमत संग्रह (Referendum): आसका अशय क्रकसी विधेयक को जनता के समक्ष


ईसके ऄनुसमथणन के वलए प्रस्तुत करने से है। यह प्लेबीसाआि (plebiscite) के समान नहीं है।
प्लेबीसाआि से अशय क्रकसी भी मुद्दे पर लोगों की राय लेने से है।
 आवनवशएरिि (Initiative): यह जनता िारा प्रारंभ एक विधेयक होता है और जनता ही आसे
विधानसभा में प्रेवषत करती है।
 प्रत्याितणन या प्रत्याशी को िापस बुलाना (Recall): आसका अशय क्रकसी भी समय प्रवतवनवध
को िापस बुलाने से है, यक्रद मतदाता ईसके कायण से संतुष्ट नहीं है।

11. भारतीय सं विधान की प्रमुख विशे ष ताएँ एिं तुल नात्मक


ऄध्ययन
11.1. ईद्दे वशका

 ईद्देवशका िस्तुतः एक संवक्षप्त पररचयात्मक कथन होता है जो पथप्रदशणक लक्ष्यों एिं ईद्देश्यों और
संिैधावनक दस्तािे़ के वसद्धांतों का िणणन करता है।
 भारतीय संविधान की ईद्देवशका की भाषा और संरचनात्मक प्रारूप को संयुक्त राज्य ऄमेररका से
ग्रहण क्रकया गया है।

51 www.visionias.in ©Vision IAS

Google it:- https://upscpdf.com


https://upscpdf.com << Download From >> https://upscpdf.com

11.2. वलवखत सं विधान

 वलवखत संविधान की ऄिधारणा संयुक्त राज्य ऄमेररका से ग्रहण की गयी है , जो विश्व का प्रथम
वलवखत संविधान है। यह सरल समझ और अिश्यकतानुसार संशोधन की ऄनुमवत प्रदान करता है।
साथ ही यह सरकार के मनमाने कानूनों से ईन्मुवक्त भी प्रदान करता है।

11.3. नाममात्र का राज्य प्रमु ख

वििेन भारत

महारानी, राज्य प्रमुख है। संिैधावनक राजतंत्र के भारत का राष्ट्रपवत:


 राज्य प्रमुख और भारत का प्रथम
कारण, िह देश पर शासन नहीं करती है, लेक्रकन
नागररक है।
सरकार के संबंध में महत्िपूणण औपचाररक और
 िह भारतीय लोकतंत्र की सभी तीन
ऄनौपचाररक भूवमकाओं का वनिणहन करती है।
शाखाओं, यथा- विधावयका,
कायणपावलका और न्यायपावलका का
"औपचाररक प्रमुख" भी होता है।
 िह भारतीय सशस्त्र बलों का सिोच्च
कमांडर भी होता है।

11.4. मं वत्रमं ड लीय प्रणाली (Cabinet System)

भारत और आंग्लैंड दोनों में प्रचवलत


 मंवत्रमंडल, प्रधानमंत्री और ईसके मंवत्रपररषद् के कु छ सदस्यों से वमलकर गरठत सरकार का
सामूवहक वनणणय वनमाणणकारी वनकाय है।
 मंवत्रपररषद् के सदस्य प्रधानमंत्री की सलाह पर राज्य प्रमुख (भारत में राष्ट्रपवत और आंग्लैंड में
सम्राि) िारा वनयुक्त क्रकए जाते हैं।
 मंवत्रमंडल के मंत्री को ईसके विभाग का अबंिन प्रधानमंत्री करता है और प्रधानमंत्री की सलाह पर
राज्य प्रमुख िारा ईसे क्रकसी भी समय हिाया जा सकता है।

11.5. विसदनात्मक सं स दीय व्यिस्था

विसदनात्मक संसद या विसदनीय विधावयका का वनमाणण दो सदनों से वमलकर होता है।


 आं ग्लैंड: हाईस ऑफ़ कॉमंस और हाईस ऑफ लॉर्डसण।
 भारत: लोकसभा (House of People) एिं राज्यसभा (Council of States)।

11.6. वनचले सदन के ऄवधक शवक्तशाली होने की ऄिधारणा

आं ग्लैंड भारत

हाईस ऑफ़ लॉर्डसण (ईच्च सदन) को विर्त्ीय विधेयकों के धन विधेयक को के िल लोकसभा में ही


मामले में सीवमत ऄवधकार प्राप्त हैं। पुरःस्थावपत क्रकया जा सकता है।

52 www.visionias.in ©Vision IAS

Google it:- https://upscpdf.com


https://upscpdf.com << Download From >> https://upscpdf.com

हाईस ऑफ कॉमंस में बहुमत खो देने पर प्रधानमंत्री वनम्न सदन में बहुमत खो देने पर
ऄपदस्थ हो जाता है। प्रधानमंत्री ऄपदस्थ हो जाता है।
ऄविश्वास प्रस्ताि के िल लोकसभा में
प्रस्तुत क्रकया जा सकता है।

ईच्च सदन के िल ऄवधकतम दो संसदीय कायणकाल के वलए धन विधेयक में राज्यसभा संशोधन

वनम्न सदन िारा पाररत विधेयकों के मामले में देरी कर नहीं कर सकती। यह के िल ऄवधकतम

सकता है, लेक्रकन आसे ऄस्िीकार नहीं कर सकता। 14 क्रदनों का विलम्ब कर सकती है।

11.7. वनम्न सदन का ऄध्यक्ष (Speaker in the Lower House)

आं ग्लैंड भारत

हाईस ऑफ कॉमंस का स्पीकर कॉमंस चैम्बर में बहस लोकसभा का ऄध्यक्ष सदन के कायों का
की ऄध्यक्षता करता है और िह एक सांसद होता है, संचालन करता है।
वजसे संसद के ऄन्य सदस्यों िारा मनोनीत क्रकया जाता
है।

स्पीकर, हाईस ऑफ कॉमंस का मुख्य ऄवधकारी होता है िह वनणणय करता है क्रक कोइ विधेयक धन
और सिोच्च प्रावधकार धारण करता है। ईससे सदैि विधेयक या नहीं।

राजनीवतक रूप से वनष्पक्ष रहने की ऄपेक्षा की जाती


है। िह बहस के दौरान व्यिस्था बनाए रखता है और
सांसदों को क्रम से बोलने का ऄिसर प्रदान करता है।

स्पीकर सम्राि, लॉर्डसण और ऄन्य ऄवधकाररयों के वलए िह सदन में ऄनुशासन और मयाणदा बनाए
हाईस ऑफ कॉमंस का प्रवतवनवधत्ि भी करता है और रखता है तथा सदस्यों को ईनके
हाईस ऑफ कॉमंस कमीशन की ऄध्यक्षता करता है। ऄवनयंवत्रत व्यिहार के वलए वनलंवबत
करके दंवडत कर सकता है।

िह ऄविश्वास प्रस्ताि, स्थगन प्रस्ताि,


बनदा प्रस्ताि और ध्यानाकषणण प्रस्ताि
जैसे विवभन्न प्रकार के प्रस्तािों एिं
संकल्पों को वनयमानुसार ऄनुमवत प्रदान
करता है।

11.8. न्यायपावलका

11.8.1. ईच्चतम न्यायालय की सं क ल्पना

 यह संयुक्त राज्य ऄमेररका से ग्रहण की गइ है। सिणप्रथम ऄमेररका में ऄपील की एक सिोच्च
ऄदालत की व्यिस्था की गयी, वजसे सुप्रीम कोिण के रूप में िर्वणत क्रकया जाता है। यह संघीय
व्यिस्था में सुव्यिवस्थत प्रशासन हेतु एक ऄवनिायण अिश्यकता है।

53 www.visionias.in ©Vision IAS

Google it:- https://upscpdf.com


https://upscpdf.com << Download From >> https://upscpdf.com

11.8.2. ईच्चतम न्यायालय की कायण प्र णाली

 यह जापान के संविधान से ऄपनाया गया है।

जापानी संविधान भारतीय संविधान

मुख्य न्यायाधीश को राज्य प्रमुख (सम्राि) िारा मुख्य न्यायाधीश राज्य प्रमुख ऄथाणत् राष्ट्रपवत
वनयुक्त क्रकया जाता है। िारा वनयुक्त क्रकया जाता है।

सुप्रीम कोिण देश का सिोच्च न्यावयक प्रावधकारण ईच्चतम न्यायालय देश का सिोच्च न्यावयक
है। प्रावधकरण है।

सुप्रीम कोिण के न्यायाधीश मामलों की सुनिाइ के ईच्चतम न्यायालय के न्यायाधीश मामलों की


वलए लघु पीठों का गठन करते हैं। सुनिाइ के वलए लघु पीठों का गठन करते हैं।

सुप्रीम कोिण मुख्य रूप से एक ऄपीलीय ऄदालत ईच्चतम न्यायालय मुख्य रूप से एक ऄपीलीय
के रूप में कायण करता है, जहाँ ऄपील की सुनिाइ न्यायालय के रूप में कायण करता है, जहाँ ऄपील
की जाती है और वनचली ऄदालतों के वनणणयों से की सुनिाइ की जाती है और वनचली ऄदालतों के
ऄसंतुष्ट िाक्रदयों की यावचकाओं पर सुनिाइ की वनणणय से ऄसंतुष्ट िाक्रदयों की यावचकाओं पर
जाती है। सुनिाइ की जाती है।

11.8.3. न्यायपावलका की स्ितं त्र ता और न्यावयक पु न र्विलोकन

 न्यायपावलका की स्ितंत्रता का वसद्धांत िर्वणत करता है क्रक न्यायपावलका को विधावयका और


कायणपावलका के हस्तक्षेप से राजनैवतक रूप से परररवक्षत क्रकया जाना चावहए। ऄथाणत्, सरकार की
ऄन्य शाखाओं, या व्यवक्तगत या पक्षपाती वहतों िारा न्यायपावलका के मामले में हस्तक्षेप नहीं
क्रकया जाना चावहए।
 न्यावयक पुनर्विलोकन एक वसद्धांत है, वजसके तहत न्यायपावलका को विधावयका और कायणपावलका
के कायों की समीक्षा (और ऄसंिैधावनक होने पर ईन्हें वनरस्त करने की शवक्त) की ऄवधकाररता
प्रदान की गयी है।
o न्यावयक पुनर्विलोकन की शवक्त से सम्पन्न विवशष्ट न्यायालयों िारा राज्य/राष्ट्र के संविधान से
ऄसंगत कृ त्यों को वनरस्त/ऄसंिैधावनक घोवषत कर क्रदया जाता है।
 आन दोनों वसद्धांतों को ऄमेररका के संविधान से ग्रहण क्रकया गया है। ये सरकार की ऄन्य दो
शाखाओं पर वनयंत्रण रखने के वलए ऄत्यंत महत्िपूणण हैं।

11.8.4. ईच्चतम/ईच्च न्यायालय के न्यायाधीशों को पद से हिाने की विवध

 यह संयुक्त राज्य ऄमेररका के संविधान से ग्रहण की गइ है।

ऄमेररकी संविधान भारतीय संविधान

 यह पृथक-पृथक राज्यों में पृथक-  कदाचार या ऄसमथणता के अधार पर ि राष्ट्रपवत के अदेश


पृथक है, जहाँ कभी-कभी जांच िारा ईच्चतम न्यायालय के न्यायाधीश को ईसके पद से
सवमवत की वसफाररश पयाणप्त होती हिाया जा सकता है।

है।  राष्ट्रपवत ऐसा तभी कर सकता है, जब आस प्रकार हिाए


 िहीं कु छ ऄन्य राज्यों में एक जाने हेतु प्रस्तुत प्रस्ताि को संविधान में वनधाणररत
न्यायाधीश पर महावभयोग के प्रक्रक्रया के ऄनुसार प्रत्येक सदन िारा ऄपनी कु ल

54 www.visionias.in ©Vision IAS

Google it:- https://upscpdf.com


https://upscpdf.com << Download From >> https://upscpdf.com

वलए दोनों सदनों के दो-वतहाइ से सदस्यता के बहुमत िारा तथा ईपवस्थत और मतदान
ऄवधक सदस्यों के बहुमत की करने िाले सदस्यों के कम से कम दो वतहाइ बहुमत िारा
पाररत क्रकया गया हो।
अिश्यकता होती है।

11.9. मू ल ऄवधकार

 मूल ऄवधकारों का ईद्देश्य न के िल कायणकाररणी की शवक्तयों पर, बवल्क विधान-मंडल की

शवक्त पर भी सीमाओं के रूप में कायण करना है।

ऄन्य देशों में भारत में

मूल ऄवधकार की ऄिधारणा ऄमेररका से भारत के संविधान में मूल ऄवधकार विश्व में सबसे
ग्रहण की गइ है। विस्तृत वििरण को संस्थावपत करता है।

िाक् -स्िातंत्र्य, संगम एिं धार्वमक स्ितंत्रता आनमें शावमल हैं:

के ऄवधकार को सोवियत संघ से ऄपनाया  समता का ऄवधकार (ऄनुच्छेद 14-18)

गया है।  स्ितंत्रता का ऄवधकार (19)

 शोषण के विरुद्ध ऄवधकार (23-24)

 धमण की स्ितंत्रता का ऄवधकार (25-28)

 संस्कृ वत और वशक्षा संबंधी ऄवधकार (29-30)

 संिैधावनक ईपचारों का ऄवधकार (32-35)

स्ितंत्रता और विवध के समक्ष समानता के


ऄवधकार को फ्ांसीसी संविधान से ग्रहण
क्रकया गया है।

11.9.1. अपातकाल के दौरान मू ल ऄवधकारों का वनलं ब न

 अपातकाल के दौरान ऄवधकारों के वनलंबन की ऄिधारणा जमणनी के िाइमर संविधान से ग्रहण


की गयी है।
 यह शवक्त राज्य प्रमुख ऄथाणत् राष्ट्रपवत में वनवहत है।
o अपातकाल के दौरान ऄवधकांश मूल ऄवधकारों का वनलंबन क्रकया जा सकता है। हालांक्रक,

कु छ मूल ऄवधकार वनलंवबत नहीं क्रकए जा सकते हैं, यथा- जीिन का ऄवधकार।

11.10. मू ल कर्त्ण व्य

 भारतीय संविधान में िर्वणत मूल कर्त्णव्य जापान, यूगोस्लाविया, चीन और सोवियत संघ

(USSR) के संविधान से ग्रहण क्रकए गए हैं।

 िास्ति में, जापान मौवलक कर्त्णव्यों को कानूनी रूप से लागू करने िाला एकमात्र लोकतांवत्रक देश है।

55 www.visionias.in ©Vision IAS

Google it:- https://upscpdf.com


https://upscpdf.com << Download From >> https://upscpdf.com

 आन्हें भारतीय संविधान में प्रत्येक नागररक को यह स्मरण कराने हेतु सवम्मवलत क्रकया गया है
क्रक िे न के िल ऄपने ऄवधकारों के प्रवत जागरूक हों, बवल्क ईन्हें ऄपने कर्त्णव्यों के प्रवत भी
जागरूक होना चावहए।

11.11. सं घीय व्यिस्था

 यह कें र और राज्यों के मध्य शवक्तयों के वितरण को संदर्वभत करता है।


 यह भारत जैसे विविधतापूणण देश के विषय में स्थानीय मुद्दों से वनपिने हेतु ऄत्यंत महत्िपूणण है।
 यह विधायी और प्रशासवनक दोनों शवक्तयों के संदभण में क्रकया गया है।

11.11.1. सु द ृढ़ कें र के साथ सं घीय व्यिस्था

 कें र और राज्य दोनों की स्िायर्त्ता एिं अपसी समायोजन, सम्मान, समझ ि सहयोग के साथ
ऄपनी शवक्तयों का प्रयोग करने िाले सहकारी ि सहयोगात्मक वनकाय के रूप में ऄपेक्षा की
गयी है।
 वििादों के वनिारण के साथ-साथ समाधान भी अिश्यक है। आस प्रकार, भारतीय संघीय व्यिस्था
में एक सुदढ़ृ कें र की संकल्पना ऄपनाइ गयी है।

11.11.2. ऄमे ररकी सं घ िाद के साथ भारतीय सं घ िाद की तु ल ना

भारतीय संविधान
 भारतीय संघ, राज्यों के मध्य क्रकसी समझौते का पररणाम नहीं है।
 राज्यों और संघ दोनों के वलए एकल नागररकता की ऄिधारणा ऄपनाइ गयी है।
 संसद में प्रत्येक राज्य को ईसकी जनसंख्या के अधार पर प्रवतवनवधत्ि प्रदान क्रकया गया है।
 आसमें राज्यों के मध्य समानता का वसद्धांत प्रचवलत नहीं है।
 आसमें तीन सूवचयां विद्यमान हैं, यथा- संघ सूची (प्रथम सूची), राज्य सूची (वितीय सूची)
और समिती सूची (तृतीय सूची)।
 संसद के िल संघ सूची और समिती सूची के विषयों पर कानून बना सकती है। राज्य संप्रभु नहीं हैं।
संघ, राज्य सूची का कु छ विषयों पर विशेष पररवस्थवतयों में कानून बना सकता है।
 कोइ राज्य भारत के राज्यक्षेत्र से पृथक नहीं हो सकता है।
 ऄिवशष्ट शवक्तयां संसद, ऄथाणत् कें र में वनवहत हैं।
 संघ और राज्यों के वलए एकल संविधान ऄपनाया गया है।
 भारत में सामान्यतः दीिानी और अपरावधक कानूनों में एकरूपता है (कु छ मामलों में वनजी
कानूनों को छोड़कर)।
 भारतीय संघ, विनाशी राज्यों का एक ऄविनाशी संघ है। संसद क्रकसी राज्य के क्षेत्र को पररिर्वतत
कर सकती है ऄथाणत् राज्यों की प्रकृ वत विनाशी हैं। लेक्रकन संघ को समाप्त नहीं जा सकता है ऄथाणत्
संघ ऄविनाशी है।
 के न्र सरकार के पास एक नए राज्य के गठन; क्रकसी राज्य के क्षेत्र में िृवद्ध करने या ईसके क्षेत्र को
कम करने; क्रकसी राज्य की सीमाओं में पररितणन; क्रकसी राज्य के नाम में पररितणन करने; और
क्रकसी राज्य से ईसके राज्य क्षेत्र को ऄलग करके ऄथिा दो या ऄवधक राज्यों या विवभन्न राज्यों के
भागों को वमलाकर या क्रकसी राज्यक्षेत्र को क्रकसी ऄन्य राज्य के साथ वमलकर नए राज्य को गरठत
करने की शवक्त है (ऄनुच्छेद 3)।
 हमारे संविधान में संघ (Federal) शब्द का प्रयोग नहीं क्रकया गया है। संविधान वनमाणताओं ने
यूवनयन शब्द का प्रयोग क्रकया है।

56 www.visionias.in ©Vision IAS

Google it:- https://upscpdf.com


https://upscpdf.com << Download From >> https://upscpdf.com

 ईच्चतम न्यायालय को ऄपीलीय (दीिानी और अपरावधक) क्षेत्रावधकार सवहत बहुत व्यापक


शवक्तयाँ प्रदान की गयी हैं।
 जनमत संग्रह का प्रािधान नहीं क्रकया गया है। संविधान में संशोधन के वलए, जनता की सहमवत
की अिश्यकता नहीं होती है। सांसदों का बहुमत प्राप्त करना पयाणप्त है और कु छ मामलों में, राज्य
विधान-मंडलों के बहुमत की भी अिश्यकता होती है।
ऄमेररकी संविधान
 ऄमेररकन फे डरेशन राज्यों के मध्य एक समझौते का पररणाम है।
 यहाँ दोहरी नागररकता (संघीय नागररकता और राज्य की नागररकता) का प्रािधान है।
 प्रत्येक राज्य िारा सीनेि में समान संख्या में प्रवतवनवधयों को भेजा जाता है।
o आसमें राज्यों के मध्य ईनकी जनसंख्या, क्षेत्र अक्रद से वनरपेक्ष समानता का वसद्धांत प्रचवलत
है।
 यहां संघीय सरकार और घिक आकाआयों के मध्य विधायी शवक्तयों का स्पष्ट विभाजन क्रकया गया
है। संघ के साथ-साथ प्रत्येक आकाइ ऄपने क्षेत्र में संप्रभु है। संघ और आकाआयां ऄपने संबंवधत
विधायी क्षेत्रों में संप्रभु हैं। कोइ भी दूसरे के क्षेत्रावधकार का ऄवतक्रमण नहीं कर सकता है।
 ऄिवशष्ट शवक्तयां राज्यों में वनवहत हैं।
 यहाँ दो संविधान (संघ और राज्यों के ) प्रचवलत हैं।
 वभन्न-वभन्न राज्यों के वलए वभन्न-वभन्न वसविल और अपरावधक कानून विद्यमान हैं।
 "फे डरल" शब्द का संविधान में कइ बार प्रयोग क्रकया गया है।
 ऄमेररका के सुप्रीम कोिण को भारत के ईच्चतम न्यायालय के समान एक विस्तृत ऄपीलीय ऄवधकार
क्षेत्र प्रदान नहीं क्रकया गया है।

11.12. व्यापार और िावणज्य की स्ितं त्र ता

ऑस्रेवलया भारत

ऑस्रेवलयाइ संविधान की धारा भारतीय संविधान के ऄनुच्छेद 301 में व्यापार, िावणज्य और
92 के ऄंतगणत मुक्त व्यापार का समागम की स्ितंत्रता के प्रािधान को ऑस्रेवलया के संविधान की
ईपबंध क्रकया गया है। धारा 92 से लगभग शब्दशः वलया गया है।

न्यायालय ने माना क्रक आस संबंध में विधावयका को न्यायपावलका


से भी ऄवधक शवक्तयां प्रदान की जानी चावहए तथा राज्य िारा
व्यापार और िावणवज्यक गवतविवधयों पर लगाए जाने िाले
युवक्तयुक्त प्रवतबंध संविधान के प्रािधानों के ऄनुसार होने
चावहए।

आसके ऄंगीकरण का महत्ि


 देश भर में िस्तुओं की मुक्त अिाजाही और ईनका अदान-प्रदान देश की अर्वथक एकता के वलए
अिश्यक है। आसवलए, अर्वथक गवतविवधयों हेतु सभी संघीय राष्ट्रों में संिैधावनक प्रािधानों के
माध्यम से ऄंतराणज्यीय व्यापार-िावणज्य की बाधाओं को दूर करने तथा अर्वथक संसाधनों
के मामले में देश में एकरूपता के वनमाणण के वलए प्रयास क्रकए गए हैं।
 आस प्रकार, व्यापार और िावणज्य की स्ितंत्रता के प्रािधान को कु छ ऄपिादों के साथ ऑस्रेवलयाइ
संविधान से ग्रहण क्रकया गया है।

57 www.visionias.in ©Vision IAS

Google it:- https://upscpdf.com


https://upscpdf.com << Download From >> https://upscpdf.com

11.13. राज्य की नीवत के वनदे श क तत्ि (DPSP)

 ये ऐसे वसद्धांत हैं वजन्हें नागररकों के वलए ईपयुक्त सामावजक और अर्वथक वस्थवत
को सृवजत करने के वलए वनधाणररत क्रकया गया है।
 भारत में, DPSP को अयरलैंड के संविधान से शब्दशः ऄपनाया गया है।
 अयरलैंड में DPSP स्पेन से ऄपनाए गए थे।

11.14. राष्ट्रपवत िारा सं स द सदस्यों को मनोनीत करना

 भारत में उपरी सदन में 250 सदस्य होते हैं, वजसमें से 12 सदस्यों को ईनके संबंवधत क्षेत्रों में
ऄनुकरणीय कायण के वलए देश के नाममात्र प्रमुख ऄथाणत् राष्ट्रपवत िारा नामवनर्ददष्ट क्रकया जाता है।
 नाम वनदेशन का ईद्देश्य यह है क्रक प्रवतवष्ठत लोगों को वनिाणचन में भाग वलए वबना राज्य सभा में
स्थान प्राप्त हो सके ।
 आस प्रणाली को अयरलैंड के संविधान से ऄपनाया गया है।

11.15. सं विधान के प्रमु ख ईपबं धों के स्रोतों की सू ची

 विवभन्न देशों के संविधान से ऄपनायी गइ कु छ प्रमुख विशेषताएं वनम्नवलवखत हैं:

वििेन  नाम मात्र राज्य प्रमुख - राष्ट्रपवत (महारानी के समान)


 मंवत्रयों की कै वबनेि प्रणाली
 प्रधानमंत्री का पद
 सरकार का संसदीय स्िरूप
 विसदनीय संसद
 वनचला सदन ऄवधक शवक्तशाली
 मंवत्रपररषद् वनचले सदन के प्रवत ईर्त्रदायी
 लोकसभा ऄध्यक्ष

ऄमेरीका  वलवखत संविधान


 राज्य का कायणकारी प्रमुख राष्ट्रपवत के रूप में जाना जाएगा और िह सशस्त्र
बलों का सिोच्च कमांडर होगा
 ईपराष्ट्रपवत राज्यसभा का पदेन ऄध्यक्ष होता है
 मूल ऄवधकार
 ईच्चतम न्यायालय
 न्यायपावलका की स्ितंत्रता और न्यावयक पुनर्विलोकन
 ईद्देवशका
 ईच्चतम न्यायालय और ईच्च न्यायालय के न्यायाधीशों को हिाने की प्रक्रक्रया

सोवियत संघ  मूल कर्त्णव्य

ऑस्रेवलया  समिती सूची


 ईद्देवशका की भाषा
 व्यापार, िावणज्य और समागम के संबंध में प्रािधान

जापान  कानून, वजसके अधार पर ईच्चतम न्यायालय कायण करता है

जमणनी का  अपातकाल के दौरान मूल ऄवधकारों का वनलंबन


िाइमर संविधान

58 www.visionias.in ©Vision IAS

Google it:- https://upscpdf.com


https://upscpdf.com << Download From >> https://upscpdf.com

कनाडा  सुदढ़ृ कें र के साथ संघीय व्यिस्था


 कें र और राज्यों के बीच शवक्तयों का वितरण और वस्थवत
 कें र के पास ऄिवशष्ट शवक्तयां
अयरलैंड  राज्य की नीवत के वनदेशक तत्ि की ऄिधारणा (अयरलैंड में स्पेन से ग्रहण
की गइ)
 राष्ट्रपवत के वनिाणचन की विवध
 राष्ट्रपवत िारा राज्यसभा में सदस्यों का नाम वनदेशन।

12. विगत िषों में Vision IAS GS मेंस िे स्ि सीरीज में पूछे
गए प्रश्न (Previous Year Vision IAS GS Mains Test
Series Questions)
1. भारतीय संविधान ने न्यावयक सिोच्चता की ऄमेररकी प्रणाली और संसदीय सिोच्चता की
विरिश प्रणाली के मध्य बहुत ही ईल्लेखनीय ढंग से मध्यम मागण ऄपनाया है। व्याख्या कीवजए।
दृवष्टकोण:
ईर्त्र में, वििेन में संसदीय सिोच्चता, ऄमेररका में न्यावयक सिोच्चता तथा भारतीय संदभण में
संसदीय संप्रभुता और न्यावयक सिोच्चता के संतुवलत मेल-जोल या संश्लेषण पर विचार क्रकया
जाना चावहए।
ईर्त्र:
 विरिश संविधान के ऄंतगणत न्यायालय, क्रकसी भी अधार पर संसद के क्रकसी भी ऄवधवनयम
को वनष्प्रभािी नहीं कर सकती है।
 दूसरी ओर ऄमेररका में , ईच्चतम न्यायालय वनम्नवलवखत अधारों पर क्रकसी कानून को ऄमान्य
घोवषत कर सकता है:
o संविधान में वनवहत विधायी शवक्त के दायरे का ऄवतक्रमण;
o वबल ऑफ़ राआट्जस (ऄवधकार-ऄवधवनयम) में क्रदए गए वनषेधों के अधार पर; और
o कु छ ऄन्य सामान्य वसद्धांतों के अधार पर, यथा- विवध की सम्यक् प्रक्रक्रया अक्रद।
 आन चरम पररवस्थवतयों के विपरीत भारतीय संविधान, न्यायपावलका को क्रकसी विवध को
ऄसंिैधावनक घोवषत करने की शवक्त प्रदान करता है, यक्रद ईक्त विवध संविधान िारा प्रदर्त्
शवक्तयों के विभाजन के ऄनुसार विधावयका के क्षेत्रावधकार से बाहर है या संविधान िारा
प्रत्याभूत मूल ऄवधकारों के विरुद्ध है। साथ ही, जहाँ तक विधायी नीवत के वििेक का संबंध है,
यह न्यायपावलका को ‘न्यावयक पुनर्विलोकन’ के ऄवधकार से भी िंवचत करता है।
 सरल रूप में, भारत का ईच्चतम न्यायालय ऄपने न्यावयक पुनर्विलोकन के ऄवधकार से
संसदीय कानून को ऄसंिैधावनक घोवषत कर सकता है और संसद ऄपनी विधायी शवक्त के
माध्यम से संविधान के बड़े भाग का संशोधन कर सकती है।
 आसके ऄवतररक्त, भारत में न्यावयक पुनर्विलोकन के ऄवधकार की संभािना, ऄमेररका में
ईच्चतम न्यायलय की तुलना में, प्रत्यक्षतः संकीणण है क्योंक्रक भारतीय संविधान के ऄनुच्छेद 21
में स्थावपत विवध िारा स्थावपत प्रक्रक्रया की ऄिधारणा के प्रवतकू ल ऄमेररकी संविधान में
विवध की सम्यक प्रक्रक्रया का प्रािधान है।
 क्रफर भी, यह ध्यान में रखा जाना चावहए क्रक भारत में न्यायपावलका ने मूल ढांचे के वसद्धांत
जैसे निाचार के माध्यम से न्यावयक पुनर्विलोकन के संबंध में संिध
ै ावनक प्रािधानों के दायरे
को विस्तृत क्रकया है।

59 www.visionias.in ©Vision IAS

Google it:- https://upscpdf.com


https://upscpdf.com << Download From >> https://upscpdf.com

2. विश्व भर में संसदों के उपरी सदन को अम तौर से ईनके वनचले सदन की तुलना में कम
शवक्तशाली माना जाता है। हालांक्रक, िे भी वनवश्चत कायों और शवक्तयों से वनवहत हैं, जो ईन्हें
एक वनणाणयक भूवमका के वनिणहन में सक्षम बनाता है। भारत के संदभण में विशेष बल देते हुए
अलोचनात्मक विश्लेषण कीवजए।
दृवष्टकोण:
 विधायी और महावभयोग प्रक्रक्रया अक्रद के विशेष संदभण में प्रदर्वशत कीवजए क्रक क्रकस
प्रकार उपरी सदन की शवक्तयाँ ि प्रवस्थवत वनचले सदन की तुलना में कम हैं।
 ऄनुच्छेद 249 और 312 के संदभण में भारत में राज्यसभा को विशेष शवक्तयाँ,
कायणकाररणी पर वनयंत्रण अक्रद।
 यह दशाणते हुए ईर्त्र समाप्त कीवजए क्रक ईच्च सदन एक महत्िपूणण संस्था है। क्रकन्तु आसे
वनचले सदन की ऄपेक्षा कम शवक्तयाँ प्रदान की गइ हैं।
ईर्त्रः
ईच्च सदन विसदनीय विधानमंडल के दो सदनों में से एक है। एकल प्रणाली के ऄंतगणत ईच्च
सदन को एक परामशणदात्री सदन माना जाता है जबक्रक संघीय प्रणावलयों में आसे वनचले सदन
के लगभग समान ही शवक्तयाँ प्रदान की गइ हैं। भारत की राज्य सभा के िल कु छ विर्त्ीय
मामलों को छोड़कर लगभग सभी पहलुओं में लोकसभा के समान शवक्तयाँ ि प्रवस्थवत धारण
करती है।
ईच्च सदन की गौण (ऄधीनस्थ) वस्थवत के कइ कारण हैं। िास्ति में, ईच्च सदन की अिश्यकता
के बारे में लगभग सभी देशों की संविधान सभा में काफी ऄवधक बहसें हुइ हैं। थॉमस जैफरसन
ने भी दो सदनों के विचार का विरोध क्रकया था। यह ऄप्रत्यक्ष रूप से वनिाणवचत वनकाय होता
है वजसे कइ विशेषज्ञ वनिाणवचत वनचले सदन के िारा व्यक्त जन भािना को समाप्त करने िाला
वनकाय मानते हैं। कु छ वििानों ने यह तकण क्रदया क्रक “यक्रद दूसरा सदन पहले का विरोध करता
है तो ईसे ईपरिी माना जाता है; यक्रद यह सहमत होता है तो आसे वनरथणक समझा जाता है। ”
वनम्नवलवखत कु छ वनवश्चत शवक्तयां और वस्थवतयां हैं वजनका संपण
ू ण विश्व में ईच्च सदन िारा
प्रयोग क्रकया जाता है:
 कु छ देशों में के िल कु छ सीवमत िैधावनक मामलों, जैसे- संविधान संशोधन हेतु आसकी
ऄनुमवत की अिश्यकता होती है। वििेन में, हाईस ऑफ लार्डसण यानी ईच्च सदन
ऄवधकांश ऄवधवनयमों को पाररत क्रकए जाने से रोक नहीं सकता। ईन देशों में जहाँ यह
क्रकसी कानून को िीिो कर सकता है (जैसे - नीदरलैंड), यह प्रस्तािों में संशोधन करने में
सक्षम नहीं हो सकता।
 वनचले सदन को जनता िारा प्रत्यक्ष रूप से वनिाणवचत क्रकया जाता है और आसवलए विर्त्
से संबंवधत मामलों में आसे विशेष शवक्तयाँ प्रदान की गयी हैं। राज्य सभा धन विधेयक को
के िल 14 क्रदनों (भारत के संदभण में) के वलए रोक सकती है।
 संसदीय प्रणाली में, ईच्च सदन सरकार के विरुद्ध ऄविश्वास प्रस्ताि पर मतदान नहीं कर
सकता। यह भारत के संदभण में भी सत्य है।
 आंग्लैंड में एक विकासिादी राजनैवतक प्रणाली है जहाँ शवक्त क्रमशः राजसर्त्ा से हाईस
ऑफ लार्डसण, ईच्च सदन से वनम्न सदन की ओर स्थानान्तररत हुइ है। ऄब, ईच्च सदन
सामान्यतः एक संशोधक सदन के रूप में कायण करता है।
हालांक्रक, कु छ संघीय प्रणाली में ईच्च सदन को कु छ विशेष शवक्तयां प्रदान की गयीं हैं। USA
का ईच्च सदन विश्व के सिाणवधक सशक्त ईच्च सदनों में से एक है। राज्यों ने ऄपनी शवक्तयाँ के न्र
को समर्वपत कर दी हैं और आसवलए ईच्च सदन कु छ ऐसी शवक्तयों का प्रयोग करता है जो वनम्न
सदन के पास नहीं हैं। भारत भी अरम्भ में सुदढ़ृ संघ का समथणन करता था। क्रकन्तु ऄब भी,
राज्य सभा को कु छ विशेष शवक्तयां प्राप्त है। आनमें से कु छ वनम्नवलवखत हैं:

60 www.visionias.in ©Vision IAS

Google it:- https://upscpdf.com


https://upscpdf.com << Download From >> https://upscpdf.com

 USA जैसे देशों के ईच्च सदन कु छ कायणकारी वनणणयों पर सलाह और सहमवत प्रदान कर
सकते हैं। (ईदाहरणाथण न्यायाधीशों की वनयुवक्त, ऄंतराणष्ट्रीय संवध या राजदूत)।
 कु छ राष्ट्रों में के िल ईच्च सदन को ही कायणपावलका के ऄवधकाररयों के विरुद्ध ऄवभयोग
चलाने का एकमात्र ऄवधकार प्राप्त है। USA में, आस मुद्दे पर के िल सीनेि ही ऄंवतम रूप
से वनणणय करती है और दोष वसद्ध करती है। भारत की राज्य सभा को भारत के ईप-
राष्ट्रपवत को पदच्युत करने का ऄवधकार प्राप्त है, वजससे लोकसभा सहमत हो।
 िषण 2009 से पूिण, वििेन का ईच्च सदन ऄंवतम न्यायालय के रूप में कायण करता था।
 ऄनुच्छेद 249 राज्य सभा को, राज्य सूची के विषय पर विवध वनमाणण के वलए संसद को
सक्षम बनाने हेतु प्रस्ताि पाररत करने की शवक्त प्रदान करता है। आसी प्रकार, राज्य सभा
ऄनुच्छेद 312 के ऄंतगणत एक नइ ऄवखल भारतीय सेिा (AIS) के सृजन के वलए प्रस्ताि
पाररत कर सकती है।
 लोक सभा के भंग रहने की वस्थवत में राज्य सभा ऄनुच्छेद 352, 356 और 360 के
ऄन्तगणत जारी क्रकसी ईद्घोषणा की ऄिवध बढ़ा सकती है।
वनष्कषणत:, ईच्च सदन की अिश्यकता सदैि िाद-वििाद का विषय रही है। कु छ वििान आसे
आसकी संरचना (सदस्यों के ऄप्रत्यक्ष चुनाि) के कारण ऄलोकतांवत्रक मानते हैं, जबक्रक ऄन्य
संविधान संशोधन और ऄन्य क्षमताओं के वलए आसका पक्ष-समथणन करते हैं। भारत में राज्य
सभा, धन विधेयक के मामलों के ऄवतररक्त लोक सभा के लगभग समान शवक्तयाँ धारण करती है।

3. संयक्त
ु राज्य ऄमेररका में विधावयका को ऄंतराणष्ट्रीय संवधयों के ऄनुसमथणन को विवनयवमत
करने का ऄवधकार प्राप्त है, जबक्रक भारत में यह मुख्यतः कायणपावलका का ऄवधकार क्षेत्र है।
ईदाहरणों सवहत आन दोनों दृवष्टकोणों के औवचत्य और लाभों का परीक्षण कीवजए।
दृवष्टकोण:
 ऄंतराणष्ट्रीय संवधयों के ऄनुसमथणन के संदभण में भारत सरकार और ऄमेररकी सरकार की
शवक्तयों का संवक्षप्त िणणन कीवजए तथा ऄन्य लोकतांवत्रक देशों, जहाँ संवधयों के
ऄनुसमथणन में विधान-मंडल की भी भूवमका होती है, से तुलना कीवजए।
 ईच्चतम न्यायालय के विवभन्न वनणणयों और संविधान की कायणप्रणाली की समीक्षा करने के
वलए गरठत राष्ट्रीय अयोग िारा की गइ वसफाररशों को दृवष्टगत रखते हुए, संबद्ध मुद्दों
का अलोचनात्मक विश्लेषण कीवजए।
ईर्त्र:
विदेशी शवक्तयों के साथ संवधयाँ और समझौते करना संप्रभु राज्य की विवभन्न विशेषताओं में
से एक है। कोइ भी राष्ट्र - विदेश संबंध, व्यापार, पयाणिरण, संचार, पाररवस्थवतकी या विर्त् -
आनमें से क्रकसी भी मामले में स्ियं को विश्व के ऄन्य वहस्सों से ऄलग नहीं रख सकता।
संयक्त
ु राज्य ऄमेररका (USA) में संवध से संबद्ध प्रािधान
 USA का संविधान यह व्यिस्था करता है क्रक राष्ट्रपवत को सीनेि की सहमवत और सलाह
से संवधयाँ करने का ऄवधकार है।
 ऄंतराणष्ट्रीय संवधयाँ करने की शवक्त को सीनेि से साझा करने के पीछे वनम्नवलवखत तकण
प्रस्तुत क्रकए गए हैं:
o राष्ट्रपवत को सीनेि की सलाह और सहमवत का लाभ पहुँचाना;
o राष्ट्रपवत की शवक्त पर वनयंत्रण रखना; और
o संवध करने की प्रक्रक्रया में प्रत्येक राज्य को एक िोि का वहस्सा प्रदान कर राज्यों की
संप्रभुता की रक्षा करना।

61 www.visionias.in ©Vision IAS

Google it:- https://upscpdf.com


https://upscpdf.com << Download From >> https://upscpdf.com

 ईल्लेखनीय है क्रक सीनेि, संवधयों का ऄनुसमथणन नहीं करती है ऄवपतु यह (सीनेि)


ऄनुसमथणन के संकल्प को स्िीकृ त या ऄस्िीकृ त करती है। ऐसे संकल्प के पाररत (ऄथाणत्
स्िीकृ त) होने के ईपरांत ही ऄंतराणष्ट्रीय संवधयों के ऄनुसमथणन को मूतण रूप क्रदया जाता है।
आसके पश्चात् ऄनुसमथणन संबंधी दस्तािेजों का औपचाररक रूप से संयुक्त राज्य ऄमेररका
और विदेशी शवक्त (यों) के मध्य अदान-प्रदान होता है।
 ईपयुणक्त (ऄथाणत् ऄमेररकी सीनेि की सलाह और सहमवत के वबना बाध्यकारी और
प्रभािी नहीं होने िाली संवधयों) के ऄवतररक्त ऄन्य प्रकार के ऄंतराणष्ट्रीय समझौते भी हैं
वजन्हें कायणकारी शाखा िारा संपन्न क्रकया जाता है और सीनेि के समक्ष प्रस्तुत नहीं क्रकया
जाता है। संयुक्त राज्य ऄमेररका में आन्हें, संवधयों के रूप में िगीकृ त नहीं क्रकया जाता है,
ऄवपतु कायणकारी समझौतों के रूप में िगीकृ त क्रकया जाता है और आनका के िल घरेलू
महत्ि होता है।
 ऄमेररका के संविधान में संवधयों को समाप्त करने संबंधी प्रािधान नहीं हैं।
भारत में संवध से संबद्ध प्रािधान
 भारतीय संविधान के तहत, संघ सूची में शावमल विषयों पर संसद को कानून बनाने

(ऄनुच्छेद 246) का विशेष ऄवधकार प्राप्त है।

 क्रकसी भी संवध को लागू करने के वलए, ऄनुच्छेद 253 के तहत संसद राज्य सूची के क्रकसी
भी विषय पर कानून बना सकती है।
 लेक्रकन संवधयों की सौदेबाजी और ईनके ऄनुसमथणन के वलए वनधाणररत की गइ एक
विवशष्ट प्रक्रक्रया के प्रािधानों की ऄनुपवस्थवत में, सामान्यतः आस शवक्त का कायाणन्ियन
कायणपावलका में वनवहत है।
 आसके विपरीत न्यायपावलका ने कइ घोषणाओं के माध्यम से आस पर बल क्रदया है क्रक
संवध करने की शवक्त ऄवनिायण रूप से राजनीवतक प्रकृ वत की है और आसवलए संवधयों को
मूतण रूप प्रदान करने के वलए एक पृथक विधान लागू क्रकया जाना चावहए।
 संविधान की कायणप्रणाली की समीक्षा करने के वलए गरठत राष्ट्रीय अयोग िारा भी
आसका समथणन क्रकया गया है और सरकार की संवध करने की शवक्त को विवनयवमत करने
के वलए संसद से कानून पाररत करने की वसफाररश की है, लेक्रकन आसने संसदीय
ऄनुसमथणन जैसी क्रकसी भी व्यिस्था को ऄस्िीकृ त क्रकया है।
 राष्ट्रीय संसद िारा ऄंतराणष्ट्रीय दावयत्िों के ऄनुसमथणन की कोइ भी प्रक्रक्रया; परस्पर
विरोधी विचारों एिं वहतों और लोकतंत्र की पररपक्वता से संबंवधत संस्थागत तंत्र के
विकास के स्तर जैसे विवभन्न ऄन्य कारकों को देखते हुए वनवश्चत रूप से लंबी हो जाएगी।
क्रफर भी, संसद िारा ऄनुसमथणन की अिश्यकता यह सुवनवश्चत करेगी क्रक दूरगामी
प्रभाि िाले ऄंतराणष्ट्रीय समझौते और संवधयां सूक्ष्म विधायी जांच और व्यापक
राजनीवतक एिं सािणजवनक चचाण के ऄधीन हों।
विधायी और कायणकारी शवक्तयों के संघीय ढांचे को देखते हुए संवध संपन्न करने की शवक्त का
प्रयोग स्िेच्छाचारी या ऄनावधकृ त रूप से नहीं क्रकया जा सकता है। ऄतः संसद को संवध करने
के विषय के संबंध में एक कानून बनाना चावहए और शावमल प्रक्रक्रयाओं को प्रभािी बनाने के
वलए संसदीय कानून के माध्यम से आसे लागू करना चावहए।
नागररकों के ऄवधकारों और दावयत्िों को प्रभावित करने िाले एिं प्रत्यक्षतः राज्य सूची के
विषयों से ऄसंगत संवधयों के मामले में संसद के प्रवतवनवधयों तथा राज्यों की ऄवधक से ऄवधक
भागीदारी होनी चावहए।

62 www.visionias.in ©Vision IAS

Google it:- https://upscpdf.com


https://upscpdf.com << Download From >> https://upscpdf.com

ऐतिहातसक अधार, तिकास, तिशेषिाएं, संशोधन, महत्िपूर्ण


प्रािधान और बुतनयादी संरचना
तिषय सूची
1. पररचय________________________________________________________________________________ 65

1.1. संतिधान क्या है? ______________________________________________________________________ 65

1.2. संतिधान के कायण ______________________________________________________________________ 65

1.3. संतिधानिाद (Constitutionalism) क्या है? ___________________________________________________ 65

1.3.1. भारि में संतिधानिाद (Constitutionalism In India) __________________________________________ 67

2. ऐतिहातसक अधार (Historical Underpinnings)__________________________________________________ 67

2.1. तिरिश शासन के ऄंिगणि संिध


ै ातनक घिनाक्रमों की भूतमका __________________________________________ 67

2.2. तिश्लेषर् ____________________________________________________________________________ 67

2.2.1. इस्ि आंतिया कं पनी के ऄधीन संिैधातनक तिकास (1773-1858) ____________________________________ 67

2.2.2. तिरिश िाज के ऄधीन संिैधातनक तिकास (1858-1947) ________________________________________ 70

2.2.3. ऄन्य महत्िपूर्ण घिनाक्रम _______________________________________________________________ 75

3. तिकास (Evolution) ______________________________________________________________________ 77

3.1. दो अयाम___________________________________________________________________________ 77

3.1.1. संतिधान को ऄंगीकृ ि, ऄतधतनयतमि और अत्मार्पपि ककए जाने से पूिण का घिनाक्रम _______________________ 77

3.1.2. एक सिि प्रकक्रया के रूप में क्रतमक तिकास ___________________________________________________ 80

4. भारिीय संतिधान की प्रमुख तिशेषिाएं (Salient Features of the Constitution of India) ____________________ 81

4.1. तितखि एिं सबसे तिस्िृि संतिधान __________________________________________________________ 81

4.2. नम्यिा (िचीिापन) एिं ऄनम्यिा (कठोरिा) का समन्िय ___________________________________________ 83

4.3. िोकिांतिक गर्राज्य ___________________________________________________________________ 83

4.4. सरकार का संसदीय स्िरूप________________________________________________________________ 84

4.5. संघीय और एकात्मक तिशेषिाओं का तमश्रर् (Mixture Of Federal And Unitary Features) ________________ 84

4.6. ऄसमतमि संघिाद (Asymmetric federalism) ________________________________________________ 86

4.7. मूि ऄतधकार (Fundamental Rights) ______________________________________________________ 86

4.8. राज्य की नीति के तनदेशक ित्ि _____________________________________________________________ 87

4.9. मूि कर्त्णव्य (Fundamental Duties) _______________________________________________________ 87

4.10. पंथतनरपेक्ष राज्य _____________________________________________________________________ 87

4.11. स्ििंि, तनष्पक्ष और एकीकृ ि न्यायपातिका ____________________________________________________ 87

4.12. एकि नागररकिा _____________________________________________________________________ 88


4.13. सािणभौतमक ियस्क मिातधकार (Universal Adult Franchise) ____________________________________ 88

63

Google it:- https://upscpdf.com


https://upscpdf.com << Download From >> https://upscpdf.com

4.14. अपािकािीन शतियां _________________________________________________________________ 88

4.15. शति का पृथक्करर् (Separation of Powers) ________________________________________________ 88

4.16. स्ििंि तनकाय _______________________________________________________________________ 89

4.17. सरकार के िीन स्िर ___________________________________________________________________ 90

5. संशोधन (Amendments) __________________________________________________________________ 90

5.1. भूतमका ____________________________________________________________________________ 90

5.2. संिध
ै ातनक प्रािधानों के संशोधन की अिश्यकिा _________________________________________________ 90

5.3. संशोधन के प्रकार ______________________________________________________________________ 90

5.3.1. ऄदृश्य या ऄनौपचाररक प्रकक्रया ___________________________________________________________ 91

5.3.2. दृश्य या औपचाररक प्रकक्रया ______________________________________________________________ 91

5.4. मूि ऄतधकारों का संशोधन ________________________________________________________________ 93

5.4.1. शंकरी प्रसाद बनाम भारि संघ िाद, 1951___________________________________________________ 93

5.4.2. सज्जन ससह बनाम राजस्थान राज्य िाद, 1965 ________________________________________________ 93


5.4.3. गोिखनाथ बनाम पंजाब राज्य िाद, 1967 __________________________________________________ 93

5.4.4. के शिानंद भारिी बनाम के रि राज्य िाद, 1973 _______________________________________________ 94

5.4.5. 42िां संतिधान संशोधन ऄतधतनयम, 1976 __________________________________________________ 95

5.4.6. तमनिाण तमल्स िाद, 1980 _______________________________________________________________ 95

5.4.7. िामन राि बनाम भारि संघ िाद, 1981 ____________________________________________________ 95

5.5. ईपयुि
ण के अधार पर तनष्कषण ______________________________________________________________ 95

5.6. संतिधान संशोधन की क्रमिार प्रकक्रया ________________________________________________________ 95


5.7. संतिधान संशोधन की प्रकक्रया की अिोचना _____________________________________________________ 96

6. मूि ढााँचा/बुतनयादी संरचना (Basic Structure) ___________________________________________________ 96

6.1. पररभाषा ___________________________________________________________________________ 96

6.2. मूि ढााँचे की ऄिधारर्ा का ईद्भि ___________________________________________________________ 97

6.2.1. पररचय ____________________________________________________________________________ 97

6.2.2. िाद-तििाद _________________________________________________________________________ 98

6.2.3. के शिानंद भारिी िाद, 1973 ____________________________________________________________ 98

6.3. ‘मूि ढााँच’े के तसद्ांि का अिोचनात्मक तिश्लेषर् ________________________________________________ 100

7. भारिीय संतिधान के स्रोि __________________________________________________________________ 101

8. तिगि िषों में Vision IAS GS मेंस िेस्ि सीरीज में पूछे गए प्रश्न (Previous Year Vision IAS
GS Mains Test Series Questions) _________________________________________________________ 103

9. तिगि िषों में संघ िोक सेिा अयोग द्वारा पूछे गए प्रश्न (Past Year UPSC Questions) ______________________ 106

64

Google it:- https://upscpdf.com


https://upscpdf.com << Download From >> https://upscpdf.com

1. पररचय
1.1. सं तिधान क्या है ?
 संतिधान तितशष्ट कानूनी िैधिा िािा एक तितधक दस्िािेज़ है। आसमें राज्य के मूिभूि संस्थानों
की स्थापना का ढांचा (framework) तनतहि होिा है। यह तितभन्न संस्थानों की कायण प्रर्ािी को
तनयंतिि करने िािे मूि तसद्ांिों को तनर्ददष्ट करिा है। आसके साथ ही, यह तितभन्न संस्थानों की
संरचना, संघिन, क्षेिातधकार एिं ईनके प्रमुख ऄतधदेश को भी तनधाणररि करिा है।
 िस्िुिः, यह तितभन्न संस्थानों के मध्य ऄंिसंबंधों को पररभातषि करिा है िथा जीिन के सभी
क्षेिों में नागररकों और राज्य के मध्य संबंधों का संचािन करिा है। संक्षेप में, यह ककसी राष्ट्र की
तनयम पुतस्िका होिी है जो ईस समाज और ईसके कानूनों को तितनयतमि करिी है।
 संतिधान, राज्य के शासकीय तनकायों द्वारा ऄनुसरर् की जाने िािी नीतियों को भी दशाणिा है।
चूंकक भारि एक िोकिांतिक राष्ट्र है, ऄि: आसका संतिधान नागररकों को ईनके मूि ऄतधकारों की
गारंिी प्रदान करिा है। राज्य की नीति के तनदेशक ित्ि मूिभूि सामातजक-राजनीतिक मूल्यों को
प्राप्त करने का मागण प्रशस्ि करिे हैं।
1.2. सं तिधान के कायण

संतिधान (तितखि या ऄतितखि) की महत्िपूर्ण राजनीतिक महर्त्ा होिी है। आसके ऄनेक प्रकायण होिे हैं,
तजनमें से कु छ तनम्नतितखि हैं:
 तिचारधारा की ऄतभव्यति: यह ककसी राष्ट्र के दशणन एिं तिचारधारा को दशाणिा है।
 मूिभूि कानून की ऄतभव्यति: संतिधान मूिभूि कानूनों को प्रदर्पशि करिा है। आन कानूनों को एक
प्रकक्रया के माध्यम से सामान्यिया संशोतधि या पररिर्पिि ककया जा सकिा है , तजसे संतिधान
संशोधन कहा जािा है। कु छ तिशेष कानून भी होिे है, जो नागररकों के ऄतधकारों पर कें किि होिे
हैं; ईदाहरर्ाथण- ऄतभव्यति, धमण, सम्मेिन, प्रेस अकद की स्ििंििा से संबंतधि ऄतधकार।
 संगठनात्मक ढांचा: यह सरकार के तिए एक संगठनात्मक ढांचा प्रदान करिा है। यह तिधातयका,
कायणपातिका और न्यायपातिका के कायों, ईनके ऄंिसंबंधों, ईनके प्रातधकारों पर ऄंकुश अकद को
पररभातषि करिा है।
 सरकार के स्िर: संतिधान, सामान्यि: सरकार के तितभन्न ऄंगों के स्िरों को प्रदर्पशि करिा है।
प्राय: यह संतिधान द्वारा तनरुतपि ककया जािा है कक िह संघीय है, पररसंघीय है या एकात्मक है।
यह राष्ट्रीय और प्रांिीय सरकारों की शतियों को भी तनरुतपि करिा है। भारि में िो यह स्थानीय
सरकार की शतियों को भी तनरुतपि करिा है।
o ईदाहरर्: सोतियि संतिधान में मुख्यिः तिचारधारा की ऄतभव्यति को प्रदर्पशि ककया गया
था। ईसमें संगठनात्मक ढांचे को ऄतभव्यि नहीं ककया गया था। आसके तिपरीि, ऄमेररकी
संतिधान में ित्कािीन सरकार के दशणन की ऄतभव्यति की िुिना में सरकार के संगठनात्मक
ढांचे एिं शासन पद्ति को तिस्िृि रूप में ऄतभव्यि ककया गया है।
1.3. सं तिधानिाद (Constitutionalism) क्या है ?
सिणप्रथम हमें संतिधान और संतिधानिाद के मध्य ऄंिर समझने की अिश्यकिा है।
 राज्य सर्त्ा में तनतहि बाध्यकारी शति का प्रयोग शासकों द्वारा मनमाने ढंग से ककया जा सकिा
है। संतिधान का तनमाणर् राज्य सर्त्ा के तिरुद् एक सुरक्षा िंि के रूप में ककया जािा है। िह
व्यिस्था, जो सरकारों या शासकों को (सामान्यि:) तितखि या ऄतितखि संतिधान के माध्यम से
क्षेिातधकार की सीमा के भीिर रहकर कायण करने हेिु बाध्य करिी है, ईसे संतिधानिाद कहा
जािा है।

65 www.visionias.in ©Vision IAS

Google it:- https://upscpdf.com


https://upscpdf.com << Download From >> https://upscpdf.com

 संतिधानिाद का िात्पयण है कक राजनीतिक शति का प्रयोग िस्िुिः सीमाओं, तनबंधनों, तनयंिर्


और तनयमों के भीिर ककया जाएगा। संतिधानिाद की ऄिधारर्ा में शति के तनरंकुश एिं
ऄतधनायकिादी प्रयोग के तिरुद् 'सीतमि सरकार' और 'तितध के शासन' के तसद्ांिों को सतम्मतिि
ककया गया है।

एक सीतमि सरकार (Limited government) िह होिी है तजसकी िैध सर्त्ा और शति,


प्रत्यायोतजि प्रातधकरर्ों के माध्यम से प्रतिबंतधि होिी है। सीतमि सरकार िािे देशों में आस संबंध
में तितधयों का तनमाणर् ककया गया होिा है कक सरकार क्या कर सकिी है और क्या नहीं कर सकिी
है। भारि में, संिध
ै ातनक रूप से सीतमि सरकार है, जो संतिधान द्वारा तनधाणररि तितशष्ट तसद्ांिों
और कायों के प्रति बाध्य है।
िायसी के तितध के शासन के तसद्ांि के ऄंिगणि तनम्नतितखि िीन प्रमुख तसद्ांि सतम्मतिि हैं:
 स्िेच्छाचारी शति की ऄनुपतस्थति या तितध की सिोच्चिा (Absence of Arbitrary Power

or Supremacy of Law): आसका िात्पयण तितध की पूर्ण सिोच्चिा से है और तसिाय देश के


न्यायाियों के समक्ष सामान्य तितध के द्वारा स्थातपि कानून के स्पष्ट ईल्िंघन के ककसी भी
व्यति को दंतिि नहीं ककया जा सकिा है। िायसी का तिचार था कक सभी व्यति चाहे िह
साधारर् व्यति हो या सरकारी प्रातधकरर्, तितध का पािन करने हेिु बाध्य है। ईनका मानना
था कक पहिे से स्थातपि कानून के ईल्िंघन के ऄतिररि ककसी भी व्यति को दंतिि नहीं ककया
जा सकिा है।
 तितध के समक्ष समानिा (Equality before law): आसका िात्पयण तितध की समानिा या सभी
िगों के िोगों पर देश की सामान्य तितध के िागू होने से है, तजसे सामान्य तितध न्यायाियों

द्वारा प्रशातसि ककया जािा है। आस ऄथण में , तितध का शासन यह तनर्ददष्ट करिा है कक सभी
व्यति तितध के समक्ष समान हैं। यहां िक कक आस तितध का ऄनुपािन करना सरकारी
ऄतधकाररयों का भी किणव्य होिा है और आनसे संबंतधि मामिों से तनपिने के तिए तितशष्ट
न्यायािय नहीं हो सकिे हैं।
 संतिधान, देश की सामान्य तितध का पररर्ाम है (Constitution is the result of the
ordinary law of the land): िायसी के ऄनुसार, कइ देशों में व्यतिगि स्ििंििा, स्ििंििा
अकद जैसे ऄतधकार देश के तितखि संतिधान द्वारा प्रदान ककए जािे हैं। िेककन आंग्िैंि में आन
ऄतधकारों की ईत्पतर्त् तितभन्न पक्षकारों के मध्य िकराि के संबंध में कदए गए न्यातयक तनर्णयों
के पररर्ामस्िरूप हुइ है। संतिधान, व्यतियों के ऄतधकारों का स्रोि नहीं बतल्क पररर्ाम है।
िेककन िायसी का यह तसद्ांि भारि में िागू नहीं होिा है क्योंकक भारि में संतिधान को तितध
का मूि अधार माना गया है, जो सभी तितधयों का स्रोि है।

के . सी. व्हेयर एिं िब्लल्यू. जी. एंड्रयूज के ऄनुसार संतिधानिाद का ऄथण है:
 शतियों का तिभाजन, न कक शतियों का के न्िीकरर्;

 समाज में बहुमि के तहिों की स्िीकृ ति;


 ऄतधनायकिादी या िानाशाही नेिृत्ि की ऄनुपतस्थति; और
 व्यतिगि स्ििंििा पर न्यूनिम प्रतिबंधों का अरोपर्।
कािण जे. फ्रेिररक के ऄनुसार, शतियों का तिभाजन संतिधानिाद का सबसे महत्िपूर्ण अधार है।
राजिंि या गर्िंि, कु िीनिंि या िोकिंि जैसी शासन प्रर्ातियों में भी संतिधानिाद की ईपतस्थति

हो सकिी है, यकद आन प्रर्ातियों में शतियों का तिभाजन तिद्यमान है।

66 www.visionias.in ©Vision IAS

Google it:- https://upscpdf.com


https://upscpdf.com << Download From >> https://upscpdf.com

1.3.1. भारि में सं तिधानिाद (Constitutionalism In India)

संतिधानिाद भारिीय संतिधान में भी तिद्यमान है। तनम्नतितखि तिशेषिाएाँ आसकी साक्ष्य हैं:
 तितखि संतिधान;
 संसदीय िोकिंि;
 तितध का शासन;
 मूि ऄतधकार;
 शतियों का पृथक्करर् िथा तनयंिर् एिं संिुिन;
 नम्य संतिधान िथा ऄपररििणनीय मूि ढांचा;
 सरकार का संघीय स्िरुप; एिं
 स्ििंि न्यायपातिका और न्यातयक पुनर्पििोकन।

2. ऐतिहातसक अधार (Historical Underpinnings)


संतिधान का ऄध्ययन करने के तिए यह अिश्यक है कक संतिधान के तनमाणर् के पूिण और तनमाणर् के
समय की घिनाओं का ऄध्ययन ककया जाए। हमारे संतिधान एिं राजव्यिस्था की ऄनेक तिशेषिाएाँ
तिरिश शासन प्रर्ािी से ग्रहर् की गइ हैं। ईल्िेखनीय है कक िषण 1946 में एक संतिधान सभा का गठन
ककया गया था और 26 जनिरी 1950 को संतिधान ऄतस्ित्ि में अया।

2.1. तिरिश शासन के ऄं ि गण ि सं िै धातनक घिनाक्रमों की भू तमका

आन घिनाक्रमों को तनम्नतितखि दो शीषणकों के िहि संक्षेप में प्रस्िुि ककया जा


सकिा है:
 इस्ि आंतिया कं पनी के ऄधीन संिैधातनक तिकास (1773-1858)
 तिरिश िाज के ऄधीन संिैधातनक तिकास (1858-1947)

2.2. तिश्ले ष र्

हम तनम्नतितखि दो दृतष्टकोर्ों से आन ऐतिहातसक घिनाक्रमों का ऄध्ययन कर सकिे हैं:


 सामान्य तिशेषिाओं के माध्यम से; िथा
 संतिधान की ििणमान प्रर्ािी के साथ ककसी न ककसी रूप में संबंतधि तिशेषिाओं के माध्यम से।
(* से तचतन्हि ककया गया है)

2.2.1. इस्ि आं तिया कं पनी के ऄधीन सं िै धातनक तिकास ( 1773-1858)

(i) 1773 का रेगि


ु टे िग एक्ि
आस ऄतधतनयम का संिैधातनक महत्ि है क्योंकक आसने भारि में कें िीय प्रशासन के तसद्ांिों को िागू
ककया।*

 आस ऄतधतनयम के द्वारा बंगाि के गिनणर को ‘बंगाि का गिनणर जनरि’ पद नाम कदया गया एिं
ईसकी सहायिा हेिु एक चार सदस्यीय कायणकारी पररषद का गठन ककया गया। ज्ञािव्य है कक
प्रथम गिनणर जनरि िारेन हेसस्िग्स थे।
 आसके िहि बंबइ और मिास के गिनणर , बंगाि के गिनणर जनरि के ऄधीन हो गए, जबकक आससे
पूिण सभी प्रेतसिेंतसयों के गिनणर पृथक-पृथक रूप से कायण करिे थे। आसके द्वारा शति के 'कें िीकरर्'
की प्रिृतर्त् प्रारम्भ हुइ, जो 1833 के चािणर ऄतधतनयम िक जारी रही।*

67 www.visionias.in ©Vision IAS

Google it:- https://upscpdf.com


https://upscpdf.com << Download From >> https://upscpdf.com

 आस ऄतधतनयम के ऄंिगणि िषण 1774 में किकर्त्ा में एक ईच्चिम न्यायािय की स्थापना की गइ,
तजसमें एक मुख्य न्यायाधीश और िीन ऄन्य न्यायाधीश थे।*
 आस ऄतधतनयम के माध्यम से तिरिश सरकार का कं पनी पर कठोर तनयंिर् स्थातपि ककया गया।
‘कोिण ऑफ़ िायरेक्िसण’ (कं पनी की गिर्ननग बॉिी) के माध्यम से भारि में आसके राजस्ि, नागररक
और सैन्य मामिों पर तिरिश सरकार को जानकारी देना अिश्यक कर कदया गया।
 आसका संिैधातनक महत्ि है क्योंकक आसने भारि में कें िीय प्रशासन के तसद्ांिों को स्थातपि ककया।*
(ii) 1784 का तपट्स आंतिया एक्ि
 आस ऄतधतनयम के द्वारा कं पनी के िातर्तज्यक और राजनीतिक कायों को पृथक ककया गया।
 आस ऄतधतनयम द्वारा राजनीतिक मामिों के प्रबंधन हेिु बोिण ऑफ़ कं ट्रोि (तनयंिर् बोिण) का गठन
ककया गया, जबकक कं पनी के िातर्तज्यक कायों के प्रबंधन हेिु कोिण ऑफ़ िायरेक्िसण (तनदेशक
मंिि) का गठन ककया गया।
 आंग्िैंि में तनयंिर् बोिण को इस्ि आंतिया कं पनी के कायों की तनगरानी का अदेश कदया गया था।
आसमें तनम्नतितखि छह सदस्य सतम्मतिि थे:

o राज्य सतचि (तिरिश कै तबनेि का एक सदस्य);

o चांसिर ऑफ़ एक्सचेकर; और

o राज्य सतचि द्वारा तनयुि तप्रिी काईं तसि (Privy Council) के चार सदस्य।
 आस ऄतधतनयम के िहि कायणकारी पररषद के सदस्यों की संख्या कम करके िीन कर दी गयी तजनमें
से एक कमांिर-आन-चीफ था। आसके द्वारा बंगाि के समान मिास और बंबइ की पररषदों को भी
संशोतधि ककया गया।
 आस ऄतधतनयम ने तनयंिर् बोिण को तिरिश तनयंतिि भारि की सभी ऄसैतनक (नागररक), सैन्य
एिं राजस्ि गतितितधयों का ऄधीक्षर् एिं तनयंिर् करने के तिए सशि बनाया।
 आस प्रकार, आसने दोहरी तनयंिर् प्रर्ािी (dual system of control) ऄथिा संयि
ु सरकार

(joint government) का मागण प्रशस्ि ककया।

 पहिी बार कं पनी के ऄधीन क्षेिों को 'भारि में तिरिश अतधपत्य का क्षेि' (British possession

in India) कहा गया।


ऄतधतनयम का तिश्लेषर्
जैसा कक उपर स्पष्ट ककया गया है, आस ऄतधतनयम के माध्यम से दोहरी तनयंिर् प्रर्ािी की स्थापना

(तनयंिर् बोिण के गठन के माध्यम से) की गइ, तजसके ऄंिगणि कं पनी और एक संसदीय बोिण द्वारा शासन

ककया जाना तनधाणररि ककया गया िथा यह िषण 1858 िक िागू रहा।
 तनयंिर् बोिण को कोइ स्ििंि कायणकारी शति प्राप्त नहीं थी।
 आसे ककसी प्रकार का संरक्षर् प्राप्त नहीं था िथा आसकी शतियां ऄस्पष्ट थी।
 बोिण को कं पनी के सभी दस्िािेज़ों िक पहुाँच प्राप्त थी एिं ईन सभी प्रेषर्ों (dispatches) के तिए

जो पूर्णिया व्यािसातयक नहीं थे , ईनके तिए बोिण का ऄनुमोदन अिश्यक हो गया था। आसके

ऄतिररि, अपािकाि की तस्थति में बोिण ऄपने मसौदे को हस्िाक्षर हेिु तनदेशकों की गुप्त सतमति

को प्रेतषि कर सकिा था।

आस प्रकार, आस ऄतधतनयम के ऄंिगणि कं पनी की नागररक और सैन्य सरकार को आंग्िैंि की सरकार के


ऄधीन ककया गया।

68 www.visionias.in ©Vision IAS

Google it:- https://upscpdf.com


https://upscpdf.com << Download From >> https://upscpdf.com

(iii) 1813 का चािणर ऄतधतनयम


 आस ऄतधतनयम के द्वारा भारि के प्रशासन में कोइ महत्िपूर्ण पररििणन नहीं अया।
 नेपोतियन द्वारा िागू की गइ महाद्वीपीय व्यिस्था (Continental System) के पररर्ामस्िरूप
तििेन का यूरोपीय देशों के साथ व्यापार बातधि हो गया था। पररर्ामस्िरूप तिरिश कं पतनयां एिं
व्यापारी यूरोप से दूर ऄन्य देशों में व्यापार करने हेिु तििश हुए। ऄिः िे भारिीय व्यापार पर
इस्ि आंतिया कं पनी के एकातधकार को समाप्त करना चाहिे थे। फििः आस चािणर एक्ि द्वारा भारि
के साथ कं पनी के व्यापाररक एकातधकार को समाप्त कर कदया गया।
 यद्यतप, यह ऄतधतनयम इस्ि आंतिया कं पनी के भारि के साथ व्यापार के एकातधकार को समाप्त
करने के कारर् महत्िपूर्ण था, िथातप कं पनी का चीन के साथ व्यापार और चाय के व्यापार पर
एकातधकार बना रहा।
 आसमें यह ईपबंध ककया गया था कक भारिीयों की तशक्षा के तिए कं पनी द्वारा प्रतििषण 1 िाख
रुपये का व्यय ककया जाएगा। हािांकक, आस प्रािधान को िागू नहीं ककया गया।
 आसके द्वारा स्थानीय तनकायों को कर िगाने और ईनके भुगिान न करने पर दंि देने का ऄतधकार
कदया गया। हािांकक, यह कायण सिोच्च न्यायािय के क्षेिातधकार के ऄधीन था।
(iv) 1833 का चािणर ऄतधतनयम
 आसे ‘सेंि हेिन
े ा ऄतधतनयम’ के नाम से भी जाना जािा है। आसके द्वारा ‘कें िीकरर्’ को ऄंतिम
स्िरूप प्रदान प्रदान ककया गया।
 तिरिश भारि में शासन के कें िीकरर् की कदशा में यह ऄतधतनयम एक तनर्ाणयक कदम था:
o बंगाि के गिनणर जनरि को भारि का गिनणर जनरि बना कदया गया, तजसमें सभी ऄसैतनक
(नागररक) और सैन्य शतियां तनतहि थीं। ज्ञािव्य है कक भारि के प्रथम गिनणर जनरि
तितियम बैंरिक थे।
o सपररषद् गिनणर जनरि (Governor General in Council) को तितध तनमाणर् की सभी
शतियां प्रदान की गइ। सपररषद् गिनणर जनरि को कं पनी के सैतनक िथा ऄसैतनक कायण का
तनयंिर्, तनरीक्षर् िथा तनदेशन सौंप कदया गया।
o आस प्रकार, आस ऄतधतनयम ने पहिी बार एक ऐसी सरकार का गठन ककया, तजसका तिरिश
ऄतधकार क्षेि िािे संपूर्ण क्षेि पर तनयंिर् था।
 पूिण तनर्पमि कानूनों को ‘तनयामक कानून या रेगि
ु श
े न’ कहा जािा था, िहीं आस नए एक्ि के ऄधीन
तनर्पमि कानूनों को ‘एक्ि या ऄतधतनयम’ कहा गया।
 आस एक्ि के िहि इस्ि आंतिया कं पनी के व्यापाररक एकातधकार को समाप्त कर कदया गया। ऄब यह
तिशुद् रूप से एक प्रशासतनक तनकाय बन गयी।
 आसके द्वारा सभी भारिीय कानूनों को संतहिाबद् करने का प्रयास ककया गया। आस संबंध में ,
सरकार को एक भारिीय तितध अयोग गरठि करने का तनदेश कदया गया था। आस प्रकार िॉिण
मैकािे की ऄध्यक्षिा में भारि के प्रथम तितध अयोग का गठन ककया गया।
 तसतिि सेिकों के चयन के तिए खुिी प्रतियोतगिा का अयोजन प्रारंभ करने हेिु प्रयास ककए गए*।
हािााँकक, कोिण ऑफ़ िायरेक्िसण के तिरोध के कारर् ईपयुणि प्रािधान को समाप्त कर कदया गया।
(v) 1853 का चािणर ऐक्ि
संिध
ै ातनक तिकास की दृतष्ट से यह ऄतधतनयम महत्िपूर्ण माना जािा है:
 पहिी बार सपररषद् गिनणर जनरि के तिधायी एिं प्रशासतनक कायों को स्पष्ट रूप से पृथक ककया गया।
o आसके िहि पररषद में छह नए सदस्यों को शातमि कर एक नए तनकाय भारिीय तिधान
पररषद् या कें िीय तिधान पररषद् (Indian Legislative Council or Central
Legislative Council) का गठन ककया गया।
o आस प्रकार, आस एक्ि द्वारा भारि में संसदीय प्रर्ािी की शुरुअि हुइ।

69 www.visionias.in ©Vision IAS

Google it:- https://upscpdf.com


https://upscpdf.com << Download From >> https://upscpdf.com

 आसने तसतिि सेिकों की भिी एिं चयन हेिु खुिी प्रतियोगी परीक्षा का शुभारम्भ ककया।*
o आस प्रकार संश्रातिि जानपद सेिा (Covenanted Civil Services) के तिए भारिीय
नागररकों को भी ऄनुमति प्रदान की गइ।
o 1854 में भारिीय तसतिि सेिा के संबंध में मैकािे सतमति की तनयुति की गइ।

 पहिी बार भारिीय तिधान पररषद में स्थानीय (प्रांिीय) प्रतितनतधत्ि प्रदान ककया। मिास,

बॉम्बे, बंगाि और अगरा की स्थानीय सरकारों द्वारा चार सदस्यों की तनयुति की गइ।
 आस ऄतधतनयम ने कं पनी के शासन ऄितध का तिस्िार ककया और तिरिश िाज की ओर से ट्रस्ि
(प्रन्यास) के रूप में कं पनी को भारिीय प्रदेशों पर ऄतधकार बनाए रखने की ऄनुमति प्रदान दी।
हािााँकक, तपछिे चािणसण के तिपरीि, आसमें कोइ तिशेष ऄितध या समय-सीमा तनधाणररि नहीं की

गयी थी। आस प्रकार, आसमें यह संकेि तनतहि था कक संसद ककसी भी समय जब चाहे कं पनी के
शासन को समाप्त कर सकिी है।

2.2.2. तिरिश िाज के ऄधीन सं िै धातनक तिकास ( 1858-1947)

(i) 1858 का भारि शासन ऄतधतनयम (Government of India Act, 1858)

 आस ऄतधतनयम को ‘भारि शासन को ऄतधक ऄच्छा बनाने के तिए ऄतधतनयम’ (The Act for

the Better Government of India) के नाम से जाना जािा है। आसके द्वारा भारिीय प्रदेशों पर

इस्ि आंतिया कं पनी के शासन को समाप्त कर कदया गया िथा शासन, प्रदेश और राजस्ि संबंधी
शतियों को तिरिश िाज को हस्िांिररि कर कदया गया।
 आस ऄतधतनयम का प्रमुख ईद्देश्य प्रशासतनक िंि में सुधार करना था। हािांकक, आसने भारि में

प्रचतिि शासन प्रर्ािी में कोइ तिशेष पररििणन नहीं ककया।

आस ऄतधतनयम की कु छ ऄन्य महत्िपूर्ण तिशेषिाएाँ तनम्नतितखि हैं:


 आस ऄतधतनयम ने बोिण ऑफ़ कं ट्रोि और कोिण ऑफ़ िायरेक्िसण को समाप्त कर भारि में दोहरी
तनयंिर् प्रर्ािी ऄथिा िबि गिनणमिें की व्यिस्था को समाप्त कर कदया।
 भारि राज्य सतचि (Secretary of State for India) नामक एक निीन पद का सृजन ककया

गया, तजसे भारिीय प्रशासन पर संपूर्ण तनयंिर् की शति प्रदान की गइ।


o भारि राज्य सतचि तिरिश कै तबनेि का एक सदस्य था और िह ऄंतिम रूप से तिरिश संसद
के प्रति ईर्त्रदायी था।
o सतचि की सहायिा के तिए एक 15 सदस्यीय पररषद् का प्रािधान ककया गया, जो एक
सिाहकारी तनकाय थी। भारि राज्य सतचि आस पररषद् का ऄध्यक्ष था।
o आस प्रकार, आसने भारिीय मामिों पर तिरिश संसद का तनयंिर् स्थातपि ककया। देश का
प्रशासन ऄब ऄत्यतधक कें िीकृ ि हो गया था।

िषण 1833 से िषण 1858 िक भारि में तिरिश शासन का प्रमुख भारि का गिनणर जनरि कहिािा था
और ईसे इस्ि आंतिया कं पनी के कोिण ऑफ़ िायरेक्िसण द्वारा तनयुि ककया जािा था एिं िह ईन्हीं के प्रति
ईर्त्रदायी भी था। िहीं िषण 1858 में कं पनी का शासन समाप्त होने के पश्चाि भारि में शासन का

प्रमुख िायसराय के नाम से जाना जाने िगा, जो तिरिश िाज का प्रतितनतध था और ईसी के प्रति
ईर्त्रदायी था।

70 www.visionias.in ©Vision IAS

Google it:- https://upscpdf.com


https://upscpdf.com << Download From >> https://upscpdf.com

(ii) भारि पररषद ऄतधतनयम (Indian Councils Acts): आसे कु ि िीन बार 1861, 1892 और
1909 में ऄतधतनयतमि ककया गया:
 1861 का भारि पररषद ऄतधतनयम
o आस ऄतधतनयम के द्वारा पहिी बार भारिीय प्रतितनतधयों को कानून तनमाणर् प्रकक्रया में
शातमि ककया गया।
o िायसराय ऄब कायणकारी पररषद में कु छ भारिीयों को गैर-सरकारी सदस्य के रूप में नातमि
(nominate) कर सकिा था।
o आस ऄतधतनयम ने मिास और बॉम्बे प्रेतसिेतन्सयों को पुनः तिधायी शतियााँ प्रदान कर
तिकें िीकरर् की प्रकक्रया की शुरुअि की।*
o आसके िहि बंगाि, ईर्त्र-पतश्चमी सीमांि प्रांि (NWFP) और पंजाब के तिए नइ तिधान
पररषदों के गठन हेिु प्रािधान ककया गया, तजन्हें क्रमशः 1862, 1866 और 1897 में
स्थातपि ककया गया।
o आसने िॉिण कै सनग द्वारा िषण 1859 में प्रारंभ की गइ ‘पोिणफोतियो’ प्रर्ािी को भी मान्यिा
प्रदान की।*
o आसने िायसराय को ऄध्यादेश जारी करने के तिए ऄतधकृ ि ककया।*
 1892 का भारि पररषद ऄतधतनयम
o आसने कें िीय और प्रांिीय तिधान पररषदों में गैर-सरकारी सदस्यों की संख्या में िृतद् की,
िेककन ईनमें ऄभी भी सरकारी सदस्यों के बहुमि को बनाए रखा गया ऄथाणि आनकी संख्या
ऄभी भी ऄतधक बनी रही।
o आस ऄतधतनयम ने कें िीय और प्रांिीय दोनों तिधान पररषदों में गैर-सरकारी सदस्यों की
तनयुति के तिए एक सीतमि और परोक्ष चुनाि का प्रािधान ककया।* हािांकक ‘चुनाि’
(elections) शब्लद के बदिे ‘मनोनयन’ (nomination) शब्लद का प्रयोग ककया गया। आस
प्रकार, पहिी बार चुनाि संबंधी प्रािधान को सतम्मतिि ककया गया।
o तिधान पररषद के सदस्यों को बजि पर बहस करने और प्रश्न पूछने की शति प्रदान की गइ,
हािााँकक मि तिभाजन का ऄतधकार नहीं कदया गया।
 1909 का भारि पररषद ऄतधतनयम (मॉिे-समिो सुधार)
o आसने प्रांिीय और कें िीय दोनों तिधान पररषदों के स्िर पर, सदस्यों की संख्या में िृतद् की।
o आसने कें िीय तिधान पररषद में सरकारी सदस्यों के बहुमि को बनाए रखा। हािांकक, प्रांिीय
तिधान पररषदों में गैर-सरकारी सदस्यों के बहुमि को स्िीकृ ति प्रदान की गइ।
o तिधान पररषदों के सदस्यों को चचाण करने का ऄतधकार प्रदान ककया गया। िे ऄनुपरू क प्रश्न
पूछ सकिे थे और बजि पर संकल्प प्रस्िुि कर सकिे थे।
o आसने कायणकारी पररषद में भारिीयों के प्रिेश का प्रािधान ककया। सत्येन्ि प्रसाद तसन्हा
िायसराय की कायणकारी पररषद के प्रथम भारिीय सदस्य बने। ईन्हें तितध सदस्य बनाया
गया था।
o आसने सांप्रदातयक प्रतितनतधत्ि (communal representation) की ऄिधारर्ा को स्िीकार
करिे हुए मुसिमानों के तिए पृथक तनिाणचक मंिि (separate electorate) की शुरुअि
की। आसके िहि, मुतस्िम सदस्यों को के िि मुतस्िम मिदािाओं द्वारा तनिाणतचि ककया जाना
था। आस प्रकार, आस ऄतधतनयम ने 'सांप्रदातयकिा को िैधिा' प्रदान की। िॉिण समिो को
सांप्रदातयक तनिाणचक मंिि का जनक माना जािा है।

71 www.visionias.in ©Vision IAS

Google it:- https://upscpdf.com


https://upscpdf.com << Download From >> https://upscpdf.com

(iii) भारि शासन ऄतधतनयम, 1919 (मोंिेग्यू-चेम्सफोिण सुधार)


िषण 1917 में तिरिश सरकार ने पहिी बार घोतषि ककया कक ईसका ईद्देश्य भारि में क्रतमक रूप से
ईर्त्रदायी शासन (राज्य सतचि मोंिेग्यू की घोषर्ा) की स्थापना करना है। िषण 1919 में भारि शासन
ऄतधतनयम तनर्पमि ककया गया। हािांकक, यह ईि ईद्देश्य के ऄनुरूप पूर्णि: संगि नहीं था। आस
ऄतधतनयम की कु छ महत्िपूर्ण तिशेषिाएाँ तनम्नतितखि हैं:
 कें िीय और प्रांिीय तिषयों की सूची की पहचान कर एिं ईन्हें पृथक कर राज्यों पर कें िीय तनयंिर्
में कमी की गइ। कें िीय और प्रांिीय तिधान पररषदों को ऄपनी-ऄपनी सूतचयों के तिषयों के संबंध
में तिधान बनाने का ऄतधकार प्रदान ककया गया।*
o हािांकक सरकार का मूि ढांचा पूर्णिया कें िीय और एकात्मक ही बना रहा।*
 प्रांिीय स्िर पर द्वैध शासन (Dyarchy) की शुरुअि की गइ।
o द्वैध शासन का ऄथण है: शासन के तिषयों को दो भागों में तिभि ककया जाना।
o ये दो भाग थे: हस्िांिररि तिषय (तजन्हें मंतियों की सहायिा से गिनणर द्वारा प्रशातसि ककया
जाना था) और अरतक्षि तिषय (आन्हें कायणकारी पररषद की सहायिा से गिनणर द्वारा प्रशातसि
ककया जाना था, जो तिधान पररषद के प्रति ईर्त्रदायी नहीं थे)।
 पहिी बार तद्वसदनीय व्यिस्था (Bicameralism) की शुरुअि की गइ। आसे के न्िीय स्िर पर िागू
ककया गया था।
 पहिी बार प्रत्यक्ष तनिाणचन की व्यिस्था प्रारंभ की गइ।
o संपति, कर और तशक्षा के अधार पर सीतमि संख्या में िोगों को मिातधकार प्रदान ककया
गया।
 कें िीय बजि को प्रांिीय बजि से पृथक ककया गया।
 आसमें एक िोक सेिा अयोग के गठन का प्रािधान ककया गया। ऄिः िषण 1926 में तसतिि सेिकों
की भिी के तिए एक कें िीय िोक सेिा अयोग (1923-24 के िी कमीशन की तसफाररश पर) का
गठन ककया गया।
 आसने तसखों, भारिीय इसाइयों, एंग्िो-आंतियन और यूरोपीय िोगों के तिए पृथक तनिाणचक मंिि
का प्रािधान करके सांप्रदातयक प्रतितनतधत्ि के तसद्ांि का तिस्िार ककया।
 आसमें यह प्रािधान ककया गया था कक आस ऄतधतनयम से हुइ प्रगति की समीक्षा और ररपोिण प्रस्िुि
करने के तिए 10 िषण पश्चाि् एक िैधातनक अयोग गरठि ककया जाएगा।
(iv) भारि शासन ऄतधतनयम, 1935 (Government of India Act, 1935)
1919 के भारि शासन ऄतधतनयम को पाररि करिे समय की गयी घोषर्ा के ऄनुसरर् में तिरिश
सरकार द्वारा आसकी समीक्षा के तिए सर जॉन साआमन की ऄध्यक्षिा में एक िैधातनक अयोग
(statutory commission) की तनयुति की घोषर्ा की गइ। आस अयोग में कु ि साि सदस्य थे।
अयोग के सभी सदस्य तिरिश थे, आसतिए सभी भारिीय दिों द्वारा अयोग का बतहष्कार ककया गया।
अयोग के प्रस्िािों पर तिचार करने के तिए तिरिश सरकार द्वारा, तिरिश भारि एिं तिरिश सरकार
िथा भारिीय ररयासिों के प्रतितनतधयों के मध्य िीन गोिमेज सम्मेिनों का अयोजन ककया गया। आन
सम्मेिनों की चचाणओं के अधार पर, ‘संिैधातनक सुधारों के संबंध में श्वेि पि’ (White Paper on
Constitutional Reforms) िैयार ककया गया और आसे तिरिश संसदीय सतमति के तिचाराथण प्रस्िुि
ककया गया। आस सतमति की तसफाररशों को कु छ पररििणनों के साथ भारि शासन ऄतधतनयम, 1935 में
सतम्मतिि कर तिया गया।

72 www.visionias.in ©Vision IAS

Google it:- https://upscpdf.com


https://upscpdf.com << Download From >> https://upscpdf.com

आस ऄतधतनयम का हमारे संतिधान के ििणमान स्िरूप के तनमाणर् में सिाणतधक महत्ि रहा है क्योंकक
आसके कइ प्रािधानों को हमारे संतिधान तनमाणिाओं द्वारा, ककसी न ककसी रूप में सतम्मतिि ककया गया
है। साथ ही, आस ऄतधतनयम के माध्यम से देश के तिए एक तितखि संतिधान का तनमाणर् ककया गया।*
आसके ऄतिररि, भारि शासन ऄतधतनयम, 1935 के द्वारा कइ िषों के पश्चाि्, भारिीयों को ऄपने देश
के प्रशासन के संचािन का ईर्त्रदातयत्ि प्राप्त करने का ऄिसर प्राप्त हुअ। आसकी कु छ प्रमुख तिशेषिाएाँ
तनम्नतितखि हैं:
 आसने एक ऄतखि भारिीय संघ (All-India Federation) की स्थापना का प्रािधान ककया,
तजसमें तिरिश शातसि प्रांिों एिं देसी ररयासिों को एक आकाइ के समान माना गया था।*
o आस ऄतधतनयम द्वारा कें ि और आकाआयों के मध्य शतियों को िीन सूतचयों - संघीय सूची,
प्रांिीय सूची और समििी सूची में तिभातजि ककया गया।*
 संघीय सूची, तजसमें कें ि सरकार को सौंपे गए तिषय शातमि थे।*
 प्रांिीय सूची, तजसमें के िि प्रांिों के क्षेिातधकार में शातमि सभी तिषय सतम्मतिि थे।*
 समििी सूची, तजसमें कें ि िथा प्रांिों दोनों के सामूतहक प्रातधकार िािे तिषय
सतम्मतिि थे।*
o ऄितशष्ट शतियां िायसराय को प्रदान की गईं।
o हािााँकक, यह फे िरेशन (संघ) कभी भी ऄतस्ित्ि में नहीं अया, क्योंकक देसी ररयासिें आसमें
शातमि नहीं हुईं। *
 आसने भारि शासन ऄतधतनयम, 1919 के द्वारा राज्यों में िागू द्वैध शासन पद्ति को समाप्त कर
(आसके स्थान पर) ‘प्रांिीय स्िायर्त्िा’ की शुरुअि की। राज्यों को ईनके तिए तनधाणररि सीमा के
िहि स्िायर्त् िरीके से प्रशासन संचािन का ऄतधकार प्रदान ककया गया। आसके ऄतिररि, आस
ऄतधतनयम ने राज्यों में ईर्त्रदायी सरकार की स्थापना की। ऄब गिनणर के तिए राज्य तिधान
पररषदों के प्रति ईर्त्रदायी मंतियों की सिाह पर कायण करना अिश्यक था। आस व्यिस्था को िषण
1937 में िागू ककया गया िथा िषण 1939 को समाप्त कर कदया गया।
 आसके ऄंिगणि कें ि में द्वैध-शासन प्रर्ािी का प्रािधान ककया गया था। आसके पररर्ामस्िरूप संघीय
तिषयों को अरतक्षि और हस्िांिररि तिषयों में तिभातजि ककया गया था। हािांकक, ऄतधतनयम के
आस प्रािधान को कभी भी िागू नहीं ककया गया।
 ग्यारह में से छ: प्रांिों में तद्वसदनीय व्यिस्था को प्रारम्भ ककया गया। आसके पररर्ामस्िरूप संयुि
प्रांि, तबहार, ऄसम, बंगाि, मिास और बॉम्बे की तिधातयकाओं में दो सदन (तिधानसभा और
तिधान पररषद्) िथा ऄन्य प्रांिों में एक सदन (तिधानसभा) िािी तिधातयका का गठन हुअ। आन
सदनों की सदस्यिा से संबंतधि मानदंि तभन्न-तभन्न प्रांिों में तभन्न-तभन्न थे। कें ि के स्िर पर संघीय
तिधातयका के दो सदनों राज्य पररषद् (Council of States) और संघीय सभा (Federal
Assembly) में क्रमशः 260 और 375 सदस्यों का प्रािधान ककया गया। राज्य पररषद् (ईच्च
सदन) एक स्थायी तनकाय थी, तजसके एक तिहाइ सदस्य प्रत्येक िीन िषों में सेिातनिृर्त् होने थे।*
 कें ि में एक संघीय न्यायािय की स्थापना की गयी।*
ईपयुि
ण के ऄतिररि, आसमें तनम्नतितखि प्रािधान भी शातमि थे:
 ससध और ईडीसा प्रांिों का गठन।
 आसने िंतचि िगों (ऄनुसतू चि जातियों), मतहिाओं िथा मजदूर िगण के तिए पृथक तनिाणचक मंिि
की व्यिस्था कर सांप्रदातयक प्रतितनतधत्ि व्यिस्था का और तिस्िार ककया। हािांकक, महात्मा
गांधी के प्रयासों से पूना समझौिा के ईपरांि ऄनुसूतचि जातियों के तिए पृथक तनिाणचक मंिि की
व्यिस्था में थोडा सा पररििणन ककया गया।

73 www.visionias.in ©Vision IAS

Google it:- https://upscpdf.com


https://upscpdf.com << Download From >> https://upscpdf.com

 बमाण को भारि से पृथक कर कदया गया।


 1858 के भारि शासन ऄतधतनयम द्वारा स्थातपि भारि पररषद् (आंतिया काईं तसि) को समाप्त कर
कदया गया िथा भारि सतचि को भारि तिषयक नीतियों पर परामशण देने हेिु 3 से 6 सदस्यों
िािी सिाहकारों की एक िीम का गठन ककया गया।
 भारि सतचि से सामान्यिः भारिीय मामिों में हस्िक्षेप न करने की ऄपेक्षा की गइ और आन
मामिों पर गिनणरों द्वारा तनर्णय ककया जाना था।
o साथ ही, भारि के िायसराय के द्वारा पााँच िषों के कायणकाि के तिए एक ईच्चायोग की
तनयुति की जानी थी।
 संघीय सरकार के संबंध में ककए गए पररििणनों के पररप्रेक्ष्य में, िायसराय आसका प्रमुख बना रहा।
आस ऄतधतनयम के िहि ईसे प्रशासतनक, तिधायी और तिर्त्ीय शतियों सतहि व्यापक शतियां
प्रदान की गईं।
 अरतक्षि तिषयों का प्रशासन ऄपने कायणकाररर्ी सदस्यों की सहायिा से िायसराय ऄपनी
आच्छानुसार कर सकिा था। आसी प्रकार, हस्िांिररि तिषय भी िायसराय के ऄधीन ककए गए,
तजन पर िह तिधातयका से चुने गए भारिीय मंतियों (तजनकी ऄतधकिम संख्या 10 हो सकिी थी)
की सिाह के ऄनुसार कायण करिा था।
 प्रांिीय सरकारों के मामिे में गिनणर द्वारा प्रांिीय तिधातयका से एक मंतिपररषद् का चयन ककया
जाना था और आसके द्वारा आन्हीं मंतियों की सहायिा से प्रशासतनक कायों का संचािन ककया जािा
था। प्रांिीय तिधातयका की संरचना भी तभन्न-तभन्न प्रान्िों में तभन्न-तभन्न रही।
 देश की मुिा एिं साख पर तनयंिर् रखने हेिु एक भारिीय ररज़िण बैंक की स्थापना का प्रािधान
ककया गया।
 आसके माध्यम से मिातधकार का तिस्िार ककया गया और िगभग 10 प्रतिशि जनसंख्या को
मिदान का ऄतधकार प्राप्त हुअ।
आस ऄतधतनयम का अिोचनात्मक तिश्लेषर्
ऐसा कहा जािा है कक यह ऄतधतनयम 'चाशनी में तिपिी कु नैन' के ऄतिररि कु छ भी नहीं था:
 संघीय व्यिस्था कभी ऄतस्ित्ि में नहीं अइ क्योंकक देसी ररयासिों ने आसमें शातमि होने से मना
कर कदया था।
 यद्यतप कें ि में द्वैध शासन िथा प्रांिों की स्िायर्त्िा का प्रािधान ककया गया था, िथातप तनिाणतचि
या मनोनीि सदस्यों की शतियां सीतमि ही रहीं। आस ऄतधतनयम ने एक प्रकार से प्रांिों में के न्ि
सरकार के तनयंिर् को बनाए रखा, क्योंकक गिनणर को ऄपने ‘तििेक’ या कु छ मामिों में ऄपने
'व्यतिगि तनर्णय' का प्रयोग करिे हुए कायण करने की शति प्रदान की गइ थी। ऐसे मामिों में
गिनणर तबना मंतियों की सिाह के िथा िायसराय एिं भारि सतचि के तनदेशन ि तनयंिर् में
कायण करने के तिए बाध्य था।
 कें िीय और प्रांिीय तिधातयकाओं की तिधायी शतियां तितभन्न सीमाओं के ऄधीन थीं िथा दोनों में
से ककसी को भी एक संप्रभु तिधान-मंिि की शतियां प्राप्त नहीं थी। ईदाहरर्ाथण:
o िायसराय द्वारा िीिो की शति के ऄतिररि, कें िीय तिधातयका द्वारा पाररि ककसी भी
ऄतधतनयम पर क्राईन द्वारा भी िीिो ककया जा सकिा था।
o यकद िायसराय अश्वस्ि हो कक तिधातयका में प्रस्िुि तिधेयक ईसके "तितशष्ट ईर्त्रदातयत्िों"
के तनिणहन को प्रभातिि कर रहा है िो िह ईस तिधेयक पर हो रही चचाण को स्थतगि कर
सकिा था और सदन की कायणिातहयों को तनिंतबि कर सकिा था।

74 www.visionias.in ©Vision IAS

Google it:- https://upscpdf.com


https://upscpdf.com << Download From >> https://upscpdf.com

o तिधातयका के सि में न रहने के दौरान ऄध्यादेश प्रख्यातपि करने के ऄतिररि िायसराय को


तिधातयका के समानांिर कानून बनाने की स्ििंि शति भी प्राप्त थी। आस प्रकार, िह ऄपने
तितशष्ट ईर्त्रदातयत्िों के तनिणहन के तिए ककसी भी समय ऄस्थायी ऄध्यादेशों के साथ-साथ
स्थायी ऄतधतनयमों के तनमाणर् की शति भी धारर् करिा था।
o कु छ मामिों में िायसराय की पूिण स्िीकृ ति के तबना तिधातयका में कोइ तिधेयक या संशोधन
प्रस्िुि नहीं ककया जा सकिा था। ईदाहरर्ाथण - यकद तिधेयक या संशोधन तिरिश संसद द्वारा
भारि में िागू ककए जाने के अशय से तनर्पमि या िायसराय ऄथिा गिनणर द्वारा पाररि ककसी
ऄतधतनयम के ऄधीन ककसी कानून को तनरस्ि करने, ईसमें सुधार करने ऄथिा ईसके तिरोध
में कानून बनाने से संबंतधि होिा था या कफर यह ककसी ऐसे मुद्दे को संबोतधि करिा था तजस
पर िायसराय द्वारा ऄपने तििेकातधकार का प्रयोग करने की अिश्यकिा थी, िो ऐसे
तिधेयक हेिु ईसकी पूिण स्िीकृ ति की अिश्यकिा होिी थी।
o आसी प्रकार प्रांिीय तिधातयकाओं पर भी ऄंकुश िगाया गया। ऄतधतनयम के िहि जारी ऄन्य
तनदेशों के ऄनुसार कु छ तिषयों, यथा- ईच्च न्यायािय की शतियों को कम करने या स्थायी
बंदोबस्ि को प्रभातिि करने िािे आत्याकद से संबंतधि तिधेयक िायसराय या गिनणर के समक्ष
ईनकी सहमति हेिु प्रस्िुि ककए जाने पर क्रमशः क्राईन एिं िायसराय के तिचाराथण हेिु
अरतक्षि ककए जाने थे।

2.2.3. ऄन्य महत्िपू र्ण घिनाक्रम

(i) साआमन कमीशन

 1919 के भारि सरकार ऄतधतनयम द्वारा स्थातपि भारिीय संतिधान की कायणप्रर्ािी पर ररपोिण
प्रस्िुि करने हेिु निंबर 1927 में तििेन के कं जिेरिि दि की सरकार द्वारा साआमन कमीशन की
तनयुति की गयी थी।
 अयोग की संरचना को िेकर भारि में आसकी व्यापक रूप से सनदा हुइ क्योंकक कोइ भी भारिीय
आसमें शातमि नहीं था। कांग्रेस सतहि ऄतधकांश भारिीय राजनीतिक दिों ने आस अयोग का
बतहष्कार ककया।
 अयोग की ररपोिण िषण 1930 में प्रकातशि हुइ। आसके द्वारा भारि में प्रांिीय स्िायर्त्िा का प्रस्िाि
रखा गया और प्रांिों में द्वैध शासन को प्रतिस्थातपि कर ईर्त्रदायी सरकार के गठन की तसफाररश
की गयी, तजसे िषण 1935 में िागू ककया गया। हािांकक, आसने कें ि में ईर्त्रदायी सरकार के तिचार
को ऄस्िीकृ ि कर कदया। आसने संघिाद को स्िीकार कर तिया और तिरिश िाज िथा भारिीय
ररयासिों के मध्य प्रत्यक्ष संपकण बनाए रखने की मांग की।
 िषण 1929 में िायसराय िॉिण आरतिन द्वारा की गयी ऄक्िूबर घोषर्ा ने आस अयोग की ऄतधकांश
तसफाररशों को ऄप्रचतिि (अईििेिि
े ) कर कदया। ईल्िेखनीय है कक आस घोषर्ा में भारिीय
संिैधातनक तिकास का िक्ष्य िोतमतनयन स्िेिस रखा गया था।

(ii) सांप्रदातयक पंचाि (Communal Award)


 ऄगस्ि 1932 में तद्विीय गोिमेज सम्मेिन के पश्चाि्, तिरिश प्रधानमंिी, रैमजे मैकिोनाल्ि ने

'सांप्रदातयक पंचाि’ की घोषर्ा की। आसके ऄनुसार, ऄगडी जातियों, तनचिी जातियों, मुतस्िम,

बौद्, तसख, भारिीय इसाइयों, अंग्ि-भारिीय, यूरोपीय और दतििों के तिए पृथक प्रतितनतधत्ि
प्रदान ककया जाना था। दतििों को कु छ तिशेष तिधानसभा क्षेिों में कु छ सीिें प्रदान की गईं तजन
पर के िि दतिि समुदाय से संबंतधि मिदािा ही िोि कर सकिे थे।

75 www.visionias.in ©Vision IAS

Google it:- https://upscpdf.com


https://upscpdf.com << Download From >> https://upscpdf.com

 महात्मा गांधी द्वारा आस पंचाि का तिरोध ककया गया। ईन्होंने आसके तिरुद् ईपिास भी ककया। आस
संबंध में गााँधीजी और ऄंबेिकर के बीच एक तिस्िृि िािाण के बाद एक समझौिा हुअ, तजसे पूना
समझौिा कहा जािा है। आस समझौिे में गााँधीजी एिं िॉ. ऄंबेिकर के ऄतिररि मदन मोहन
माििीय एिं एम. सी. रजा भी शातमि थे। आस समझौिे में यह व्यिस्था की गइ थी कक सहदुओं का
एक ही मि होगा, ईनमें सांप्रदातयक पंचाि के अधार पर तिभाजन नहीं ककया जाएगा और दतििों
के तिए आसी के ऄंिगणि सीिें अरतक्षि होंगी।
(iii) कक्रप्स तमशन (Cripps Mission)
 माचण 1942 में, तिरिश मंतिमंिि के सदस्य, सर स्िैनफ़ोिण कक्रप्स, तिरिश सरकार द्वारा प्रस्िुि एक
मसौदा प्रस्िाि के साथ भारि अए। आन प्रस्िािों को, तद्विीय तिश्व युद् की समातप्त के पश्चाि िागू
ककया जाना था, बशिे कक कांग्रेस और मुतस्िम िीग द्वारा ईि प्रस्िाि को स्िीकृ ति प्रदान की
जाए।
 कक्रप्स प्रस्िाि के ऄनुसार:
o भारिीय िोगों द्वारा तनिाणतचि संतिधान सभा द्वारा भारि के संतिधान का तनमाणर् ककया
जाएगा।
o ईि संतिधान, भारि को ऄतधराज्य (िोतमतनयन) का दजाण प्रदान करेगा।
o सभी प्रांिों और भारिीय राज्यों के तिए एक भारिीय संघ की स्थापना की जाएगी।
o जो भी प्रांि (या देसी ररयासि) आस संतिधान को ऄस्िीकृ ि करेंगे, ईन्हें यह ऄतधकार होगा
कक िह ऄपनी यथातस्थति को बनाए रखें। ईि संतिधान को न स्िीकारने िािे राज्यों के
तिए, तिरिश सरकार पृथक संिैधातनक व्यिस्था स्थातपि कर सकिी है।
(iv) कै तबनेि तमशन (Cabinet Mission)
 माचण 1946 में, िािण एििी द्वारा िीन कै तबनेि मंतियों से तनर्पमि एक तशष्टमंिि (कै तबनेि तमशन)
को भारि भेजा गया, तजसमें िािण पेतथक िॉरेंस, सर स्िेफिण कक्रप्स और ए. िी. ऄिेक्जेंिर शातमि थे।
 कै तबनेि तमशन का ईद्देश्य भारि की यथाशीघ्र स्ििंििा प्रातप्त में सहायिा करना और एक
संतिधान सभा का गठन करना था।
 कै तबनेि तमशन योजना के ऄनुसार, तिरिश भारि और देसी ररयासिों को शातमि कर भारि संघ
का गठन ककया जाना था, तजसके ऄतधकार क्षेि में तिदेश, रक्षा और संचार संबंधी तिषय होंगे।
जबकक सभी ऄितशष्ट शतियााँ प्रांिों और राज्यों में तनतहि होंगी।
 संघ में एक कायणपातिका और एक तिधातयका का गठन ककया जाना था, तजसमें ररयासिों एिं
प्रांिों के प्रतितनतध शातमि होने थे।
 प्रत्येक प्रांि कायणपातिका और तिधातयका के साथ समूह बना सकिे थे और प्रत्येक समूह को प्रांिीय
तिषयों के संबंध में तनर्णय िेने का ऄतधकार था।
(v) माईं िबेिन योजना (Mountbatten Plan)
आस योजना में भारिीयों को सर्त्ा का हस्िांिरर् और देश के तिभाजन संबंधी प्रािधान ककए गए थे। 3
जून 1947 को तिरिश सरकार द्वारा प्रस्िुि एक औपचाररक घोषर्ा के माध्यम से आसे िागू ककया गया।
(vi) भारिीय स्ििंििा ऄतधतनयम, 1947
20 फरिरी 1947 को ित्कािीन तिरिश प्रधानमंिी क्िीमेंि एििी ने घोषर्ा की थी कक 30 जून
1948 िक भारि में तिरिश शासन समाप्त हो जाएगा। आसी को ध्यान में रखिे हुए भारिीय स्ििंििा
ऄतधतनयम, 1947 पाररि ककया गया। भारिीय स्ििंििा ऄतधतनयम, 1947 के िहि तनम्नतितखि
प्रािधान थे:

76 www.visionias.in ©Vision IAS

Google it:- https://upscpdf.com


https://upscpdf.com << Download From >> https://upscpdf.com

 तिरिश शासन की समातप्त।


 भारि और पाककस्िान के रूप में देश का तिभाजन।
 िायसराय के पद की समातप्त िथा भारि और पाककस्िान दोनों के तिए एक गिनणर जनरि की
तनयुति।
 भारि और पाककस्िान दोनों िोतमतनयनों की संतिधान सभाओं को सशि करना िाकक िे दोनों
देशों के तिए संतिधान का तनमाणर् एिं ऄंगीकरर् कर सकें ।

3. तिकास (Evolution)
3.1. दो अयाम

आसे हम तनम्नतितखि दो अयामों के ऄंिगणि समझ सकिे हैं:


 संतिधान को ऄंगीकृ ि, ऄतधतनयतमि और अत्मार्पपि ककए जाने से पूिण का घिनाक्रम (िषण 1950
के पूिण की घिनाएाँ); एिं
 एक सिि प्रकक्रया के रूप में तिकास (िषण 1950 के पश्चाि् की घिनाएाँ)।

3.1.1. सं तिधान को ऄं गीकृ ि, ऄतधतनयतमि और अत्मार्पपि ककए जाने से पू िण का


घिनाक्रम
 संतिधान सभा
o पररचय
 एक संप्रभु िोकिांतिक राष्ट्र का संतिधान िैयार करने का कायण आसके िोगों की एक
प्रतितनतध संस्था द्वारा ककया जािा है। संतिधान पर तिचार करने एिं ईसे ऄंगीकृ ि करने
के ईद्देश्य से िोगों द्वारा तनिाणतचि (प्रत्यक्ष/ऄप्रत्यक्ष) ऐसे तनकाय को संतिधान सभा कहा
जािा है।
o संतिधान सभा के तिचार की ईत्पतर्त्
 संतिधान सभा का तिचार िषण 1906 में ही भारिीय राष्ट्रीय कांग्रेस द्वारा स्िराज के तिए
की गइ मांग में तनतहि था।
 भारि में संतिधान सभा के गठन का तिचार सिणप्रथम िषण 1934 में एम. एन. रॉय द्वारा
प्रस्िुि ककया गया।
 िषण 1936, में कांग्रेस ने यह संकल्प ककया कक, “कांग्रेस भारि में एक ऐसे तिशुद्
िोकिांतिक राज्य हेिु दृढ़ संकतल्पि है, जहां सम्पूर्ण सर्त्ा जनिा में तनतहि होगी िथा
सरकार ईनके प्रभािी तनयंिर् में कायण करेगी। ऐसा राज्य के िि एक संतिधान सभा के
माध्यम से ही ऄतस्ित्ि में सकिा है, तजसे (संतिधान सभा) देश के तिए संतिधान तनर्पमि
करने की ऄंतिम शति प्राप्त होगी।”
 15 माचण 1946 को िेबर पािी के प्रधानमंिी ऐििी द्वारा स्पष्ट रूप से स्ियं ऄपना
संतिधान तनर्पमि करने संबंधी भारिीयों के ऄतधकार को स्िीकार ककया गया।
 िदनुपरांि तिरिश संसद द्वारा भारिीय स्ििंििा ऄतधतनयम, 1947 पाररि ककया गया।
15 ऄगस्ि 1947 से प्रभािी होने िािे आस ऄतधतनयम के िहि दो स्ििंि िोतमतनयन
(भारि एिं पाककस्िान) का तनमाणर् ककया गया।
 पहिे से ही गरठि संतिधान सभा ने कै तबनेि तमशन योजना के ऄनुसार कायण करना
अरम्भ कर कदया। संपूर्ण भारि के तिए आसकी सदस्य संख्या 389 थी। आसके ऄंिगणि 93
सदस्य देसी ररयासिों और 292 सदस्य तिरिश भारिीय प्रांिों से तनिाणतचि (ऄप्रत्यक्ष
रीति से) ककए गए थे िथा 4 चीफ कतमश्नर िािे क्षेिों के प्रतितनतध सतम्मतिि थे।

77 www.visionias.in ©Vision IAS

Google it:- https://upscpdf.com


https://upscpdf.com << Download From >> https://upscpdf.com

o 9 कदसंबर 1946 को संतिधान सभा की प्रथम बैठक हुइ थी। ज्ञािव्य है कक प्रथम बैठक के
समय संतिधान सभा एक संप्रभु तनकाय नहीं थी। आसे तिरिश संसद के कै तबनेि तमशन द्वारा
तनधाणररि एक तनतश्चि प्रकक्रया का ऄनुपािन करना था।
o 11 कदसंबर 1946 को भारिीय राष्ट्रीय कांग्रेस ने िॉ. राजेंि प्रसाद को संतिधान सभा के
स्थायी ऄध्यक्ष के रूप में चुना।
(i) संतिधान सभा का गठन
o संतिधान सभा अंतशक रूप से तनिाणतचि और अंतशक रूप से नातमि एक तनकाय थी। आसके
सदस्यों का चयन ऄप्रत्यक्ष रूप से प्रांिीय तिधान सभाओं के सदस्यों द्वारा ककया गया था।
ईल्िेखनीय है कक प्रांिीय तिधान सभाओं के सदस्यों का तनिाणचन सीतमि मिातधकार (कर,

संपतर्त् और तशक्षा के अधार पर) के अधार पर हुअ था।


o यद्यतप, यह ऄप्रत्यक्ष रूप से तनिाणतचि तनकाय थी, िथातप, आसमें भारिीय समाज के सभी

िगों (यथा- सहदू, मुतस्िम, तसख, पारसी, अंग्ि-भारिीय) के प्रतितनतध शातमि थे। आसके

ऄतिररि, गााँधीजी और तजन्ना को छोडकर संतिधान सभा में ित्कािीन भारि के िगभग
सभी प्रमुख व्यतित्ि (नेिा) शातमि थे।
(ii) संतिधान सभा की सतमतियां
संतिधान सभा ने संतिधान तनमाणर् से संबंतधि तितभन्न कायों के सरििापूिणक तनिणहन हेिु 8 प्रमुख
सतमतियों और कइ ऄन्य छोिी सतमतियों का गठन ककया था। प्रमुख सतमतियां और ईनके ऄध्यक्षों के
नाम आस प्रकार हैं:
 प्रारूप सतमति - िॉ. बी. अर. ऄंबि
े कर
o यह संतिधान सभा की सबसे महत्िपूर्ण सतमति थी। आसमें साि सदस्य थे और आसे नए
संतिधान का एक प्रारूप िैयार करने का ईर्त्रदातयत्ि सौंपा गया था। संतिधान का प्रारूप छः
माह से भी कम समय में बनकर िैयार हुअ जो स्ियं में एक बडी ईपितब्लध है।
 संघ शति सतमति - जिाहर िाि नेहरू
 संघीय संतिधान सतमति - जिाहर िाि नेहरू
 प्रांिीय संतिधान सतमति - सरदार िल्िभभाइ पिेि
 मूि ऄतधकार, ऄल्पसंख्यकों एिं जनजािीय और ऄपिर्पजि क्षेिों संबध
ं ी सिाहकारी सतमति -
सरदार िल्िभभाइ पिेि
आसकी तनम्नतितखि ईप-सतमतियां थीं:
o ऄल्पसंख्यक ईप-सतमति- एच. सी. मुखजी
o मूि ऄतधकार ईप-सतमति- जे. बी. कृ पिानी
o ऄपिर्पजि और अंतशक रूप से ऄपिर्पजि क्षेिों (ऄसम के क्षेिों को छोडकर) संबध
ं ी
ईपसतमति- ए. िी. ठक्कर
o पूिोर्त्र सीमांि जनजािीय क्षेिों और असाम के ऄपिर्पजि और अंतशक रूप से ऄपिर्पजि
क्षेिों संबध
ं ी ईपसतमति- गोपीनाथ बारदोिोइ
 राज्यों के तिए सतमति - जिाहर िाि नेहरू
 प्रकक्रया तनयम सतमति - िॉ. राजेंि प्रसाद
 संचािन सतमति - िॉ. राजेंि प्रसाद
छोिी सतमतियों में से कु छ तनम्नतितखि थीं:
 कायण संचािन सतमति - के . एम. मुंशी
 राष्ट्रीय ध्िज संबध
ं ी िदथण सतमति - िॉ. राजेंि प्रसाद
 संतिधान सभा के कायों से संबध
ं ी सतमति - जी. िी. माििंकर

78 www.visionias.in ©Vision IAS

Google it:- https://upscpdf.com


https://upscpdf.com << Download From >> https://upscpdf.com

 सदन सतमति - बी. पट्टातभसीिारमैया


 क्रेिेंतशयि सतमति - सर ऄल्िादी कृ ष्र्ा स्िामी ऄय्यर
 तिर्त् एिं कार्पमक सतमति - िॉ. राजेंि प्रसाद
 चीफ कतमश्नर िािे प्रांिों की सतमति - बी. पट्टातभसीिारमैया
 सिोच्च न्यायािय संबध
ं ी िदथण सतमति- एस. िदाणचाररयर
(iii) कायणप्रर्ािी

संतिधान सभा की प्रथम बैठक 9 कदसम्बर 1946 को हुइ, तजसका मुतस्िम िीग द्वारा बतहष्कार ककया
गया और पृथक पाककस्िान की मांग पर बि कदया गया।
 ईद्देश्य प्रस्िाि (Objective Resolutions)

13 कदसम्बर 1946 को जिाहर िाि नेहरू ने संतिधान सभा के समक्ष ऐतिहातसक 'ईद्देश्य

प्रस्िाि’ प्रस्िुि ककया। 'ईद्देश्य प्रस्िाि' का मूि तिचार संिैधातनक संरचना के मूि तसद्ांिों एिं
दशणन को प्रतिष्ठातपि करना था। आसके प्रमुख ऄंश आस प्रकार थे:
o संतिधान सभा भारि को एक स्ििंि एिं संप्रभु गर्राज्य घोतषि करने संबंधी ऄपने दृढ़ एिं
महत्त्िपूर्ण संकल्प को व्यि करिी है।
o संप्रभु स्ििंि भारि की सभी शतियां एिं प्रातधकार, आसके ऄतभन्न ऄंग िथा सरकार के ऄंग,
सभी का स्रोि भारि की जनिा होगी। (िोकिंि का समथणन)
o भारि के सभी िोगों के तिए न्याय, सामातजक, अर्पथक एिं राजनीतिक स्ििंििा एिं सुरक्षा,

ऄिसर की समानिा, तितध के समक्ष समिा, तिचार एिं ऄतभव्यति की स्ििंििा, तिश्वास,

अस्था, ईपासना, व्यिसाय करने, संगठन बनाने की स्ििंििा िथा िोक सदाचार की
स्थापना सुतनतश्चि की जाएगी। (मूि ऄतधकार)
o ऄल्पसंख्यकों, तपछडे िगों िथा जनजािीय क्षेिों के िोगों को पयाणप्त सुरक्षा ईपाय प्रदान की

जाएगी। (भाग X, भाग XVI)


o कें िीय सरकार भारिीय गर्राज्य के क्षेिों की एकिा िथा तिश्व के सभ्य देशों के न्याय एिं
तितधयों के अधार पर भू-क्षेि, समुि और िायु क्षेि में ऄपनी संप्रभुिा को ऄक्षुण्र् बनाए रखेगी।
o एक प्राचीन भूतम होने के कारर् भारि तिश्व में ऄपना ईतचि और सम्मानजनक स्थान रखिा है
िथा यह तिश्व शांति एिं मानि कल्यार् को बढ़ािा देने में ऄपना पूर्ण योगदान सुतनतश्चि करेगा।
 'ईद्देश्य प्रस्िाि', ििणमान भारिीय संतिधान की ईद्देतश्यका का अधार बना।

(iv) ऄतधतनयमन और प्रििणन

 संतिधान के प्रारूप के दो बार िाचन के पश्चाि् तितभन्न पररििणनों को आसमें शातमि ककया गया,

ित्पश्चाि िॉ. बी. अर. ऄंबेिकर ने 26 निंबर 1949 को एक प्रस्िाि प्रस्िातिि ककया। 26 निंबर

1949 के कदन ही भारि के िोगों ने 'संतिधान सभा में संतिधान को ऄंगीकृ ि, ऄतधतनयतमि एिं

अत्मार्पपि ककया'।

 ऄपने (26 जनिरी के ) ऐतिहातसक महत्ि के कारर् 26 जनिरी 1950 को संतिधान के 'िागू होने

की तितथ' के रूप में चुना गया। (हािांकक, संतिधान के कु छ प्रािधानों को 26 निंबर 1949 से ही
िागू कर कदया गया था)
(v) संतिधान सभा की अिोचना
 यह प्रतितनतध तनकाय नहीं थी: संतिधान सभा के सदस्यों का चुनाि िोगों द्वारा सािणभौतमक
ियस्क मिातधकार के अधार पर प्रत्यक्ष रूप से नहीं ककया गया था।

79 www.visionias.in ©Vision IAS

Google it:- https://upscpdf.com


https://upscpdf.com << Download From >> https://upscpdf.com

 यह एक संप्रभु तनकाय नहीं थी: यह कहा जािा है कक संतिधान सभा का गठन तिरिश सरकार के
प्रस्िािों के अधार पर ककया गया था। आसके ऄतिररि, आसके द्वारा तिरिश सरकार की ऄनुमति के
पश्चाि् ही ऄपने सि का अयोजन ककया जािा था।
 कांग्रस
े के सदस्यों का िचणस्ि: तिरिश संतिधान तिशेषज्ञ ग्रेनतििे ऑतस्िन ने रिप्पर्ी की:
"संतिधान सभा एक-दिीय देश का एक-दिीय तनकाय थी। सभा ही कांग्रेस थी और कांग्रेस ही
भारि थी।"
 िकीिों एिं राजनीतिज्ञों का िचणस्ि: यह माना जािा है कक िकीिों और राजनेिाओं का
तिधानसभाओं में प्रभुत्ि था। ईल्िेखनीय है कक आसे संतिधान की तिस्िृि और जरिि प्रकृ ति के
पीछे मुख्य कारर् के रूप में ईद्धृि ककया गया है।
3.1.2. एक सिि प्रकक्रया के रूप में क्रतमक तिकास

 न्यायमूर्पि श्री एच. अर. खन्ना ने ऄपनी पुस्िक ‘मेककग ऑफ़ कॉतन्स्िट्यूशन’ में कहा है कक:
"संतिधान का तनमाणर् कायण ईच्चिम शासन कौशि की मांग करिा है। तजन्हें आस कायण का दातयत्ि
सौंपा जािा है, ईन्हें सरकार की व्यिहाररक अिश्यकिाओं का ज्ञान होना चातहए, साथ ही ईन्हें
राष्ट्र तनमाणर् को प्रेररि करने िािे अदशों को भी ध्यान में रखना होिा है”।
 ककसी संतिधान को जीिंि दस्िािेज़ के रूप में होना चातहए। आसे एक या दो पीकढ़यों के तिए नहीं
बतल्क भािी पीकढ़यों के तिए एक जीिंि दस्िािेज़ होना चातहए। आसका कारर् यह है कक संतिधान
के प्रािधान को सामान्य शब्लदों में िर्पर्ि ककया जािा है, ऄसाधारर् सामान्योति के तिए संतिधान
की तिषय-िस्िु और महत्ि समय-समय पर पररिर्पिि होिे रहिे हैं और साथ ही ईसमें
ऄनुभिािीि तनरंिरिा भी शातमि होिी है।
 एक संतिधान को भूिकाि के तिए तनयमों का तनमाणर् नहीं करना चातहए बतल्क ईसे भतिष्य के
तिए तसद्ांिों का तनमाणर् करना चातहए।
 संतिधान की तनम्नतितखि दो ऄंिजाणि तिशेषिाओं के कारर् आसे जीिंि माना गया है:
(i) भारिीय संतिधान की एक प्रमुख तिशेषिा यह है कक यह सिि पररििणन के तिए ईपिब्लध है , िाकक
समय के साथ आसमें कु छ अिश्यक पररििणन ककए जा सकें । निीन संशोधनों द्वारा संतिधान को ऄतभपुष्ट
(ratify) करके या ककसी तिद्यमान प्रािधान को तनरस्ि करके आसमें पररििणन ककया जा सकिा है। आसके
कु छ ईदाहरर् तनम्नतितखि हैं:
 42िें संतिधान संशोधन ऄतधतनयम द्वारा संतिधान के एक ऄंग के रूप में स्पष्ट रूप से
पंथतनरपेक्षिा और समाजिाद की ऄिधारर्ाओं का ईल्िेख ककया गया।
 ऄनुच्छेद 15(4) को प्रथम संतिधान संशोधन ऄतधतनयम द्वारा जोडा गया है और आसके द्वारा
सामातजक एिं शैक्षतर्क रूप से तपछडे िगों या ऄनुसूतचि जाति/ऄनुसूतचि जनजातियों के तिए
सकारात्मक कारणिाइ करने का प्रािधान ककया गया था।
 आसी प्रकार, ऄनुच्छेद 15(5) तशक्षर् संस्थानों (चाहे िे सरकारी सहायिा प्राप्त हो या गैर-सहायिा
प्राप्त) में सामातजक और शैक्षतर्क रूप से कमजोर िगो के तिए सकारात्मक कारणिाइ करने का
प्रािधान करिा है। ऄनुच्छेद 15(5) को 93िें संतिधान संशोधन ऄतधतनयम द्वारा जोडा गया है।
(ii) आसके ऄतिररि, संतिधान ईच्चिम न्यायािय द्वारा तनरंिर व्याख्या के तिए खुिा है। संतिधान
ईच्चिम न्यायािय को ऐसी व्याख्याओं की ऄनुमति प्रदान करिा है जो:
 ईर्त्रोर्त्र समय और समकािीन िास्ितिकिा की प्रिृतर्त् के तिए प्रासंतगक हों।
 ऄतधकिम सीमा िक िोगों की अिश्यकिाओं और अकांक्षाओं को प्रतिसबतबि करिा हो।
पुनः तनम्नतितखि ईदाहरर् व्याख्या करिे हैं कक कै से नागररकों की अकांक्षाओं और समय की
अिश्यकिाओं को पूरा करने के तिए संिैधातनक प्रािधानों को जोडा, संशोतधि और पुनव्याणख्यातयि
ककया गया:

80 www.visionias.in ©Vision IAS

Google it:- https://upscpdf.com


https://upscpdf.com << Download From >> https://upscpdf.com

 तशक्षा का ऄतधकार (ऄनुच्छेद 21-A): तशक्षा के ऄतधकार को ईच्चिम न्यायािय द्वारा मोतहनी जैन
और ईन्नीकृ ष्र्न िाद में ऐतिहातसक तनर्णय के पश्चाि् भारि की संसद द्वारा पाररि 86िें संतिधान
संशोधन के माध्यम से सतम्मतिि ककया गया था।
 ईच्चिम न्यायािय ने ऄनुच्छेद 21 या ‘प्रार् और दैतहक स्ििंििा का संरक्षर्’ की सीमा को तनरंिर
तिस्िृि करके िोगों की अकांक्षाओं को पूरा करने िथा संतिधान को जीिंि बनाए रखने में
सहायिा की है। िषण 1978 के मेनका गांधी बनाम भारि संघ िाद के ऄपने तनर्णय के साथ अरंभ
करिे हुए ईच्चिम न्यायािय ने समय-समय पर िोगों के सामान्य जीिन को प्रभातिि करने िािे
मुद्दों के समाधान और समकातिक ऄन्य अिश्यकिाओं से तनपिने के तिए तितभन्न तनर्णय कदए हैं।
कदल्िी का CNG मामिा, गंगा नदी संरक्षर् मामिा, पयाणप्त अश्रय के ऄतधकार से संबंतधि
मामिा आत्याकद के संबंध में कदए गए तनर्णय आसके ईदाहरर् हैं।
आस प्रकार, संतिधान एक जीतिि जीि (जो ऄपने ऄनुभि और असपास के पयाणिरर् से सीखिे हैं) के
समान तनरंिर तिकतसि होिा है।
4. भारिीय संतिधान की प्रमुख तिशे ष िाएं (Salient
Features of the Constitution of India)
 संतिधान की प्रमुख तिशेषिाएं स्पष्ट रूप से दृतष्टगोचर होने िािे िक्षर् या तिशेष िथ्य होिे हैं।
भारिीय संतिधान आन्हीं तितशष्ट िक्षर्ों के माध्यम से तितभन्न िैचाररक तनयमों और मूल्यों के
मध्य समन्िय स्थातपि करिा है। ये सब तितशष्ट िक्षर् भारिीय स्ििंििा संग्राम के ऐतिहातसक
मूल्यों से प्रेररि रहे हैं, जो भारिीय संतिधान की सफििा को अगे बढ़ाने में महत्त्िपूर्ण भूतमका का
तनिणहन कर रहे हैं।
4.1. तितखि एिं सबसे तिस्िृ ि सं तिधान
 भारिीय संतिधान एक तितखि संतिधान है। आसके साथ ही यह तिश्व के सभी देशों के संतिधान की
िुिना में सबसे ऄतधक तिस्िृि भी है। मूि संतिधान में 395 ऄनुच्छेद एिं 8 ऄनुसूतचयां
सतम्मतिि थी, तजनमें संतिधान संशोधनों के माध्यम से कइ पररििणन ककए गए हैं। ऄप्रैि 2017
िक आसमें 25 भाग, 12 ऄनुसूतचयों और 5 पररतशष्टों सतहि 448 ऄनुच्छेद थे। िषण 1950 में
ऄतधतनयतमि होने के पश्चाि् संतिधान संशोधन हेिु 125 संतिधान संशोधन तिधेयक प्रस्िुि ककए
गए हैं िथा 103 संतिधान संशोधन कानून पाररि ककए जा चुके हैं।

तितखि और ऄतितखि संतिधान


 तितखि संतिधान तितधयों के रूप में तितधिि प्रिर्पिि तिधायी दस्िािेज होिा है। यह स्पष्ट,
तनतश्चि और व्यितस्थि होिा है। यह िोगों के सचेिन एिं सुतिचाररि प्रयासों का पररर्ाम होिा
है िथा आसे देश के राजनीतिक आतिहास की एक तितशष्ट ऄितध में िोगों द्वारा तितधिि रूप से
चयतनि एक प्रतितनतधत्ि तनकाय द्वारा तनर्पमि ककया जािा है। आसे भूिकाि में एक तनर्ददष्ट तितथ
को प्रख्यातपि ककया गया होिा है। एक तितखि संतिधान सामान्यिया ऄनम्य (rigid) होिा है
िथा आसके संशोधन या पुनरीक्षर् हेिु साधारर् कानून को ऄतधतनयतमि करने की प्रकक्रया से पृथक
एक प्रकक्रया होिी है ऄथाणि् संिैधातनक तितध एिं सामान्य तितध के मध्य एक तिभेद स्थातपि
ककया जािा है। संयुि राज्य ऄमेररका का संतिधान एक प्रतितनतधत्िकारी संतिधान सभा
(representative constituent assembly) द्वारा तनर्पमि प्रथम तितखि संतिधान है। फ्रांस में
भी तितखि संतिधान का ऄनुपािन ककया जािा है। ईल्िेखनीय है कक 19िीं शिाब्लदी के दौरान
ऄनेक देशों ने ऄपने-ऄपने तितखि संतिधान का तनमाणर् ककया था। के िि आंग्िैंि ही एक ऄपिाद
है, तजसने तितखि संतिधान का तनमाणर् नहीं ककया है। भारिीय संतिधान भी तितखि संतिधान का
एक ईदाहरर् है।

81 www.visionias.in ©Vision IAS

Google it:- https://upscpdf.com


https://upscpdf.com << Download From >> https://upscpdf.com

 ऄतितखि संतिधान में सरकार के ऄतधकांश तसद्ांिों को तितधयों के रूप में ऄतधतनयतमि नहीं
ककया जािा है। आसमें रीतियों, प्रथाओं, परंपराओं और तितभन्न तितथयों में िागू ककए गए तितखि
कानूनों को शातमि ककया जािा है। यह ऄव्यितस्थि एिं ऄतनतश्चि होिा और आसकी प्रकृ ति तिशुद्
नहीं होिी है। ऐसा संतिधान िोगों के सचेिन एिं सुतिचाररि प्रयासों का पररर्ाम नहीं होिा
बतल्क यह सामान्यिया ऐतिहातसक तिकास का पररर्ाम होिा है। आसे आतिहास के एक तनयि
चरर् पर एक प्रतितनतधत्िकारी संतिधान सभा द्वारा तनर्पमि नहीं ककया जािा िथा यह न ही एक
तितशष्ट तितथ पर प्रख्यातपि होिा है। आसतिए, आसे एक “तिकतसि ऄथिा संचयी संतिधान” कहा
जािा है। आंग्िैि का संतिधान ऄतितखि संतिधान का एक ईत्कृ ष्ट ईदाहरर् है , जो मुख्यिया
ऐतिहातसक तिकास का पररर्ाम है।
हािांकक, तितखि और ऄतितखि संतिधान में कोइ तितशष्ट तिभेद नहीं होिा है। ऐसा कोइ भी संतिधान
नहीं है जो पूर्णिया तितखि स्िरूप में हो। साथ ही, कोइ भी संतिधान पूर्णिया ऄतितखि नहीं होिा है।
प्रत्येक तितखि संतिधान में कु छ ऄतितखि ऄियि होिे हैं िथा आसी प्रकार प्रत्येक ऄतितखि संतिधान
के भी कु छ तितखि घिक होिे हैं।

महत्िपूर्ण सबदु
हािांकक, ऄनुच्छेद 395, संतिधान का ऄंतिम ऄनुच्छेद है, परन्िु भारिीय संतिधान में ऄनुच्छेदों की

कु ि संख्या 448 है। संशोधन के माध्यम से जोडे गए नए ऄनुच्छेद, मूि संतिधान के संबंतधि भाग में
सतम्मतिि कर कदए गए हैं। ऄनुच्छेदों के मूि क्रम में ऄत्यतधक पररििणन न करने के ईद्देश्य से नए
ऄनुच्छेद ऄल्फान्यूमेररक सूची के ऄनुसार सतम्मतिि ककए गए हैं। ईदाहरर् के तिए, तशक्षा के ऄतधकार

से संबंतधि ऄनुच्छेद 21A, को 86िें संशोधन ऄतधतनयम द्वारा संतिधान में जोडा गया था।

संतिधान के तिस्िृि होने के पीछे तितभन्न कारक तजम्मेदार हैं:


 सबसे प्रमुख कारकों में से एक यह है कक संतिधान तनमाणिाओं ने तितभन्न स्रोिों और तिश्व के कइ
ऄन्य संतिधानों से कइ ईपबंध ग्रहर् ककए, जैसे- संतिधान तनमाणिाओं ने प्रशासतनक तििरर् से

संबंतधि तिषयों से संबद् प्रािधानों के तनमाणर् हेिु भारि शासन ऄतधतनयम, 1935 (गिनणमेंि

ऑफ़ आंतिया एक्ि, 1935) का ऄनुसरर् ककया िथा ईसकी कइ तिशेषिाओं को बनाए रखा गया।

 दूसरा, भारि से संबंतधि तितशष्ट मुद्दों के तिए तितभन्न प्रािधानों को संतिधान में शातमि करना

अिश्यक था, जैसे- ऄनुसूतचि जाति, ऄनुसूतचि जनजाति और तपछडे क्षेिों से संबंतधि तितभन्न
प्रािधान।
 िीसरा, कें ि-राज्य संबंधों में ईनके प्रशासतनक, तिधायी एिं तिर्त्ीय संबंधों िथा ऄन्य गतितितधयों
के सभी महत्िपूर्ण पहिुओं के तिए तिस्िृि प्रािधान ककए गए हैं।
 चौथा, चूंकक भारिीय राज्यों के तिए पृथक संतिधान नहीं है, ऄिः राज्य प्रशासन से संबंतधि
प्रािधान भी भारिीय संतिधान में सतम्मतिि ककए गए हैं।
 पााँचिां, स्थानीय स्िशासी संस्थाओं से संबंतधि प्रािधान भी भारिीय संतिधान में सतम्मतिि ककए
गए हैं।
 आसके ऄतिररि, संतिधान को अम नागररकों के तिए सुस्पष्ट बनाने हेिु व्यतिगि ऄतधकारों,
राज्य की नीति के तनदेशक ित्िों की एक तिस्िृि सूची एिं प्रशासकीय प्रकक्रया की जानकारी
संतिधान में समातिष्ट की गयी है।

82 www.visionias.in ©Vision IAS

Google it:- https://upscpdf.com


https://upscpdf.com << Download From >> https://upscpdf.com

4.2. नम्यिा (िचीिापन) एिं ऄनम्यिा (कठोरिा) का समन्िय


 भारिीय संतिधान तिशुद् रूप से न िो कठोर या ऄनम्य है और न ही नम्य या िचीिा है। आसमें
कठोरिा और िचीिेपन का समािेश है। संतिधान के कु छ भागों को संसद के साधारर् कानून
बनाने की प्रकक्रया द्वारा संशोतधि ककया जा सकिा है। हािांकक, कु छ प्रािधानों में संशोधन िभी
ककया जा सकिा है, जब आस ईद्देश्य के तिए एक तिधेयक संसद के प्रत्येक सदन के कु ि सदस्यों के
बहुमि िथा सदन में ईपतस्थि एिं मिदान में भाग िेने िािे सदस्यों के कम से कम दो-तिहाइ
बहुमि से पाररि हो जािा है।
 साथ ही, कु छ ऄन्य ऐसे प्रािधान भी हैं जो ईपयुणि तितध द्वारा संशोतधि ककए जा सकिे हैं परन्िु
ईन्हें राष्ट्रपति के समक्ष स्िीकृ ति हेिु प्रस्िुि करने से पूिण कम से कम अधे राज्यों का ऄनुसमथणन
प्राप्त होना अिश्यक है। यह भी ध्यान कदया जाना चातहए कक संतिधान संशोधन तिधेयक प्रस्िुि
करने की शति के िि संसद में तनतहि है, राज्य तिधान-मंििों में नहीं।
 संतिधान सभा में पंतिि नेहरू के शब्लद: "यद्यतप हम संतिधान को आिना दृढ़ और स्थायी बनाना
चाहिे हैं तजिना हम बना सकिे हैं, िथातप संतिधान में कोइ स्थातयत्ि नहीं है। संतिधान में कु छ
िचीिापन होना चातहए। यकद अप कु छ भी कठोर और स्थायी बनािे हैं, िो अप राष्ट्र के तिकास,
जीिन के तिकास... अकद को रोक देिे हैं। ककसी भी तस्थति में, हम आस संतिधान को आिना कठोर
नहीं बना सकिे थे कक बदििी पररतस्थतियों के ऄनुसार आसका पािन नहीं ककया जा सके । जब
तिश्व में ऄशांति है और हम ऄत्यंि िीव्र संक्रमर् काि से गुजर रहे हैं, िब हम जो अज करिे है
संभििः िह भतिष्य में पूर्णिया िागू नहीं हो सकिा है।"
हािांकक, ईच्चिम न्यायािय ने मूि ढांचे के एक भाग के रूप में संतिधान संशोधन हेिु संसद की सीतमि
शतियााँ तनधाणररि की है। ऄन्य शब्लदों में, संसद, संतिधान के सभी भागों में संशोधन नहीं कर सकिी है।

4.3. िोकिां तिक गर्राज्य

 भारि एक िोकिांतिक गर्राज्य है। आसका िात्पयण यह है कक भारि की संप्रभुिा भारि के िोगों में
तनतहि है। भारि के नागररक सािणभौतमक ियस्क मिातधकार के अधार पर तनिाणतचि
प्रतितनतधयों के माध्यम से स्ियं को प्रशातसि करिे हैं। भारि के राष्ट्रपति (जो देश का सिोच्च
ऄतधकारी होिा है) को एक तनतश्चि समयाितध के तिए चुना जािा है।
 यद्यतप, भारि एक संप्रभु गर्राज्य है, िथातप आसकी राष्ट्रमंिि (कॉमनिेल्थ) की सदस्यिा जारी
है। राष्ट्रमंिि का प्रमुख तिरिश साम्राज्ञी हैं। राष्ट्रमंिि में भारि की सदस्यिा एक संप्रभु गर्राज्य
के रूप में ईसकी तस्थति को प्रभातिि नहीं करिी है क्योंकक राष्ट्रमंिि, मुि और स्ििंि राष्ट्रों की
एक संस्था है। तिरिश साम्राज्ञी आस संस्था की माि प्रिीकात्मक प्रमुख हैं।
क्या भारिीय संसद “संप्रभु” ऄथिा “गैर-संप्रभु” ऄथिा दोनों तिशेषिाओं से संपन्न एक तिधातयका है?
संसदीय संप्रभुिा की ऄिधारर्ा
 ईपयुणि तस्थति के सूक्ष्म तिभेदों का मूल्यांकन करने के तिए “संसदीय संप्रभुिा” के तनतहिाथों को
समझने की अिश्यकिा है।
 संसदीय संप्रभुिा: आसे संसदीय सिोच्चिा ऄथिा तिधायी सिोच्चिा के रूप में भी जाना जािा है।
यह संसद को सिणप्रमुख तिधायी प्रातधकरर् के रूप में स्थातपि करिी है, जो ककसी भी तितध का
तनमाणर् ऄथिा समापन कर सकिी है। आसके ऄतिररि, न्यायपातिका ऐसे तिधान को तनरस्ि नहीं
कर सकिी है िथा कोइ भी संसद ऐसा कानून पाररि नहीं कर सकिी, तजन्हें भािी संसद
पररिर्पिि न कर सके ।

83 www.visionias.in ©Vision IAS

Google it:- https://upscpdf.com


https://upscpdf.com << Download From >> https://upscpdf.com

 ससंदीय संप्रभुिा के समक्ष तनम्नतितखि रूप में कु छ बाधाएं तिद्यमान हैं:


o संिैधातनक सिोच्चिा का तसद्ांि;
o शतियों के पृथक्करर् का तसद्ांि (यह सामान्य तितध तनमाणर् हेिु तिधातयका के कायण क्षेि को
सीतमि करिा है); एिं
o न्यातयक पुनर्पििोकन का तसद्ांि (तिधातयका द्वारा पाररि तितधयों को कु छ पररतस्थतियों में
ऄमान्य ऄथिा शून्य घोतषि ककया जा सकिा है)।
 संसदीय संप्रभुिा िस्िुिः यूनाआिेि ककगिम (UK) के संतिधान का एक तसद्ांि है, जो UK की
संसद को UK में सिोच्च तिधायी प्रातधकरर् बनािा है।
क्या भारिीय संसद संप्रभु है?
 UK के तिपरीि भारिीय संसद पूर्णिया संप्रभु नहीं है, क्योंकक यह संतिधान के प्रािधानों के
ऄधीन है। आसका िात्पयण यह है कक भारिीय संसद भी ऄपनी शति और प्रातधकार संतिधान से ही
ग्रहर् करिी है।
 भारिीय ससंद की पूिण-तनधाणररि सीमाएं हैं, तजनका नीचे िर्णन ककया गया है:
o संसद के िि संघ सूची और समििी सूची में सूचीबद् मामिों के संबंध में ही कानून पाररि
कर सकिी है।
o संसद द्वारा तनर्पमि कानून ईच्चिम न्यायािय की न्यातयक पुनर्पििोकन की शति के ऄधीन हैं।
आसका ऄथण है कक यकद कोइ कानून संतिधान के प्रािधानों के प्रतिकू ि है िो आसे न्यायपातिका
द्वारा ऄमान्य घोतषि ककया जा सकिा है। आस प्रकार, भारि में UK के संसद की सिोच्चिा
तसद्ांि के तिपरीि संतिधान की सिोच्चिा तसद्ांि को ऄपनाया गया है।
4.4. सरकार का सं स दीय स्िरूप
 भारि ने तििेन द्वारा ऄपनाइ गयी िेस्िसमस्िर प्रर्ािी को ऄपनाया है। यह सरकार की एक
िोकिांतिक संसदीय प्रर्ािी है। आस प्रर्ािी में कायणकाररर्ी, तिधातयका के प्रति ईर्त्रदायी होिी
है। यह सर्त्ा में के िि िब िक बनी रहिी है जब िक आसे तिधातयका का तिश्वास प्राप्त होिा है।
 भारि का राष्ट्रपति नाममाि का संिैधातनक प्रमुख होिा है। कें िीय मंतिपररषद का गठन
तिधातयका से ही ककया जािा है। आसका प्रमुख प्रधानमंिी होिा है। कें िीय मंतिपररषद सामूतहक
रूप से िोकसभा के प्रति ईर्त्रदायी होिी है। यकद कें िीय मंतिपररषद सदन में बहुमि खो देिी है
िो ईसके द्वारा आस्िीफ़ा देना अिश्यक होिा है।
 राष्ट्रपति जो नाममाि का कायणकारी प्रमुख होिा है, कें िीय मंतिपररषद ऄथाणि् िास्ितिक
कायणकाररर्ी की सिाह के ऄनुसार ऄपनी शतियों का प्रयोग करिा है। राज्यों में भी सरकार का
स्िरूप संसदीय प्रर्ािी जैसा ही है।
4.5. सं घीय और एकात्मक तिशे ष िाओं का तमश्रर् (Mixture Of Federal And Unitary
Features)

भारिीय संतिधान के ऄनुच्छेद 1 के ऄनुसार: ‘भारि, ऄथाणि् आंतिया, राज्यों का संघ’ होगा। यद्यतप,
संतिधान में कही भी 'पररसंघ' (federation) शब्लद का प्रयोग नहीं ककया गया है, िथातप, भारि एक
संघीय गर्िंि (federal republic) है।
कोइ राज्य संघीय (federal) होिा है, यकद:
 सरकार दो स्िरों में तिभि होिी है िथा दोनों के मध्य शतियों का तििरर् होिा है;
 तितखि संतिधान होिा है, जो देश का सिोच्च कानून होिा है; िथा
 संतिधान की व्याख्या और कें ि एिं राज्यों के मध्य तििादों के समाधान के तिए एक स्ििंि
न्यायपातिका का प्रािधान होिा है।

84 www.visionias.in ©Vision IAS

Google it:- https://upscpdf.com


https://upscpdf.com << Download From >> https://upscpdf.com

भारिीय संतिधान में संघीय व्यिस्था के िक्षर् तनम्नतितखि हैं:


 ईपयुणि सभी संघीय तिशेषिाएाँ भारिीय संतिधान में तनतहि हैं। सरकार के दो स्िर तिद्यमान हैं ,
एक कें ि स्िर पर एिं दूसरा राज्य स्िर पर। आन दोनों के मध्य शतियों के तििरर् का तिस्िृि
तििरर् हमारे संतिधान में ककया गया है (73िें एिं 74िें संतिधान संशोधनों के पश्चाि् शतियों
को स्थानीय स्िर िक तिकें िीकृ ि ककया गया है)।
 भारि का संतिधान तितखि है और संतिधान ही देश का सिोच्च कानून है।
 एकि एकीकृ ि न्यातयक प्रर्ािी के शीषण पर, ईच्चिम न्यायािय तिद्यमान है, जो कायणपातिका और
तिधातयका के तनयंिर् से स्ििंि है।
भारिीय संतिधान में एकात्मकिा के िक्षर् तनम्नतितखि हैं:
 संघीय राज्य की आन सभी अिश्यक तिशेषिाओं के बािजूद, भारिीय संतिधान में कु छ एकात्मक
प्रिृतर्त्यां भी तिद्यमान हैं। ऄन्य संघ , जैसे- संयुि राज्य ऄमेररका में दोहरी नागररकिा का
प्रािधान ककया गया है जबकक भारि के संतिधान में एकि नागररकिा का प्रािधान है।
 सम्पूर्ण देश के तिए एकीकृ ि न्यायपातिका है।
 ऄतखि भारिीय सेिाओं, जैसे- भारिीय प्रशासतनक सेिा, भारिीय पुतिस सेिा एिं भारिीय िन
सेिा से संबंतधि प्रािधान, एक ऄन्य एकात्मक तिशेषिा प्रदर्पशि करिे हैं। आन सेिाओं के सदस्य,
संघ िोक सेिा अयोग द्वारा ऄतखि भारिीय अधार पर तनयुि ककए जािे हैं। चूंकक आन सेिाओं
पर कें ि सरकार का तनयंिर् होिा है, ऄि: कु छ सीमा िक ये राज्यों की स्िायर्त्िा में बाधा ईत्पन्न
करिी हैं।
 भारिीय संतिधान की एक महत्िपूर्ण एकात्मक तिशेषिा अपािकाि का प्रािधान है। अपािकाि
के दौरान कें ि सरकार और ऄतधक शतिशािी हो जािी है िथा संघीय संसद को राज्यों हेिु कानून
बनाने की शति प्राप्त हो जािी है।
 राज्यपाि राज्य का संिैधातनक प्रमुख होिा है। यह कें ि सरकार के एजेंि के रूप में कायण करिा है
और कें ि सरकार के तहिों की रक्षा करने के तनतमर्त् होिा है। ईपयुणि प्रािधान, हमारे संघ की
एकात्मक प्रिृतर्त् को प्रकि करिे हैं।
तनष्कषण:
 प्रोफे सर के . सी. व्हेयर के ऄनुसार, भारिीय संतिधान, "सरकार की एक ऄद्ण-संघीय प्रर्ािी िथा
सहायक एकात्मक तिशेषिाओं के साथ एक एकात्मक राज्य" का प्रािधान करिा है।
 संतिधान तनमाणिाओं ने स्पष्ट रूप से व्यि ककया था कक संघिाद और एकात्मकिा के मध्य
सामंजस्य तिद्यमान है। िॉ. ऄंबेिकर के ऄनुसार, “संतिधान में ऄपनाइ गइ राजनीतिक प्रर्ािी,
समय एिं पररतस्थतियों की अिश्यकिा के ऄनुसार एकात्मक के साथ-साथ संघीय हो सकिी है"।
 संयुि राज्य ऄमेररका िास्ितिक संघीय संतिधान को ऄपनाने िािा पहिा देश था। आसकी संघीय
संरचना को ििणमान में भी यह संदर्पभि करने हेिु प्रयुि ककया जािा है कक कोइ संतिधान संघीय है
या नहीं।
 भारिीय संतिधान का तनमाणर् तजन पररतस्थतियों में ककया गया था िे िषण 1787 में तनर्पमि
ऄमेररकी संतिधान की पररतस्थतियों से तबिकु ि तभन्न थीं। स्ििंििा के समय भारि दुखद
तिभाजन और देशभर में तिद्यमान तिभाजनकारी प्रिृतर्त्यों का साक्षी रहा है। आसतिए राज्य को
एक आकाइ के रूप में बनाए रखने िथा ऄंि में ऄपने िोगों को एकि राष्ट्र के ऄंिगणि एकीकृ ि रखने
के तिए एक सुदढ़ृ कें ि का तनमाणर् करना िात्कातिक अिश्यकिा थी।
 हािांकक, संतिधान में कु छ कें िीय प्रिृतर्त्यां भी तिद्यमान हैं, परन्िु भारिीय राज्यों में भी शति
और स्िायर्त्िा का एक ईतचि स्िर तनतहि है। भारिीय तितध अयोग के ऄनुसार, एक सुदढ़ृ संघ
और सुदढ़ृ राज्यों के मध्य कोइ तिरोधाभास नहीं होिा है।

85 www.visionias.in ©Vision IAS

Google it:- https://upscpdf.com


https://upscpdf.com << Download From >> https://upscpdf.com

 बोम्मइ िाद (1994) में ईच्चिम न्यायािय ने यह तनधाणररि ककया कक संतिधान की प्रकृ ति संघीय
(federal) है िथा न्यायािय ने संघिाद (federalism) को आसके “मूि ढांच”े के रूप में भी
तिशेषीकृ ि ककया। न्यायािय ने कहा कक कें ि में ऄत्यतधक शतियों के तनतहि होने का िात्पयण यह
नहीं है कक राज्य, कें ि के सहायक माि हैं। राज्यों का एक स्ििंि ऄतस्ित्ि है। िे कें ि के ऄनुगामी
या ऄतभकिाण माि नहीं हैं। ऄपने ऄतधकार क्षेि के भीिर प्रत्येक राज्य की तस्थति सिोच्च है।
संतिधान में संघिाद प्रशासतनक सुतिधा का मामिा नहीं है, परन्िु यह तसद्ांि का एक तिषय है।
ईपयुणि चचाण को संभििः प्रोफे सर ऄिेक्सेंिरोंतिज़ (Alexanderowicz) के शब्लदों में सिणश्रेष्ठ रूप
से संक्षेतपि ककया जा सकिा है, तजसमें ईन्होंने कहा है कक "भारि एक संघ है, परंिु यह एक
ऄतद्विीय (sui generis) संघ है, ऄथाणि् यह ऄपने ही प्रकार का संघ है।"
 तनष्कषण के रूप में यह कहा जा सकिा है कक भारि में कें िीय तनदेशन और राज्य ऄनुपािन के साथ
“सहकारी संघिाद” तिद्यमान है। हातिया समय में “प्रतिस्पधाणत्मक संघिाद” की ऄिधारर्ा
तिकतसि हुइ है, तजसके ऄंिगणि भारि के तिकास हेिु ऄपने संयुि प्रयासों में राज्य, कें ि के साथ
और कें ि, राज्यों के साथ िथा प्रत्येक राज्य एक-दूसरे के साथ प्रतिस्पधाण करिे हैं।

4.6. ऄसमतमि सं घ िाद (Asymmetric federalism)

 भारि के संतिधान में कु छ राज्यों के तिए तिशेष प्रािधान ककए गए हैं। जम्मू-कश्मीर राज्य को
ऄनुच्छेद 370 (हाि ही में ऄनुच्छेद 370 के ऄतधकांश प्रािधानों को समाप्त कर कदया गया है) के
ऄंिगणि तिशेष ऄतधकार प्रदान ककए गए थे। साथ ही भारिीय संतिधान के सभी ईपबन्ध जम्मू -
कश्मीर राज्य पर िागू नहीं होिे थे। ईल्िेखनीय है कक हाि ही में सरकार द्वारा ककए गए
प्रािधानों के ऄनुसार संतिधान के सभी प्रािधान ऄब जम्मू और कश्मीर पर भी िागू होंगे। आसके
ऄतिररि, महाराष्ट्र एिं गुजराि (ऄनुच्छेद 371), नागािैंि (ऄनुच्छेद 371-A), ऄसम (ऄनुच्छेद
371-B), मतर्पुर (ऄनुच्छेद 371-C), अंध्र प्रदेश (ऄनुच्छेद 371-D एिं 371-E), तसकक्कम
(ऄनुच्छेद 371-F), तमज़ोरम (ऄनुच्छेद 371-G), ऄरुर्ाचि प्रदेश (ऄनुच्छेद 371-H) और गोिा
(ऄनुच्छेद 371-I) के तिए भी तिशेष ईपबन्ध तितभन्न राज्यों की प्रादेतशक समस्याओं और मांगो के
कारर् ककए गए हैं। आन सब तिशेषिाओं के कारर् ही भारिीय संघिाद को ऄसमतमि संघिाद के
नाम से जाना जािा है।

4.7. मू ि ऄतधकार (Fundamental Rights)

 प्रो. एच. जे. िास्की के ऄनुसार "प्रत्येक राज्य को ईन ऄतधकारों से जाना जािा है तजन्हें िह
बनाए रखिा है। भारिीय संतिधान आस मूि तसद्ांि की पुतष्ट करिा है कक प्रत्येक व्यति को कु छ
अधारभूि ऄतधकार प्राप्त हैं। आन ऄतधकारों की चचाण संतिधान के भाग III में की गयी है। आन्हें मूि
ऄतधकारों के रूप में जाना जािा है। मूि रूप से संतिधान में 7 मूि ऄतधकारों की श्रेतर्यां थीं,
परन्िु ऄब यह संख्या 6 रह गयी है। ये हैं: (i) समिा का ऄतधकार (ii) स्ििंििा का ऄतधकार (iii)
शोषर् के तिरुद् ऄतधकार (iv) धमण की स्ििंििा का ऄतधकार (v) संस्कृ ति और तशक्षा संबंधी
ऄतधकार िथा (vi) सांतिधातनक ईपचारों का ऄतधकार। संपतर्त् का ऄतधकार (ऄनुच्छेद 31),
मूििः, एक मूि ऄतधकार था, तजसे 44िें संतिधान संशोधन ऄतधतनयम, 1978 द्वारा समाप्त कर
कदया गया। ििणमान में यह के िि एक तितधक ऄतधकार है।
 मूि ऄतधकार राज्य के नकारात्मक दातयत्िों के रूप में िर्पर्ि हैं और ये राज्य की सर्त्ा के तिरुद्
सीमाओं के रूप में कायण करिे हैं।

86 www.visionias.in ©Vision IAS

Google it:- https://upscpdf.com


https://upscpdf.com << Download From >> https://upscpdf.com

 मूि ऄतधकार प्रििणनीय (justiciable) हैं। आन ऄतधकारों में से ककसी एक का भी ऄतिक्रमर् होिा
है िो कोइ भी व्यति ईच्चर्त्र न्यायपतिका, ऄथाणि ईच्चिम न्यायािय और ईच्च न्यायािय में ऄपीि
दायर कर सकिा है। मूि ऄतधकारों के प्रििणन के तिए ऄनुच्छेद 32 (सांतिधातनक ईपचारों का
ऄतधकार) के िहि सीधे ईच्चिम न्यायािय में जाने के ऄतधकार की गारंिी प्रदान की गइ है।
हािांकक, भारि में मूि ऄतधकार ऄसीतमि नहीं हैं। राज्य और समाज की सुरक्षा िथा ऄन्य
अिश्यकिाओं को ध्यान में रखिे हुए आन पर युतियुि प्रतिबंध अरोतपि ककया जा सकिा है।

4.8. राज्य की नीति के तनदे श क ित्ि

 संतिधान की एक ऄतद्विीय तिशेषिा आसमें सतम्मतिि राज्य की नीति के तनदेशक ित्ि हैं। ये
तसद्ांि सरकार के तिए तनदेशात्मक प्रकृ ति के हैं, तजन्हें सरकार सामातजक और अर्पथक िोकिंि
की स्थापना के तिए िागू कर सकिी है।
 आनके ऄंिगणि जीतिका के तिए पयाणप्त साधन; पुरुषों और मतहिाओं को समान कायण के तिए समान
िेिन; िोकतहि के तिए धन का तििरर्; तन:शुल्क और ऄतनिायण प्राथतमक तशक्षा; कायण का
ऄतधकार; िृद्ािस्था, बेरोजगारी, बीमारी और कदव्यांगिा की तस्थति में सािणजतनक सहायिा;
ग्राम पंचायिों का गठन; अर्पथक रूप से तपछडे िगों के तिए तिशेष प्रािधान अकद महत्िपूर्ण
तनदेशक ित्ि सतम्मतिि हैं।
 आन तसद्ांिों में से ऄतधकांश भारि को एक कल्यार्कारी राज्य के रूप में स्थातपि करने में
सहायिा करिे हैं। हािांकक, ये प्रििणनीय नहीं हैं, कफर भी आन तसद्ांिों को “देश के शासन के तिए
अधारभूि” माना गया है।

4.9. मू ि कर्त्ण व्य (Fundamental Duties)

 42िें संतिधान संशोधन ऄतधतनयम, 1976 द्वारा संतिधान में राज्य की नीति के तनदेशक ित्िों के
पश्चाि् मूि कर्त्णव्यों से संबंतधि एक नया भाग IV(A) जोडा गया। संतिधान में मूि कर्त्णव्यों को
सतम्मतिि करने का ईद्देश्य नागररकों को यह स्मरर् कराना है कक तजस प्रकार नागररक ऄपने
ऄतधकारों का ईपयोग करिे हैं, ईसी प्रकार ईन्हें ऄपने कर्त्णव्यों का भी पािन करना चातहए
क्योंकक ऄतधकार और किणव्य सहसंबद् होिे हैं।

4.10. पं थ तनरपे क्ष राज्य

 एक पंथतनरपेक्ष राज्य न िो धार्पमक और न ही ऄधार्पमक या धमण तिरोधी होिा है। बतल्क यह धमण
के मामिों में ििस्थ होिा है। भारि में कइ धमों की तिद्यमानिा के कारर् संतिधान के संस्थापकों
ने आसे पंथतनरपेक्ष राज्य के रूप में स्थातपि करना ईतचि समझा।
 भारि एक पंथतनरपेक्ष राज्य है क्योंकक यह धमण के अधार पर िोगों के मध्य ककसी भी प्रकार का
भेदभाि नहीं करिा है। यह न िो ककसी तिशेष धमण को प्रोत्सातहि करिा है और न ही ककसी धमण
को हिोत्सातहि करिा है। आसके तिपरीि, संतिधान में धमण की स्ििंििा का ऄतधकार सुतनतश्चि
ककया गया है और ककसी भी धार्पमक समूह से संबंतधि िोगों को ईनकी पसंद के धमण को मानने,
अचरर् करने या प्रसार करने का ऄतधकार प्रदान ककया गया है।

4.11. स्ििं ि , तनष्पक्ष और एकीकृ ि न्यायपातिका

 हमारे संतिधान में न्यायपातिका को एक महत्िपूर्ण स्थान प्राप्त है िथा आसे तिधातयका और
कायणपातिका से स्ििंि भी रखा गया है। भारि का ईच्चिम न्यायािय एकि एकीकृ ि न्यातयक
प्रर्ािी के शीषण पर है।

87 www.visionias.in ©Vision IAS

Google it:- https://upscpdf.com


https://upscpdf.com << Download From >> https://upscpdf.com

 यह भारिीय नागररकों के मूि ऄतधकारों के रक्षक और संतिधान के संरक्षक (गार्पजयन) के रूप में
कायण करिा है।
 यकद तिधातयका द्वारा पाररि कोइ कानून या कायणपातिका द्वारा की गयी कोइ भी कायणिाही
संतिधान के प्रािधानों का ईल्िंघन करिी है िो ईसे ईच्चिम न्यायािय द्वारा ऄमान्य ऄथिा शून्य
घोतषि ककया जा सकिा है। आस प्रकार, ईच्चिम न्यायािय के पास न्यातयक पुनर्पििोकन की शति
है।

4.12. एकि नागररकिा

 भारि का संतिधान एकि नागररकिा को मान्यिा प्रदान करिा है। संयुि राज्य ऄमेररका में ,
दोहरी नागररकिा का प्रािधान है। भारि में हम के िि भारि के नागररक है, न कक ईस संबंतधि
राज्य के तजससे हम तनिास करिे हैं। यह प्रािधान राष्ट्र की एकिा और ऄखंििा को बढ़ािा देने में
सहायिा करिा है िथा तभन्न-तभन्न क्षेिों के िोगों के मध्य बंधुत्ि को बढ़ािा देिा है।
4.13. सािण भौतमक ियस्क मिातधकार (Universal Adult Franchise)

 भारि के संतिधान में सािणभौतमक ियस्क मिातधकार का ईल्िेख ककया गया है। सािणभौतमक
ियस्क मिातधकार के िहि सभी व्यस्क नागररकों को ईनके धमण , जाति, मूििंश, रंग और सिग के
अधार पर कोइ तिभेद ककए तबना चुनाि प्रकक्रया में भाग िेने का ऄतधकार प्रदान ककया गया है।
 मूि संतिधान में मिदान करने की अयु 21 िषण तनधाणररि की गयी थी, तजसे 61िें संतिधान
संशोधन ऄतधतनयम, 1989 द्वारा घिाकर 18 िषण कर ककया गया।

4.14. अपािकािीन शतियां

 भारिीय संतिधान में अपािकािीन शतियों का प्रािधान ककया गया है। ये प्रािधान देश को आसके
समक्ष ईपतस्थि ककसी भी अपाि तस्थति से तनपिने में सक्षम बनािे हैं।
 अपािकािीन शतियां भारि के राष्ट्रपति में तनतहि हैं। संतिधान में िीन प्रकार के अपािकाि का
ईल्िेख ककया गया है: राष्ट्रीय अपािकाि (ऄनुच्छेद 352); संिैधातनक िंि की तिफििा
(ऄनुच्छेद 356) और तिर्त्ीय अपािकाि (ऄनुच्छेद 360)।

4.15. शति का पृ थ क्करर् (Separation of Powers)

 शतियों के पृथक्करर् की संकल्पना के पीछे की मूि धारर्ा यह है कक शतियों का के न्िीकरर् जब


ककसी एक व्यति या व्यतियों के समूह में हो जािा है िो िे सरकारी मशीनरी का ईपयोग जनतहि
के स्थान पर व्यतिगि तहि में करने िगिे हैं। शतियों का पृथक्करर् ककसी भी व्यति या समूह में
सर्त्ा के संकेन्िर् को रोकने का एक िरीका है। आससे सर्त्ा का दुरुपयोग करना करठन हो जािा है।
 आसके ऄनुसार, राज्य सर्त्ा एकि आकाइ नहीं ऄतपिु तितभन्न राज्य तनकायों द्वारा एक-दूसरे से
स्ििंि रहकर ककए जाने िािे तितभन्न सरकारी कायों (ऄथाणि् तिधायी, कायणकारी और न्यातयक)
का संयोजन है। तिधातयका कानून का तनमाणर् करिी है; कायणपातिका कानून को िागू करिी है और
न्यायपातिका ईन कानूनों की व्याख्या करिी है।
 शति के पृथक्करर् का परंपरागि तिचार मोंिेस्क्यू द्वारा िषण 1748 में प्रकातशि ऄपनी पुस्िक “द
तस्पररि ऑफ द िॉ” (The Spirit of the Laws) में कदया गया है। आन्होंने सरकार के िीनों ऄंगों
ऄथाणि् कायणपातिका, तिधातयका और न्यायपातिका के मध्य शति और प्रकायों के कठोर एिं
तनरपेक्ष पृथक्करर् की िकािि की है। आन िीनों पृथक तनकायों के मध्य शतियों का तििरर् आस
प्रकार होना चातहए कक ईनके प्रकायण पूर्णिया पृथक हों और आनके प्रकायों में कोइ ऄतिव्यापन
(ओिरिेसपग) न हो।

88 www.visionias.in ©Vision IAS

Google it:- https://upscpdf.com


https://upscpdf.com << Download From >> https://upscpdf.com

 हािांकक, भारि का संतिधान शतियों के तिभाजन के तसद्ांि के संदभण में एक समकािीन


दृतष्टकोर् प्रस्िुि करिा है। हमारे संतिधान के िहि शतियों का कठोर पृथक्करर्, तसद्ांि और
व्यिहार दोनों में व्याप्त नहीं है। चूंकक संसदीय िोकिंिों में कायणपातिका या मंिीपररषद् (जैसे-
भारि या तििेन में) तिधातयका का भी भाग होिी है, आसतिए, यहााँ शति का कठोर पृथक्करर्
मौजूद नहीं हो सकिा है।
 िस्िुिः भारि के संतिधान ने शति के पृथक्करर् के तसद्ांि के साथ तनयंिर् और संिुिन के तसद्ांि
को ऄपनाया है। आस तसद्ांि के िहि सरकार के तितभन्न ऄंग (जैसे- तिधातयका, कायणपातिका,
न्यायपातिका) एक-दूसरे को तनयंतिि करिे हैं। आसतिए भारि में सरकार का प्रत्येक ऄंग ऄपने
कक्रयाकिापों के दौरान दूसरे ऄंगों के कायण -क्षेि में हस्िक्षेप नहीं करिा है। िेककन ठीक ईसी समय
यह सुतनतश्चि करना भी अिश्यक होिा है कक सरकार के दूसरे ऄंग शतियों का दुरुपयोग न करें
या ऄपने तिए तनधाणररि ऄतधदेश (मैंिेि) की सीमा का ऄतिक्रमर् न करें।
 आस सम्बन्ध में एक महत्िपूर्ण प्रश्न राज्य के आन िीनों ऄंगों के मध्य संबंध का है ऄथाणि क्या आन
िीन ऄंगों के मध्य शतियााँ पूर्ण रूप से पृथक होनी चातहए या ईनके मध्य समन्िय होना चातहए।
िॉ. दुगाण दास बसु के शब्लदों में,
 “जहााँ िक न्यायपातिका का संबंध है, आस तसद्ांि (शतियों का पृथक्करर् का तसद्ांि) के ऄनुप्रयोग
में दो प्रस्िाि तनतहि हो सकिे हैं:
o सरकार के िीनों ऄंगों, यथा- तिधातयका, कायणपातिका और न्यायपातिका में से कोइ भी एक
ऄंग समुतचि (properly) रूप से ऄन्य दोनों ऄंगों में से ककसी के भी ऄंिगणि अने िािी
शतियों का ईपयोग नहीं कर सकिा है।
o तिधातयका ऄपनी शतियों का प्रत्यायोजन (Delegation) नहीं कर सकिी है। ज्ञािव्य है कक
ककसी ईच्च ऄतधकारी द्वारा ऄधीनस्थ ऄतधकारी को तितशष्ट सर्त्ा एिं प्रातधकार प्रदान करना
प्रत्यायोजन कहिािा है।
 यहााँ महत्िपूर्ण “समुतचि” शब्लद है और आसतिए शतियों के व्यापक पृथक्करर् को प्रदर्पशि करिा है
जहााँ मुख्य (कोर) प्रकायण एक है जो तिशेष रूप से सरकार के ककसी ऄंग को प्रदर्त् है, यद्यतप िहां
कदए गए कु छ तिषयों के सीमाििी क्षेिों को िेकर कु छ व्यातप्त हो सकिी है। न्यायाियों के द्वारा
कानून के आन पहिुओं पर राय यह है कक भारि के संतिधान के िहि शतियों का व्यापक पृथक्करर् है।

4.16. स्ििं ि तनकाय

 भारिीय संतिधान सरकार (कें ि एिं राज्य) के के िि तिधायी, कायणकारी और न्यातयक ऄंगों का ही
प्रािधान नहीं करिा, ऄतपिु, यह तनिाणचन अयोग, तनयंिक एिं महािेखा परीक्षक और संघ िोक
सेिा अयोग जैसे स्ििंि तनकायों की स्थापना का भी प्रािधान करिा है। संतिधान में आनकी
पररकल्पना भारि में सरकार की िोकिांतिक प्रर्ािी के एक रक्षक के रूप में की गइ है।
o तनिाणचन अयोग: यह संसद, राज्य तिधान-मंििों, राष्ट्रपति िथा ईप-राष्ट्रपति के तनिाणचन
की व्यिस्था करिा है। आस संस्था को कायणपातिका के तनयंिर् से मुि रखने के तिए संतिधान
में कु छ प्रािधान ककए गए हैं।
o तनयंिक िथा महािेखा परीक्षक: यह संघ िथा राज्यों के तिर्त् एिं िेखाओं का परीक्षर्
करिा है और ईनको तनयंतिि करिा है। आसे भी संघीय और राज्यों की कायणपातिका के
तनयंिर् से मुि रखने के तिए संतिधान में व्यिस्था की गइ है।
o संघ और राज्य िोक सेिा अयोग: ये क्रमशः कें िीय और राज्य सरकारों की ईच्चिर सेिाओं के
तिए ऄभ्यर्पथयों की भिी हेिु परीक्षाओं का संचािन एिं ईनकी तनयुति की संस्िुतियााँ करिे हैं।

89 www.visionias.in ©Vision IAS

Google it:- https://upscpdf.com


https://upscpdf.com << Download From >> https://upscpdf.com

4.17. सरकार के िीन स्िर

 मूि रूप से, ऄन्य संघीय संतिधानों की भांति भारिीय संतिधान में भी एक द्वैध राजव्यिस्था का
प्रािधान ककया गया था िथा कें ि एिं राज्यों के संगठनों और शतियों से संबंतधि ईपबंधों को भी
शातमि ककया गया था। कािांिर में 73िें एिं 74िें संतिधान संशोधनों के माध्यम से पंचायिों एिं
नगरपातिकाओं के रूप में सरकार के िृिीय स्िर का सृजन ककया गया, जो तिश्व के ककसी भी
संतिधान में तिद्यमान नहीं है।
ईपयुणि भारिीय संतिधान की महत्िपूर्ण तिशेषिाएं हैं, जो आसे तिश्व का सिाणतधक ऄतद्विीय एिं
तितशष्ट संतिधान बनािी हैं।

5. संशोधन (Amendments)
5.1. भू तमका

 पररििणन ही प्रकृ ति का तनयम है। समय के साथ-साथ राजनीतिक समाज में पररििणन होिा रहा है।
नइ समस्याओं और चुनौतियों का सामना करने के तिए राष्ट्रीय जीिन के सभी अयामों में
पररििणन और ईपांिरर् अिश्यक हो जािा है। तजस प्रकार समाज की अिश्यकिाओं के ऄनुसार
तशक्षा पद्ति पररिर्पिि होिी है, औद्योतगक नीति में पररििणन होिा है, ईसी प्रकार तितधयों और
संतिधान में भी पररििणन की अिश्यकिा होिी है।
 यकद कोइ संतिधान, संशोधन का ऄतधकार नहीं देिा है िो आस बाि की संभािना बढ़ जािी है कक
नइ पीढ़ी ईसे नष्ट कर ईसके स्थान पर नया संतिधान स्थातपि कर देगी। ऄिः यह अिश्यक हो
जािा है कक संतिधान में पररििणन की प्रकक्रया को संतिधान में ही समातिष्ट कर कदया जाए।
भारिीय संतिधान में ये तिशेषिाएाँ तिद्यमान हैं, आसतिए यह एक जीिंि दस्िािेज कहिािा है।
 के शिानंद भारिी बनाम के रि राज्य िाद में ईच्चिम न्यायािय ने ‘संशोधन’ शब्लद के क्षेि तिस्िार
एिं पररभाषा का सबसे बेहिर तििरर् प्रस्िुि ककया। न्यायािय ने 'संशोधन' शब्लद की व्यापक
पररभाषा प्रस्िुि की है, जहााँ संशोधन शब्लद में ककसी भी पररििणन या संशोधन को शातमि ककया
जािा है।
o 'संशोधन' शब्लद जब संतिधान के संबंध में ईपयोग ककया जािा है, िो यह एक नए और स्ििंि
तिषय से संबंतधि प्रािधान को जोडने को संदर्पभि कर सकिा है। ये प्रािधान ऄपने अप में
पूर्ण हो सकिे हैं और ऄन्य प्रािधानों से पूर्ण रूप से पृथक हो सकिे हैं, या ककसी तिशेष
ऄनुच्छेद या ईपबंध के समान हो सकिे हैं, और ित्पश्चाि आसका ईपयोग आस तिशेष ऄनुच्छेद
या ईपबंध में पररिधणन, पररििणन या तनरसन को संदर्पभि करने हेिु ककया जा सकिा है।

5.2. सं िै धातनक प्रािधानों के सं शोधन की अि श्यकिा

 संतिधान एक जीिंि और पररििणनशीि दस्िािेज़ है। समकािीन पररििणनों के ऄनुकूि संतिधान


में पररििणन ऄतिअिश्यक होिे हैं। आसका संतिधान के मूि दशणन में भी ईल्िेख ककया गया है।
संतिधान को बदििी पररतस्थतियों और अिश्यकिाओं के ऄनुसार समायोतजि करने के तिए
संशोधन की ऄनुमति प्रदान करना ऄत्यािश्यक है।

5.3. सं शोधन के प्रकार

 ऄदृश्य या ऄनौपचाररक; एिं


 दृश्य या औपचाररक।

90 www.visionias.in ©Vision IAS

Google it:- https://upscpdf.com


https://upscpdf.com << Download From >> https://upscpdf.com

5.3.1. ऄदृ श्य या ऄनौपचाररक प्रकक्रया


 संतिधान का संशोधन करने की ऄदृश्य या ऄनौपचाररक प्रकक्रया की भूतमका और दायरा सीतमि
होिा है। आस प्रकार के पररििणन तनम्नतितखि िरीके से ककए जािे हैं:
o न्यायािय द्वारा तनिणचन के माध्यम से;
o तिधायन द्वारा संतिधान की ऄनुपूर्पि करके या ररि स्थानों को पूररि करके ; िथा
o ऄतभसमय और संिैधातनक परंपराओं में पररििणन करके ।
 न्यातयक तनिणचन द्वारा ककए गए पररििणन संतिधान के पाठ को तबना बदिे ईसे एक नया या
पररिर्पिि ऄथण प्रदान करिे हैं। यहााँ संतिधान की भाषा नहीं बदििी ककिु तिद्यमान पररतस्थतियों
और समाज की अिश्यकिाओं को दृतष्ट में रखिे हुए न्यायािय नया ऄथण प्रदान करिा है। न्यातयक
तनिणचन की ईन संतिधानों में बहुि महत्िपूर्ण भूतमका होिी है जहां संशोधन की प्रकक्रया कठोर
और करठन है।
 कइ बार ऄतभसमय के कारर् संतिधान के ईपबंध तनष्प्रभािी हो जािे हैं। ऄतभसमय का प्रििणन
संतिधान की सीमा के भीिर होिा है, कफर भी िह संतिधान को प्रभातिि और ईपांिररि कर देिा है।

5.3.2. दृश्य या औपचाररक प्रकक्रया


 प्रत्येक संतिधान में यह ईल्िेख होिा है कक ककस रीति से ईसे पररिर्पिि ककया जा सके गा। संशोधन
द्वारा संतिधान के पाठ को पररिर्पिि ककया जािा है तजससे कक समाज में हुए पररििणन के तिए या
राष्ट्र के तिकास के तिए ऄपेतक्षि नया ऄथण संतिधान में प्रतिसबतबि हो सके ।
 ऐसे संशोधन की प्रकक्रया औपचाररक होिी है। यह संशोधन की एक घोतषि और स्पष्ट प्रकक्रया है।
संतिधान को नइ समस्याओं का सामना करने के तिए ऄनुकूि बनाने हेिु यह सिाणतधक स्िीकृ ि
मागण है। भारि में औपचाररक संशोधन की प्रकक्रया को मुख्यिः िीन भागों में बांिा जािा है। तजसे
नीचे सतिस्िार िर्पर्ि ककया गया है।
 संशोधन की प्रकक्रया ने संतिधान को न ही पूर्णिः कठोर और न ही पूर्णिः िचीिा बनाया है,
यद्यतप यह दोनों का तमश्रर् है। कु छ ईपबंधों को सरििा से संशोतधि ककया जा सकिा है और कु छ
के तिए तितशष्ट प्रकक्रया का ऄनुकरर् ककया जािा है। भारि के एक संघीय राज्य होने के बािजूद ,
संतिधान संशोधन का प्रस्िाि के िि संसद के ककसी एक सदन में ही िाया जा सकिा है, राज्य
तिधानमंििों के पास ऐसी कोइ शति नहीं है।
 साधारर् तिधेयक के मामिे में यकद संसद के दोनों सदनों में सहमति नहीं होिी है, िो संयुि बैठक
का प्रािधान है। परन्िु संतिधान संशोधन तिधेयक के मामिे में जब िक प्रत्येक सदन में सहमति
नहीं बनिी है, िब िक तिधेयक पाररि नहीं हो सकिा है, क्योंकक आस मामिे में संसद की संयुि
बैठक का प्रािधान नहीं है।
संतिधान को तनम्नतितखि िीन प्रकार से संशोतधि ककया जा सकिा है:
(i) साधारर् बहुमि द्वारा संशोधन
संतिधान के ऄनेक ऐसे ईपबंध हैं, तजनके संशोधन हेिु संसद के दोनों सदनों में के िि साधारर् बहुमि
की अिश्यकिा होिी है। आन्हें दो िगों में बांि सकिे हैं:
 जहााँ संतिधान के मूि पाठ में पररििणन नहीं होिा है ककिु तितध में पररििणन हो जािा है:
o ऄनुच्छेद 11 संसद को नागररकिा के संबंध में तितध ऄतधतनयतमि करने की शति प्रदान
करिा है। आस शति के ऄनुसरर् में जो ऄतधतनयम तनर्पमि ककया जाएगा, िह नागररकिा से
संबंतधि तितध को पररिर्पिि कर देगा ककिु ऄनुच्छेद 5 से 10 िक के ऄनुच्छेद यथािि रहेंगे।
o ऄनुच्छेद 124 में ईल्िेख है कक ईच्चिम न्यायािय, मुख्य न्यायमूर्पि और साि से ऄनतधक
न्यायाधीशों से तमिकर बनेगा। ककिु संसद ने न्यायाधीशों की संख्या 7 से बढ़ाकर 34 कर दी है।

91 www.visionias.in ©Vision IAS

Google it:- https://upscpdf.com


https://upscpdf.com << Download From >> https://upscpdf.com

 जहां संतिधान का मूि पाठ पररिर्पिि हो जािा है:


o नए राज्यों का गठन, ऄनुसूची 1 और 4 का संशोधन अकद साधारर् तितध द्वारा ककए जा
सकिे हैं। संसद तितध बनाकर पांचिीं और छठी ऄनुसूची को संशोतधि कर सकिी है।
o जो ईपबंध साधारर् तितध द्वारा पररिर्पिि ककए जा सकिे हैं ईनमें (ईपयुणि िर्पर्ि के
ऄतिररि) शातमि हैं: तिधान पररषदों का सृजन और ईत्सादन; संघ राज्य क्षेिों के तिए
मंतिपररषद् का सृजन; ऄनुच्छेद 343 में ऄंग्रेजी भाषा के प्रयोग के तिए 15 िषण की ऄितध
का तिस्िार; संसदीय तिशेषातधकारों को पररतनश्चि करना; राष्ट्रपति, ईपराष्ट्रपति,
न्यायाधीशों अकद के िेिन और भर्त्े अकद।
(ii) तिशेष बहुमि द्वारा संशोधन
 आस संशोधन प्रकक्रया में प्रत्येक सदन के सदस्यों की कु ि संख्या का बहुमि िथा ईस सदन में
ईपतस्थि और मिदान करने िािे सदस्यों के कम से कम दो-तिहाइ बहुमि की अिश्यकिा होिी
है। प्रथम श्रेर्ी (साधारर् बहुमि से संशोतधि होने िािे ईपबंधों) और िृिीय श्रेर्ी (तिशेष बहुमि
के साथ-साथ कम से कम भारि के अधे राज्यों के तिधानमंििों द्वारा संशोतधि होने िािे ईपबंधों)
में शातमि ऄनुच्छेदों के ऄतिररि ऄन्य सभी ऄनुच्छेद ऐसे हैं, तजन्हें संसद तिशेष बहुमि द्वारा ही
संशोतधि कर सकिी है।

नोि: ‘कु ि सदस्यिा’ का िात्पयण सदन के कु ि सदस्यों की संख्या से है, भिे ही सदन में ररतियां हों
या सदस्य ऄनुपतस्थि हैं। हािांकक, राष्ट्रपति के महातभयोग की प्रकक्रया एकमाि ऄपिाद है, जहााँ
संकल्प को प्रत्येक सदन की कु ि सदस्यिा के दो-तिहाइ बहुमि से पाररि ककया जाना चातहए।

(iii) तिशेष बहुमि िथा कम से कम अधे राज्य तिधान मंििों की स्िीकृ ति द्वारा संशोधन
 आस प्रकक्रया के िहि संतिधान के कु छ तितशष्ट ऄनुच्छेद शातमि हैं, तजन्हें संशोतधि करने हेिु करठन
प्रकक्रया ऄपनायी जािी है। आन ऄनुच्छेदों में संशोधन करने के तिए संसद के तिशेष बहुमि के साथ-
साथ भारि के कम से कम अधे राज्यों के तिधानमंििों की स्िीकृ ति अिश्यक होिी है।
 यह ध्यान कदया जाना चातहए कक सभी राज्यों की भागीदारी अिश्यक नहीं है। जैसे ही अधे राज्य
ऄपनी सहमति (ईपयुणि तितध के माध्यम से) प्रदान करिे हैं, प्रकक्रया पूर्ण हो जािी है।
 आस हेिु राज्यों को ऄपनी सहमति प्रदान करने के तिए कोइ समय सीमा का प्रािधान नहीं है।
 आस श्रेर्ी में तनम्नतितखि ऄनुच्छेद सतम्मतिि हैं:

o ऄनुच्छेद 54- राष्ट्रपति का तनिाणचन

o ऄनुच्छेद 55- राष्ट्रपति के तनिाणचन की तितध


o ऄनुच्छेद 73–संघ की कायणपातिका शति का तिस्िार

o ऄनुच्छेद 162- राज्यों की कायणपातिका शति का तिस्िार

o ऄनुच्छेद 241- संघ राज्य क्षेिों के तिए ईच्च न्यायािय


o संघीय न्यायपातिका (भाग-5 ऄध्याय-4)

o राज्यों के तिए ईच्च न्यायािय (भाग-VI ऄध्याय-V)

o संघ-राज्य-संबंध (तिधायी) (भाग-XI ऄध्याय-I)


o साििीं ऄनुसूची का कोइ भी तिषय
o संसद में राज्यों का प्रतितनतधत्ि
o संतिधान-संशोधन से संबंतधि ऄनुच्छेद-368

92 www.visionias.in ©Vision IAS

Google it:- https://upscpdf.com


https://upscpdf.com << Download From >> https://upscpdf.com

5.4. मू ि ऄतधकारों का सं शोधन

 मूि ऄतधकारों में संशोधन ऄत्यतधक तििादास्पद तिषय रहा है, आसके पररर्ामस्िरूप
न्यायपातिका और संसद दोनों के मि में समय-समय पर पररििणन देखा गया है। आस संबंध में कु छ
महत्िपूर्ण प्रश्न तनम्नतितखि हैं:
o मूि ऄतधकार संशोधनीय हैं या नहीं, ऄथाणि् क्या संसद संतिधान द्वारा प्रदर्त् मूि ऄतधकार
को समाप्त कर सकिी है?
o संतिधान संशोधन के संदभण में संसद का ऄतधकार, ईसकी सीमा और तिस्िार क्या है?
 ईच्चिम न्यायािय द्वारा समय-समय पर आस संदभण में िकण प्रस्िुि ककए गए हैं। आसे तनम्नतितखि
मामिों िथा पररर्ामी संसदीय प्रतिकक्रया के अिोक में समझा जा सकिा है:
5.4.1. शं क री प्रसाद बनाम भारि सं घ िाद, 1951

 िषण 1951 में, संतिधान के प्रिृर्त् होने के एक िषण के भीिर, प्रथम संतिधान संशोधन ऄतधतनयम
पाररि ककया गया। आस ऄतधतनयम से ऄनुच्छेद 31 द्वारा प्रत्याभूि संपतर्त् के ऄतधकार को सीतमि
ककया गया। आस संशोधन की संिैधातनकिा पर शंकरी प्रसाद िाद में प्रश्नतचन्ह िगाया गया।
 यातचकाकिाण ने यह िकण प्रस्िुि ककया कक ऄनुच्छेद 13(2) राज्य को ऐसी तितध तनमाणर् से
प्रतितषद् करिा है जो मूि ऄतधकार को समाप्त या न्यून करिी है।
 यातचकाकिाण का िकण था कक ऄनुच्छेद 13(2) में प्रयुि शब्लद ‘तितध’ के ऄधीन सभी ऄतधतनयम
अिे हैं ऄथाणि् ईसमें संतिधान संशोधन ऄतधतनयम भी सतम्मतिि हैं। आस िकण को ऄस्िीकार करिे
हुए ईच्चिम न्यायािय ने यह ऄतभतनधाणररि ककया कक ऄनुच्छेद 368 द्वारा प्रदर्त् शति का प्रयोग
करिे हुए जो संतिधान संशोधन ऄतधतनयम पाररि ककया जािा है, िह ऄनुच्छेद 13(2) के ऄंिगणि
तितध नहीं है। न्यायािय ने यह व्यिस्था दी कक आस ऄनुच्छेद में तितध का ऄथण है- सामान्य तितध,
संतिधान संशोधन ऄतधतनयम नहीं (ऄथाणि् सांतिधातनक तितध नहीं)।
 ऄनुच्छेद 368 के प्रािधानों के ऄंिगणि संसद, मूि ऄतधकारों में संशोधन कर सकिी है। यकद दूसरे
शब्लदों में कहा जाए िो यह कहना ईतचि होगा कक साधारर् तितधयों के माध्यम से मूि ऄतधकारों
का संशोधन नहीं ककया जा सकिा है ककिु संिैधातनक तितधयों द्वारा ककया जा सकिा है।
5.4.2. सज्जन ससह बनाम राजस्थान राज्य िाद, 1965

 17िें संतिधान संशोधन ऄतधतनयम, 1964 की िैधिा को ईसी अधार पर चुनौिी दी गइ, जैसे
शंकरी प्रसाद मामिे में दी गइ थी।
 सज्जन ससह िाद में ईच्चिम न्यायािय ऄपने पूिणििी शंकरी प्रसाद िािे तनर्णय पर दृढ़ रहा।
5.4.3. गोिखनाथ बनाम पं जाब राज्य िाद, 1967

 िषण 1967 में न्यायािय ने गोिकनाथ िाद में ऄपने पूिणििी तितनश्चयों को ईिि कदया। बहुमि ने
यह दृतष्टकोर् ऄपनाया कक संतिधान में मूि ऄतधकारों को एक सिोच्च तस्थति प्रदान की गयी है।
 ऄनुच्छेद 368 के ऄधीन कायण करिे हुए संसद या ककसी ऄन्य प्रातधकारी को यह शति नहीं है कक
िह मूि ऄतधकारों को न्यून या समाप्त कर सके । न्यायािय ने आस संदभण में संसद की तिधायी शति
और संतिधान संशोधन करने की शति के मध्य भेद करने से ऄस्िीकृ ि कर कदया।
 यह तनर्णय 11 न्यायाधीशों की पीठ द्वारा कदया गया था। 6 न्यायाधीश बहुमि में थे और 5
ऄल्पमि में। ऄल्पमि िािे न्यायाधीशों ने यह माना कक मूि ऄतधकारों के संशोधन से संबंतधि पूिण
मि सही है ऄथाणि् मूि ऄतधकारों का संशोधन ककया जा सकिा है।

93 www.visionias.in ©Vision IAS

Google it:- https://upscpdf.com


https://upscpdf.com << Download From >> https://upscpdf.com

गोिकनाथ िाद में प्रदर्त् तनर्णय की प्रतिकक्रया स्िरूप संसद ने 24िां संतिधान संशोधन ऄतधतनयम,
1971 पाररि ककया। आस ऄतधतनयम द्वारा ऄनुच्छेद 13 में खंि (4) और ऄनुच्छेद 368 में एक नया
खंि (1) ऄंिःस्थातपि ककया गया। आस संशोधन द्वारा यह घोषर्ा की गइ कक ऄनुच्छेद 368 के ऄनुसार
पाररि संतिधान का संशोधन ऄनुच्छेद 13 के ऄंिगणि तितध नहीं होगी। आस प्रकार, ऄनुच्छेद 13
संतिधान का संशोधन करने िािे ऄतधतनयमों पर िागू नहीं होगा।

 24िां संतिधान संशोधन ऄतधतनयम (1971): ईच्चिम न्यायािय द्वारा गोिकनाथ िाद में कदए
तनर्णय के प्रतिकक्रयास्िरूप यह संशोधन ऄतधतनयम पाररि ककया गया। संसद द्वारा पाररि 24िें
संशोधन ऄतधतनयम ने ऄनुच्छेद 368 के पुराने शीषणक - "संतिधान संशोधन के तिए प्रकक्रया" - को
पररिर्पिि कर आसे एक निीन शीषणक - "संतिधान का संशोधन करने की संसद की शति और ईसके
तिए प्रकक्रया" - प्रदान ककया।
o तितध तनमाणिाओं की सुतिधा (ऄथिा ईन्हें िाभ की तस्थति प्रदान करने) के तिए आस
ऄतधतनयम ने संसद के संतिधान में संशोधन करने की शति को पुन: स्थातपि ककया िथा
आसके (संसद) दायरे का तिस्िार ककया। आस हेिु तनम्नतितखि शब्लदों को जोडा गया “....संसद
ऄपनी संतिधायी शति का प्रयोग करिे हुए आस संतिधान के ककसी ईपबंध का पररिधणन,
पररििणन या तनरसन के रूप में संशोधन आस ऄनुच्छेद में ऄतधकतथि प्रकक्रया के ऄनुसार कर
सके गी"।
 आसके ऄतिररि, आस संशोधन के माध्यम ऄनुच्छेद 368 में एक ऄन्य खंि 3 को जोड कर यह
प्रािधान ककया गया कक "ऄनुच्छेद 13 की कोइ बाि आस ऄनुच्छेद के ऄधीन ककए गए ककसी भी
संशोधन को िागू नहीं होंगी"।

5.4.4. के शिानं द भारिी बनाम के रि राज्य िाद, 1973

 िषण 1973 में के शिानंद भारिी िाद में यह तिषय पुन: ईच्चिम न्यायािय के समक्ष प्रस्िुि हुअ।
आस मामिे में ईठाए गए तितभन्न प्रश्नों में से एक, ऄनुच्छेद 368 के िहि संसद के संतिधान
संशोधन की शति की सीमा का प्रश्न था। ईच्चिम न्यायािय के ित्कािीन मुख्य न्यायाधीश, एस.
एम. तसकरी की ऄध्यक्षिा में 13 न्यायाधीशों की एक संिैधातनक पीठ द्वारा आस मामिे पर तिचार
ककया गया। बहुमि िािे न्यायाधीशों (7 न्यायाधीश) ने 24िें संतिधान संशोधन ऄतधतनयम को
तितधमान्य ठहरािे हुए गोिकनाथ िाद में कदए तनर्णय को ईिि कदया। ककिु, साथ ही एक नया
तसद्ांि भी प्रतिपाकदि ककया।
 न्यायािय ने यह कहा कक संसद मूि ऄतधकारों िािे भाग का संशोधन करने के तिए ईिनी ही
सक्षम है तजिनी कक संतिधान के ककसी ऄन्य भाग का। ककिु संतिधान का संशोधन करके संसद,
संतिधान के मूि ढांचे (अधाररक संरचना/अधारभूि िक्षर्) (Basic Structure) को न िो
संतक्षप्त कर सकिी है, न ही समाप्त कर सकिी है और न नष्ट कर सकिी है।
 गोिकनाथ िाद के पश्चाि् ककसी भी मूि ऄतधकार को न िो समाप्त ककया जा सकिा था और न ही
न्यून ककया जा सकिा था। के शिानंद िाद के पश्चाि् न्यायािय को यह तितनश्चय करना है कक कोइ
मूि ऄतधकार मूि ढ़ाचे का भाग है या नहीं। यकद िह मूि ढ़ाचे का भाग है िो ईसे हिाया नहीं जा
सकिा।

गोिकनाथ िाद में ईच्चिम न्यायािय ने शंकरी प्रसाद िाद और सज्जन ससह िाद में कदए तनर्णयों को
ईिि कदया। अगे, के शिानंद िाद में गोिकनाथ िाद में कदए तनर्णय को पिि कदया गया।

94 www.visionias.in ©Vision IAS

Google it:- https://upscpdf.com


https://upscpdf.com << Download From >> https://upscpdf.com

5.4.5. 42िां सं तिधान सं शोधन ऄतधतनयम, 1976

 संसद ने ऄनुच्छेद 368 में संशोधन कर ईपयुणि तनर्णय पर पुनः प्रतिकक्रया व्यि की। यह घोषर्ा
की गइ कक संसद की संतिधान संशोधन करने की संिैधातनक शति पर ककसी प्रकार का तनबंधन
नहीं है और संतिधान का कोइ भी संशोधन ककसी भी न्यायािय में ककसी भी अधार (मूि
ऄतधकारों में से ककसी के ईल्िंघन सतहि) पर प्रश्नगि नहीं ककया जा सकिा है।
 आस प्रकार न्यायपातिका ने जो मूि ढांचे का तसद्ांि (के शिानंद भारिी िाद) प्रस्िुि ककया था ईसे
तनरस्ि करने के तिए 42िें संतिधान संशोधन ऄतधतनयम, 1976 द्वारा ऄनुच्छेद 368 में खंि (4)
ऄंिःस्थातपि ककया गया। आस खंि का ईद्देश्य न्यातयक पुनर्पििोकन की शति पर ऄंकुश िगाना था।
 आस खंि से यह ऄतधतनयतमि ककया गया कक संतिधान के (तजसके ऄंिगणि भाग 3 के ईपबंध
शातमि हैं) आस ऄनुच्छेद के ऄधीन ककया गया कोइ संशोधन ककसी न्यायािय में ककसी भी अधार
पर प्रश्नगि नहीं ककया जाएगा।
5.4.6. तमनिाण तमल्स िाद, 1980

 ईच्चिम न्यायािय ने तमनिाण तमल्स िाद में यह ऄतभतनधाणररि ककया कक 42िें संतिधान संशोधन
द्वारा ऄनुच्छेद 31(C) में ककया गया संशोधन ऄतितधमान्य है, क्योंकक यह संतिधान के मूि ढांचे
को क्षति पहुाँचिा है।
 ऄनुच्छेद 368 का खंि 4 और 5 ऄतितधमान्य हैं क्योंकक िे संतिधान के मूि ढांचे में शातमि दो
िक्षर्ों का ईल्िंघन करिे हैं। ये िक्षर् हैं- संशोधन करने की शति का सीतमि होना और न्यातयक
पुनर्पििोकन। न्यायाियों को न्यातयक पुनर्पििोकन की शति से िंतचि नहीं ककया जा सकिा है।
5.4.7. िामन राि बनाम भारि सं घ िाद, 1981

 आस मामिे में न्यायािय ने यह भी स्पष्ट कर कदया कक मूि ढांचे का तसद्ांि 24-4-1973


(के शिानंद भारिी िाद में तनर्णय सुनाए जाने की िारीख) के पश्चाि् पाररि संतिधान संशोधन
ऄतधतनयमों पर िागू होगा ऄथाणि् भतिष्यिक्षी रूप से िागू होगा। यह पूिणििी तिधान पर ऄथाणि्
भूििक्षी रूप से िागू नहीं होगा।
5.5. ईपयुण ि के अधार पर तनष्कषण
आन संशोधनों और तितनश्चयों का पररर्ाम यह है कक:
 संसद मूि ऄतधकारों सतहि संतिधान के ककसी भी भाग का संशोधन कर सकिी है।
 प्रत्येक मामिे में न्यायािय यह तिचार कर सकिा है कक क्या मूि ऄतधकारों के संशोधन से
संतिधान के ककसी अधाररक िक्षर् (मूि ढांचे) का तनराकरर् या ईत्सादन या समापन हो रहा है।
यकद ‘हााँ’ िो संशोधन ईस सीमा िक शून्य होगा।
 हािांकक, ईच्चिम न्यायािय द्वारा भारिीय संतिधान के 'मूि ढांचे' की सभी तिशेषिाओं को
पररभातषि करना और ईसकी स्पष्ट व्याख्या करना ऄभी शेष है।
5.6. सं तिधान सं शोधन की क्रमिार प्रकक्रया

ऄनुच्छेद 368 में संतिधान संशोधन हेिु ऄनुसरर् की जाने िािी प्रकक्रयाओं का िर्ण न है। ये
तनम्नतितखि हैं:
 संशोधन का प्रारंभ ईस प्रयोजन के तिए तिधेयक पुरः स्थातपि करके ककया जािा है।
 तिधेयक संसद के ककसी भी सदन में प्रस्िुि ककया जा सकिा है।
 तिधेयक को प्रत्येक सदन द्वारा अिश्यकिानुसार साधारर् या तिशेष बहुमि द्वारा पाररि ककया
जाना चातहए।
 कु छ तिशेष सुरतक्षि ईपबंधों की दशा में तिधेयक का कम से कम अधे राज्य तिधान-मंििों द्वारा
ऄनुसमथणन ककया जाना चातहए।

95 www.visionias.in ©Vision IAS

Google it:- https://upscpdf.com


https://upscpdf.com << Download From >> https://upscpdf.com

 आस प्रकार सम्यक् रूप से जब तिधेयक पाररि कर कदया जािा है और जहााँ ऄपेक्षा है िहां
ऄनुसमर्पथि कर कदया जािा है, िब ईसे राष्ट्रपति को प्रस्िुि ककया जािा है।
 राष्ट्रपति स्िीकृ ति देने हेिु बाध्य है। साधारर् तिधेयक की तस्थति में राष्ट्रपति आसे पुनर्पिचार के
तिए िौिा सकिा है या ऄनुमति रोक कर सकिा है।
 यकद दोनों सदनों के मध्य ऄसहमति होिी है िो आस संबंध में संयि
ु बैठक का कोइ ईपबंध नहीं है।
तिधेयक प्रत्येक सदन द्वारा पृथक रूप से पाररि ककया जाना चातहए।
 संतिधान संशोधन तिधेयक को प्रस्िुि करने से पूिण राष्ट्रपति की पूिण स्िीकृ ति की अिश्यकिा नहीं
होिी है।

5.7. सं तिधान सं शोधन की प्रकक्रया की अिोचना

 भारि में संतिधान संशोधन के तिए कोइ तिशेष तनकाय नहीं है।
 संयुि राज्य ऄमेररका {जहााँ पर एक तिशेष संस्था (एमेंिमेंि कन्िेंशन) स्थातपि है} की िुिना में
भारि में आस प्रकार की कोइ संस्था नहीं है। ऄिः संतिधान को कइ बार राजनीतिक स्िाथण और
ईद्देश्य की प्रातप्त के तिए भी संशोतधि ककया गया है।
 राज्य तिधान-मंिि, संतिधान संशोधन तिधेयक को प्रारंभ नहीं कर सकिे (ऄमेररका के तिपरीि)।
यह भारि के संघीय संरचना के अधार पर की जाने िािी अिोचना है। ईपयुणि सबदु में भी एक
ऄपिाद है (राज्य तिधान-मंिि राज्य तिधान पररषद् के सृजन और ईत्सादन के तिए प्रस्िाि िा
सकिे हैं), यहां भी संसद आसे या िो पाररि कर सकिी है या नहीं या आस पर कोइ कायणिाही नहीं
कर सकिी है। संतिधान के ऄतधकांश भाग को संसद द्वारा या िो तिशेष बहुमि से ऄथिा साधारर्
बहुमि से संशोतधि ककया जा सकिा है। के िि कु छ मामिों में, राज्य तिधान-मंििों की सहमति
अिश्यक है।
 राज्य तिधान-मंिि द्वारा ऄनुसमथणन और ऄस्िीकृ ति के तिए कोइ समय सीमा तनधाणररि नहीं की
गइ है।
 ककसी संतिधान संशोधन ऄतधतनयम के संदभण में गतिरोध हो िो संसद के दोनों सदनों की संयुि
बैठक का कोइ प्रािधान नहीं है, जैसाकक साधारर् तिधेयक के मामिे में ईपिब्लध है।
 संशोधन प्रकक्रया से संबद् व्यिस्था ऄपयाणप्त है, ऄिः आन्हें न्यायपातिका को संदर्पभि करने के
व्यापक ऄिसर प्राप्त हो जािे हैं (ईपयुणि ईल्िेतखि)। आसने न्यायपातिका और संसद के मध्य
िकराि में िृतद् की है, तजससे भारिीय राज्य-व्यिस्था का संिुिन कमजोर हुअ है।

6. मूि ढााँचा/बु तनयादी संर चना (Basic Structure)


6.1. पररभाषा

मूि ढााँचे (बुतनयादी संरचना) का तसद्ांि एक न्यातयक निाचार है। आसके ऄनुसार, संतिधान की कु छ
ऐसी मूिभूि तिशेषिाएाँ हैं, तजन्हें संसद ऄनुच्छेद 368 के ऄंिगणि ऄपनी शतियों का प्रयोग करिे हुए

संतिधान संशोधन ऄतधतनयम के माध्यम से पररिर्पिि या समाप्त नहीं कर सकिी है। आसतिए, यकद

कोइ संतिधान संशोधन ऄतधतनयम, संतिधान के मूि ढााँचे या संरचना को पररिर्पिि करने का प्रयास
करिा है, िो न्यायािय के पास ईि ऄतधतनयम को ऄतधकारािीि (ultra vires) ठहरािे हुए तनरस्ि

करने का ऄतधकार है। ऄनुच्छेद 368 में प्रयुि 'संशोधन' शब्लद, ऐसे संशोधन जो संतिधान के मूि ढााँचे

में पररििणन के फिस्िरूप नया संतिधान बनाने के समिुल्य हों, को छोडकर संतिधान में संशोधन की
ऄनुमति प्रदान करिा है।

96 www.visionias.in ©Vision IAS

Google it:- https://upscpdf.com


https://upscpdf.com << Download From >> https://upscpdf.com

मूि ढााँचे का तसद्ांि िस्िुिः के शिानंद भारिी िाद (1973) में ईच्चिम न्यायािय द्वारा दी गइ
न्यातयक व्याख्या का पररर्ाम है। यह तसद्ांि संसद द्वारा ऄतधतनयतमि ऐसे संतिधान संशोधनों को
पुनर्पििोककि (समीक्षा) करने और ईन्हें ऄमान्य घोतषि करने की ईच्चिम न्यायािय की सीतमि शति
के अधार का सृजन करिा है, जो संतिधान के "मूि ढााँच"े के प्रतिकू ि हैं या ईसमें पररििणन करना
चाहिे हैं।
आस तसद्ांि (मूि ढााँचे) के संबंध में कु छ ईल्िेखनीय सबदु तनम्नतितखि हैं:
 मूि ढााँचे का तसद्ांि के िि संतिधान संशोधनों पर िागू होिा है।
o यह संसद द्वारा पाररि साधारर् ऄतधतनयमों (तजन्हें स्ियं संतिधान के ऄनुरूप होना चातहए)
पर िागू नहीं होिा है।
 ईल्िेखनीय है कक संतिधान के मूि ढााँचे को न्यायपातिका ने पूर्णि: पररभातषि नहीं ककया है। ऐसे
में संतिधान की कोइ तितशष्ट तिशेषिा “मूि ढााँचे” का तहस्सा है या नहीं, आसका तनधाणरर्
न्यायािय द्वारा ऄपने तितभन्न तनर्णयों में ककया जािा रहा है।
हािांकक, मूि ढााँचे के ऄंिगणि संतिधान की िे तिशेषिाएं सतम्मतिि हैं, तजन्हें ईच्चिम न्यायािय ने
ऄिंघनीय माना है। तिगि िषों के दौरान (िषण 1973 के बाद से) तितभन्न न्यातयक तनर्णयों के माध्यम
से आन्हें रेखांककि ककया गया है। आन मूि तिशेषिाओं (ऄथाणि् मूि ढााँचे) में तनम्नतितखि सतम्मतिि हैं:
 संतिधान की सिोच्चिा
 भारिीय राजव्यिस्था का संप्रभु, िोकिांतिक और गर्िंिात्मक स्िरूप
 संतिधान का पंथतनरपेक्ष चररि
 तिधातयका, कायणपातिका और न्यायपातिका के मध्य शतियों का पृथक्करर्
 संतिधान का संघीय चररि
 राष्ट्र की एकिा और ऄखंििा
 कल्यार्कारी राज्य (सामातजक-अर्पथक न्याय)
 न्यातयक पुनर्पििोकन
 व्यति की स्ििंििा और गररमा
 संसदीय प्रर्ािी
 तितध का शासन
 मूि ऄतधकारों और तनदेशक तसद्ांिों के मध्य सामंजस्य और संिुिन
 समानिा का तसद्ांि
 स्ििंि और तनष्पक्ष चुनाि
 न्यायपातिका की स्ििंििा
 संतिधान में संशोधन करने की संसद की सीतमि शति
 न्याय िक प्रभािी पहुाँच
 युतियुििा/िकण संगििा
 ऄनुच्छेद 32, 136, 141 और 142 के ऄंिगणि ईच्चिम न्यायािय की शतियााँ

6.2. मू ि ढााँ चे की ऄिधारर्ा का ईद्भि

6.2.1. पररचय

जैसा कक उपर ईल्िेख ककया गया है, ‘मूि ढााँच’े की ऄिधारर्ा को सिणप्रथम के शिानंद भारिी िाद
(1973) में ईच्चिम न्यायािय के तनर्णय के माध्यम से प्रतिपाकदि ककया गया था। ईल्िेखनीय है कक
'मूि ढााँचा' िाक्यांश सिणप्रथम गोिकनाथ िाद (1967) में यातचकाकिाणओं की ओर से बहस करिे हुए
एम. के . नांतबयार और ऄन्य ऄतधिक्िाओं द्वारा प्रयुक्ि ककया गया था, िेककन िषण 1973 में यह
ऄिधारर्ा शीषण न्यायािय के तनर्णय द्वारा प्रस्िुि की गइ।

97 www.visionias.in ©Vision IAS

Google it:- https://upscpdf.com


https://upscpdf.com << Download From >> https://upscpdf.com

6.2.2. िाद-तििाद

'मूि ढााँचे' की ऄिधारर्ा का ईद्भि संसदीय संशोधनों और ईच्चिम न्यायािय के तनर्णयों की श्रृंखिा की
चरम पररर्ति के रूप में देखा जा सकिा है। आन्हें प्रथम संशोधन ऄतधतनयम, 1951 से िेकर िषण 1971
और िषण 1972 में हुए ऄतधकातधक संशोधनों से व्युत्पन्न िाद-तििाद से समझा जा सकिा है। िाद-
तििाद के मूि सबदु तनम्नतितखि हैं, तजन्होंने ईच्चिम न्यायािय और संसद के तिचारों में तिभेद का
मागण प्रशस्ि ककया:
 क्या संसद की ‘संतिधानी शति (constituent power)’ (संतिधान में संशोधन करने की शति)
ईसकी ‘तिधायी शति (legislative power)’ (तितध तनमाणर् की शति) के समान है?
 क्या संतिधान में संशोधन करने की संसद की शति ऄसीतमि है?
 साथ ही, DPSPs (राज्य की नीति के तनदेशक ित्ि) और मूि ऄतधकारों के सापेतक्षक महत्ि को
िेकर ईच्चिम न्यायािय और संसद, दोनों के मि एक-दूसरे से तिपरीि थे। जहााँ, संसद मूि
ऄतधकारों का ऄतधक्रमर् (supersede) करिे हुए राज्य के नैतिक दातयत्ि के रूप में DPSPs का
प्रििणन चाहिी थी, िहीं ईच्चिम न्यायािय ने मूि ऄतधकारों को DPSPs की िुिना िरीयिा
प्रदान की।
 एक ऄन्य प्रश्न संतिधान की व्याख्या करने और ईसे ऄतस्ित्ि में बनाए रखने हेिु न्यायाियों की
शति की िुिना में संसद की सिोच्चिा से संबंतधि था।
के शिानंद भारिी िाद में ईपयुणि प्रश्न से संबंतधि िकण -तििकों का समाधान करने का प्रयास ककया गया
और िभी से ईच्चिम न्यायािय ने आस िाद में कदए गए तनर्णय का बाद के न्यातयक तनर्णयों में पािन
ककया है।

6.2.3. के शिानं द भारिी िाद, 1973

 संसद ने न्यातयक पुनर्पििोकन से कु छ कानूनों को प्रतिरतक्षि करने के साधन के रूप में िषण 1951
में प्रथम संतिधान संशोधन ऄतधतनयम के माध्यम से संतिधान में नौिीं ऄनुसूची को सतम्मतिि
ककया। ऄनुच्छेद 31 के प्रािधानों (तजसमें कक बाद में स्ियं कइ बार संशोधन ककया गया) के
ऄनुसार, नौिीं ऄनुसूची में तितनर्ददष्ट कानूनों (राज्य द्वारा तनजी संपतर्त् के ऄतधग्रहर् और आस
प्रकार के ऄतधग्रहर् के तिए देय क्षतिपूर्पि कानून) को आस अधार पर न्यायािय में चुनौिी नहीं दी
जा सकिी कक िे ईन नागररकों के मूि ऄतधकारों का ईल्िंघन करिे हैं, तजनकी संपतर्त् का
ऄतधग्रहर् ककया गया है।
o संतिधान (तिशेष रूप से नागररकों के मूि ऄतधकारों से संबंतधि आस खंि) में संशोधन करने
की संसद के प्रातधकार को िषण 1951 में ही चुनौिी दी गइ थी।
o ईल्िेखनीय है कक स्ििंििा के ईपरांि, भू-स्िातमत्ि और काश्िकारी व्यिस्था में सुधार के
ईद्देश्य से तितभन्न राज्यों में कइ कानून पाररि ककए गए थे। यह संतिधान के समाजिादी
िक्ष्यों {DPSPs के ऄनुच्छेद 39 (b) और (c) में तनतहि} को कायाणतन्िि करने के सर्त्ारुढ़
कांग्रेस पािी के चुनािी िादे के ऄनुरूप था। ये ऄनुच्छेद ईत्पादन के संसाधनों का सभी
नागररकों के मध्य समान तििरर् और कु छ िोगों में धन-संपतर्त् के ऄतहिकारी संकेिर् की
रोकथाम को अिश्यक बनािे हैं। आन कानूनों से प्रतिकू ि रूप से प्रभातिि संपतर्त्धारकों
(ऄथाणि् भू-स्िातमयों) ने न्यायाियों में यातचका दायर की। अरंभ में, न्यायािय ने यह कहिे
हुए भूतम सुधार कानूनों को तनरस्ि कर कदया कक िे संतिधान द्वारा प्रदर्त् संपतर्त् के मूि
ऄतधकार का ईल्िंघन करिे हैं।

98 www.visionias.in ©Vision IAS

Google it:- https://upscpdf.com


https://upscpdf.com << Download From >> https://upscpdf.com

 प्रतिकू ि तनर्णयों की प्रतिकक्रया स्िरूप, संसद ने आन कानूनों को संतिधान की नौिीं ऄनुसूची में

प्रथम और चिुथण संतिधान संशोधन ऄतधतनयम (क्रमशः िषण 1951 और 1952 में ऄतधतनयतमि)
के माध्यम से सतम्मतिि कर कदया। आस प्रकार ईन्हें प्रभािी ढंग से न्यातयक पुनर्पििोकन के दायरे
से बाहर रखा गया।
o हािााँकक, िषण 1952 (शंकरी प्रसाद ससह देि बनाम भारि संघ िाद) और िषण 1955 (सज्जन
ससह बनाम राजस्थान राज्य िाद) में ईच्चिम न्यायािय ने आस िकण को ऄस्िीकृ ि कर कदया कक
ईपयुणि संसदीय कारणिाइ संतिधान के ऄनुच्छेद 13(2) (नागररकों के मूि ऄतधकारों की

सुरक्षा के तिए प्रािधातनि) का ईल्िंघन करिी है। आस प्रकार, ईच्चिम न्यायािय ने संतिधान

के ककसी भी भाग को संशोतधि करने की संसद की शति को यथािि रखा, तजसमें नागररकों
के मूि ऄतधकारों को प्रभातिि करने िािे संशोधन भी सतम्मतिि थे।
 िेककन, िषण 1967 में, गोिकनाथ िाद में, ईच्चिम न्यायािय ने ऄपने पूिण के मि के तिपरीि
तनर्णय सुनाया। बहुमि के तनर्णय ने संतिधान में संशोधन करने की संसद की शति पर ऄंिर्पनतहि
सीमाओं की ऄिधारर्ा का सूिपाि ककया। िब, ईच्चिम न्यायािय का यह दृतष्टकोर् था कक
संतिधान, नागररकों की मूिभूि स्ििंििा को स्थातयत्ि प्रदान करिा है। बहुमि के तनर्णय के
ऄनुसार, ऄनुच्छेद 13, संसद की शतियों पर यह सीमा अरोतपि करिा है।
o ईच्चिम न्यायािय के ईि तनर्णय का प्रतिकार करने के तिए संसद ने पुन: कइ ईपाय ककए।
जुिाइ 1971 और जून 1972 के मध्य ऄनेक संशोधनों के द्वारा संसद ने संतिधान में संशोधन
करने की ऄपनी शति को पुन: स्थातपि करने का प्रयास ककया।
 संसद ने ऄपने तिए मूि ऄतधकारों से संबंतधि (संतिधान के ) भाग 3 सतहि संतिधान के
ककसी भी भाग में संशोधन करने की पूर्ण शति पुनस्थाणतपि की।
 राष्ट्रपति को संसद के दोनों सदनों द्वारा पाररि ककसी भी संतिधान संशोधन तिधेयक पर
ऄपनी सहमति देने के तिए बाध्य ककया गया।
 पाररि ककए गए ऄनेक कानून में संपतर्त् के ऄतधकार पर कइ ऄंकुश िगाए गए। तितध के
समक्ष समिा और तितध का समान संरक्षर् (ऄनुच्छेद 14) के ऄतधकार िथा ऄनुच्छेद 19
के ऄंिगणि प्रत्याभूि मूि स्ििंििाओं को DPSPs के ऄंिगणि अने िािे ऄनुच्छेद 39 (b)
और (c) के ऄधीनस्थ बनाया गया।
o पूिणििी राजाओं को प्रदर्त् तप्रिी पसण समाप्त कर कदया गया।
o भूतम सुधारों से संबंतधि कानूनों की संपूर्ण श्रेर्ी को न्यातयक पुनर्पििोकन के दायरे से बाहर
कर नौिीं ऄनुसूची में रखा गया।
िषण 1971 और िषण 1972 के मध्य हुए ऄतधकातधक संशोधनों के ईपयुणि घिनाक्रमों को के शिानंद
भारिी बनाम के रि राज्य िाद (1973) में ईच्चिम न्यायािय के समक्ष चुनौिी दी गइ। आस िाद की

सुनिाइ करने के तिए पूर्ण पीठ (13 न्यायाधीशों) का गठन ककया गया।
के शिानंद भारिी िाद में प्रदर्त् तनर्णय के मूि ित्ि तनम्नतितखि थे:
 सभी न्यायाधीशों ने यह कहिे हुए 24िें संतिधान संशोधन ऄतधतनयम की िैधिा को यथािि रखा

कक संसद के पास संतिधान के ककसी भी या सभी भाग में संशोधन करने की शति है।
o न्यायाधीशों ने यह माना कक गोिकनाथ िाद में संिैधातनक प्रािधानों के संदभण में ईच्चिम
न्यायािय द्वारा प्रदर्त् तनर्णय िुरिपूर्ण था और ऄनुच्छेद 368 में संतिधान में संशोधन करने की
शति और प्रकक्रया दोनों सतम्मतिि है।

99 www.visionias.in ©Vision IAS

Google it:- https://upscpdf.com


https://upscpdf.com << Download From >> https://upscpdf.com

 हािांकक, संतिधान में संशोधन करना, के िि तितध तनमाणर् के समान नहीं है, जैसा कक ऄनुच्छेद
13(2) के ऄनुरूप समझा जािा है। ऄतपिु, आसका मूि तनतहिाथण यह है कक संसद की ‘संिैधातनक
शति (constituent power)’ ईसकी 'तिधायी शति (legislative power)' के समान नहीं है।
संसद की 'संिैधातनक शति' ऄंिर्पनतहि सीमाओं के ऄधीन है। संसद, ऄनुच्छेद 368 के ऄंिगणि संतिधान
संशोधन संबंधी ऄपनी शतियों का ईपयोग कर, संतिधान के 'मूि ढााँच'े को 'क्षतिग्रस्ि', 'क्षीर्', 'नष्ट',
'तनरस्ि', 'पररिर्पिि' या ‘रूपांिररि’ नहीं कर सकिी है।

6.3. ‘मू ि ढााँ चे ’ के तसद्ां ि का अिोचनात्मक तिश्ले ष र्


 तिपक्ष में िकण
o संतिधान में मूि ढााँचे के संबंध में कोइ प्रािधान नहीं है। मूि संतिधान में आसे पररितक्षि करने
िािा कोइ प्रािधान भी नहीं है कक संतिधान तनमाणिा, संतिधान के ककसी भी प्रािधान को पूर्ण
रूप से ऄसंशोधनीय रखना चाहिे थे।
o ऄभी िक, आस संबंध में कोइ सिणसम्मति नहीं बनी है कक िास्िि में कौन-से प्रािधान ‘मूि
ढााँच’े के भाग हैं। साथ ही, कु छ तनर्णयों के संदभण में भी न्यायाधीशों के मध्य ही आस संबंध में
मिभेद रहा है कक कौन-सा तिशेष ित्ि मूि ढााँचे का तहस्सा है।
o ईच्चिम न्यायािय ने स्ियं ही यह रिप्पर्ी की है कक, संतिधान में प्रत्येक शब्लद ऄपने साधारर्
या सामान्य ऄथण में ईपयोग ककया गया माना जाना चातहए और ईसे सामान्य ऄथणबोध कदया
जाना चातहए। हािांकक, ‘मूि ढााँच’े की ऄिधारर्ा आससे तिचिन की ओर संकेि करिी है
और यह संतिधान में िर्पर्ि के बजाय सिोच्च न्यायािय द्वारा की गइ व्याख्या पर ऄतधक
अधाररि है।
o प्रतितनतधत्िकारी िोकिंि (representative democracy) में, संप्रभु िोगों का
प्रतितनतधत्ि करने िािी सिोच्च संस्था संसद होिी है। िेककन, ‘मूि ढााँचे’ का तसद्ांि,
संिैधातनक शतियों को तनिाणतचि प्रतितनतधयों से ईच्चिम न्यायािय के न्यायाधीशों को
हस्िांिररि करिा है।
 पक्ष में िकण
o ऐसा कहा जािा है कक के शिानंद भारिी िाद (1973) में यकद ईच्चिम न्यायािय के बहुमि ने
यह तनर्णय कदया होिा (जैसा कक 6 न्यायाधीशों ने िास्िि में ककया था) कक संसद, संतिधान
के ककसी भी भाग में पररििणन कर सकिी है, िो भारि संभििः सिणसर्त्ािादी राज्य
(Totalitarian State) में पररिर्पिि हो गया होिा या एकदिीय शासन िािा राज्य बन
गया होिा।
o आसके ऄतिररि, संभििः संतिधान ने ऄपनी सिोच्चिा खो दी होिी। यह सिणस्िीकृ ि िथ्य है
कक मूि ढााँचे के तसद्ांि ने भारिीय िोकिंि को संरतक्षि रखा है।
o मूि ढााँचे के तसद्ांि के पक्ष में कदए गए कु छ ऄन्य िकण तनम्नतितखि हैं:
 यह कहा जािा है कक के िि अपाि-काि के दौरान ककए गए संशोधनों का परीक्षर्
करके , कोइ भी व्यति 'मूि ढााँचे' के तसद्ांि का महत्ि समझ सकिा है।
 39िें संतिधान संशोधन ऄतधतनयम ने चुनािी कदाचार के बािजूद राष्ट्रपति, ईप-
राष्ट्रपति, ऄध्यक्ष और प्रधानमंिी के चुनाि को ककसी भी अधार पर चुनौिी देने से
तनतषद् कर कदया था।
 41िें संतिधान संशोधन ऄतधतनयम ने राष्ट्रपति, ईप-राष्ट्रपति, प्रधानमंिी या राज्यपािों
के तिरूद् न के िि ईनके कायणकाि के दौरान, ऄतपिु सदैि के तिए कोइ भी िाद
(दीिानी या अपरातधक) दायर करने से प्रतिबंतधि कर कदया था।

100 www.visionias.in ©Vision IAS

Google it:- https://upscpdf.com


https://upscpdf.com << Download From >> https://upscpdf.com

 आस प्रकार, यकद कोइ व्यति के िि एक कदन के तिए राज्यपाि के पद पर असीन होिा, िो िह


जीिन-भर के तिए ककसी भी कानूनी कायणिाही से संरक्षर् प्राप्ि कर िेिा।
 मूि ढााँचे का तसद्ांि तिकासशीि िोकिंि के तिए अिश्यक है। यह आस िथ्य के अिोक में तिशेष
रूप से महत्िपूर्ण हो जािा है कक भारिीय िोकिंि ऄभी भी ऄपनी शैशिास्था में है और तजन
संस्थानों को तिरासि के िौर पर आसे संरतक्षि रखने का दातयत्ि सौंपा गया है , ईन्हें पररपक्व होने
के तिए कु छ और समय प्रदान करने की अिश्यकिा है।

7. भारिीय संतिधान के स्रोि


 भारिीय संतिधान आस संदभण में भी ऄतद्विीय है कक आसके तनमाणर् में तिश्व के कइ देशों के संतिधानों
को दृतष्टगि रखा गया है। हमारे संतिधान तनमाणिा सभी ज्ञाि शासन-तिधानों के कायणकरर् से प्राप्त
ऄनुभिों को ऄपने संतिधान में समातिष्ट करना चाहिे थे। यहााँ यह ईल्िेखनीय है कक ऄन्य देशों के
संतिधानों से तितभन्न प्रािधानों को ग्रहर् करना ककसी नकिची मानतसकिा का पररचायक नहीं है ,
ऄतपिु, आसका ईद्देश्य भारिीय पररप्रेक्ष्य, समस्याओं और अकांक्षाओं की पूर्पि करने िािे तिश्व के
सिोर्त्म संिैधातनक प्रािधानों को ग्रहर् करना था।
 हमारे संतिधान के मूि ऄतधकार और ईच्चिम न्यायािय संबंधी व्यिस्थाओं पर संयुि राज्य
ऄमेररका का, राज्य की नीति के तनदेशक ित्िों पर अयरिैंि का, अपािकािीन व्यिस्थाओं पर
जमणनी का, तिधायी शतियों के तििरर् पर कनािा का िथा संसदीय संस्थाओं पर तििेन (के
संतिधान) का स्पष्ट प्रभाि दृतष्टगोचर होिा है।
 आनके ऄतिररि, हमारे संतिधान तनमाणिाओं ने भारि शासन ऄतधतनयम, 1935 के कइ प्रािधानों
को ऄक्षरशः ग्रहर् कर तिया था। 1935 के ऄतधतनयम के प्रमुख प्रािधानों में संघ िथा राज्यों के
मध्य शतियों का तिभाजन, राष्ट्रपति की अपािकािीन शतियां, ऄल्पसंख्यक िगों के तहिों की
रक्षा, ईच्चिम न्यायािय का ऄधीनस्थ न्यायाियों पर तनयंिर्, संघ का राज्य के शासन में
हस्िक्षेप, तद्वसदनीय तिधातयका अकद सतम्मतिि हैं।
भारिीय संतिधान के तिदेशी स्रोिों को संक्षेप में तनम्नतितखि िातिका में दशाणया गया है:

देश ग्रहर् ककये गए प्रािधान

 सांकेतिक प्रमुख - राष्ट्रपति (जैसा कक तििेन में सम्राि/साम्राज्ञी की तस्थति


है)
 तितध का शासन
 तितध तनमाणर् की पद्ति
 मंतियों की कै तबनेि प्रर्ािी
 प्रधानमंिी का पद
यूनाआिेि ककगिम  सरकार का संसदीय स्िरुप
(UK)
 तद्वसदनीय तिधातयका
 एकि नागररकिा
 ऄतधक शतिशािी तनम्न सदन
 मंतिपररषद्, तनम्न सदन के प्रति ईर्त्रदायी
 िोकसभा ऄध्यक्ष की शतियां और ईसकी भूतमका
 सिाणतधक मि के अधार पर चुनािों में जीि का तनर्णय (फस्िण पास्ि द
पोस्ि तसस्िम)

101 www.visionias.in ©Vision IAS

Google it:- https://upscpdf.com


https://upscpdf.com << Download From >> https://upscpdf.com

 तितखि संतिधान

 ईद्देतशका
 मूि ऄतधकार
 राज्य के कायणकारी प्रमुख और सशस्त्र बिों के सिोच्च सेनापति के रूप में
राष्ट्रपति की तस्थति
 राज्यसभा के पदेन सभापति के रूप में ईप-राष्ट्रपति
 राज्यों से संबंतधि प्रािधान
संयुि राज्य ऄमेररका  राष्ट्रपति पर महातभयोग
(USA)  ईच्चिम न्यायािय

 न्यायपातिका की स्ििंििा और न्यातयक पुनर्पििोकन की शति

 ईच्चिम न्यायािय और ईच्च न्यायािय के न्यायाधीशों की पदच्युति

USSR  मूि कर्त्णव्य

 ईद्देतश्यका में न्याय (सामतजक, अर्पथक एिं राजतनतिक) का अदशण

 समििी सूची का प्रािधान


ऑस्ट्रेतिया  ईद्देतश्यका की भाषा
 व्यापार, िातर्ज्य और समागम संबंधी प्रािधान

जापान  तितध द्वारा स्थातपि प्रकक्रया


 िह तितध तजसके अधार पर ईच्चिम न्यायािय कायण करिा है

जमणनी का िाइमर  अपाि-काि के दौरान मूि ऄतधकारों का तनिंबन


संतिधान

कनािा  एक सुदढ़ृ कें ि सतहि संघीय व्यिस्था


 कें ि और राज्यों के मध्य शतियों का तििरर्
 ऄितशष्ट शतियों का कें ि में तनतहि होना

अयरिैंि  राज्य की नीति के तनदेशक ित्िों की ऄिधारर्ा (अयरिैंि ने यह


ऄिधारर्ा स्पेन से ग्रहर् की है)
 राष्ट्रपति के तनिाणचन की तितध
 राष्ट्रपति द्वारा राज्यसभा में सदस्यों का नामतनदेशन

फ्रांस  गर्िंिात्मक शासन प्रर्ािी

दतक्षर् ऄफ्रीका  संतिधान संशोधन की प्रकक्रया

102 www.visionias.in ©Vision IAS

Google it:- https://upscpdf.com


https://upscpdf.com << Download From >> https://upscpdf.com

8. तिगि िषों में Vision IAS GS मेंस िे स्ि सीरीज में पू छे


गए प्रश्न (Previous Year Vision IAS GS Mains Test
Series Questions)
1. संतिधान को त्िररि ढंग से बार-बार संशोतधि नहीं ककया जाना चातहए, जब िक ऐसा करना
ऄतनिायण न हो। कै से भारिीय संतिधान में ककए गए संशोधन ऐसी अिश्यकिा का ऄनुपािन
करिे हैं?

दृतष्टकोर्:

 सिणप्रथम प्रश्न के कथन पर तिचार कीतजए। क्या यह न्यायसंगि है? ईर्त्र देने का सही
िरीका आस पर तनभणर होना चातहए कक क्या कथन ईतचि है? यकद अप कथन का खंिन

करिे हैं, िो प्रश्न के दूसरे भाग की प्रासंतगकिा का ऄतधक महत्ि नहीं रह जाएगा।
 जो भी तनर्णय अप िेिे हैं ईसके समथणन में िकण भी दीतजए। यहााँ अप संतिधान की
सत्यात्मक प्रर्ािी की अिश्यकिा पर भी चचाण कर सकिे हैं। आस िरह, आस पर भी
तिचार करना अिश्यक है कक संशोधन त्िररि और बार-बार नहीं होने चातहए।
 ित्पश्चाि आस पर तिचार कीतजए कक भारिीय संतिधान आस कथन की सत्यिा पर खरा
ईिरिा है। जब अप आस पर तिचार कर रहे हों िब अपका यह मानना होगा कक यह
कथन ईतचि है।
 कु छ संतिधान संशोधनों का ईदाहरर् भी दीतजए और तिचार कीतजए कक संशोधन
त्िररि रूप से ककए गए हैं या के िि िब ककए गए जब पररतस्थतििश अिश्यक हो गए थे।
ईर्त्रः
साधरर्िः यह माना जािा है कक संतिधान एक गतिशीि दस्िािेज़ है। आसे समाज की
अिश्यकिाओं के ऄनुसार पररििणनों को स्िीकार करने योग्य होना चातहए। कभी-कभी नइ
सामातजक और अर्पथक शतियों के प्रभाि में सरकार बडे बदिाि करिी है क्योंकक
राजनीतिक तस्थति समय के ऄनुसार ऄररिार्पिि होिी रहिी है। आसतिए ित्कािीन
पररतस्थति के साथ सामंजस्य स्थातपि करना भी संिैधातनक दस्िािेज़ों का िाभ होना चातहए
और आसको ऄपनी प्रासंतगकिा तनरंिर सुतनतश्चि करनी चातहए। संिैधातनक पररििणन यह
प्रदर्पशि करिे हैं कक नागररक ककस िरह से शातसि हो रहे हैं।
कफर भी, यह ऄनुभि होना चातहए कक संतिधान सरकार के तिए कोइ ऐसा ईपकरर् नहीं हैं

जो िोगों के ऄतधकारों को तनयंतिि करे। आसके तिपरीि, यह ऐसा ईपकरर् है जो िोगों द्वारा
सरकार को गिि नीतियों के तनमाणर् से तनतषद् करिा है। यकद संतिधान में पररििणन त्िररि
रूप से होिे हैं िो यह ऄपना महत्ि खो देगा क्योंकक जो संरक्षर् संतिधान के द्वारा प्रदान की
गयी है िह बार-बार के संशोधन से ऄपना महत्ि खो देगी। यह संतिधान को ककसी भी
साधारर् कानून के जैसा बना सकिा है। आससे संतिधान की शतियों के महत्ि में कमी हो
सकिी हैं। ऄि: संतिधान के जो मूिभूि ऄिधारर् हैं जैसे शतियों का पृथक्करर्, जााँच और
संिुिन, सरकार पर तनयंिर् आत्याकद को तनतश्चि रूप से ऄक्षुण्र् रखना चातहए।
सरकार के प्रचतिि स्िरूप में ऄनेक दोष तिद्यमान हैं क्योंकक आसका संचािन िोगों द्वारा
ककया जा रहा है। िेककन आन दोषों का संतिधान में पररििणन के तबना ही समाधान ककया जा
सकिा है। तजसको पंतिि नेहरू ने ईतचि ही कहा था कक - संतिधान में तनरंिर पररििणन नहीं
होना चातहए। आसमें पररतस्थति की ऄतनिायणिा के ऄनुरूप तनतश्चि रूप से पररििणन ककया
जाना चातहए।

103 www.visionias.in ©Vision IAS

Google it:- https://upscpdf.com


https://upscpdf.com << Download From >> https://upscpdf.com

आस िथ्य को भी दृतष्टगि रखना चातहए कक संतिधान तनमाणिाओंंं ने भारिीय संतिधान में


ऄनुच्छेद 368 को भी शातमि ककया है। यह ऄनुच्छेद संतिधान संशोधन की प्रकक्रया से
संबंतधि है। ऄनुच्छेद 368 के कारर् ही भारिीय संतिधान को न िो बहुि कठोर और न बहुि
िचीिा कहा जािा है, बतल्क अंतशक रूप से कठोर और अंतशक रूप से िचीिा कहा जा
सकिा है।
भारिीय संतिधान के ऄनुच्छेद तनम्नतितखि िरीकों से संशोतधि हो सकिे हैं:
 संसद में साधारर् बहुमि से,
 तिशेष बहुमि से तजसका ऄथण है, प्रत्येक सदन के सभी सदस्यों के बहुमि के द्वारा और
प्रत्येक सदन के ईपतस्थि और मि देने िािे दो तिहाइ सदस्यों के बहुमि के द्वारा।
 कम से कम अधे राज्यों की तिधानसभा के ऄनुसमथणन के द्वारा, तजसमें तिशेष बहुमि भी
शातमि हो।
 कु छ बडे संशोधनों के ईदाहरर् जो नागररक और समाज की अिश्यकिाओं के संदभण में
ककए गए हैं:
o संतिधान के 52 िें संशोधन के द्वारा दसिीं ऄनुसूची को जोडा गया है, तजसमें
सांसदों और तिधायकों को दि-बदि के अधार पर ऄयोग्य घोतषि ककया जा सकिा
है। आस कानून का मुख्य ईद्देश्य राजनीतिक दि-बदि की समस्या का समाधान
करना था तजसकी ईत्पतर्त् गठबंधन की राजनीति से हुइ है।
o 73िां संतिधान संशोधन जो पंचायिी राज्य प्रर्ािी को संस्थातपि करिा है। यह
मूि और िाभ-रतहि िगण को प्रतितनतधत्ि प्रदान कर जनसाधरर् की शति को मं च
प्रदान करिा है। आस प्रकार यह भारि में िोकिांतिक तिके न्िीकरर् की सबसे बडी
शुरूअि है।
कु छ संशोधनों ने तनतश्चि रूप से संतिधान के दस्िािेज़ों की ऄन्िणिस्िु और स्िभाि के
पररििणन के संदभण में और सामातजक, अर्पथक एिं राजनीतिक अिश्यकिाओं के ऄनुसार
बेहिर कायण ककया है, जो नीति-तनर्णयों के कक्रयान्ियन के तिए या िो ऄतनिायण या पररर्ाम
थे। कफर भी, ईनमें से कु छ ईपेक्षा करने योग्य, गैर-अिश्यक और के िि राजनीतिक एिं
दिगि (बहुमि प्राप्त सर्त्ा दि) भािनाओं से ककए गए हैं, ईदाहरर्ाथण- 42िां संतिधान
संशोधन। सैद्ांतिक रूप से ककसी भी राजनीतिक दि द्वारा संतिधान का सरििा से संशोधन
नहीं ककया जाना चातहए।
यह समझना अिश्यक है कक सरकारी काम-काज के प्रति िोगों का मोहभंग होना संिैधातनक
ईपचारों की मांग करिा है न कक संशोधनों की। चुनािी कानून एिं ईसकी प्रकक्रया िथा
राजनीतिक दिों की कायणप्रर्ािी में सबसे ऄतधक सुधार की अिश्यकिा है और आसके तिए
संतिधान संशोधन की अिश्यकिा नहीं है। यकद राजनीतिक आच्छाशति है िो संबंतधि
समस्या या मुद्दे का समाधान करिे हुए तिधेयक को िब भी प्रस्िुि ककया जा सकिा है जब
आसकी अिश्यकिा महसूस हो।

2. यद्यतप संतिधान-तनमाणिाओं ने संतिधान संशोधन का ऄतधकार संसद में तनतहि कर कदया था,
ककिु, के शिानंद भारिी िाद के बाद से, सिोच्च न्यायािय आस ऄतधकार में तनर्ाणयक
भागीदार बन चुका है। रिप्पर्ी कीतजए।
दृतष्टकोर्ः
ईर्त्र में के शिानंद भारिी िाद के पूिण की तस्थति को रेखांककि करना चातहए और आस िथ्य
को भी प्रकि करना चातहए कक ककस प्रकार आस तनर्णय ने संतिधान में संशोधन की ऄिधारर्ा
को बदि कदया। संतिधान के मूि ढााँचे का संतक्षप्त तििरर् भी प्रस्िुि करना चातहए।

104 www.visionias.in ©Vision IAS

Google it:- https://upscpdf.com


https://upscpdf.com << Download From >> https://upscpdf.com

ईर्त्रः
हािांकक, संतिधान ने संतिधान या आसके ककसी भी भाग में संशोधन (ऄनुच्छेद 368) का
ऄतधकार जनिा के प्रतितनतधयों से तनर्पमि संसद को प्रदान ककया है। ईच्चिम न्यायािय ने एक
ऐतिहातसक तनर्णय के िहि यह तनर्पर्ि ककया कक कोइ भी संशोधन, जो संतिधान के मूि ढााँचे

का ईल्िंघन करिा है, िह प्रभाि-शून्य होगा। आस प्रकार आसने संसद के संतिधान में संशोधन
के ऄतधकार में काफी हद िक किौिी कर दी।
यकद ऄनुच्छेद 368 का पठन सामान्य रूप से ककया जािा है िो आसमें संतिधान के ककसी भी
भाग को संशोतधि करने के संसद के ऄतधकार पर कोइ प्रतिबंध या सीमा िागू होिी प्रिीि
नहीं होिी है। गोिकनाथ िाद में, सिोच्च न्यायािय की यह राय थी कक आसे संतिधान के

ऄनुच्छेद 13 के साथ जोडकर पढ़ा जाना चातहए, िेककन, िषण 1973 में के शिानंद भारिी
िाद में आसने ऄपने तनर्णय को बदि कदया।
के शािानंद भारिी िाद में, न्यायपातिका ने आस प्रश्न पर तिचार करने की चेष्टा की कक ‘‘क्या

संतिधान को संशोतधि करने का संसद का ऄतधकार ऄसीतमि था?” आस प्रश्न पर तिचार करिे
हुए आसके द्वारा मूि ढांचे के तसद्ांि को प्रस्िुि ककया गया तजसके िहि यह माना गया कक
संसद संतिधान के ककसी भी तहस्से में िब िक संशोधन कर सकिी है, जब िक ईससे संतिधान

के मूि ढााँचे में बदिाि या संशोधन न हो।

ईच्चिम न्यायािय ने ‘मूि ढााँच’े के ऄंिगणि शातमि ित्िों का स्पष्ट रूप से ईल्िेख नहीं ककया
है। यह अगे आस बाि की अशंका ईत्पन्न करिा है कक यकद ककसी कानून को चुनौिी दी जािी
है िो क्या ईसका न्यातयक पुनर्पििोकन ककया जा सके गा या नहीं।

ककिु बाद के तनर्णयों से यह समझा जा सकिा है कक ईद्देतश्यका, संघिाद, मूि ऄतधकार, पंथ-

तनरपेक्षिा आत्याकद आसके अधारभूि घिकों में शातमि हैं।

एक समीक्षक के ऄनुसार, संतिधान की िास्ितिकिा यह रही है कक जब संतिधान की व्याख्या

की बाि अिी है, िो यह संभि है कक तिधातयका और न्यायपातिका दोनों के तिपरीि तिचार


हो। आसका तनतश्चि रूप से तनधाणरर् नहीं ककया गया है कक ककसका तनर्णय ऄंतिम होगा। संसद
कोइ भी कानून पास कर सकिी है और न्यायािय को आसकी संिैधातनकिा पर तनर्णय िेने का
ऄतधकार होगा। संसद संतिधान में संशोधन कर न्यायािय के ऄतधकार में गतिरोध ईत्पन्न
करने की चेष्टा कर सकिी है और न्यायािय आसकी घोषर्ा कर सकिा है कक संसद के
ऄतधकार सीतमि हैं। ऐसे ऄतधतनयमन के ईदाहरर् हैं जो मूि ढााँचे का ईल्िंघन करने के
कारर् प्रभाि-शून्य हो गए, जबकक 44िें संतिधान संशोधन ऄतधतनयम के द्वारा संपतर्त् के
ऄतधकार का ईन्मूिन संबंधी संशोधन न्यायािय की समीक्षा में सही तसद् हुअ।
तनष्कषणिः यह कहा जा सकिा है कक ऄन्य सरकारी संस्थाओं के तिपरीि संिैधातनक शति के
के न्ि के रूप में संसद की शति में कमी करना, न्यायपातिका के समक्ष सिाणतधक प्रबि प्रिीि
होिा है।

105 www.visionias.in ©Vision IAS

Google it:- https://upscpdf.com


https://upscpdf.com << Download From >> https://upscpdf.com

9. तिगि िषों में संघ िोक सेिा अयोग द्वारा पूछे गए प्रश्न
(Past Year UPSC Questions)
1. Describe the emergence of Basic Structure concept in the Indian

Constitution. (150 words) (94/II/4b/20)


भारिीय संतिधान में मूि ढााँचे की ऄिधारर्ा के ईद्भि का िर्णन कीतजए।
2. What is a Constitution? What are the main sources of the Indian

Constitution? (250 words) (07/I/6a/30)

संतिधान क्या है? भारिीय संतिधान के मुख्य स्रोि क्या हैं?

3. 'Separation of Powers is essential to ensure individual liberty' Discuss this

with regard to the provisions in the Constitution and practices adopted so

far.

'व्यति की स्ििंििा सुतनतश्चि करने के तिए शतियों का पृथक्करर् अिश्यक है' संतिधान में
ककए गए प्रािधानों और ऄब िक ऄपनाइ गइ पररपारियों के संबंध में आसकी तििेचना
कीतजए।
4. Is the Indian Parliament a "Sovereign" or a "non-Sovereign" legislature or

both? (84/II/8b(B)/3)

क्या भारिीय संसद "संप्रभु" या "गैर- संप्रभु" तिधातयका या दोनों है?

5. Write on Significance of 26th November in the country's polity in about 20

words. (09/I/10a/2)

िगभग 20 शब्लदों में देश की राजनीति में 26 निंबर के महत्ि पर तितखए।

6. Highlight the significance of the Twenty Fourth Amendment to the

Constitution of India. (in about 150 words) (99/II/4c/20)


भारि के संतिधान के चौबीसिें संशोधन के महत्ि पर प्रकाश िातिए।
7. What are the main difference between the passage of a Constitution

Amendment Bill and other Legislative Bills? (in about 250 words)

(01/I/7b/30)
संतिधान संशोधन तिधेयक और ऄन्य तिधायी तिधेयक पाररि ककए जाने के मध्य मुख्य ऄंिर
क्या हैं?

8. How is the Constitution of Indian amended? Do you think that the

procedure for amendment makes the Constitution a play-thing in the

hands of the Centre? (in about 250 words) (02/I/7b/30)

भारि का संतिधान कै से संशोतधि ककया जािा है? क्या अप मानिे हैं कक संशोधन की प्रकक्रया

संतिधान को कें ि के हाथों का तखिौना बना देिी है?

106 www.visionias.in ©Vision IAS

Google it:- https://upscpdf.com


https://upscpdf.com << Download From >> https://upscpdf.com

9. How would you differentiate between the passage of a Constitution

Amendment Bill and of an Ordinary Legislative Bill? (06/I/8a/15)


अप संतिधान संशोधन तिधेयक और साधारर् तिधायी तिधेयक पाररि ककए जाने के मध्य
ककस प्रकार तिभेदन करेंगे?

10. What is meant by 'Sovereignty of Parliament'? Consider whether the

Indian Parliament is a sovereign body. (in about 150 words) (82/II/5d/20)

'संसद की संप्रभुिा' से क्या ऄतभप्राय है? तिचार कीतजए कक क्या भारिीय संसद संप्रभु
तनकाय है।
11. How will you define 'judicial review'? (82/II/8c(A)/3)

अप 'न्यातयक पुनर्पििोकन’ को कै से पररभातषि करेंगे?

12. What constitutes the doctrine of 'basic features' as introduced into the

Constitution of India by the Judiciary? (in about 150 words) (00/I/7b/30)

न्यायपातिका द्वारा कदया गया भारिीय संतिधान के 'अधारभूि ऄतभिक्षर्' का तसद्ांि क्या

है?

13. Why is the Indian Constitution called quasi-federal? (87/II/8c(B)/3)

भारिीय संतिधान को ऄद्ण-संघीय क्यों कहा जािा है?

Copyright © by Vision IAS


All rights are reserved. No part of this document may be reproduced, stored in a retrieval system or transmitted
in any form or by any means, electronic, mechanical, photocopying, recording or otherwise, without prior
permission of Vision IAS.

107 www.visionias.in ©Vision IAS

Google it:- https://upscpdf.com


https://upscpdf.com << Download From >> https://upscpdf.com

संविधान की ईद्देविका
विषय सूची
1. संविधान की ईद्देविका: एक पररचय ____________________________________________________________ 109
1.1. ईद्देविका की विषय-िस्तु ________________________________________________________________ 110
2. ईद्देविका की वस्थवत ______________________________________________________________________ 110
2.1. पारंपररक दृविकोण ___________________________________________________________________ 110
2.2. अधुवनक दृविकोण ____________________________________________________________________ 110
1.2. संविधान की ईद्देविका में वनवहत मूल तत्ि _____________________________________________________ 110
1.3. क्या ईद्देविका संविधान का ऄंग है? _________________________________________________________ 111
1.4. ईद्देविका के वनिवचन एिं संिोधन से संबवं धत विवभन्न िाद (Cases) ___________________________________ 111
1.4.1. बेरूबारी संघ िाद (1960) _____________________________________________________________ 111
1.4.2. गोलकनाथ िाद (1967) _______________________________________________________________ 112
1.4.3. के ििानंद भारती िाद (1973) __________________________________________________________ 112
1.4.4. रघुनाथ राि बनाम भारत संघ िाद (1993) _________________________________________________ 112
1.4.5. एस. अर. बोम्मइ िाद (1994) __________________________________________________________ 112
1.4.6. एल. अइ. सी. ऑफ़ आंवडया िाद (1995) ___________________________________________________ 112
1.5. ईद्देविका में ईवललवखत प्रमुख िब्दों की व्याख्या _________________________________________________ 112
1.5.1. संपूणव प्रभुत्ि-संपन्न (Sovereign) ________________________________________________________ 112
1.5.1.1. संप्रभुता और िैश्वीकरण ____________________________________________________________ 113
1.5.2. समाजिादी (Socialist) _______________________________________________________________ 113
1.5.3. पंथ-वनरपेक्ष (Secular) _______________________________________________________________ 114
1.5.4. लोकतंत्रात्मक (Democratic)___________________________________________________________ 115
1.5.5. गणराज्य (Republic) ________________________________________________________________ 116
1.5.6. न्याय (Justice) ____________________________________________________________________ 116
1.5.7. स्ितंत्रता (Liberty) __________________________________________________________________ 117
1.5.8. समता (Equality) ___________________________________________________________________ 117
1.5.9. बंधुता (Fraternity) _________________________________________________________________ 118
2. ईद्देविका, मूल ऄवधकार तथा राज्य की नीवत के वनदेिक तत्त्िों के मध्य पारस्पररक संबध
ं _________________________ 118
3. ईद्देविका से संबवं धत हावलया वििाद ___________________________________________________________ 118
4. ईपसंहार ______________________________________________________________________________ 120
5. विगत िषों में Vision IAS GS मेंस टेस्ट सीरीज में पूछे गए प्रश्न (Previous Year Vision IAS
GS Mains Test Series Questions) _________________________________________________________ 120
6. विगत िषों में संघ लोक सेिा अयोग द्वारा पूछे गए प्रश्न (Past Year UPSC Questions) ______________________ 121

108

Google it:- https://upscpdf.com


https://upscpdf.com << Download From >> https://upscpdf.com

1. सं विधान की ईद्दे विका: एक पररचय


'ईद्देविका' िब्द संविधान के पररचय या भूवमका को संदर्भभत करता है। यह संविधान का दिवन या सार

होता है। वलवखत संविधान की 'ईद्देविका' ईन लक्ष्यों को दिावती है, वजन्हें संविधान स्थावपत करना
चाहता है। यह संविधान की कानूनी व्याख्या को भी व्यापक अधार प्रदान करती है और वििेषतः जहां
व्याख्या ऄस्पि हो िहां समझने में सहायता करती है। आसवलए, हमारे संविधान में वनवहत ईद्देश्यों या

लक्ष्यों और अकांक्षाओं के समुवचत प्रोत्साहन ऄथिा आन्हें बढ़ािा देने हेतु , हमें ईद्देविका में वनवहत
विवभन्न ऄवभव्यवियों को ऄपनाने की अिश्यकता है।

 ईद्देविका का िावब्दक ऄथव होता है, भूवमका ऄथिा प्राक्कथन।

o सिवप्रथम, संयुि राज्य ऄमेररका द्वारा ऄपने संविधान में ईद्देविका को सवम्मवलत ककया गया।

 भारतीय संविधान की ईद्देविका का संबंध आसके ईद्देश्यों, लक्ष्यों, अदिों तथा मूलभूत वसद्धान्तों से
है।
 संविधान की ईद्देविका, संविधान सभा द्वारा 22 जनिरी 1947 को पाररत ईद्देश्य प्रस्ताि पर
अधाररत है।
 ध्यातव्य है कक जब ऄन्य सभी ईपबंध ऄवधवनयवमत ककए जा चुके थे, ईसके पश्चात् ईद्देविका को
पृथक रूप से पाररत ककया गया था।
 संविधान सभा के संिैधावनक सलाहकार बी. एन. राि ने ईपयुवि प्रस्ताि के अधार पर ईद्देविका
का प्रारूप तैयार ककया। संविधान की प्रारूप सवमवत द्वारा आस प्रारूप पर विचार ककया गया तथा
आसमें अिश्यक संिोधन करके संविधान सभा के कायों के ऄंवतम चरण में आसे पाररत ककया ताकक
यह संविधान के विवभन्न प्रािधानों के ऄनुरूप हो।
 भारतीय संविधान की ईद्देविका की विविि भाषा अस्रेवलया के संविधान से ग्रहण की गइ है।
प्रख्यात न्यायविद ि संविधान वििेषज्ञ एन. ए. पालकीिाला ने ईद्देविका को ‘संविधान का

पररचय पत्र’ कहा है।

109 www.visionias.in ©Vision IAS

Google it:- https://upscpdf.com


https://upscpdf.com << Download From >> https://upscpdf.com

1.1. ईद्दे विका की विषय-िस्तु

‘हम भारत के लोग, भारत को एक संपण


ू व प्रभुत्ि-संपन्न, समाजिादी, पंथवनरपेक्ष, लोकतंत्रात्मक
गणराज्य बनाने के वलए तथा ईसके समस्त नागररकों को:
सामावजक, अर्भथक और राजनैवतक न्याय,
विचार, ऄवभव्यवि, विश्वास, धमव और ईपासना की स्ितंत्रता,
प्रवतष्ठा और ऄिसर की समता
प्राप्त कराने के वलए,
तथा ईन सब में व्यवि की गररमा और
राष्ट्र की एकता और ऄखंडता
सुवनवश्चत करने िाली बंधतु ा बढ़ाने के वलए
दृढ़संकलप होकर ऄपनी आस संविधान सभा में अज तारीख 26 निंबर, 1949 इ. (वमवत मागविीषव
िुक्ला सप्तमी, संित् दो हजार छह विक्रमी) को एतद्द्वारा आस संविधान को ऄंगीकृ त, ऄवधवनयवमत
और अत्मार्भपत करते हैं।’
हमारे संविधान की ईद्देविका वनम्नवलवखत दो ईद्देश्यों को पूणव करती है:
 यह ईन स्रोतों को आंवगत करती है वजनसे संविधान िवि प्राप्त करता है; एिं
 यह ईन ईद्देश्यों का भी िणवन करती है, वजन्हें संविधान स्थावपत करना तथा प्रोत्साहन प्रदान
करना चाहता है।

कु छ ऄवत महत्त्िपूणव तथ्य


 ईद्देविका की प्रकृ वत ऄप्रितवनीय है ऄथावत् आसकी व्यिस्थाओं को न्यायालय द्वारा प्रिृत्त नहीं कराया
जा सकता है।
 ईद्देविका में कोइ भी संिोधन के िल ऄनुच्छेद 368 के तहत ईपबंवधत प्रकक्रया के ऄनुसार ही हो
सकता है।
 ऄब तक ईद्देविका को के िल एक बार 42िें संविधान संिोधन ऄवधवनयम, 1976 द्वारा 03-01-
1977 को संिोवधत कर, आसके पहले पैरा में दो िब्द समाजिादी और पंथवनरपेक्ष एिं छठे
पररच्छेद खंड में ऄखंडता िब्द जोड़ा गया।

2. ईद्दे विका की वस्थवत


2.1. पारं पररक दृ विकोण

पारंपररक दृविकोण यह है कक यह संविधान का ऄवभन्न ऄंग नहीं है, बवलक आसे ऄलग से जोड़ा गया है।
आसका अिय यह है कक यकद हम आसे संविधान से हटा देते हैं, तो यह कानून या ऄवधवनयम के प्रािधानों
को प्रभावित नहीं करेगा।

2.2. अधु वनक दृ विकोण

अधुवनक दृविकोण यह है कक ईद्देविका संविधान का एक ऄवभन्न ऄंग है और आसमें संसद द्वारा संिोधन
ककया जा सकता है।

1.2. सं विधान की ईद्दे विका में वनवहत मू ल तत्ि

संविधान की ईद्देविका में वनवहत चार मूल तत्ि वनम्नवलवखत हैं :


 संविधान की िवि का स्रोत: ईद्देविका में ईललेख है कक संविधान भारत के लोगों से िवि
ऄवधग्रवहत करता है।

110 www.visionias.in ©Vision IAS

Google it:- https://upscpdf.com


https://upscpdf.com << Download From >> https://upscpdf.com

 भारत की प्रकृ वतः ईद्देविका में ईललेख है कक भारत एक संप्रभु, समाजिादी, पंथवनरपेक्ष,
लोकतांवत्रक एिं गणतांवत्रक व्यिस्था िाला देि है।
 संविधान के ईद्देश्य: न्याय, स्ितंत्रता, समता एिं बंधत
ु ा।
 संविधान लागू होने की वतवथः 26 निंबर, 1949

1.3. क्या ईद्दे विका सं विधान का ऄं ग है ?

ईद्देविका संविधान का ऄंग है या नहीं, यह वििाद का विषय रहा है। बेरुबारी िाद (1960) में ईच्चतम
न्यायालय ने ईद्देविका को संविधान का ऄंग नहीं माना था। हालााँकक, के ििानंद भारती बनाम के रल
राज्य (1973) िाद में ईच्चतम न्यायालय ने ऄपने पूिव के संिैधावनक वनणवयों में संिोधन करते हुए कहा
कक ईद्देविका संविधान का ऄवभन्न ऄंग है।

1.4. ईद्दे विका के वनिव च न एिं सं िोधन से सं बं वधत विवभन्न िाद ( Cases)

बेरूबारी संघ, गोलकनाथ, के ििानंद भारती, रघुनाथ राि िाद अकद।

1.4.1. बे रू बारी सं घ िाद (1960)

 ईच्चतम न्यायालय ने कहा कक ईद्देविका, संविधान में वनवहत सामान्य प्रयोजनों को दिावती है। ऄत:
"यह संविधान वनमावताओं के विचारों को समझने की कुं जी है।" आसके ऄवतररि संविधान के
विवभन्न ऄनुच्छेदों में की गयी व्यिस्थाओं के ऄनेक ऄथव वनकलते हैं। आस व्यिस्था के ईद्देश्य को
संविधान में िावमल ककया गया है। ईच्चतम न्यायालय ने यह भी कहा कक ईद्देविका संविधान का
भाग नहीं है।
 प्रस्तािना के महत्ि को मान्यता प्रदान करने के बािजूद , ईच्चतम न्यायालय ने यह माना कक
प्रस्तािना संविधान का ऄंग नहीं है। पुनः यह संविधान द्वारा स्पि तौर पर प्रदत्त िवियों को
प्रवतबंवधत करने का स्रोत भी नहीं है। आसके ऄवतररि, प्रस्तािना न्यायालय में प्रितवनीय नहीं है।
 संविधान की व्याख्या में ईद्देविका का सीवमत महत्ि है।
 संविधान के प्रािधान के स्पि होने की वस्थवत में ईद्देविका को लागू नहीं ककया जा सकता है (ए. के .
गोपालन िाद, 1950)। गोपालन िाद में, ईच्चतम न्यायालय ने स्पि ककया कक ईद्देविका का
ईपयोग स्पि प्रािधानों की व्याख्या या िणवन करने के वलए नहीं ककया जाएगा। आस मामले में,
ईच्चतम न्यायालय ने कहा कक ऄनुच्छेद 21 (जीिन और स्ितंत्रता) में 'विवध' िब्द का ईपयोग
ककया गया है जो 'प्राकृ वतक विवध' को आंवगत नहीं करता बवलक राज्य द्वारा बनायी गइ 'विवध' को
संदर्भभत करता है।
 हालांकक, यह स्िीकार ककया कक संविधान के प्रािधानों में ऄस्पिता होने पर ईद्देविका का ईपयोग
ककया जा सकता है। ईच्चतम न्यायालय के ऄनुसार, ईद्देविका संविधान सभा के लोगों के ऄंतरमन
को समझने की कुं जी है।
ईच्चतम न्यायालय द्वारा प्रदत्त स्पिीकरण:
 यद्यवप संविधान की ईद्देविका साधारण विधान का ऄवभन्न ऄंग नहीं है, तथावप यह संिैधावनक
विधान का एक ऄवनिायव ऄंग है।
 ईच्चतम न्यायालय ने यह स्िीकार ककया कक बेरुबारी िाद में ईद्देविका से संबद्ध कु छ तथ्यों पर
ध्यान नहीं कदया गया। ये तथ्य ईद्देविका को संविधान के एक ऄंग के रूप में स्थावपत करते हैं।
o संविधान सभा ने संविधान के ऄन्य भागों के ऄनुरूप ही ईद्देविका को भी ऄंगीकृ त ककया था।
o िेष संविधान के ऄवधवनयमन के ईपरांत ईद्देविका को ऄवधवनयवमत ककया गया था। यह
सुवनवश्चत करने हेतु कक ईद्देविका और संविधान के ऄन्य प्रािधानों के मध्य कोइ ऄसंगवत नहीं

111 www.visionias.in ©Vision IAS

Google it:- https://upscpdf.com


https://upscpdf.com << Download From >> https://upscpdf.com

है, ईद्देविका को सबसे ऄंत में सवम्मवलत ककया गया। आसके विपरीत संयुि राज्य ऄमेररका में
संविधान की ईद्देविका को सबसे पहले ऄवधवनयवमत ककया गया था।

1.4.2. गोलकनाथ िाद (1967)

ईच्चतम न्यायालय ने कहा कक जहााँ संविधान की भाषा ऄस्पि या संकदग्ध हो, िहां ईसके ऄथव को स्पि
करने के वलए ईद्देविका का संदभव वलया जा सकता है।

1.4.3. के ििानं द भारती िाद (1973)

ईच्चतम न्यायालय ने बेरूबारी संघ िाद में कदए गए स्ियं के वनणवय को ऄस्िीकार कर कदया और यह
व्यिस्था दी कक ईद्देविका संविधान का एक ऄंग है। न्यायालय ने यह भी कहा कक संसद ऄनुच्छेद 368
के तहत आसमें संिोधन भी कर सकती है, लेककन ईद्देविका में वनवहत मूल ढांचे को संिोवधत नहीं ककया
जा सकता है।
 ईद्देविका का ऄत्यवधक महत्ि है और संविधान को ईद्देविका में वनवहत व्यापक और विविि
संकलपों के सन्दभव में परठत और वििेवचत ककया जाना चावहए।
 ईच्चतम न्यायालय ने स्पि ककया कक ईद्देविका का ईपयोग मूल ऄवधकारों और राज्य की नीवत के
वनदेिक वसद्धांतों के मध्य संबंधों की व्याख्या करने में ककया जा सकता है।
 ईच्चतम न्यायालय द्वारा 25िें संविधान संिोधन ऄवधवनयम, 1971 (वजसके तहत ऄनुच्छेद 31C
को जोड़ा गया) की िैधता की जांच में ईपरोि दृविकोण का ईपयोग ककया गया।

1.4.4. रघु नाथ राि बनाम भारत सं घ िाद ( 1993)

ईच्चतम न्यायालय ने वनम्नवलवखत वनणवय कदया:


 ईद्देविका िवि का स्रोत नहीं है, विवध की िवि का स्रोत विवनर्ददि ऄनुच्छेद ही हो सकता है।
 ईद्देविका विधानमंडल की िवियों पर प्रवतबंध अरोवपत करने का स्रोत नहीं है।
 संविधान के संकदग्ध तथा वद्वऄथी ईपबंधों का ऄथव स्पि करने के वलए ईद्देविका ईपयोगी है।

1.4.5. एस. अर. बोम्मइ िाद (1994)

“ईद्देविका संविधान का ऄवभन्न ऄंग है। सरकार का प्रजातांवत्रक स्िरुप, संघीय संरचना, राष्ट्रीय एकता
और ऄखंडता, पंथवनरपेक्षता, समाजिादी स्िरूप, सामावजक न्याय एिं न्यावयक पुनर्भिलोकन भी आसके
मूल ढांचे में सवम्मवलत हैं।”

1.4.6. एल. अइ. सी. ऑफ़ आं वडया िाद ( 1995)

ईच्चतम न्यायालय ने पुनः व्यिस्था दी कक ईद्देविका संविधान का एक ऄवभन्न ऄंग है।

1.5. ईद्दे विका में ईवललवखत प्रमु ख िब्दों की व्याख्या

ईद्देविका में कु छ प्रमुख िब्दों का ईललेख ककया गया है, जो आसमें वनवहत मूलयों एिं दिवन के द्योतक हैं।
ये हैं: संप्रभु, समाजिादी, पंथवनरपेक्ष, लोकतंत्रात्मक, गणराज्य, न्याय, स्ितंत्रता, समता ि बंधत
ु ा।
आनकी संक्षेप में वनम्नवलवखत व्याख्या की गइ है:

1.5.1. सं पू णव प्रभु त्ि-सं प न्न (Sovereign)

 संप्रभु िब्द का अिय है कक भारत ऄपने अंतररक तथा बाह्य मामलों का वनधावरण करने के वलए
स्ितंत्र है। यद्यवप िषव 1949 में भारत ने राष्ट्रमंडल की सदस्यता स्िीकार करते हुए विटेन को

112 www.visionias.in ©Vision IAS

Google it:- https://upscpdf.com


https://upscpdf.com << Download From >> https://upscpdf.com

आसका प्रमुख माना, तथावप राष्ट्रमंडल एिं संयुि राष्ट्र की सदस्यता ककसी भी तरह भारतीय
संप्रभुता को प्रभावित नहीं करती।
 एक संप्रभु राष्ट्र होने के कारण भारत ककसी विदेिी सीमा का ऄवधग्रहण कर सकता है ऄथिा ककसी
ऄन्य देि के पक्ष में ऄपनी सीमा के ककसी वहस्से से दािा छोड़ सकता है।
 िस्तुतः यह पद वबना ककसी बाह्य दबाि या प्रभाि के अत्मवनणवय की िवि का द्योतक है।
संप्रभुता िब्द ककसी राष्ट्र के स्ितंत्र स्िरूप को संदर्भभत करता है। आसका अिय है कक राष्ट्र को ककसी भी
विषय पर विवध वनमावण की िवि प्राप्त है; और यह ककसी भी ऄन्य राष्ट्र या बाहरी िवि के वनयंत्रण के
ऄधीन नहीं है।
पररणामतः भारत की ईद्देविका में ईवललवखत संप्रभु िब्द का ऄथव है कक भारत न तो ककसी ऄन्य राष्ट्र
पर वनभवर और न ही ऄवधराज्य (डोवमवनयन) है, बवलक यह एक स्ितंत्र राष्ट्र है। राष्ट्र से सिोच्च कोइ नहीं
है और यह ऄपने अंतररक एिं बाह्य दोनों मामलों को स्ियं वनस्तारण करने हेतु स्ितंत्र है।
िषव 1949 में राष्ट्रमंडल राष्ट्र की ऄपनी पूणव सदस्यता जारी रखने हेतु भारत की घोषणा ककसी भी
तरीके से भारत की संप्रभुता को प्रभावित नहीं करती है। यह घोषणा विवधयेत्तर है एिं संविधान में
आसका कोइ ईललेख नहीं है। यह एक स्िैवच्छक घोषणा है। यह संगठन स्ितंत्र राज्यों के मध्य एक
प्रवतवष्ठत संघ था।
यह विरटि ताज को के िल राष्ट्रमंडल के प्रतीकात्मक प्रमुख के रूप में स्िीकार करता है और भारत के
नागररकों की वनष्ठा पर कोइ प्रश्न नहीं करता। जैसा कक पंवडत जिाहरलाल नेहरू ने कहा था: "यह
स्ितंत्र आच्छा से ककया हुअ एक समझौता है, वजसे स्ितंत्र आच्छा द्वारा समाप्त ककया जा सकता है।"

1.5.1.1. सं प्र भु ता और िै श्वीकरण


 भारत हमेिा से ही ऄंतरावष्ट्रीय संस्थानों का समथवक रहा है। भारत संयुि राष्ट्र संघ का संस्थापक
सदस्य भी है। आसने ऄंतरावष्ट्रीय विवधयों के विकास में भी सकक्रय भूवमका वनभाइ है। हालााँकक यह
प्रश्नगत है कक क्या यह हमारे देि की संप्रभुता को सीवमत करता है? यद्यवप यह महसूस ककया
जाना चावहए कक संप्रभुता एक विवधक ऄिधारणा है। ककन्तु व्यिहार में, प्रायः आसके साथ
समझौता होता रहा है।
 हालााँकक, यह दृविकोण कक िैश्वीकरण संप्रभुता को सीवमत करता है, आस पर पुनः चचाव करने की
अिश्यकता है। िैश्वीकरण से अिय संप्रभुता का ह्रास होना नहीं है, बवलक यह ईस तरीके में एक
संिोधन है वजस रीवत से संप्रभुता का प्रयोग ककया जाता है। िैश्वीकरण की दौर में संप्रभुता को
ऄत्यवधक ईत्तरदायी पूणव ईपयोग करने की अिश्यकता है।
 भारत सरकार ऄंतरावष्ट्रीय क्षेत्र में ऄपनी रणनीवतक स्िायत्तता सुवनवश्चत करने और भारत के लोगों
के वहतों की रक्षा के प्रवत सतकव है। खाद्य सवब्सडी के संबंध में विश्व व्यापार संगठन में हावलया
वििाद आसका ईदाहरण है।

1.5.2. समाजिादी (Socialist)

 मूल ईद्देविका में, समाजिादी िब्द का ईललेख नहीं था, क्योंकक संविधान हमारे देि को ककसी
विविि अर्भथक संरचना के रूप में वनरुवपत नहीं करता है। लेककन िषव 1976 में 42िें संविधान
संिोधन ऄवधवनयम के द्वारा संविधान में समाजिादी िब्द जोड़ा गया।
 दृिव्य है कक भारतीय समाजिाद ‘लोकतांवत्रक समाजिाद’ है न कक ‘राज्य समाजिाद’ (‘साम्यिादी
समाजिाद’) वजसमें ईत्पादन और वितरण के सभी साधनों का राष्ट्रीयकरण तथा वनजी संपवत्त का
ईन्मूलन सवम्मवलत है।

113 www.visionias.in ©Vision IAS

Google it:- https://upscpdf.com


https://upscpdf.com << Download From >> https://upscpdf.com

 िस्तुतः लोकतांवत्रक समाजिाद, वमवित ऄथवव्यिस्था का समथवक होता है, जहां सािवजवनक ि
वनजी क्षेत्र साथ-साथ मौजूद रहते हैं। भारतीय समाजिाद, माक्सविाद और गांधीिाद का एक
वमवित रूप है, वजसमें ‘गांधीिादी समाजिाद’ की ओर ऄवधक झुकाि है।
यद्यवप “समाजिाद” िब्द का अिय ऄस्पि प्रतीत होता है, तथावप ईच्चतम न्यायालय के दृविकोण में
आसका मुख्य ईद्देश्य अय, सामावजक वस्थवत एिं जीिन-स्तर में व्याप्त विषमता को समाप्त करना है,
साथ ही, कामकाजी लोगों को एक ईत्कृ ि जीिन स्तर प्रदान करना है। आसवलए, भारतीय संविधान
वनजी संपवत्त को पूरी तरह से समाप्त करने के पक्ष में नहीं है, बवलक आसे संतुवलत करना चाहता है ताकक
आसका ईपयोग राष्ट्र के वहत में ककया जा सके , वजसमें ग़रीबों का ईत्थान भी िावमल है। संपवत्त तथा
ईद्योगों के पूणव राष्ट्रीयकरण के बजाय, यह एक 'वमवित ऄथवव्यिस्था' की पररकलपना करता है, साथ ही,
आसका ईद्देश्य सभी को 'समान ऄिसर' प्रदान करना एिं 'वनवहत स्िाथों' को समाप्त करना है।
हालााँकक, कु छ लोगों द्वारा यह तकव कदया जाता है कक भारत ऄपने समाजिाद के मागव से विचवलत हो
रहा है। आस तकव के पक्ष में वनम्नवलवखत कारणों को ईद्धृत ककया जाता है:
 नि-ईदारिादी अर्भथक नीवत का ऄपनाया जाना: ईदारीकरण, वनजीकरण तथा िैश्वीकरण की नइ
अर्भथक नीवत (1991) ने भारत के समाजिादी साख को कमजोर ककया है।
 िषव 1991 के ईपरांत, समाजिाद से दूर जाने और वनजीकरण की ओर ऄग्रसर होने की प्रिृवत्त रही
है। कइ सािवजवनक ईद्यमों में वनजी व्यवियों एिं ईद्योगों के पक्ष में विवनिेि ककया गया है, साथ
ही, सािवजवनक क्षेत्र के वलए अरवक्षत कइ क्षेत्रकों को वनजी ईद्यवमयों के वलए खोल कदया गया है।
 1990 के दिक के सुधारों के ईपरांत हुइ संिृवद्ध के पररणामस्िरूप ऄंत:राज्यीय और ऄंतर-
राज्यीय विषमताओं में िृवद्ध हुइ है। आसके ऄवतररि, आस संिृवद्ध से ककसी प्रकार का साथवक
रोजगार सृजन नहीं हुअ है (रोजगारविहीन संिृवद्ध)।
 भारत व्यापक रूप से वनधवनता ईन्मूलन में विफल रहा है।
हालांकक, आस तथ्य पर भी ध्यान कदया जाना चावहए कक सोवियत संघ एिं पूिी यूरोपीय राष्ट्रों में
समाजिाद के पतन के पश्चात् ये विकास-क्रम विश्व के कइ राष्ट्रों में दृविगत हुए हैं। साथ ही, राज्य द्वारा
भूवम ऄवधग्रहण ककए जाने की वस्थवत में वनजी भूस्िावमयों को क्षवतपूर्भत का भुगतान करने की
संिैधावनक बाध्यता को 44िें संविधान संिोधन ऄवधवनयम, 1978 द्वारा ऄनुच्छेद 31 को वनरवसत
करके समाप्त कर कदया गया है। आसके ऄवतररि, नि-ईदारिादी अर्भथक मॉडल की सीमाओं को महसूस
ककया जा रहा है तथा समािेिी विकास के अदिव को नीवत-वनमावण की प्राथवमकता में पुनः जगह दी जा
रही है।

1.5.3. पं थ -वनरपे क्ष (Secular)

 भारत के संदभव में पंथ-वनरपेक्ष का ऄवभप्राय है कक भारत ककसी एक धमव या धार्भमक विचारधारा
से वनदेवित नहीं होता है। ककतु आसका ऄथव यह नहीं है कक राज्य (देि) ककसी धमव वििेष के विरुद्ध
है। यह ऄपने सभी नागररकों को ककसी भी धमव को मानने, अचरण करने और प्रचार करने की
स्ितंत्रता प्रदान करता है। साथ ही, संविधान धार्भमक अधार पर ककसी भी प्रकार के भेद -भाि पर
भी रोक लगाता है। आस प्रकार, पंथवनरपेक्षता से अिय धमव के अधार पर भेद-भाि का वनषेध
और सभी धमों के प्रवत समान भाि से भी है।
 पंथवनरपेक्षता की पवश्चमी ऄिधारणा के ऄनुसार धमव और राज्य (राष्ट्र) दोनों पृथक-पृथक हैं। आसके
ऄनुसार धार्भमक संस्थानों एिं पदावधकाररयों का राज्य के प्रावधकाररयों से पृथक्करण होना
चावहए।

114 www.visionias.in ©Vision IAS

Google it:- https://upscpdf.com


https://upscpdf.com << Download From >> https://upscpdf.com

 िषव 1974 में ईच्चतम न्यायालय ने स्पि ककया कक यद्यवप ‘पंथवनरपेक्ष राज्य’ िब्द का स्पि रूप से
संविधान में ईललेख नहीं ककया गया है तथावप आसमें कोइ संदह
े नहीं है कक संविधान वनमावता आसी
प्रकार के राज्य की स्थापना करना चाहते थे। आसवलए, संविधान में ऄनुच्छेद 25 से 28 तक
धार्भमक स्ितंत्रता से संबंवधत ऄवधकारों को जोड़ा गया।
 कालांतर में पंथ-वनरपेक्ष िब्द को 42िें संविधान संिोधन ऄवधवनयम, 1976 द्वारा ईद्देविका में
जोड़ा गया।
पंथवनरपेक्षता के संबध
ं में संिध
ै ावनक प्रािधान
 राष्ट्र का ऄपना कोइ धमव नहीं है।
 विवध के समक्ष सभी नागररक समान हैं। ऄनुच्छेद 14, 15 और 27 राज्य की धमववनरपेक्ष प्रकृ वत
को स्थावपत करते हैं। ऄनुच्छेद 15 के तहत विविि वनदेि है कक राज्य के िल धमव , मूलिंि, जावत,
ललग या जन्म स्थान के अधार पर नागररकों के बीच भेदभाि नहीं करेगा। हालांकक सकारात्मक
कारविाइ की ऄनुमवत है, लेककन के िल धमव के अधार पर नहीं।
 मूल ऄवधकार में िर्भणत ऄनुच्छेद 25-28, सभी व्यवियों को ऄपने धमव को मानने, अचरण और
प्रचार करने की स्ितंत्रता तथा सभी धमों के प्रवत राज्य एिं ईसके संस्थानों की ओर से ककसी
प्रकार के भेदभाि न करने का अश्वासन देते हैं।
 राज्य धमव के ऄधीन होने के बजाय धमव राज्य के ऄधीन है। आसका तात्पयव यह भी है कक सामावजक
सुधारों के ईद्देश्य से राज्य धार्भमक मामलों में हस्तक्षेप कर सकता है।
 ऐवतहावसक रूप से विकवसत भारतीय पंथवनरपेक्षता की एक ऄवद्वतीय वििेषता यह है कक
ऄलपसंख्यकों को ईनकी संस्कृ वत और परंपराओं के संरक्षण के संबंध में वििेष सुरक्षा प्रदान की गइ
है।

1.5.4. लोकतं त्रात्मक (Democratic)

 संविधान की ईद्देविका में एक लोकतांवत्रक राजव्यिस्था की पररकलपना की गइ है, जहााँ सिोच्च


िवि जनता में वनवहत है। लोकतंत्र में प्रत्येक नागररक को ऄपने देि के िासन में भाग लेने का
ऄवधकार है। लोकतंत्र जनता द्वारा, जनता के वलए, जनता का िासन है।
 लोकतंत्र के दो प्रमुख प्रकार हैं- प्रत्यक्ष ि ऄप्रत्यक्ष। प्रत्यक्ष लोकतंत्र में लोग ऄपनी राजनीवतक एिं
प्रिासवनक िवियों का प्रयोग प्रत्यक्ष रूप से करते हैं, जैसे - वस्िटज़रलैंड में। प्रत्यक्ष लोकतंत्र को
जनमत संग्रह (Referendum), आवनविएरटि (Initiative), प्रत्याितवन या प्रत्यािी को िापस
बुलाना (Recall) तथा प्लेबीसाआट (Plebiscite) के माध्यम से सुवनवश्चत ककया जा सकता है।
दूसरी ओर ऄप्रत्यक्ष लोकतंत्र में लोगों द्वारा चुने गए प्रवतवनवध जनता की ओर से सिोच्च िवि का
प्रयोग करते हैं और सरकार चलाते हुए विवध वनमावण करते हैं। आस प्रकार के लोकतंत्र को
प्रवतवनवध/प्रवतवनवधत्िकारी लोकतंत्र भी कहा जाता है। यह दो प्रकार का होता है: संसदीय और
ऄध्यक्षीय।
 भारतीय संविधान में प्रवतवनवधत्िकारी संसदीय लोकतंत्र (Representative Parliamentary
Democracy) की व्यिस्था है, वजसमें कायवपावलका ऄपनी सभी नीवतयों और कायों के वलए
विधावयका के प्रवत जिाबदेह होती है। ियस्क मतावधकार, सामवयक चुनाि, विवध की सिोच्चता,
न्यायपावलका की स्ितंत्रता ि भेदभाि का ऄभाि भारतीय राजव्यिस्था में वनवहत लोकतांवत्रक
लक्षणों को वनर्ददि करते हैं।
 संविधान की ईद्देविका में लोकतांवत्रक िब्द का प्रयोग व्यापक ऄथव में ककया गया है, वजसमें न
के िल राजनीवतक लोकतंत्र बवलक सामावजक ि अर्भथक लोकतंत्र को भी िावमल ककया गया है।
लोकतंत्र के तहत के िल लोकतांवत्रक सरकार की ही कलपना नहीं गइ है बवलक ऐसे समाज की भी
पररकलपना की गइ है वजसमें विचारों का मुि अदान-प्रदान हो और प्रत्येक व्यवि की समाज में
समान प्रवतष्ठा हो।

115 www.visionias.in ©Vision IAS

Google it:- https://upscpdf.com


https://upscpdf.com << Download From >> https://upscpdf.com

1.5.5. गणराज्य (Republic)

 गणराज्य से ऄवभप्राय ऐसी व्यिस्था से है जहााँ राजनीवतक संप्रभुता ककसी एक व्यवि (जैसे- राजा)
में के वन्ित होने के स्थान पर जनता में वनवहत होती है और कोइ भी वििेषावधकार प्राप्त िगव
विद्यमान नहीं होता है।
 प्रत्येक सािवजवनक कायावलय वबना ककसी भेदभाि के प्रत्येक नागररक के वलए खुला होता है।
 गणतंत्र में राज्य प्रमुख को सदैि प्रत्यक्ष ऄथिा ऄप्रत्यक्ष रूप से एक वनवश्चत समय के वलए चुना
जाता है (जैस-े ऄमेररका ऄथिा भारत में)। भारत में राज्य के प्रमुख (राष्ट्रपवत) को वनिावचन द्वारा
पद प्राप्त होता है, अनुिांविकता के अधार पर नहीं। ईसका चुनाि पांच िषव के वलए ऄप्रत्यक्ष
रीवत से ककया जाता है।

1.5.6. न्याय (Justice)

 न्याय का सामान्य ऄथव एक ऐसी वस्थवत से है जहााँ ककसी भी प्रकार का भेदभाि नहीं हो और
सबको ईनके ईवचत ऄवधकार प्राप्त हों। ईद्देविका में तीन प्रकार के न्याय की संकलपना की गयी है:
सामावजक, अर्भथक ि राजनीवतक। सामावजक, अर्भथक ि राजनीवतक न्याय के तत्िों को िषव
1917 की रूसी क्रांवत से ग्रहण ककया गया है। आनकी सुरक्षा मूल ऄवधकार ि नीवत वनदेिक तत्त्िों
के विवभन्न ईपबंधों द्वारा की जाती है।
 सामावजक न्याय से ऄवभप्राय ऐसी व्यिस्था से है जहााँ जावत, मूलिंि, ललग, जन्म स्थान, धमव या
भाषा अकद में से ककसी भी अधार पर ककसी के साथ भेद -भाि न ककया जाए तथा समाज में
सबको समान ऄिसर/स्थान प्राप्त हो। आसका ऄथव है समाज में ककसी िगव वििेष के वलए
वििेषावधकारों की ऄनुपवस्थवत और ऄनुसूवचत जावत, जनजावत, ऄन्य वपछड़े िगों तथा मवहलाओं
की वस्थवत में सुधार ककया जाना।
 अर्भथक न्याय का ऄथव है कक अर्भथक कारणों के अधार पर ककसी भी व्यवि से भेदभाि नहीं ककया
जाएगा। आसमें संपदा, अय ि संपवत्त की ऄसमानता को समाप्त करना भी िावमल है।
 राजनीवतक न्याय का ऄथव है कक प्रत्येक व्यवि को समान राजनीवतक ऄवधकार प्राप्त होंगे। सभी
नागररकों को समान रूप से मतदान का, चुनाि लड़ने का तथा लोक पद धारण करने का ऄवधकार
प्राप्त हो।
ईद्देविका में वनवहत न्याय िब्द के भाि से संबंवधत कु छ मूल ऄवधकार / नीवत वनदेिक तत्त्ि / ऄन्य
ऄनुच्छेद
सामावजक न्याय
ऄनुच्छेद 17: ऄस्पृश्यता/छु अछू त का ऄंत; भारतीय संसद ने ऄस्पृश्यता वनषेध ऄवधवनयम, 1955
ऄवधवनयवमत कर आसे दंडनीय ऄपराध घोवषत ककया है।
ऄनुच्छेद 18: ईपावधयों का ऄंत ककया गया है। राज्य, सैन्य और िैवक्षक क्षेत्र के ऄवतररि कोइ ईपावध
प्रदान नहीं करेगा (भारत रत्न एिं पद्म ऄलंकरण, पाने िाले के नाम के अगे या पीछे प्रयोग नहीं ककए
जा सकते, आसवलए िे ईपावध नहीं हैं)।
ऄनुच्छेद 46: ऄनुसूवचत जावतयों, ऄनुसूवचत जनजावतयों तथा ऄन्य दुबवल िगों के विक्षा और ऄथव
संबंधी वहतों की ऄवभिृवद्ध।
राजनीवतक न्याय
ऄनुच्छेद 14-18 में कदए गए समता के ऄवधकार राजनीवतक न्याय के अधार हैं। आसके ऄवतररि
वनम्नवलवखत ऄनुच्छेद राजनीवतक न्याय सुवनवश्चत करते हैं:
ऄनुच्छेद 38: राज्य, लोक कलयाण की ऄवभिृवद्ध के वलए सामावजक व्यिस्था करेगा।
ऄनुच्छेद 39: राज्य द्वारा ऄनुसरणीय कु छ नीवत वनदेिक तत्त्ि।
ऄनुच्छेद 39A: समान न्याय और वन:िुलक विवधक सहायता।
ऄनुच्छेद 41: कु छ दिाओं में काम, विक्षा और लोक सहायता पाने का ऄवधकार।

116 www.visionias.in ©Vision IAS

Google it:- https://upscpdf.com


https://upscpdf.com << Download From >> https://upscpdf.com

1.5.7. स्ितं त्र ता (Liberty)

 ईद्देविका में विचार, ऄवभव्यवि, विश्वास, धमव ि ईपासना की स्ितंत्रता ईवललवखत है। आसका
ऄवभप्राय यह है कक सभी नागररकों को समान रूप से आच्छानुसार ऄपने धमव का पालन करने और
ऄपने विचारों को व्यि करने की स्ितंत्रता है और राज्य आन विषयों में तब तक हस्तक्षेप नहीं
करेगा जब तक कक दूसरों की स्ितंत्रता ऄथिा ऄवधकार बावधत न हों।
 हालांकक स्ितंत्रता का ऄवभप्राय यह नहीं है कक प्रत्येक व्यवि को कु छ भी करने का ऄवधकार वमल
गया है। स्ितंत्रता के ऄवधकार का प्रयोग संविधान में ईवललवखत सीमाओं के ऄंतगवत ही ककया जा
सकता है। संक्षेप में कहा जाए तो ईद्देविका में प्रदत्त स्ितंत्रताएाँ (एिं मूल ऄवधकार) वनरपेक्ष नहीं
हैं। ईद्देविका में ईवललवखत स्ितंत्रता, समता और बंधत ु ा के अदिों को फ्ांसीसी क्रांवत (1789-
1799 इ.) से ग्रहण ककया गया है।

ईद्देविका में वनवहत स्ितंत्रता िब्द के भाि से संबंवधत कु छ मूल ऄवधकार / नीवत वनदेिक तत्त्ि/ ऄन्य
ऄनुच्छेद
ऄनुच्छेद 19: िाक् एिं ऄवभव्यवि की स्ितंत्रता, िांवतपूिवक और वनरायुध सम्मेलन की स्ितंत्रता,
संगम या संघ बनाने की स्ितंत्रता, भारत के राज्यक्षेत्र में सिवत्र ऄबाध संचरण की स्ितंत्रता, भारत के
राज्यक्षेत्र के ककसी भाग में वनिास करने और बस जाने की स्ितंत्रता तथा कोइ िृवत्त, ईपजीविका,
व्यापार या कारोबार करने की स्ितंत्रता।
ऄनुच्छेद 25: ऄंत:करण एिं धमव के ऄबाध रूप से मानने, अचरण और प्रचार करने की स्ितंत्रता।
ऄनुच्छेद 26: धार्भमक कायों के प्रबंध की स्ितंत्रता।
ऄनुच्छेद 27: ककसी विविष्ट धमव की ऄवभिृवद्ध के वलए करों के संदाय के बारे में स्ितंत्रता।
ऄनुच्छेद 28: कु छ विक्षा संस्थाओं में धार्भमक विक्षा या धार्भमक ईपासना में ईपवस्थत होने के बारे में
स्ितंत्रता।

1.5.8. समता (Equality)

 समता का ऄवभप्राय है कक न्याय, कराधान, सािवजवनक पद और वनयोजन के संबंध में सभी के साथ
एक समान व्यिहार ककया जाना। आसके ऄंतगवत, समाज के ककसी भी िगव के वलए वििेषावधकार
का न होना भी िावमल है। हमारे संविधान में मूल ऄवधकारों से संबंवधत ऄनुच्छेद 14 - 18 के
ऄंतगवत आस वसद्धांत को प्रभािी बनाया गया है।
 भारतीय संविधान की ईद्देविका प्रत्येक नागररक को प्रवतष्ठा और ऄिसर की समता प्रदान करती
है। आस ईपबंध में समता के तीन अयाम िावमल हैं: नागररक, राजनीवतक ि अर्भथक।

मूल ऄवधकारों के तहत वनम्नवलवखत प्रािधान नागररक समता को सुवनवश्चत करते हैं:
ऄनुच्छेद 14 - 18
 विवध के समक्ष समता (ऄनुच्छेद-14)
 धमव, मूलिंि, जावत, ललग या जन्म स्थान के अधार पर विभेद का प्रवतषेध (ऄनुच्छेद-15)
 लोक वनयोजन के विषय में ऄिसर की समता (ऄनुच्छेद-16)
 ऄस्पृश्यता का ऄंत (ऄनुच्छेद-17)
 ईपावधयों का ऄंत (ऄनुच्छेद-18)
संविधान में दो ऄन्य ईपबंध हैं जो राजनीवतक समता को सुवनवश्चत करते प्रतीत होते हैं:
 प्रथम; धमव, जावत, ललग ऄथिा िगव के अधार पर ककसी व्यवि को मतदाता सूची में िावमल होने
से ऄयोग्य न ठहराना (ऄनुच्छेद-325); तथा
 दूसरा; लोकसभा और विधानसभाओं के वलए ियस्क मतदान का प्रािधान (ऄनुच्छेद-326)।
अर्भथक समता के तहत राज्य की नीवत के वनदेिक तत्त्ि, मवहला तथा पुरुष को जीिन यापन के वलए
पयावप्त साधन और समान काम के वलए समान िेतन के ऄवधकार को सुरवक्षत करते हैं (ऄनुच्छेद-39)।

117 www.visionias.in ©Vision IAS

Google it:- https://upscpdf.com


https://upscpdf.com << Download From >> https://upscpdf.com

1.5.9. बं धु ता (Fraternity)

 बंधुता का ऄथव है: भाइचारे की भािना। मूल कत्तवव्य (ऄनुच्छेद-51क) में भी यह ईललेख है कक
“प्रत्येक भारतीय नागररक का यह कत्तवव्य होगा कक िह धार्भमक, भाषायी, क्षेत्रीय ऄथिा िगव पर
अधाररत सभी भेदभाि से परे होकर सौहािव और अपसी भाइचारे की भािना को प्रोत्सावहत
करेगा”।
 ईद्देविका में बंधुता के संबंध में वनम्नवलवखत दो बातों का ईललेख है: (a) व्यवि की गररमा
सुवनवश्चत करना एिं (b) देि की एकता और ऄखंडता सुवनवश्चत करना। आन्हीं कारणों से संविधान
में ऄस्पृश्यता का वनषेध (ऄनुच्छेद 17) ककया गया है और ऐसे व्यिहार को ऄपराध घोवषत ककया
गया है।

2. ईद्दे विका, मूल ऄवधकार तथा राज्य की नीवत के वनदे ि क


तत्त्िों के मध्य पारस्पररक संबंध
ईद्देविका, मूल ऄवधकार तथा राज्य की नीवत के वनदेिक तत्त्ि, भारत के संविधान के ऄवभन्न ऄंग हैं।
 ईद्देविका संविधान का भाग है और आसवलए संिोधनीय भी है, ककतु आसमें ईवललवखत संविधान के
मूल ढांचे में कोइ संिोधन नहीं ककया जा सकता (के ििानंद भारती िाद, 1973)।
 संविधान के भाग-3 (ऄनुच्छेद 12-35) में प्रदत्त मूल ऄवधकार ‘स्ितंत्रता तथा समानता’ के संकलप
की व्यािहाररक प्रावप्त को सुवनवश्चत करने िाले साधन हैं। ये न्यायालयों द्वारा प्रितवनीय ऄवधकार
हैं। ये प्रवतषेधकारी हैं तथा राज्य पर नकारात्मक दावयत्ि अरोवपत करते हैं।
 संविधान के भाग-4 में कदए गए राज्य की नीवत के वनदेिक तत्त्ि, ‘सामावजक तथा अर्भथक न्याय’
के लक्ष्य की प्रावप्त हेतु राज्य के सकारात्मक कत्तवव्य हैं। आनमें ऄन्तर्भनवहत वसद्धांत, राष्ट्र के िासन के
मूलभूत वसद्धांत हैं। ये न्यायालय द्वारा प्रितवनीय नहीं हैं। ये िासन, प्रिासन ि विधावयका द्वारा
सुवनवश्चत ककए जाने योग्य संिैधावनक ऄपेक्षाएं हैं।
 ऄतः ईद्देविका, मूल ऄवधकार तथा राज्य की नीवत के वनदेिक तत्त्ि एक ही संिैधावनक ढांचे के
ऄवभन्न ऄंग हैं और समान रूप से महत्िपूणव हैं।

3. ईद्दे विका से संबंवधत हावलया वििाद


पृिभूवम
 26 निंबर 2015 एिं 26 जनिरी 2016 (क्रमिः संविधान के ऄंगीकृ त एिं लागू होने की 66िीं
िषवगाठ) को दो ऄलग-ऄलग सरकारी विज्ञापनों में मूल ईद्देविका का विज्ञापन प्रकावित ककया
गया था।
 आस प्रकार ईि विज्ञापन में 42िें संविधान संिोधन द्वारा ईद्देविका में जोड़े गए तीन िब्दों -
समाजिादी, पंथवनरपेक्ष एिं ऄखंडता - को स्थान नहीं कदया गया।
 आसके बाद विवभन्न विपक्षी दलों एिं नागररक समाज के कु छ कायवकतावओं द्वारा सरकार के आस
कदम की अलोचना की गयी।
 आसी दौरान कें ि सरकार के एक मंत्री के आस बयान पर कक, “आस बात पर ऄब बहस होनी चावहए
कक, क्या ‘समाजिादी’ एिं ‘पंथवनरपेक्ष’ जैसे िब्दों का ईद्देविका में स्थान कदया जाना चावहए
ऄथिा नहीं।” यह वििाद सुर्भियों में रहा था।

118 www.visionias.in ©Vision IAS

Google it:- https://upscpdf.com


https://upscpdf.com << Download From >> https://upscpdf.com

समाजिादी एिं पंथवनरपेक्ष िब्दों को हटाने के पक्ष में तकव


 समाजिाद िब्द का अिय है: “पूाँजी, सम्पवत्त, भूवम अकद ईत्पादन के साधनों पर राज्य का
स्िावमत्ि एिं आनके वितरण में समानता”। यही कारण है कक संविधान वनमावताओं ने जान-बूझकर
समाजिादी िब्द का प्रयोग ईद्देविका में नहीं ककया था, क्योंकक िे देि को ककसी विविि अर्भथक
संरचना से संबद्ध नहीं करना चाहते थे।
 पंथवनरपेक्ष का अिय है: “धार्भमक ऄथिा अध्यावत्मक मामलों से राज्य का संबद्ध न होना एिं
राज्य का संचालन ककसी धार्भमक वनयम से न होकर संविधान द्वारा होना। ईद्देविका के लहदी
रुपांतरण में ऄंग्रेजी के सेक्युलर िब्द के वलए ऄत्यवधक विचार-विमिव के पश्चात् धमववनरपेक्ष के
स्थान पर पंथवनरपेक्ष िब्द का प्रयोग ककया गया है, क्योंकक हमारा संविधान एिं देि के विवभन्न
कानून, राज्य से धमव से वनरपेक्ष न होकर पंथ से वनरपेक्ष होने की ऄपेक्षा करते हैं।
 मूल ईद्देविका में ‘भारत के समस्त नागररकों को सामावजक, अर्भथक एिं राजनीवतक न्याय
सुवनवश्चत करने का लक्ष्य रखा गया है। आस प्रकार, सामावजक एिं अर्भथक न्याय िस्तुतः एक
समाजिादी राज्य को ही वनरुवपत करते हैं। राज्य की नीवत के वनदेिक तत्त्िों (DPSPs) के
ऄंतगवत आन्हें (सामावजक एिं अर्भथक न्याय) विविि स्थान प्रदान ककया गया है, ऄतः ऐसे में
ईद्देविका में वनर्ददि समाजिादी एिं पंथवनरपेक्ष िब्द राजनीवतक ईद्देश्यों को आंवगत करते हैं।
 मूल ईद्देविका में “विचार, ऄवभव्यवि, विश्वास, धमव और ईपासना की स्ितंत्रता” का भी ईललेख
ककया गया है। हम जानते हैं कक एक सेक्युलर राज्य में ही आन स्ितंत्रताओं की प्रावप्त संभि है। मूल
ऄवधकार (FR) एिं DPSP के विवभन्न ईपबंध आसकी रक्षा करते हैं।
 आन िब्दों को हटाये जाने के पीछे यह तकव भी कदया जाता है कक, “03-01-1977 को 42िें
संविधान संिोधन द्वारा समाजिादी एिं पंथवनरपेक्ष िब्दों को ईद्देविका में समाविि करने से पूिव
क्या भारत एक समाजिादी एिं पंथवनरपेक्ष राज्य नहीं था।”
 िषव 1991 के अर्भथक सुधारों एिं िैश्वीकरण के पश्चात्, सैद्धांवतक रूप में ितवमान में समाजिाद
िब्द लगभग ऄप्रासंवगक हो गया है और िषव 1991 के पश्चात् यह व्यिहार में कम ही पररलवक्षत
हुअ है।
समाजिादी एिं पंथवनरपेक्ष िब्दों को न हटाने के पक्ष में तकव
 समाजिादी एिं पंथवनरपेक्ष जैसे िब्द हमारे संविधान के अदिव हैं और ये संपूणव संविधान में
ऄंतर्भनवहत हैं। आन्हें हटाने से राष्ट्रीय एकता एिं बंधुता नकारात्मक रूप से प्रभावित होगी।
 पंथवनरपेक्ष िब्द को हटाने से देि में सांप्रदावयक विभाजन को बढ़ािा वमल सकता है।
 FR, DPSP, अरक्षण, वििेष िगों के वलए वििेष प्रािधान, पांचिीं एिं छठिीं ऄनुसूवचयां अकद
एक लोकतांवत्रक समाजिादी राज्य की पररकलपना करते हैं। ऄतः ईद्देविका में समाजिादी िब्द
आन्हें स्पिता प्रदान करता है। ईच्चतम न्यायालय ने भी कहा है कक ईद्देविका संविधान की व्याख्या में
सहायक है।
 42िें संविधान संिोधन द्वारा समाजिादी एिं पंथवनरपेक्ष के ऄवतररि “ऄखंडता” िब्द को भी
समाविि ककया गया था। ऄत: क्या ऄखंडता िब्द को भी हटा कदया जाना चावहए? आसका ईत्तर
है, नहीं, ऄवपतु हमें आन्हें बनाए रखने पर ध्यान कें कित करना चावहए, न कक आनका राजनीवतकरण
करना चावहए।
अगे की राह
 ईद्देविका में ईवललवखत समाजिादी एिं पंथवनरपेक्ष िब्दों पर बहस करने की बजाए गुणित्तापूणव
विक्षा, स्िास्थ्य, गरीबी ईन्मूलन, अिास, गररमापूणव जीिन, रोजगार अकद पर बहस होनी
चावहए।

119 www.visionias.in ©Vision IAS

Google it:- https://upscpdf.com


https://upscpdf.com << Download From >> https://upscpdf.com

 समाजिादी िब्द को हटाने के बजाए आस पर बहस होनी चावहए कक “सभी को िास्तविक समता
की वस्थवत” प्राप्त हो एिं समाज के िोवषत िगव को गररमापूणव जीिन एिं ऄवधकारविहीन िगव को
ऄवधकार प्राप्त हों।
 ईद्देविका में ईवललवखत न्याय (सामावजक, अर्भथक एिं राजनीवतक) िब्द पर बहस होनी चावहए।
 ऄंततः यह कहा जा सकता है कक ईद्देविका में ईवललवखत ये िब्द संकीणव न होकर व्यापक ऄथव की
ओर हमारा ध्यान अकर्भषत करते हैं। ऄतः हमें आनका सम्मान करना चावहए एिं आन अदिों को
प्राप्त करने की ओर ऄग्रसर होना चावहए।

4. ईपसंहार
 ईद्देविका का लक्ष्य एक ऐसी सामावजक व्यिस्था स्थावपत करना है जहां जनता संप्रभु हो; सरकार
वनिाववचत हो और जनता के प्रवत ईत्तरदायी हो; िासन की सत्ता, जनता के मूल ऄवधकारों से
सीवमत हो तथा जनता को ऄपने विकास का समुवचत ऄिसर प्राप्त हो।
 यद्यवप िैधावनक रूप से ईद्देविका न्यायालयों द्वारा लागू नहीं की जा सकती है तथावप यह
संविधान के विवभन्न प्रािधानों की व्याख्या करने में ईपयोगी वसद्ध हो सकती है तथा
कककतवव्यविमूढ़ता की वस्थवत में यह पथ प्रदिवक के रूप में कायव करती है।

5. विगत िषों में Vision IAS GS मेंस टे स्ट सीरीज में पू छे


गए प्रश्न (Previous Year Vision IAS GS Mains Test
Series Questions)
1. ईद्देविका पर वििाद के अलोक में, आसकी प्रवस्थवत और संिोधनीयता पर ईभरे दृविकोणों की
वििेचना कीवजए। आस संदभव में, ईस ईद्देश्य पर चचाव कीवजए जो सलिधान की ईद्देविका
हमारी राजव्यिस्था को प्रदान करती है।
दृविकोण:
 संविधान की ईद्देविका के बारे में संक्षेप में चचाव कीवजए।
 ईच्चतम न्यायालय द्वारा विवभन्न मामलों में व्यि की गयी राय के अधार पर ईद्देविका
की प्रवस्थवत एिं ईसकी संिोधनीयता के संबंध में चचाव कीवजए।
 हाल ही के वििाद के संबंध में आस पर संक्षेप में चचाव कीवजए।
 भारतीय राजव्यिस्था में संविधान की ईद्देविका में ईवललवखत ईद्देश्यों की वििेचना
कीवजए।
ईत्तर:
ईद्देविका, संविधान के मूल दिवन एिं अधारभूत मूलयों को मूतव रूप प्रदान करती है, वजन पर
संविधान अधाररत है। स्ितंत्रता पश्चात् ईद्देविका के संबंध में ईत्पन्न वििादों में से एक वििाद
यह था कक क्या यह संविधान का भाग है ऄथिा नहीं। न्यायपावलका ने विवभन्न मामलों में
ऄलग-ऄलग व्याख्याएं की हैं। ईदाहरणाथव:
 बेरुबारी संघ मामले में ईच्चतम न्यायालय ने कहा कक ईद्देविका, संविधान के विवभन्न
प्रािधानों में वनवहत सामान्य ईद्देश्यों को प्रदर्भित करती है, परन्तु ईच्चतम न्यायालय ने
आसे संविधान का भाग नहीं माना।
 के ििानन्द भारती मामले में, ईच्चतम न्यायालय ने ऄपने पूिव की राय को वनरस्त कर
कदया और यह माना कक ईद्देविका संविधान का भाग है।
ईद्देविका की संिोधनीयता
 के ििानन्द भारती मामले में, ईच्चतम न्यायालय ने माना कक ईद्देविका को संिोवधत
ककया जा सकता है ककतु ितव यह है कक ईस संिोधन द्वारा संविधान के मूल ढांचे में
पररितवन नहीं होना चावहए।

120 www.visionias.in ©Vision IAS

Google it:- https://upscpdf.com


https://upscpdf.com << Download From >> https://upscpdf.com

ईद्देविका को ऄभी तक के िल एक बार िषव 1976 में 42िें संिैधावनक संिोधन ऄवधवनयम
द्वारा संिोवधत ककया गया है। आस संिोधन से ईद्देविका में तीन नए िब्दों - समाजिादी,
पंथवनरपेक्ष और ऄखंडता - को सवम्मवलत ककया गया।
हाल ही में, सरकार द्वारा पंथवनरपेक्ष एिं समाजिादी िब्दों को सवम्मवलत न करने िाली मूल
ईद्देविका का ईललेख करने िाला विज्ञापन जारी ककया गया। वजस कारण, ईद्देविका की
प्रवस्थवत के संबंध में वििाद ईत्पन्न हो गया। आस वििाद को मुख्य रूप से िोट बैंक के साधन के
रूप में पार्टटयों के वलए राजनीवतक प्रचार का हथकण्डा माना जा सकता है। आसे वनम्नवलवखत
वबन्दुओं से समझा जा सकता है:
 ईच्चतम न्यायालय की राय के ऄनुसार पंथवनरपेक्षता, भारतीय संविधान की मूल
वििेषता है। ऄनुच्छेद 25 के तहत भारत को एक पंथवनरपेक्ष राष्ट्र के रूप में स्थावपत
ककया गया है। पंथवनरपेक्षता का तत्ि मूल ऄवधकारों में वनवहत है न कक ईद्देविका में।
 चूाँकक ितवमान में ऄथवव्यिस्था में वनजी क्षेत्र का प्रभुत्ि है, ऄतः समाजिादी िब्द का
महत्ि ऄत्यंत कम हो गया है।
भारतीय राजव्यिस्था में ईद्देविका का महत्ि:
 यह संविधान के मूल स्रोत ऄथावत भारतीय जनता का प्रतीक है।
 ईद्देविका में ईवललवखत संप्रभु , समाजिादी, पंथवनरपेक्ष, लोकतांवत्रक, गणराज्य अकद
जैसे िब्द राज्य की प्रकृ वत को प्रदर्भित करते हैं।
 न्याय, स्ितंत्रता, समानता ि बंधुत्ि के अदिव संविधान के ईद्देश्य को प्रकट करते हैं।
 यह संविधान सभा के महान और ईत्कृ ष्ट दिवन को समाविष्ट करती है तथा संविधान
वनमावताओं के स्िप्न और अकांक्षाओं को प्रवतवबवम्बत करती है। आस प्रकार यह
न्यायाधीिों के वलए संविधान वनमावताओं के मनोभािों को समझने हेतु मागवदिवन का भी
स्रोत है।

6. विगत िषों में संघ लोक सेिा अयोग द्वारा पू छे गए प्रश्न


(Past Year UPSC Questions)
1. भारतीय संविधान की प्रमुख प्रवतबद्धताएाँ क्या हैं जो आसकी ईद्देविका के रूप में सवम्मवलत की
गयी हैं?
2. भारत को गणतंत्र क्यों कहा जाता है?
3. संविधान की ईद्देविका का ईद्देश्य ईन मूलभूत मूलयों और दिवन को समाविि करना है, वजन
पर संविधान अधाररत है। स्पि कीवजए।
4. ककसी संविधान में ईद्देविका का क्या महत्ि है? भारतीय संविधान की ईद्देविका में
प्रवतष्ठावपत भारतीय राज्य-व्यिस्था के दिवन को सुस्पि कीवजए।

Copyright © by Vision IAS


All rights are reserved. No part of this document may be reproduced, stored in a retrieval system or transmitted
in any form or by any means, electronic, mechanical, photocopying, recording or otherwise, without prior
permission of Vision IAS.

121 www.visionias.in ©Vision IAS

Google it:- https://upscpdf.com


https://upscpdf.com << Download From >> https://upscpdf.com

संघ और ईसका राज्य क्षेत्र


विषय सूची

1. पररचय_______________________________________________________________________________ 123

1.1. पररसंघ बनाम संघ (Federation Vs Union) _________________________________________________ 123

1.2. भारत का राज्य क्षेत्र (Territory of India) ____________________________________________________ 123

1.3. ऄनुच्छेद 2 : नए राज्यों का प्रिेश या स्थापना___________________________________________________ 124

1.4. ऄनुच्छेद 3 : नए राज्यों का वनमााण और ितामान राज्यों के क्षेत्रों, सीमाओं या नामों में पररितान __________________ 124

1.4.1. ऄनुच्छेद 4 ________________________________________________________________________ 125

1.5. भारतीय क्षेत्र के ऄध्यपाण से संबवं धत प्रािधान __________________________________________________ 125

1.6. राज्यों एिं संघशावसत प्रदेशों का विकास ______________________________________________________ 126

1.7. राज्यों के पुनगाठन की मांग का आवतहास_______________________________________________________ 127

1.7.1. राज्य पुनगाठन हेतु गरठत विवभन्न अयोग____________________________________________________ 127

1.7.2. भारत में नए राज्यों का ईद्भि: कालानुक्रम __________________________________________________ 129

1.7.3. भारतीय संघ में सवममवलत संघ शावसत प्रदेश _________________________________________________ 130

1.8. राज्यों के पुनगाठन से संबवं धत मुद्द_े __________________________________________________________ 131

1.9. राज्यों के नाम पररितान से जुड़े मुद्दे__________________________________________________________ 134

2. समकालीन मुद्द_े _________________________________________________________________________ 135

2.1. तेलग
ं ाना मुद्दा _______________________________________________________________________ 135

2.2. गोरखालैंड मुद्दा ______________________________________________________________________ 136

2.3. ऄनुच्छेद 370 और 35A का वनरसन (Removal Of Article 370 And 35A) ____________________________ 138

3. विगत िषों में Vision IAS GS मेंस टेस्ट सीरीज में पूछे गए प्रश्न (Previous Year Vision IAS GS Mains Test

Series Questions) _______________________________________________________________________ 141

4. विगत िषों में संघ लोक सेिा अयोग द्वारा पूछे गए प्रश्न (Past Year UPSC Questions) _____________________ 143

122

Google it:- https://upscpdf.com


https://upscpdf.com << Download From >> https://upscpdf.com

1. पररचय
भारतीय संविधान के भाग 1 के ऄंतगात ऄनुच्छेद 1 से 4 तक संघ एिं ईसके राज्य क्षेत्रों से संबंवधत
प्रािधानों की चचाा की गयी है।
ऄनुच्छेद - 1 : संघ का नाम और राज्यक्षेत्र
 ऄनुच्छेद 1(1) - भारत, ऄथाात् आंवडया, राज्यों का संघ होगा।
 ऄनुच्छेद 1(2) - राज्य और ईनके राज्य क्षेत्र िे होंगे जो पहली ऄनुसूची में विवनर्ददष्ट हैं।
 ऄनुच्छेद 1(3) - भारत के राज्यक्षेत्र में समाविष्ट होंगे:
(क) राज्यों के राज्यक्षेत्र;
(ख) पहली ऄनुसूची में विवनर्ददष्ट संघ राज्य क्षेत्र; और
(ग) ऐसे ऄन्य राज्य क्षेत्र जो ऄर्जजत ककए जाएँ।

1.1. पररसं घ बनाम सं घ (Federation Vs Union)

 प्रारूप सवमवत द्वारा ‘पररसंघ’ शब्द के स्थान पर ‘संघ’ (यूवनयन) शब्द को िरीयता देने के पीछे
एक विशेष ईद्देश्य वनवहत था। सवमवत का मानना था कक ‘संघ’ शब्द आस तथ्य को बेहतर तरीके
से ऄवभव्यक्त कर सकता है कक (a) भारतीय संघ, प्रांतों के मध्य समझौते का पररणाम नहीं है
और (b) कोइ भी राज्य या राज्यों का समूह संघ से ऄलग होने के वलए स्ितंत्र नहीं है। आसके साथ
ही कोइ भी राज्य स्िेच्छा से, ऄपने राज्य की सीमा में पररितान भी नहीं कर सकता है।
 आस प्रकार भारत एक संघ है तथा यह विभक्त नहीं हो सकता। हालांकक प्रशासवनक सुविधा के
वलए देश को विवभन्न राज्यों में बाँटा जा सकता है, ककन्तु देश एक ऄखंड आकाइ है, जहाँ लोग एक
ही स्रोत से व्युत्पन्न, एकल सत्ता के तहत रह रहे हैं।

1.2. भारत का राज्य क्षे त्र (Territory of India)


 राज्यों एिं संघ शावसत प्रदेशों के नाम एिं ईनके क्षेत्र विस्तार को संविधान की पहली ऄनुसूची
में दशााया गया है। ितामान में, भारत में 28 राज्य एिं 9 के न्रशावसत प्रदेश हैं।
 ईल्लेखनीय है कक भारत का राज्य क्षेत्र (Territory of India), यूवनयन ऑफ़ आंवडया से ऄवधक
व्यापक ऄथा समेटे हुअ है। जहाँ ‘यूवनयन ऑफ़ आंवडया’ में के िल राज्य सवममवलत हैं, िहीं ‘भारत
के राज्य क्षेत्र’ में न के िल राज्य ऄवपतु संघ शावसत प्रदेश एिं िे क्षेत्र, वजन्हें कें र सरकार द्वारा
भविष्य में कभी भी ऄवधगृहीत ककया जा सकता है, भी सवममवलत हैं।
 संघीय व्यिस्था में राज्य आसके सदस्य हैं और कें र के साथ शवक्तयों के विभाजन में वहस्सेदार हैं।
दूसरी तरफ, संघ शावसत प्रदेश एिं कें र द्वारा ऄवधगृहीत क्षेत्र सीधे कें र सरकार द्वारा प्रशावसत
होते हैं।

ऄवधगृहीत क्षेत्र (Acquired Territory)


 ककसी क्षेत्र को तब ऄवधगृहीत क्षेत्र कहा जाता है जब भारतीय संघ ऄंतरााष्ट्रीय कानूनों द्वारा
मान्यता प्राप्त पद्धवत के तहत ईस क्षेत्र पर संप्रभुता ऄर्जजत कर ले। ऄजान द्वारा, युद्ध में जीत कर,
ककसी संवध के ऄनुसरण में , ऄध्यपाण (Cession) द्वारा या स्िामीविहीन क्षेत्र पर कब्ज़ा करके
ऐसा ककया जा सकता है।
 यकद ककसी सािाजवनक ऄवधसूचना, दािे या घोषणा के द्वारा भारत सरकार ककसी क्षेत्र को
भारत का ऄवभन्न ऄंग घोवषत करती या मानती है, तब न्यायालय के वलए ऐसे ‘ऄवधग्रहण’ को
मान्यता प्रदान करना ऄवनिाया हो जाएगा, वजसके पररणामस्िरूप यह क्षेत्र ऄनुच्छेद 1(3)(C)
के तहत संघ के राज्य क्षेत्र का वहस्सा होगा।

123 www.visionias.in ©Vision IAS

Google it:- https://upscpdf.com


https://upscpdf.com << Download From >> https://upscpdf.com

1.3. ऄनु च्छे द 2 : नए राज्यों का प्रिे श या स्थापना

संसद, विवध द्वारा, ऐसे वनबंधनों और शतों पर, जो िह ठीक समझे, संघ में नए राज्यों का प्रिेश या
ईनकी स्थापना कर सके गी (ऄनुच्छेद 2)।
आस प्रकार ऄनुच्छेद 2 संसद को वनम्नवलवखत दो शवक्तयां प्रदान करता है:
(i) नए राज्यों को संघ में सवममवलत करने की शवक्त; और
(ii) नए राज्यों की स्थापना करने की शवक्त।
 पहली शवक्त ईन राज्यों के प्रिेश से संबंवधत है जो पहले से ऄवस्तत्ि में हैं, जबकक दूसरी शवक्त
नए राज्यों, जो ऄवस्तत्ि में नहीं हैं, की स्थापना से संबंवधत है, ऄथाात् ऄनुच्छेद 2 ईन राज्यों, जो
भारतीय संघ का वहस्सा नहीं हैं, के प्रिेश और स्थापना से संबंवधत है।
 आसके ऄवतररक्त, यह ध्यान रखा जाना चावहए कक ऄनुच्छेद 2 संसद को पूणा शवक्त देता है कक ऐसे
वनबंधनों और शतों पर, जो “िह ठीक समझे” (it thinks fit), संघ में नए राज्यों का प्रिेश या
स्थापना करे। परन्तु, ये वनबंधन और शतें, ऄवनिायात: संविधान के अधारभूत वसद्धांत या मूल
ढाँचे के ऄनुरूप होने चावहए। संविधान में ऐसा कोइ प्रािधान नहीं है जो ‘एक नए राज्य’ को संघ
में प्रिेश या स्थापना के पश्चात्, पहले से विद्यमान राज्य के समान दजे का ऄवधकार प्रदान करे।

1.4. ऄनु च्छे द 3 : नए राज्यों का वनमाा ण और िता मान राज्यों के क्षे त्रों , सीमाओं या
नामों में पररिता न
संसद, विवध, द्वारा -
(i) ककसी राज्य में से ईसका राज्य क्षेत्र ऄलग करके ऄथिा दो या ऄवधक राज्यों को या राज्यों के
भागों को वमलाकर ऄथिा ककसी राज्य क्षेत्र को ककसी राज्य के भाग के साथ वमलाकर नए राज्य का
वनमााण कर सके गी;
(ii) ककसी राज्य के क्षेत्र को बढ़ा सके गी;
(iii) ककसी राज्य का क्षेत्र घटा सके गी;
(iv) ककसी राज्य की सीमाओं में पररितान कर सके गी; और
(v) ककसी राज्य के नाम में पररितान कर सके गी।
ऄनुच्छेद 3 के ऄंतगात पुरःस्थावपत ककसी विधेयक को वनम्नवलवखत दो शतों का पालन करना ऄवनिाया
है:
(i) ऐसे विधेयक को राष्ट्रपवत की वसफाररश के वबना, दोनों में से ककसी भी सदन में , पुरःस्थावपत नहीं
ककया जा सकता है।
(ii) यकद ककसी विधेयक में ऄंतर्जिष्ट प्रस्थापना का प्रभाि राज्यों में से ककसी के क्षेत्र, सीमाओं या नाम
पर पड़ता है तो राष्ट्रपवत द्वारा ईक्त विधेयक को, प्रभावित राज्य के विधान-मंडल के विचार जानने के
वलए विवनर्ददष्ट ककया जाएगा।
 राष्ट्रपवत िह ऄिवध तय कर सकता है वजसके भीतर राज्य विधान-मंडल को ऄपना विचार व्यक्त
करना होता है। आस विवनर्ददष्ट ऄिवध या ऄन्य ककसी ऄिवध (वजसकी राष्ट्रपवत द्वारा ऄनुमवत
प्रदान की गइ हो) के भीतर राज्य विधान-मंडल के विचार प्राप्त न होने की वस्थवत में भी विधेयक
को पुरःस्थावपत ककया जा सकता है।
 संसद (या राष्ट्रपवत), राज्य विधान-मंडल के विचार को स्िीकार करने या ईनके ऄनुसार काया
करने के वलए बाध्य नहीं है और संसद आसे स्िीकार या ऄस्िीकार कर सकती है , भले ही यह
विचार तय समय-सीमा के भीतर प्राप्त हो गया हो। यकद पूिा में पुरःस्थावपत विधेयक में संसद

124 www.visionias.in ©Vision IAS

Google it:- https://upscpdf.com


https://upscpdf.com << Download From >> https://upscpdf.com

द्वारा संशोधन ककया जाता है तो प्रत्येक बार राज्य विधान-मंडल के वलए नया सन्दभा बनाना
(ऄथाात् राज्य विधान-मंडल को आसे पुन: िापस भेजना) अिश्यक नहीं है।
 ऄनुच्छेद 3, संसद को यह ऄवधकार प्रदान करता है कक िह नए राज्यों के वनमााण, ितामान राज्यों
के क्षेत्रों, सीमाओं या नाम में पररितान के संबंध में वबना राज्यों की ऄनुमवत से कदम ईठा सकती
है। यह भारतीय संघीय प्रणाली को ऄन्य परंपरागत संघात्मक प्रणाली से पृथक करती है। दूसरे
शब्दों में, संसद ऄपने ऄनुसार भारत के राजनीवतक मानवचत्र का पुनर्जनधाारण कर सकती है। आस
प्रकार संविधान द्वारा क्षेत्रीय ऄखंडता या राज्य के ऄविभाज्य ऄवस्तत्ि की गारंटी नहीं दी गयी
है। ऄतः भारत को ‘विनाशी राज्यों का ऄविनाशी संघ’ (an indestructible Union of
destructible states) कहना सही है।
 ईल्लेखनीय है कक संयुक्त राज्य ऄमेररका की पररसंघीय प्रणाली (फ़े डरल वसस्टम) स्ितंत्र राज्यों
के मध्य समझौते का पररणाम है। ऄमेररका में पररसंघीय सरकार नए राज्यों का वनमााण या
ईनकी सीमाओं में पररितान, संबंवधत राज्यों की ऄनुमवत के वबना नहीं कर सकती है। आसवलए
ऄमेररका को ‘ऄविनाशी राज्यों का ऄविनाशी संघ’ (an indestructible Union of
indestructible States) कहा जाता है।
 भारतीय संदभा में डी. डी. बसु के ऄनुसार भारतीय संसद को आस तरह की ईदार शवक्तयां कदए
जाने के पीछे प्रमुख तका यह है कक भारत सरकार ऄवधवनयम के ऄधीन प्रांतों का समूहीकरण
ऐवतहावसक और राजनीवतक कारकों के अधार पर था, न कक स्ियं लोगों के सामावजक,
सांस्कृ वतक और भाषायी विभाजन के अधार पर। संविधान के वनमााण के समय प्रांतों के प्राकृ वतक
संरेखण के ऄनुसार ईनके पुनगाठन का प्रश्न ईत्पन्न हुअ था लेककन ईस समय समस्या की जरटलता
को देखते हुए आस विशाल काया का ईत्तरदावयत्ि नहीं वलया जा सका।
 ऐसे कइ ईदाहरण हैं जहां राज्य विधान-मंडलों ने नए राज्यों के गठन के वलए प्रस्ताि पाररत
ककए हैं। लेककन संिैधावनक रूप से कोइ राज्य, नए राज्य के वनमााण आत्याकद की प्रकक्रया की
शुरुअत नहीं कर सकता है। आसकी शुरूअत कें रीय मंवत्रपररषद की सलाह से राष्ट्रपवत की
पूिाानुमवत द्वारा ऐसे विधेयक को संसद में पुरःस्थावपत करने से होती है। निंबर, 2011 में ईत्तर
प्रदेश विधानसभा द्वारा राज्य को चार भागों - पूिाांचल, पवश्चम प्रदेश, ऄिध प्रदेश और बुंदल
े खंड
- में विभावजत करने का प्रस्ताि पाररत ककया गया, वजसका के िल सांकेवतक मूल्य था जबकक
संिैधावनक रूप से आसका कोइ महत्ि नहीं था।

1.4.1. ऄनु च्छे द 4

 पहली और चौथी ऄनुसूची के संशोधन तथा आस संदभा में ऄनुपूरक, अनुषंवगक और पाररणावमक
विषयों का ईपबंध करने के वलए ऄनुच्छेद 2 और ऄनुच्छेद 3 के ऄधीन बनायी गयी विवधयों को
ऄनुच्छेद 368 के ऄंतगात संविधान संशोधन नहीं माना जाएगा। आसका ऄथा यह है कक ऐसी
विवधयाँ साधारण बहुमत एिं साधारण विधायी प्रकक्रया के माध्यम से पाररत की जा सकती हैं।

1.5. भारतीय क्षे त्र के ऄध्यपा ण से सं बं वधत प्रािधान


 क्या ककसी भारतीय क्षेत्र के ऄध्यपाण (ककसी भारतीय क्षेत्र को ककसी ऄन्य देश को देना) के वलए
संविधान संशोधन की अिश्यकता है? यह प्रश्न िषा 1960 में, राष्ट्रपवत द्वारा एक परामशा के रूप
में ईच्चतम न्यायालय के समक्ष प्रस्तुत ककया। कें र सरकार द्वारा िषा 1958 के नेहरु-नून समझौते
के कक्रयान्ियन हेतु बेरुबारी संघ (पवश्चम बंगाल) पाककस्तान को हस्तांतररत कर कदया गया
वजसने राजनीवतक हलचल और िाद-वििाद को जन्म कदया। आसी अलोक में राष्ट्रपवत को ईच्चतम
न्यायालय से यह परामशा लेना पड़ा।

125 www.visionias.in ©Vision IAS

Google it:- https://upscpdf.com


https://upscpdf.com << Download From >> https://upscpdf.com

 ईच्चतम न्यायालय ने वनणाय कदया कक ककसी राज्य के राज्यक्षेत्र को घटाने की शवक्त के संदभा में
ऄनुच्छेद 3 संसद को ककसी भारतीय क्षेत्र को ककसी ऄन्य देश को ऄध्यर्जपत करने (सौंपने) की
शवक्त नहीं देता है। चूँकक, समझौते के कायाान्ियन से देश के क्षेत्रफल में कमी होगी, ऄतः संविधान
के ऄनुच्छेद 1 तथा साथ ही साथ पहली ऄनुसूची के कु छ अिश्यक प्रािधानों में संशोधन करना
ऄवनिाया होगा। आस प्रकार, के िल ऄनुच्छेद 368 के ऄंतगात संविधान संशोधन के माध्यम से ही
ककसी भारतीय क्षेत्र को ककसी ऄन्य देश को ऄध्यार्जपत ककया जा सकता है। पररणामस्िरूप, ईक्त
क्षेत्र को पाककस्तान को हस्तांतररत करने के वलए 9िाँ संविधान संशोधन ऄवधवनयम पाररत
ककया गया।
 िषा 2011 में भारत एिं बांग्लादेश ने दोनों राष्ट्रों के मध्य िषा 1974 में संपन्न भूवम सीमा
समझौते (Land Boundary Agreement: LBA) के तहत विदेशी ऄंतः क्षेत्रों (enclaves) के
परस्पर अदान-प्रदान के वलए एक प्रोटोकॉल पर हस्ताक्षर ककए। आस समझौते के तहत 111
भारतीय एन्क्लेि बांग्लादेश को तथा 51 बांग्लादेशी एन्क्लेिों को भारत को हस्तांतररत करने
की पररकल्पना की गइ। हालांकक, आस हेतु संसद के ऄनुसमथान की अिश्यकता थी। आसी
अिश्यकता को ध्यान में रखते हुए 119िां संविधान संशोधन विधेयक, 2013 पुरःस्थावपत
ककया गया था तथा वजसे िषा 2015 में पाररत ककया गया। आस विधेयक को दोनों सदनों एिं
राष्ट्रपवत द्वारा स्िीकृ वत वमलने के पश्चात् 100िां संविधान संशोधन के रूप में ईवल्लवखत ककया
गया है। LBA को प्रभािी बनाने के वलए आस विधेयक के माध्यम से संविधान की प्रथम ऄनुसूची
को संशोवधत ककया गया। यह विधेयक प्रथम ऄनुसूची में ऄसम, पवश्चम बंगाल, मेघालय और
वत्रपुरा के क्षेत्रों से संबंवधत पररच्छेदों में संशोधन करती है।
 दूसरी ओर, िषा 1969 में ईच्चतम न्यायालय ने यह वनणाय कदया था कक भारत और ककसी ऄन्य
देश के मध्य सीमा-वििाद के वनपटारे के वलए संिैधावनक संशोधन की अिश्यकता नहीं है। यह
कायापावलका कायािाही (executive action) के माध्यम से ककया जा सकता है, यकद आसके द्वारा
ककसी दूसरे राष्ट्र को भारतीय क्षेत्र का ऄध्यपाण नहीं ककया जाता है।
 कच्चावतिु द्वीप: भारत और श्रीलंका के मध्य िषा 1974 और िषा 1976 में हुइ संवधयों के माध्यम
से भारत ने कच्चावतिु द्वीप श्रीलंका को ऄध्यर्जपत कर कदया। बेरुबारी मामले में कदए गए वनदेश के
ऄनुसार ककसी भारतीय भू -भाग का ककसी ऄन्य देश को हस्तान्तरण के िल संिैधावनक संशोधन
के माध्यम से ककया जा सकता है। श्रीलंका को कच्चावतिु द्वीप के हस्तांतरण को ईच्चतम न्यायालय
द्वारा स्थावपत प्रकक्रया और वनदेश का ईल्लंघन मानते हुए िषा 2012 में ईच्चतम न्यायालय में
चुनौती दी गयी थी। यह मामला न्यायालय के समक्ष ऄभी भी लंवबत है।

1.6. राज्यों एिं सं घ शावसत प्रदे शों का विकास

 स्ितंत्रता के पश्चात्, भाषायी अधार पर राज्यों का पुनगाठन राष्ट्रीय समेकन और एकीकरण का


एक मुख्य पक्ष बन गया। देसी ररयासतों के एकीकरण की प्रकक्रया के साथ ईन प्रांतों, वजनकी
सीमाओं का वनधाारण ऄंग्रेजों द्वारा ऄतार्दकक रूप से ककया गया था, को एकीकृ त करने का काया
ऄत्यंत श्रमसाध्य था। हालांकक आस प्रकक्रया ने ऄंततः स्िातंत्रोत्तर भारत की विविधता को बढ़ाया।
विरटश भारत में प्रांत दो प्रकार के थे:
o गिनार-जनरल के प्रवत ईत्तरदायी विरटश ऄवधकाररयों द्वारा प्रत्यक्षतः शावसत प्रांत, एिं
o स्थानीय िंशानुगत शासकों के ऄधीन देसी ररयासतें , वजनके सन्दभा में विरटश सरकार संप्रभु
तो थी ककन्तु संवध के अधार पर ईन्हें स्िायत्तता प्रदान की गयी थी।

126 www.visionias.in ©Vision IAS

Google it:- https://upscpdf.com


https://upscpdf.com << Download From >> https://upscpdf.com

 15 ऄगस्त 1947 को जब भारत ने स्ितंत्रता प्राप्त की तो विरटश सरकार ने ईन 600 से ऄवधक


ररयासतों के साथ ऄपने संवध अधाररत संबंधों को समाप्त कर कदया वजनके पास भारत या
पाककस्तान में से ककसी एक में विलय का विकल्प था। ऄवधकांश ररयासतें या तो स्िेच्छा से या
सशस्त्र हस्तक्षेप के माध्यम से भारत में सवममवलत हुईं।
 1947-1950 की ऄिवध के दौरान, आन प्रांतों को राजनीवतक रूप से भारतीय संघ में या तो
संलग्न प्रांतों के साथ विलय द्वारा या नए प्रांतों के रूप में आनके गठन द्वारा एकीकृ त ककया गया।
26 जनिरी 1950 को जब भारतीय संविधान ऄवस्तत्ि में अया तो ईस समय भारतीय संघ की
घटक आकाआयों को वनम्नवलवखत चार िगों में विभावजत ककया गया था:
o श्रेणी-A के राज्यों में तत्कालीन गिनार के ऄधीन शावसत प्रांत सवममवलत थे। आस श्रेणी में
ऄसम, वबहार, बॉमबे, मध्य प्रदेश, मरास, ईड़ीसा, पंजाब, ईत्तर प्रदेश और पवश्चम बंगाल
सवममवलत थे।
o श्रेणी-B के राज्यों में पूिा की ररयासतें या ररयासतों के समूह सवममवलत थे, जो राजप्रमुख
द्वारा शावसत थे। यह राजप्रमुख प्रायः कोइ पूिा राजकु मार होता था तथा यहाँ वनिाावचत
विधावयकाएँ थीं। राजप्रमुख को भारत के राष्ट्रपवत द्वारा वनयुक्त ककया जाता था। आस श्रेणी
में शावमल राज्य थे - हैदराबाद, जममू एिं कश्मीर, मध्य भारत, मैसूर, परटयाला एंड इस्ट
पंजाब स्टेट्स यूवनयन (PEPSU), राजस्थान, सौराष्ट्र, त्रािणकोर-कोचीन और विध्य प्रदेश।
o श्रेणी-C के 10 राज्यों में पूिा मुख्य अयुक्त के शासनाधीन प्रांत थे और ऄंडमान एिं
वनकोबार द्वीप समूह को छोड़कर कु छ ररयासतें शावमल थीं। आसमें सवममवलत राज्यों में
ऄजमेर, भोपाल, वबलासपुर, कू च-वबहार, कु गा, कदल्ली, वहमाचल प्रदेश, कच्छ, मवणपुर और
वत्रपुरा थे।
o श्रेणी-D में के िल ऄंडमान और वनकोबार द्वीप समूह शावमल था वजसे लेवटटनेंट गिनार द्वारा
प्रशावसत ककया जाता था।

1.7. राज्यों के पु न गा ठ न की मां ग का आवतहास

 भारत में ऄंग्रेजों का विजय ऄवभयान लगभग 100 िषों से ऄवधक समय तक चला वजसके
पररणामस्िरूप राज्यों की अतंररक सीमाएँ ऄव्यिवस्थत एिं ऄतार्दकक ढंग से वनर्जमत हुईं। आन
राज्यों के गठन में भाषाइ और सांस्कृ वतक एकजुटता का ध्यान नहीं रखा गया वजसके चलते ये
बहुभावषक और बहुसांस्कृ वतक चररत्र िाले राज्य बन गए।
 भाषाइ अधार पर राज्यों के पुनगाठन की मांग वनरंतर बनी हुइ है। यह स्ितंत्रता प्रावप्त से बहुत
पहले से ईठती रही है। लगभग एक सदी के प्रयासों के बाद विरटश शासन में ही सिाप्रथम भाषाइ
अधार पर िषा 1936 में ईड़ीसा प्रांत का गठन हुअ। आसमें मधुसूदन दास की प्रमुख भूवमका थी।
 भारतीय राष्ट्रीय कांग्रेस ने भी स्ितंत्रता के पूिा आस अधार पर राज्यों के पुनगाठन की मांग को
ईवचत माना। िषा 1917 में ही कांग्रेस की राज्य आकाआयों को भाषायी अधार पर संरवचत करने
का वनणाय वलया गया। िषा 1946 के चुनाि में कांग्रेस के चुनािी घोषणापत्र में भी आसे स्थान
कदया गया।
 ककन्तु स्ितंत्रता प्रावप्त के ईपरांत विभाजन के कटु ऄनुभि के कारण, भाषाइ जैसे ककसी भी
अधार पर राज्यों के गठन को विभाजनकारी तत्िों के आरादों को बढ़ािा देने िाला माना गया
और आसे संशयात्मक दृवष्ट से देखा जाने लगा।

1.7.1. राज्य पु न गा ठ न हे तु गरठत विवभन्न अयोग

 नए राज्यों के गठन हेतु सभी िगों की ओर से वनरंतर ईठने िाली मांगों के चलते , िषा 1948 में
भारतीय संविधान सभा के ऄध्यक्ष द्वारा भारत में राज्यों के पुनगाठन के प्रश्न पर विचार करने के

127 www.visionias.in ©Vision IAS

Google it:- https://upscpdf.com


https://upscpdf.com << Download From >> https://upscpdf.com

वलए एस. के . धर की ऄध्यक्षता में एक भाषाइ प्रांत अयोग (धर अयोग) का गठन ककया गया।
ऄपनी ररपोटा में अयोग ने वसफाररश की कक राज्यों का पुनगाठन भाषाइ अधार के स्थान पर
प्रशासवनक सुविधा के अधार पर ही होना चावहए। भारतीय राष्ट्रीय कांग्रेस ने िषा 1949 के
ऄपने जयपुर ऄवधिेशन में धर अयोग की वसफाररशों पर विचार करने के वलए एक ईच्चस्तरीय
भाषाइ राज्य सवमवत का गठन ककया, वजसमें जिाहर लाल नेहरू, िल्लभभाइ पटेल तथा पट्टावभ
सीतारमैया शावमल थे। आसे JVP सवमवत कहा गया। आस सवमवत ने भी ऄपनी ररपोटा में राज्यों
के भाषायी पुनगाठन के प्रस्ताि के साथ अगे बढ़ने में ऄत्यंत सािधानी बरतने की सलाह दी।
 हालाँकक, िषा 1952 तक भाषाइ अधार पर राज्यों के पुनगाठन की मांग काफ़ी मज़बूत हो चुकी
थी। ऄक्टूबर 1953 में भारत सरकार को भाषा के अधार पर प्रथम राज्य का गठन करने के वलए
मजबूर होना पड़ा और मरास से 16 तेलगू भाषी वजलों को पृथक कर (वजसमें तटीय अंध्र एिं
रायलसीमा क्षेत्र सवममवलत थे) एक नया राज्य अंध्र प्रदेश वनर्जमत ककया गया। आस दौरान 56
कदनों की भूख हड़ताल के बाद कांग्रेस कायाकताा पोट्टी श्रीरामुलू की मृत्यु हो गयी थी। (श्रीरामुलू
गाँधीिादी स्ितंत्रता सेनानी थे, वजन्होंने दवलत ईत्थान के वलए ईल्लेखनीय काया ककया था और
नमक सत्याग्रह में भी भाग वलया था)।
 आसके पश्चात, जिाहर लाल नेहरू ने पूरे मामले की जांच करने के वलए, फजल ऄली की
ऄध्यक्षता में तीन सदस्यीय राज्य पुनगाठन अयोग (1953) की वनयुवक्त की। अयोग के ऄन्य दो
सदस्य के . एम. पवणक्कर और एच. एन. कुं जरू थे। िषा 1955 में अयोग ने ऄपनी ररपोटा प्रस्तुत
की। अयोग ने राज्यों के पुनगाठन हेतु प्रशासवनक तथा अर्जथक कारकों को ध्यान में रखा, साथ ही
भाषाइ वसद्धांत को भी स्िीकार करते हुए भाषा को एक प्रमुख अधार के रूप में माना।
 राज्य पुनगाठन अयोग द्वारा ककसी राज्य के गठन के वलए ककसी क्षेत्र की मांग स्िीकार करने के
वलए वनम्नवलवखत चार मापदंड वनधााररत ककए गए:
o भाषाइ और सांस्कृ वतक एकता के अधार पर राज्यों को वनर्जमत ककया जाना चावहए;
o राज्यों का गठन आस प्रकार हो वजससे राष्ट्रीय एकता की रक्षा तथा ईसे मजबूत बनाया जा
सके ;
o नए राज्यों का गठन वित्तीय, प्रशासवनक और अर्जथक व्यिहायाता को ध्यान में रखकर ककया
जाना चावहए; तथा
o राज्यों का गठन आस प्रकार हो वजससे पंचिषीय योजनाओं के कक्रयान्ियन की प्रकक्रया में
सहायता वमले।
 निंबर 1956 में संसद द्वारा राज्य पुनगाठन ऄवधवनयम पाररत ककया गया। आसके द्वारा चौदह
राज्यों और छह के न्र प्रशावसत प्रदेशों का वनमााण ककया गया। साथ ही, सातिें संविधान संशोधन
ऄवधवनयम द्वारा पूिा में राज्यों की चार श्रेवणयों (A, B, C और D) में हुए िगीकरण को राज्यों
के एकल िगा से प्रवतस्थावपत कर कदया गया।
 िषा 1950 में भारतीय संघ के समेकन के बाद से मौजूदा राज्य की सीमाओं के पुनगाठन को मोटे
तौर पर पुनगाठन के वनम्नवलवखत चार व्यापक चरणों के ऄंतगात िगीकृ त ककया जा सकता है:
o पहला प्रमुख पुनगाठन, सुगरठत भाषायी प्रांतों के वनमााण के वलए एक राष्ट्रव्यापी अंदोलन के
बाद िषा 1956 में हुअ। कश्मीर को पहले से ही भारतीय संविधान के ऄनुच्छेद 370 के
द्वारा, आसे दी गइ विशेष वस्थवत के अधार पर, संघ में शावमल कर वलया गया था।
o दूसरी बड़ी पहल 1970 के दशक में हुइ, जब पूिोत्तर भारत में नए राज्यों का गठन ककया
गया। िषा 1963 में नागालैंड की स्थापना के बाद कु छ नए राज्यों का गठन हुअ।

128 www.visionias.in ©Vision IAS

Google it:- https://upscpdf.com


https://upscpdf.com << Download From >> https://upscpdf.com

o तीसरे चरण में भारत के ईत्तरी प्रांतों में झारखंड, ईत्तरांखंड और छत्तीसगढ़ की स्थापना की
गयी। िषा 2014 में तेलंगाना, अन्ध्र प्रदेश राज्य से ऄलग होकर बना भारत का निीनतम
राज्य है।
o हाल ही में, संसद द्वारा जममू और कश्मीर पुनगाठन ऄवधवनयम, 2019 (Jammu and
Kashmir Reorganization Act, 2019) को पाररत कर जममू और कश्मीर को दो संघ
शावसत प्रदेशों में विभावजत कर कदया गया है, यथा- जममू और कश्मीर वडिीज़न
(विधानसभा युक्त) तथा लद्दाख (विधानसभा रवहत)।
1950 के दशक के दौरान हुए भाषाइ पुनगाठन ने राजनीवतक और प्रशासवनक आकाआयों में सांस्कृ वतक
पहचान को सवममवलत करने में प्रमुख योगदान कदया। भारतीय लोकतंत्र द्वारा प्रारंभ से ही जनता के
एक बड़े िगा की अकांक्षाओं का सममान ककया गया और भाषाइ अधार पर राज्यों का पुनगाठन करके ,
राष्ट्रीय नेतृत्ि ने एक ऐसी प्रमुख वशकायतों का समाधान ककया जो विभाजनकारी प्रिृवत्तयों को
ईकसाने में समथा थीं।
पुनगाठन का प्रत्येक चरण कें र तथा संघीय आकाआयों के मध्य राजनीवतक शवक्त के संतुलन पर अधाररत
था। राज्यों के पुनगाठन ने गंभीरता से राष्ट्र की एकता को कमजोर ककए वबना भारत के राजनीवतक
मानवचत्र को युवक्तसंगत बनाया है। आसवलए राज्यों के पुनगाठन को 'सिाश्रष्ठ
े तरीके से प्राप्य राष्ट्रीय
एकता’ के वलए अधार तैयार करने िाला कारक माना जाता है।

1.7.2. भारत में नए राज्यों का ईद्भि: कालानु क्र म

अंध्र प्रदेश मरास राज्य से कु छ क्षेत्रों को पृथक कर, अंध्र राज्य ऄवधवनयम
(1953) द्वारा आस राज्य का गठन ककया गया। अरंभ में आसकी
राजधानी कु नुाल थी और ईच्च न्यायालय गुंटूर में स्थावपत ककया गया
था।

के रल राज्य पुनगाठन ऄवधवनयम, 1956 द्वारा के रल राज्य का गठन ककया


गया। आसमें त्रािणकोर और कोचीन क्षेत्र सवममवलत ककए गए।

कनााटक राज्य पुनगाठन ऄवधवनयम, 1956 द्वारा मैसूर ररयासत से आसका


सृजन ककया गया। िषा 1973 में मैसूर का नया नाम कनााटक रखा
गया।

गुजरात और महाराष्ट्र बॉमबे (पुनगाठन) ऄवधवनयम, 1960 द्वारा बॉमबे राज्य को दो राज्यों
ऄथाात् महाराष्ट्र और गुजरात में विभक्त ककया गया।

नागालैंड आसका गठन नागालैंड राज्य ऄवधवनयम, 1962 द्वारा ऄसम राज्य से
आसे पृथक कर ककया गया था।

हररयाणा पंजाब (पुनगाठन) ऄवधवनयम, 1966 द्वारा पंजाब राज्य से आसे पृथक
कर गठन ककया गया था।

वहमाचल प्रदेश संघ शावसत प्रदेश वहमाचल प्रदेश को वहमाचल प्रदेश राज्य
ऄवधवनयम, 1970 द्वारा पूणा राज्य का दजाा प्रदान ककया गया।

129 www.visionias.in ©Vision IAS

Google it:- https://upscpdf.com


https://upscpdf.com << Download From >> https://upscpdf.com

मेघालय, मवणपुर और आसे सिाप्रथम 22िें संविधान संशोधन ऄवधवनयम, 1969 द्वारा ऄसम
वत्रपुरा राज्य के भीतर एक 'ईप-राज्य' या 'स्िायत्त राज्य' के रूप में पृथक
ककया गया। तत्पश्चात् िषा 1971 में, ईत्तर पूिी क्षेत्र (पुनगाठन)
ऄवधवनयम, 1971 द्वारा मेघालय को एक पूणा राज्य का दजाा प्राप्त
हुअ। पूिोत्तर क्षेत्र (पुनगाठन) ऄवधवनयम, 1971 द्वारा दोनों कें र
शावसत प्रदेशों मवणपुर एिं वत्रपुरा को पूणा राज्य का दजाा प्रदान
ककया गया। वमज़ोरम और ऄरुणाचल प्रदेश (मूलत: आसे नाथा इस्ट
फ्रंरटयर एजेंसी - NEFA के नाम से जाना जाता था) नामक दो कें र
शावसत प्रदेश भी ऄवस्तत्ि में अए।

वसकक्कम पहले वसकक्कम को 35िें संविधान संशोधन ऄवधवनयम, 1974 द्वारा


समबद्ध राज्य (Associate State) का दजाा कदया गया था, जब यह
'चोग्याल' शासन के ऄधीन था। िषा 1975 में 36िें संशोधन
ऄवधवनयम, 1975 द्वारा आसे एक पूणा राज्य का दजाा प्राप्त हुअ।

वमज़ोरम वमज़ोरम राज्य ऄवधवनयम, 1986 द्वारा आसे एक पूणा राज्य का दजाा
कदया गया।

ऄरुणाचल प्रदेश आसे ऄरुणाचल प्रदेश ऄवधवनयम, 1986 द्वारा एक पूणा राज्य का दजाा
प्राप्त हुअ।

गोिा गोिा, दमन और दीि पुनगाठन ऄवधवनयम, 1987 द्वारा कें र शावसत
प्रदेश गोिा, दमन और दीि से पृथक कर गोिा को पूणा राज्य का दजाा
प्रदान ककया गया था, परन्तु दमन और दीि को कें र शावसत प्रदेश ही
रखा गया।

छत्तीसगढ़ 1 निंबर 2000 को मध्य प्रदेश को विभक्त कर आसका गठन ककया


गया।

ईत्तराखंड 9 निंबर 2000 को ईत्तर प्रदेश को विभक्त कर आसका गठन ककया


गया।

झारखंड 15 निंबर 2000 को वबहार को विभक्त कर आसका गठन ककया गया।

तेलग
ं ाना 2 जून 2014 को अंध्र प्रदेश को विभक्त कर आसका गठन ककया गया।

1.7.3. भारतीय सं घ में सवममवलत सं घ शावसत प्रदे श

 िषा 1987 के बाद से संघ शावसत प्रदेशों की कु ल संख्या सात थी। ये हैं- कदल्ली, ऄंडमान और
वनकोबार द्वीप समूह, दादरा और नगर हिेली, लक्षद्वीप, दमन और दीि, पांवडचेरी तथा
चंडीगढ़।

130 www.visionias.in ©Vision IAS

Google it:- https://upscpdf.com


https://upscpdf.com << Download From >> https://upscpdf.com

o ितामान में संघ शावसत प्रदेशों की कु ल संख्या 9 है (जममू-कश्मीर एिं लद्दाख दो निीन संघ
शावसत प्रदेश हैं)।
 संसद ने पांवडचेरी संघ शावसत प्रदेश के वलए ऄनुच्छेद 239A के तहत एक कानून बनाकर ऄथाात्
पांवडचेरी (प्रशासन) ऄवधवनयम, 1962 द्वारा विधावयका अकद के वलए प्रािधान ककया।
 कदल्ली में विधावयका और मंत्रालय की स्थापना के वलए िषा 1992 में संविधान संशोधन द्वारा दो
नए ऄनुच्छेद ऄथाात् ऄनुच्छेद 239AA एिं 239AB जोड़े गए। ऄनुच्छेद 239AA द्वारा आसे
कदल्ली राष्ट्रीय राजधानी क्षेत्र (NCR) के रूप में नावमत ककया गया।
 शेष संघ शावसत प्रदेश , के न्र द्वारा प्रशावसत क्षेत्र हैं तथा राष्ट्रपवत, ऄपने द्वारा वनयुक्त 'प्रशासक'
के माध्यम से और आन प्रदेशों में सुशासन के वलए विवनयम के अदेश जारी कर प्रशासन करते हैं।
(ऄनुच्छेद 239-240)।
पुदच
ु रे ी (पांवडचेरी) का ऄवधग्रहण एिं भारत संघ में प्रिेश
 यह क्षेत्र पूिा फ्रांसीसी ईपवनिेश था। िषा 1956 में भारत और फ्रांस के मध्य एक ऄध्यपाण संवध
पर हस्ताक्षर हुअ। िषा 1962 तक, जब फ्रांसीसी संसद ने समझौते की पुवष्ट की, तब तक
आसे ‘ऄवधगृहीत क्षेत्र' का दजाा कदया गया था। ऄंतत: िषा 1962 में भारत और फ्रांस के द्वारा
ऄनुसमथान प्रपत्रों का अदान-प्रदान ककया गया। आसके तहत फ्रांस ने ईसके द्वारा ऄवधकृ त क्षेत्रों
की पूणा संप्रभुता भारत को सौंप दी। तत्पश्चात् आसे संघ शावसत प्रदेश का दजाा प्रदान ककया गया।
 आसके ऄवतररक्त िषा 2006 में, संसद ने आस संघ शावसत प्रदेश के लोगों की अकांक्षाओं के ऄनुरूप
पांवडचेरी कें र शावसत प्रदेश का नाम पररिर्जतत कर पुदच
ु ेरी करने के वलए एक विधेयक पाररत
ककया। पुदच
ु ेरी में चार क्षेत्र ऄथाात् पुदच
ु ेरी, कराइकल, माहे और यनम सवममवलत हैं।
वसकक्कम का भारत संघ के साथ एकीकरण
 वसकक्कम मूलत: भारत का एक संरवक्षत राज्य था। िषा 1974 में आसे संविधान में संशोधन (35िां
संशोधन ऄवधवनयम) द्वारा भारतीय संघ के एक 'समबद्ध राज्य' (Associate state) का दजाा
कदया गया था। िषा 1975 में वसकक्कम विधानसभा में चोग्याल (शाही) शासन को समाप्त करने
और वसकक्कम को भारत के एक संघटक क्षेत्र के रूप में घोवषत करने के प्रस्ताि को स्िीकृ त ककया
गया।
 एक सामान्य जनमत संग्रह द्वारा आस राज्य के भारतीय संघ के साथ एकीकरण का मागा प्रशस्त
ककया गया। पररणामस्िरूप संसद द्वारा िषा 1975 में 36िां संविधान संशोधन ऄवधवनयम
पाररत ककया गया और वसकक्कम, भारतीय संघ का 22िां राज्य बन गया।

संरवक्षत राज्य (Protectorate)


ऄंतरााष्ट्रीय कानून के तहत एक संरवक्षत राज्य िह राजनीवतक आकाइ है जो औपचाररक रूप से संवध
द्वारा ककसी ऄन्य सुदढ़ृ राज्य वजसे संरक्षक (protector) राज्य कहा जाता है, से संबंध स्थावपत करती
है। यह सुदढ़ृ संरक्षक राज्य ककसी तीसरे पक्ष से ईक्त संरवक्षत राज्य की रक्षा (कू टनीवतक या सैन्य)
करने के वलए सहमत होता है। आसके बदले में संरवक्षत राज्य सामान्यत: वनर्ददष्ट दावयत्िों को स्िीकार
करता है।

1.8. राज्यों के पु न गा ठ न से सं बं वधत मु द्दे

क्या नए राज्यों की मांग राष्ट्रीय एकता के समक्ष चुनौती प्रस्तुत करती है?
 रामचंर गुहा का तका है कक भाषाइ राज्यों के गठन ने भारत की एकता की रक्षा की है।
पाककस्तान विभावजत हो गया और श्रीलंका में दीघाकालीन गृह-युद्ध हुअ क्योंकक पाककस्तान के

131 www.visionias.in ©Vision IAS

Google it:- https://upscpdf.com


https://upscpdf.com << Download From >> https://upscpdf.com

मामले में बंगाली भावषयों और श्रीलंका के मामले में तवमल भावषयों को ईस स्िायत्तता और
गररमा से िंवचत रखा गया वजसके िे हक़दार थे।
 दूसरी ओर, भारत में नागररकों को ईनकी ही भाषा में वशवक्षत होने और स्ियं को प्रशावसत करने
की स्िंत्रतता ने ईनमें अश्वासन, ईत्तरदावयत्ि और सुरक्षा की भािना पैदा की है तथा ‘पहचान के
संकट’ (Identity Crisis) को ईत्पन्न होने से रोका।
 प्रख्यात विद्वानों एिं कइ ऄन्य का मानना है कक भाषाइ राज्य, भारतीय स्ितंत्रता के प्रारंवभक
दौर में अिश्यक थे, लेककन ऄब राज्यों के ‘एक और पुनगाठन’ का समय अ गया है। हाल ही में
तेलंगाना का वनमााण आसका एक ईदाहरण है। विदभा और गोरखालैंड अकद के समथाकों के पास
भी मजबूत तका हैं। ये क्षेत्र पयाािरणीय और सांस्कृ वतक ऄथा में बेहतर रूप से पररभावषत हैं तथा
ऐवतहावसक रूप से राज्यों के ऄवधक शवक्तशाली या समृद्ध वहस्से द्वारा ईपेवक्षत रहे हैं।
 अजादी के 70 िषों बाद, भारत की एकता के बारे में ककसी भय की अिश्यकता नहीं रह गइ है।
भारत िषा ऄखंड रहेगा। अज भारत की िास्तविक समस्या शासन की गुणित्ता है तथा
ऄपेक्षाकृ त छोटे राज्य, आस समस्या का एक समाधान प्रस्तुत कर सकते हैं।
क्या वद्वतीय राज्य पुनगाठन अयोग की अिश्यकता है?
 नए राज्यों की बढ़ती मांग ने भारत की संघीय लोकतांवत्रक व्यिस्था को सुदढ़ृ बनाने के संबंध में
कु छ प्रश्न ईत्पन्न ककए हैं। संघीय पुनगाठन के मुद्दे का समाधान करने के वलए ककसी भी ढांचे को
तैयार करते समय वनम्नवलवखत चार ईपायों पर विचार ककया जाना चावहए:
o एक स्थायी राज्य पुनगाठन अयोग का गठन;
o यह सुवनवश्चत करने के वलए संविधान में संशोधन ककया जाना चावहए, कक नए राज्यों के
गठन की मांग राज्य विधावयका द्वारा ईत्पन्न हो न कक कें र की ओर से;
o राजनीवतक विचार के स्थान पर अर्जथक और सामावजक व्यिहायाता का परीक्षण ककया
जाना चावहए; तथा
o धमा, जावत और भाषा को नए राज्यों के गठन का िैध अधार बनाने के बजाय विकास एिं
सुशासन जैसे लोकतांवत्रक मुद्दों को प्रोत्सावहत करने पर ध्यान कें करत ककया जाना चावहए।
 भारत के राज्यों के अकार और ढांचे का एक ऄवधक व्यापक ऄिलोकन करने के वलए लंबे समय
से वद्वतीय राज्य पुनगाठन अयोग (SRC) की स्थापना की मांग की जा रही है। यकद एक नए
SRC का गठन ककया जाता है तो ईसे वनम्नवलवखत प्रश्नों का समाधान करना होगा:
(i) क्या भारत को और राज्यों की अिश्यकता है?
 भारत प्रवत राज्य जनसंख्या के ऄनुसार राज्यों की संख्या के मामले में संघीय देशों की तावलका में
सबसे नीचे अता है। आसके प्रवत राज्य जनसंख्या का औसत 35 वमवलयन है। जोकक तुलनात्मक
रूप से िाज़ील में 7 वमवलयन, ऄमेररका में 6 वमवलयन, नाआजीररया में 4 वमवलयन है।
 आसके ऄवतररक्त, भौगोवलक दृवष्ट से आसके राज्यों का अकार ऄवधक बड़ा नहीं है। ऄमेररका के
लगभग 2,00,000 िगा ककलोमीटर और िाजील के 3,00,000 िगा ककलोमीटर की तुलना में
भारत के राज्यों का अकार औसतन 1,10,000 िगा ककलोमीटर है। जमान राज्यों (Lander) का
औसत क्षेत्रफल लगभग 22,000 िगा ककलोमीटर, जबकक वस्िस कैं टन का औसत अकार के िल
1,588 िगा ककलोमीटर है।
 ऄतः प्रवत राज्य जनसंख्या के ऄनुसार राज्यों की संख्या की दृवष्ट से , यह ऄन्य संघीय देशों से पीछे
है, परंतु भौगोवलक क्षेत्र की दृवष्ट से यह ईनके लगभग बराबर या कम ही है।
(ii) क्या छोटे राज्य का ऄथा बेहतर शासन प्रदान करना है?
 नए राज्य पुनगाठन अयोग को आस पर अिश्यक रूप से विचार करना होगा कक क्या छोटे राज्यों
के वनमााण से प्रशासन में सुधार की संभािना है। नए राज्यों के गठन से राज्यों की संख्या में िृवद्ध

132 www.visionias.in ©Vision IAS

Google it:- https://upscpdf.com


https://upscpdf.com << Download From >> https://upscpdf.com

होगी तथा नए राज्यों को नइ राजधावनयों, प्रशासवनक संरचना, ऄदालतों और कर्जमयों की


अिश्यकता होती है।
 G-20 देशों के मध्य भारत में सािाजवनक क्षेत्र के ऄंतगात रोजगार की दर सबसे कम है। सरकारी
कमाचाररयों की संख्या में कमी के कारण करारोपण, न्याय प्रदायगी, सुरक्षा और वशक्षा एिं
स्िास्थ्य जैसी बुवनयादी सुविधाओं की ईपलब्धता के संदभा में भारतीय राज्यों की क्षमता प्रवतकू ल
रूप से प्रभावित होती है। यद्यवप, ररक्त पदों को भरना कौशल सुधार और ईच्च वशक्षा पर वनभार
करता है, तथावप नए राज्य आस संबंध में कोइ गारंटी प्रदान नहीं करते हैं।
 राज्यों के अकार को कम करने और सुशासन के मध्य कोइ ऄवनिाया संबंध नहीं है। राज्यों के
अकार को सुशासन से संबद्ध करने िाले प्रायः आस पूिााग्रह से ग्रवसत होते हैं कक छोटे राज्य
भौगोवलक दृवष्ट से ऄवधक सुसंगरठत और सामावजक रूप से ऄवधक एकजुट होगें तथा आससे
सािाजवनक खचा की दक्षता में सुधार करने में मदद वमलेगी।
 यह भी माना जाता है कक छोटे राज्य वनिाावचत प्रवतवनवधयों और मतदाताओं के मध्य की दूरी को
कम करके जिाबदेही में िृवद्ध कर सकते हैं। कफर भी यकद, गरीबी में कमी या अर्जथक संिृवद्ध के
संदभा में राज्यों के प्रदशान की तुलना की जाए, तो हमें राज्यों के अकार और ईनके प्रदशान के
मध्य कोइ स्पष्ट सह-संबंध दृष्टीगोचर नहीं होता है। भारत में निवनर्जमत नए राज्यों, यथा-
तेलंगाना, छत्तीसगढ़, झारखंड और ईत्तराखंड का वमवश्रत ऄनुभि यह दशााता है कक नए राज्यों
के वनमााण में विकास को बढ़ािा देने की संभािना का ऄंतर्जनवहत होना अिश्यक नहीं है।
(iii) नए राज्यों के वनमााण के ऄवतररक्त ऄन्य ईपलब्ध विकल्प क्या हैं?
 नए SRC के समक्ष तीसरा प्रश्न यह है कक यकद नए राज्यों के वनमााण का अधार प्रशासवनक
दक्षता है तो क्या नए राज्यों के वनमााण का कोइ ऄन्य विकल्प भी है जोकक संबंवधत मुद्दों का
वनिारण कर सके ?
 कु छ मामलों में कें रीय संसाधनों का बेहतर वितरण या राज्य स्तर पर पंचायती राज संस्थाओं को
सत्ता का बेहतर हस्तांतरण, नए राज्यों के वनमााण से ऄवधक िांवछत पररणाम दे सकता है। यहाँ
यह ईल्लेख ककया जा सकता है कक नगर वनगमों, स्िायत्त क्षेत्रीय पररषदों या पंचायती राज
संस्थाओं के रूप में ईप-राज्य संस्थाओं को ऄवधकार देने में राज्यों का बहुत हद तक वमवश्रत
प्रदशान रहा है।
(iv) नए राज्यों के गठन का वनधाारण कौन करेगा?
 नइ SRC को वनवश्चत रूप से आस प्रश्न का जिाब देना चावहए कक यकद नए राज्यों का गठन करना
है तो आसका वनधाारण कौन करेगा। भारत की संिैधावनक व्यिस्था की एक कदलचस्प विशेषता
यह है कक ऄनुच्छेद 3 (जो प्रभािी रूप से कें र सरकार को नए राज्यों के वनमााण की शवक्त देता है)
की कें रीकृ त प्रकृ वत के बािजूद, राज्य विभाजन पर िास्तविक संघषा राज्य स्तर पर देखने को
वमलता है। नए SRC को आस पर विचार करना चावहए कक क्या विभाजन का समथान करने के
वलए राज्य विधानसभा के संकल्प को ऄवनिाया बनाया जाना चावहए।
(v) पूणा राज्य के दजे की मौजूदा मांगों पर ऄवधवनणायन
 ऄंत में, भविष्य में गरठत होने िाले SRC को गोरखालैंड, विदभा, बोडोलैंड, बुंदल
े खंड, हररत
प्रदेश और ऄन्य स्थानों पर राज्य के दजे हेतु ईठी ितामान मांगों पर वनणाय करने की अिश्यकता
होगी। भविष्य में ककसी भी SRC के वलए एक ऄनुत्तररत प्रश्न, राज्य के 'ईवचत' अकार, का ईत्तर
खोजना असान नहीं होगा। ककस क्षेत्र को राज्य बनाया जाना चावहए, आसका सािाभौम या सबके
वलए स्िीकाया ईत्तर नहीं है।

133 www.visionias.in ©Vision IAS

Google it:- https://upscpdf.com


https://upscpdf.com << Download From >> https://upscpdf.com

1.9. राज्यों के नाम पररिता न से जु ड़े मु द्दे

ऄगस्त 2016 में, पवश्चम बंगाल विधानसभा ने एक प्रस्ताि पाररत ककया, वजसमें पवश्चम बंगाल राज्य
का नाम बदलकर बंगाली (भाषा) में “बंग्ला” और ऄंग्रेजी में “बंगाल” ककए जाने के संबंध में प्रािधान
है।
राज्यों के नाम पररितान से संबवं धत संिध
ै ावनक प्रािधान:
 ऄनुच्छेद 3, संसद को ककसी भी राज्य के नाम में पररितान करने की शवक्त प्रदान करता है।
 ऄनुच्छेद 3 के ऄधीन विवध बनाने की जो प्रकक्रया विवहत की गयी है, ईसका यहाँ पालन ककया
जाता है। ऄथाात,् संसद विवध बनाकर ही यह काया करती है।
 ऄनुच्छेद 3 के ऄधीन संविधान संशोधन के वबना ऐसा ककया जा सकता है। लेककन, यकद ईक्त
राज्य की भाषा में पररितान (जैसे- ईवड़या के स्थान पर ओवडया; 96िां संविधान संशोधन) के
संबंध में प्रािधान हो तथा जो संविधान की अठिीं ऄनुसूची में पररितान से संबंवधत हो तो आस
हेतु संविधान संशोधन विधेयक की अिश्यकता होती है।
राज्यों के नाम पररितान की अिश्यकता क्यों?
 विरटश शासन के दौरान ऄंग्रेजों ने ऄपनी प्रशासवनक सुविधा के दृवष्टकोण से कइ भारतीय प्रांतों,
क्षेत्रों एिं शहरों का नाम पररिर्जतत ककया था।
 आनके नामकरण में विरटश प्रभाि भी झलकता है, जहां ईनके ऐवतहावसक नामकरण को ऄनदेखा
ककया गया है।
 ऐसे प्रांतों, शहरों अकद की स्थानीय भाषा एिं ऄंग्रेजी भाषा में ईच्चारण एिं ध्िन्यात्मक
(Phonetic) समस्या भी आनके नाम में पररितान के कारण रही हैं। जैसे- यूनाआटेड प्रोविस,
वमवडल प्रोविस अकद।

ऄनुच्छेद 3 के ऄधीन जो ऄनेक विधान बनाए गए हैं ईनमें से कु छ ईदाहरणस्िरूप नीचे कदए गए हैं:
(i) नए राज्यों का वनमााण
 अंध्र राज्य ऄवधवनयम, 1953 (अंध्र प्रदेश)।
 राज्य पुनगाठन ऄवधवनयम, 1956 (के रल, मध्य प्रदेश अकद)।
 मुंबइ पुनगाठन ऄवधवनयम, 1960 (महाराष्ट्र और गुजरात)।
(ii) विलय
 वहमाचल प्रदेश और वबलासपुर (नया राज्य) ऄवधवनयम, 1954 (वबलासपुर का वहमाचल प्रदेश में
विलय)।
 ऄर्जजत राज्य क्षेत्र (विलय) ऄवधवनयम, 1960 (कु छ राज्य क्षेत्रों का ऄसम, पंजाब और पवश्चम
बंगाल में विलय)।
(iii) विभाजन
 पंजाब पुनगाठन ऄवधवनयम, 1966 (पंजाब को, पंजाब और हररयाणा में विभावजत ककया गया
तथा चंडीगढ़ कें र शावसत प्रदेश का वनमााण ककया गया)।
 पूिोत्तर क्षेत्र (पुनगाठन) ऄवधवनयम, 1971 (आसके द्वारा मवणपुर, वत्रपुरा, मेघालय, वमजोरम और
ऄरूणाचल प्रदेश का वनमााण हुअ)।
(iv) ऄध्यपाण
 भारत और पाककस्तान के मध्य हुए करार के ऄनुसरण में 9िें संविधान संशोधन ऄवधवनयम,
1960 द्वारा कु छ राज्य क्षेत्र पाककस्तान को ऄध्यर्जपत ककए गए।
 100िां संविधान संशोधन ऄवधवनयम: 119िाँ संविधान संशोधन विधेयक, 2013 भारत एिं

134 www.visionias.in ©Vision IAS

Google it:- https://upscpdf.com


https://upscpdf.com << Download From >> https://upscpdf.com

बांग्लादेश के मध्य भूवम सीमा समझौते से संबंवधत है। यह विधेयक भारतीय संविधान के 100िें
संशोधन का अधार बना। यह संविधान संशोधन विधेयक दोनों देशों की सीमा के मध्य मौजूद कु छ
विदेशी ऄंतःक्षेत्रों (एन्क्लेि) के अदान-प्रदान से संबंवधत है। आसके तहत बांग्लादेश को भारत से
111 एन्क्लेि (17,160 एकड़), जबकक भारत को बांग्लादेश से 51 एन्क्लेि (7,110 एकड़) प्राप्त
हुए।
(v) ऄजान (नए क्षेत्रों का प्रिेश)
 36िें संविधान संशोधन ऄवधवनयम, 1975 द्वारा वसकक्कम का भारत संघ में प्रिेश।
(vi) नाम पररितान
 मरास राज्य (नाम पररितान) ऄवधवनयम, 1968 (मरास का नाम तवमलनाडु ककया गया)।
 मैसूर राज्य (नाम पररितान) ऄवधवनयम, 1973; मैसूर राज्य का नाम कनााटक कर कदया गया।
 ईड़ीसा (नाम पररितान) ऄवधवनयम, 2011; ईड़ीसा का नाम बदलकर ओवडशा ककया गया।

2. समकालीन मुद्दे
2.1. ते लं गाना मु द्दा

तेलंगाना राज्य के वनमााण के मामले में, अंध्र प्रदेश पुनगाठन विधेयक, 2013 को अंध्र प्रदेश की

विधान सभा और विधान पररषद द्वारा वनणाायक रूप से ऄस्िीकार कर कदया गया। लेककन, अंध्र

प्रदेश विधान-मंडल का यह कदम भी कें र सरकार को संसद में अंध्र प्रदेश पुनगाठन ऄवधवनयम, 2014
पाररत करने से नहीं रोक पाया।
ऄनुच्छेद 3 की विवधक व्याख्या से स्पष्ट है कक अंध्र प्रदेश की विधानसभा के विचारों का कोइ विवधक
महत्ि नहीं था और तेलंगाना का गठन पूणातया भारत सरकार का विशेषावधकार था। लेककन भारतीय
संघिाद के विकासमान ऄथा के प्रकाश में आस वस्थवत पर पुनर्जिचार ककया जाना चावहए।
हालाँकक, जैसा कक व्यापक रूप से सुझाि कदया गया है कक राज्य विधान-मंडल के विचारों को के िल

औपचाररक महत्ि का मानना, राष्ट्रीय महत्ि के मामले में परामशा की संिैधावनक प्रकक्रया को पूणातया

वनरथाक बना देगा। ऄनुच्छेद 3 का ितामान प्रािधान संविधान (पांचिां संशोधन) ऄवधवनयम, 1955

द्वारा स्थावपत ककया गया था। आस संशोधन से पहले, राष्ट्रपवत संबवं धत राज्य विधानसभाओं को ईनके
विचार के वलए संदर्जभत करने के बाद ही संसद में संशोधन विधेयक प्रस्तुत कर सकता था। यह
समयसाध्य प्रकक्रया थी, जो राज्यों को प्रत्युत्तर देने में समय लेने की ऄनुमवत देती थी, वजससे कें र
सरकार के प्रयास विफल हो जाते थे। यह संशोधन राज्य पुनगाठन अयोग की ररपोटा को सुचारू रूप से
लागू करने के वलए अिश्यक था वजसने राज्यों की सीमाओं को नए वसरे से वनधााररत करने और नए
राज्यों के वनमााण की ऄनुशंसा की थी। आस संबंध में राज्य विधान-मंडल की शवक्त को पयााप्त रूप से
आस प्रकार सीवमत ककया गया कक कोइ भी एक राज्य, पुनगाठन की प्रकक्रया में ऄिरोध ईत्पन्न न कर
पाए।
संघीय चररत्र पर प्रभाि डालने िाली संविधान के प्रािधानों की पुनव्यााख्या की यही भािना एस.
अर. बोममइ बनाम भारत संघ में भी प्रदर्जशत हुइ थी, जब राष्ट्रपवत शासन अरोवपत करने की
राष्ट्रपवत की शवक्त को न्यावयक पुनर्जिलोकन से उपर और परे नहीं माना गया था।

135 www.visionias.in ©Vision IAS

Google it:- https://upscpdf.com


https://upscpdf.com << Download From >> https://upscpdf.com

भारत की संघीय संरचना में कें रीयकरण की प्रिृवत्त को ईस समय ऄपनाया गया था जब भारत के
वभन्न-वभन्न तत्िों को एक साथ एक राष्ट्र में वपरोना अिश्यक था। यह ऐसा काया था वजसके वलए
भारत सरकार को विवशष्ट रूप से तैयार ककया गया था और वजसके वलए एक सहायक संिैधावनक
संरचना की अिश्यकता थी।
आस प्रकार, संविधान के संघीय प्रािधान राजनीवतक रणनीवत के समान ईत्कृ ष्ट दृवष्ट िाले थे। आस
प्रकार की रणनीवत के वलए ऄब सािधानीपूिाक पुन: ऄंशांकन की अिश्यकता है, क्योंकक राष्ट्र वनमााण
के वलए आस प्रकार के ईपकरण का ईपयोग यकद वििेकपूणा तरीके से नहीं ककया जाता है, तो आसे
विशुद्ध रूप से कें रीय अवधपत्य के रूप में देखा जाएगा। ऐसी बलात एकात्मक संरचना के नकारात्मक
पररणाम भी हो सकते हैं।

2.2. गोरखालैं ड मु द्दा


पृथक गोरखालैंड राज्य के वनमााण की मांग को लेकर ऄक्सर दार्जजवलग अकद क्षेत्रों में पूणा बंदी की
वस्थवत देखने को वमलती है और कइ बार यह मांग वहसक प्रदशान का स्िरुप भी ग्रहण कर लेता है।
हाल ही में, ऐसे प्रदशान देखेने को वमले।
अंदोलन क्यों?
 तत्कावलक कारण: राज्य सरकार द्वारा 9िीं कक्षा तक बंगाली भाषा को ऄवनिाया बनाया जा रहा
है। गोरखा लोग राज्य सरकार के आस कदम को ऄपनी पहचान एिं ऄवस्मता के वलए खतरे के रूप
में देखते हैं क्योंकक ईनकी मातृभाषा नेपाली है।
 दीघाकावलक कारण: GTA (गोरखालैंड टेररटोररयल एडवमवनस्रेशन) के कामकाज में समस्याएं।
नेताओं ने राज्य सरकार पर हस्तक्षेप और GTA को पयााप्त वित्तीय संसाधन हस्तांतररत नहीं
करने का अरोप लगाया है।
गोरखालैंड की मांग का आवतहास
गोरखालैंड में दार्जजवलग, कावलमपोंग, कु र्जसओंग और ऄन्य पहाड़ी वजलों के नेपाली भाषी क्षेत्र
सवममवलत हैं। आन क्षेत्रों के लोगों का बंगाली समुदाय से कदावचत ही कोइ प्रत्यक्ष संबंध रहा है और िे
नृजातीयता, संस्कृ वत और भाषा के मामले में वभन्न हैं।
गोरखालैंड की मांग क्यों?
 भाषा और संस्कृ वत में वभन्नता।
 भारतीय गोरखा पहचान की अकांक्षा: क्योंकक िषा 1988 में दार्जजवलग गोरखा वहल काईं वसल के
गठन और िषा 2012 में गरठत GTA ने आस अकांक्षा को पूरा नहीं ककया और िे ऄसफल रहे।
 सापेवक्षक रूप से अर्जथक वपछड़ापन।
 बंगाली भाषी लोगों द्वारा कवथत दुर्वयाहार और सरकार द्वारा आसकी ऄनदेखी अकद।
गोरखालैंड के प्रवत ऄनुकक्रयाएं
दार्जजवलग गोरखा वहल काईं वसल (DGHC): िषा 1986 के अंदोलन को देखते हुए, जुलाइ 1988 में
भारत सरकार, पवश्चम बंगाल सरकार और गोरखा नेशनल वलबरेशन फ्रंट के मध्य एक वत्रपक्षीय
समझौता हुअ। आसके ऄंतगात, "दार्जजवलग वजले के पहाड़ी क्षेत्रों में रहने िाले लोगों की सामावजक,
अर्जथक, शैवक्षक और सांस्कृ वतक ईन्नवत" के वलए राज्य ऄवधवनयम के ऄंतगात स्िायत्त रूप से
दार्जजवलग गोरखा वहल काईं वसल (DGHC) की स्थापना की गइ थी। आस काईं वसल में दार्जजवलग वजले
के तीन पहाड़ी सब-वडविजन (ईप-संभाग) और वसलीगुड़ी सब-वडविजन के कु छ क्षेत्र सवममवलत थे।
समस्याएं:
 काईं वसल को सीवमत कायाकारी शवक्तयां प्रदान की गईं थीं। ऐसे में विधायी शवक्तयों के ऄभाि के
कारण क्षेत्र के लोगों की अकांक्षाओं को पूरा नहीं ककया जा सका।
 काईं वसल में दुअर क्षेत्र का गैर-समािेशन, ऄसंतोष का एक प्रमुख कारण बन गया था।

136 www.visionias.in ©Vision IAS

Google it:- https://upscpdf.com


https://upscpdf.com << Download From >> https://upscpdf.com

गोरखालैंड टेररटोररयल एडवमवनस्रेशन (GTA): िषा 2012 में भारत सरकार, पवश्चम बंगाल सरकार
और गोरखा जनमुवक्त मोचाा (GJM) के मध्य हस्ताक्षररत एक वत्रपक्षीय समझौते के माध्यम से GTA
की स्थापना की गयी। आसने दार्जजवलग गोरखा वहल काईं वसल का स्थान ग्रहण ककया है। यह एक ऄधा-
स्िायत्त प्रशासवनक वनकाय है। आसके पास प्रशासवनक, कायाकारी और वित्तीय शवक्तयां हैं लेककन
विधायी शवक्तयां नहीं हैं। GTA में ितामान में आसके प्रावधकार के ऄंतगात तीन पहाड़ी सब-वडविजन
दार्जजवलग, कु र्जसओंग और वमररक तथा दार्जजवलग वजले के वसलीगुड़ी सब-वडविजन के कु छ क्षेत्र एिं
संपूणा कावलमपोंग वजला सवममवलत हैं।
समस्याएं:
 GTA के पास विधायी शवक्तयों का ऄभाि है। आसका ऄथा है यह कक क्षेत्र के लोगों का ऄपने उपर
वनयंत्रण करने िाले विवधयों पर कोइ वनयंत्रण नहीं है।
 दुअर (Dooars) क्षेत्र को आसमें सवममवलत नहीं ककया गया है। हालांकक, दुअर क्षेत्र में "गोरखा
बहुसंख्यक" क्षेत्रों की पहचान करने के वलए एक सत्यापन टीम का गठन ककया गया था।
समाधान
पृथक गोरखालैंड राज्य की मांग करने िाले अंदोलन को शीघ्र समाप्त करने हेतु एक ईपयुक्त समाधान
प्रस्तुत ककया जाना चावहए। यह समाधान ऐसा होना चावहए जो बंगाली भाषी बहुमत की भािनाओं
को अहत ककए वबना नेपाली भाषी लोगों की अकांक्षाओं को पूरा करे। ध्यातव्य है कक बंगाली भाषी
बहुमत राज्य के विभाजन के विरूद्ध है। ऐसे समाधान हेतु संभावित कदमों में वनम्नवलवखत सवममवलत
हैं:
 ईपयुक्त शवक्त साझाकरण समझौता: पृथक गोरखालैंड राज्य की मांग ऄपने अप में अर्जथक रूप
से व्यिहाया नहीं है। पयाटन को छोड़कर आस क्षेत्र के पास ऄपना कोइ ऄन्य महत्िपूणा संसाधन
नहीं है। यहाँ का चाय ईद्योग भी संकट का सामना कर रहा है। GTA के कामकाज में सुधार करने
और ईसे जिाबदेह बनाने की अिश्यकता है।
 कें र के साथ-साथ राज्य सरकार को भी गोरखाओं की अिश्यकताओं और अकांक्षाओं के प्रवत
ऄवधक संिेदनशील होने की अिश्यकता है, जैसे- बंगाली भाषा को थोपने के बजाय आसे
िैकवल्पक बनाया जा सकता है।
 क्षेत्र का अर्जथक विकास: ऄस्पताल, विद्यालय एिं ऄन्य सािाजवनक संस्थाओं की स्थापना की
जानी चावहए तथा विद्यमान र्वयिस्था में सुधार लाया जाना चावहए।
 ऄविभावजत पवश्चम बंगाल के भीतर एक स्िायत्त गोरखालैंड राज्य के वनमााण पर विचार ककया
जा सकता है। ऄनुच्छेद 244A ऄसम में कु छ जनजातीय क्षेत्रों के वलए ऄपनी स्ियं की विधावयका
और मंवत्रपररषद के साथ स्िायत्त राज्य का प्रािधान करता है। संिैधावनक संशोधन द्वारा, आस
ऄनुच्छेद की प्रयोज्यता को पवश्चम बंगाल हेतु विस्तृत ककया जा सकता है। िैकवल्पक रूप से ,
संविधान संशोधन के माध्यम से, ऄनुच्छेद 244A के समान एक ऄनुच्छेद को संविधान के भाग
VI में एक नए ऄध्याय के रूप में ऄंतर्जिष्ट ककया जा सकता है। यह पवश्चम बं गाल को विभावजत
ककए गए वबना ितामान राज्य के भीतर विधावयका और मंवत्रपररषद से युक्त एक स्िायत्तशासी
गोरखालैंड की स्थापना को संभि बनाएगा।
साथ ही, अंदोलनकाररयों को यह भी ध्यान में रखना चावहए कक वनरंतर संकीणा होती जा रही
नृजातीय पहचान के अधार पर छोटे-छोटे राज्य वनर्जमत नहीं ककये जा सकते हैं। ऐसा समाधान तैयार
करने की अिश्यकता है जो व्यािहाररक होने के साथ-साथ सभी वनिावसयों के वहतों का भी समािेश
करे। नहीं तो, नए राज्यों की वनरंतर बढ़ती मांग वहसा, जिाबी वहसा और विरोध प्रदशानों का चक्र
ऄविरत रखते हुए देश की वस्थरता के वलए खतरा पैदा करेगी।

137 www.visionias.in ©Vision IAS

Google it:- https://upscpdf.com


https://upscpdf.com << Download From >> https://upscpdf.com

2.3. ऄनु च्छे द 370 और 35A का वनरसन (Removal of Article 370 and 35A)

हाल ही में, कें र सरकार ने संविधान के ऄनुच्छेद 370 के तहत जममू और कश्मीर को प्रदत्त विशेष दजे
को समाप्त कर कदया।
संबवं धत तथ्य

 राष्ट्रपवत ने “जममू और कश्मीर राज्य सरकार की सहमवत” से संविधान (जममू और कश्मीर में

लागू) अदेश, 2019 {The Constitution (Application to Jammu and Kashmir)

Order, 2019} प्रख्यावपत ककया है। आस अदेश में यह ईवल्लवखत है कक भारतीय संविधान के
सभी प्रािधान राज्य में प्रितानीय होंगे। आसका तात्पया यह है कक िे सभी प्रािधान जो जममू और
कश्मीर हेतु एक पृथक संविधान के अधार का वनमााण करते हैं, ईन्हें वनरस्त कर कदया गया है।

आस प्रकार, ऄनुच्छेद 35A स्ित: वनरवसत हो गया है।

 आसके साथ ही, राष्ट्रपवत के ईक्त अदेश के प्रभाि से व्युत्पन्न प्रावधकार का प्रयोग करते हुए संसद

द्वारा एक सांविवधक संकल्प को भी ऄनुमोकदत ककया गया, वजसमें यह ऄनुशंसा की गइ कक

राष्ट्रपवत ऄनुच्छेद 370 (के ऄवधकांश प्रािधान) को वनष्प्रभािी (abrogate) करते हैं।

 साथ ही, संसद द्वारा जममू और कश्मीर पुनगाठन ऄवधवनयम, 2019 (Jammu and Kashmir

Reorganization Act, 2019) को भी पाररत ककया गया है। आसके द्वारा जममू और कश्मीर को

दो संघ शावसत प्रदेशों में विभावजत कर कदया गया है, यथा- जममू और कश्मीर वडिीज़न

(विधानसभा युक्त) तथा लद्दाख (विधानसभा रवहत)।

ऄनुच्छेद 370 और ऄनुच्छेद 35A : एक संवक्षप्त पृष्ठभूवम

 जममू और कश्मीर की विवशष्ट वस्थवत ईन पररवस्थवतयों का पररमाण थी, वजसमें राज्य का भारत
में विलय हुअ। भारत सरकार ने यह घोषणा की थी कक जममू और कश्मीर की जनता ऄपनी
संविधान सभा के माध्यम से काया करते हुए ऄपने संविधान और भारत सरकार के क्षेत्रावधकार
का वनधाारण करेगी।

 जममू और कश्मीर के संदभा में संविधान के प्रािधानों की ऄनुप्रयोज्यता (applicability) एक

ऄंतररम व्यिस्था की भांवत थी। यह भारत के संविधान के ऄनुच्छेद 370 में वनवहत प्रािधान का

सार था।

 ऄनुच्छेद 370 जममू और कश्मीर राज्य के संबध


ं में एक ऄस्थायी ईपबंध था, जो राज्य को आसके

पृथक संविधान होने की ऄनुमवत के साथ-साथ विवशष्ट शवक्तयाँ (special powers) भी प्रदान
करता था।
 ऄनुच्छेद 370 के ऄनुसार रक्षा, विदेश मामले, वित्त और संचार को छोड़कर ऄन्य सभी कानूनों

के प्रितान हेतु संसद को राज्य सरकार की सहमवत की अिश्यकता होती थी।

 भारतीय संविधान का ऄनुच्छेद 35A, जो ऄनुच्छेद 370 से ही व्युत्पन्न हुअ था, राज्य के स्थायी

वनिावसयों, ईनके विशेषावधकारों तथा विवशष्ट ऄवधकारों को पररभावषत करने हेतु जममू और

कश्मीर विधान सभा को शवक्तयाँ प्रदान करता था।

138 www.visionias.in ©Vision IAS

Google it:- https://upscpdf.com


https://upscpdf.com << Download From >> https://upscpdf.com

ऄनुच्छेद 370 और 35A का वनरसन कै से संभि हुअ?

 राष्ट्रपवत ने संविधान के ऄनुच्छेद 370 (1) के ऄंतगात एक राष्ट्रपतीय अदेश (presidential


order) जारी ककया था। यह खंड राष्ट्रपवत को जममू और कश्मीर सरकार की सहमवत से राज्य में

प्रितानीय मामलों को वनर्ददष्ट करने की शवक्त प्रदान करता है।

 आस अदेश द्वारा ऄनुच्छेद 367 में भी संशोधन ककया गया। ऄनुच्छेद 367 में कु छ प्रािधानों के
पठन ऄथिा ईनकी व्याख्या संबंधी रीवत का समािेश है। संशोवधत ऄनुच्छेद यह घोषणा करता है
कक ऄनुच्छेद 370 (3) में ईवल्लवखत राज्य की “संविधान सभा” ऄवभव्यवक्त को राज्य की “विधान
सभा” पढ़ा जाएगा। ईल्लेखनीय है कक, ऄनुच्छेद 370 (3) में यह प्रािधावनत था कक ऄनुच्छेद

370 को राज्य की विधान सभा की सहमवत से ही संशोवधत ककया जाएगा। हालांकक, आस


संशोधन के कारण ऄब आसे राज्य विधान-मंडल की ऄनुशंसा के अधार पर भी समपाकदत ककया
जा सकता है।

 दूसरे शब्दों में, सरकार ने संविधान के एक प्रािधान (ऄनुच्छेद 367) में संशोधन करने हेतु
ऄनुच्छेद 370 (1) द्वारा प्रदत्त शवक्तयों का प्रयोग ककया तथा तत्पश्चात ऄनुच्छेद 370 (3) को
संशोवधत ककया गया। पररणामस्िरूप यह सांविवधक संकल्प (भारतीय संविधान के ऄनुच्छेद
370 के वनरसन हेतु संकल्प) को प्रस्तुत करने का कारक बना। चूँकक, जममू और कश्मीर राष्ट्रपवत

शासन के ऄधीन था, आसवलए राज्यपाल की सहमवत को ही “जममू और कश्मीर सरकार” की

सहमवत स्िीकार कर वलया गया।

ईठाए गए कदम के संभावित वनवहताथा


 जममू और कश्मीर पर भारतीय संविधान की पूणा प्रितानीयता।
 पृथक ध्िज के विशेषावधकार का ईन्मूलन।
 जममू और कश्मीर विधान सभा के पूिािती छह िषीय कायाकाल के स्थान पर पांच िषीय
कायाकाल का प्रािधान।
 रणबीर दंड संवहता (जममू और कश्मीर हेतु पृथक दंड संवहता) का भारतीय दंड संवहता द्वारा
प्रवतस्थापन।
 ऄनुच्छेद 356, वजसके तहत ककसी भी राज्य में राष्ट्रपवत शासन लागू ककया जा सकता है,
पुनगारठत जममू और कश्मीर संघ शावसत प्रदेश हेतु भी प्रितानीय होगा।
 विद्यालय-महाविद्यालयों में एडवमशन और राज्य की सरकारी नौकररयों में के न्रीय कोटा संबंधी
कानून लागू होंगे।
 ऄन्य राज्यों के लोग समपवत्त और वनिास ऄवधकार प्राप्त करने के पात्र होंगे।
 सूचना का ऄवधकार ऄवधवनयम प्रितानीय होगा।
 जममू और कश्मीर के संविधान के कु छ प्रािधान जो ककसी ऄन्य राज्य के व्यवक्त से वििाह करने
िाली राज्य की मवहलाओं को समपवत्त के ऄवधकारों से िंवचत करते हैं, ऄिैध घोवषत हो सकते हैं।

ऄनुछेद 370 का वनरसन: संिध


ै ावनक और विधायी चुनौवतयाँ
जममू और कश्मीर के संबंध में कें र सरकार की हावलया कायािाही को चुनौती देते हुए ईच्चतम
न्यायालय में कइ यावचकाएं दायर की गइ हैं। ईल्लेखनीय है कक वनम्नवलवखत िैधावनक मुद्दे न्यावयक
वििेचनाओं के दौरान चचाा का विषय हो सकते हैं:
 राष्ट्रपतीय अदेश की िैधावनकता (Legality of the Presidential order): ऄनुच्छेद 370 को

राष्ट्रपतीय अदेश द्वारा संशोवधत नहीं ककया जा सकता। यद्यवप, यह अदेश ऄनुच्छेद 367 को

139 www.visionias.in ©Vision IAS

Google it:- https://upscpdf.com


https://upscpdf.com << Download From >> https://upscpdf.com

संशोवधत करता है, तथावप आन संशोधनों के विषय ऄनुच्छेद 370 में भी संशोधन करते हैं और

जैसा कक विवभन्न ऄिसरों पर ईच्चतम न्यायालय द्वारा यह वनणाय कदया गया है कक ‘वजसे अप
प्रत्यक्ष रूप से नहीं कर सकते, ईसे अप परोक्ष रूप से भी नहीं कर सकते’। आस प्रकार, आस अदेश

की िैधावनकता - जहाँ तक यह ऄनुच्छेद 370 में संशोधन करता है - संदह


े ास्पद है।
 राष्ट्रपवत शासन का दुरुपयोग और राज्यपाल को वनिाावचत विधान सभा का एक प्रवतवनवध
बनाना: राज्यपाल, राज्य में कें र सरकार का प्रवतवनवध होता है। आस प्रकार, िास्ति में आस

मामले में कें र सरकार ने स्ियं से ही परामशा ककया है।


o आसके ऄवतररक्त, राष्ट्रपवत शासन ऄस्थायी होता है तथा यह माना जाता है कक आसका

ऄवस्तत्ि के िल वनिाावचत सरकार की पुनस्थाापना से पहले तक ही है। ऐसे में, वनिाावचत


विधान सभा की सहमवत के वबना के िल राज्यपाल की सहमवत से एक राज्य के समपूणा दजे
में पररितान जैसे स्थायी चररत्र के वनणाय ऄपने अप में संकदग्ध हैं।

 राज्य विधान सभा की संविधान सभा के साथ सामयता: दोनों के मध्य प्रमुख ऄंतर यह है कक,
जहाँ विधान सभा को संविधान के ऄनुसार ऄपनी शवक्तयों का प्रयोग करना होता है, िहीं

संविधान सभा संविधान का वनमााण करती है। यह विभेद, जो कक भारतीय संविधान के मूल ढांचे

के वसद्धांत का ममा है, आस अधार पर कु छ संिैधावनक संशोधनों को वनवषद्ध करता है कक संसद


जो प्रवतवनवधत्ि प्रावधकार का प्रयोग करती है, ईसकी शवक्तयाँ सीवमत हैं तथा िह एक निीन
संविधान का वनमााण नहीं कर सकती है और आसवलए संप्रभु प्रावधकार का प्रयोग करने में भी
ऄसक्षम है।
 जममू और कश्मीर की संिध
ै ावनक वस्थवत के विरुद्ध ईठाया गया कदम: राष्ट्रपतीय अदेश में यह
स्िीकृ त ककया गया है कक विधान सभा को ऄनुच्छेद 370 को वनरवसत करने की शवक्त प्राप्त है।

जबकक, जममू और कश्मीर के संविधान का ऄनुच्छेद 147 आस तरह के कदम को वनवषद्ध करता है।

ऄनुच्छेद 147 यह स्पष्ट करता है कक जममू और कश्मीर के संविधान में ककसी भी प्रकार के
पररितान हेतु विधान सभा के दो-वतहाइ सदस्यों के ऄनुमोदन की अिश्यकता होगी।

वनष्कषा

 ज्ञातव्य है कक, जब जममू और कश्मीर राज्य की संविधान सभा ने काया करना बंद ककया था, तब
ऄनुच्छेद 370 की प्रकृ वत के संबंध में दीघाकावलक िाद-वििाद अरमभ हुए। विघटन से पूिा

संविधान सभा ने न तो ऄनुच्छेद 370 के ईन्मूलन की ऄनुशंसा की थी तथा न ही आसके स्थावयत्ि


का समथान ककया था। ऄभी भी यह पयािेक्षण करना शेष है कक क्या िह रीवत वजसके तहत
ऄनुच्छेद 370 का वनरसन ककया गया है, न्यावयक पुनर्जिलोकन के ऄधीन है।

 ईल्लेखनीय है कक ऄनुच्छेद 370 राज्य के लोगों को शासन के विषयों में ऄपने विचार प्रस्तुत
करने का ऄवधकार प्रदान करता था जोकक ऄपनी पहचान के मामलों में ऄत्यवधक सुभेद्य ऄनुभि
करते हैं और ऄपने भविष्य के बारे में ऄवनवश्चत हैं। हालांकक, ये वचताएं भी प्रकट की गइ हैं कक न
तो यह कदम जममू और कश्मीर राज्य के सामान्य-जन के ऄनुकूल होगा तथा न ही यह शेष भारत
के साथ राज्य के एकीकरण को सुविधाजनक बनाएगा। तथावप, यह ऄपेक्षा की गइ है कक यह
कदम जममू और कश्मीर हेतु विकास एिं समािेशन के एक नि प्रभात का सृजन करेगा, जो राज्य
के िंवचत ि हावशये पर रह रहे लोगों को मुख्यधारा में सवममवलत करेगा।

140 www.visionias.in ©Vision IAS

Google it:- https://upscpdf.com


https://upscpdf.com << Download From >> https://upscpdf.com

3. विगत िषों में Vision IAS GS मेंस टे स्ट सीरीज में पू छे


गए प्रश्न (Previous Year Vision IAS GS Mains Test
Series Questions)
1. ‘भारतीय संघ (यूवनयन ऑफ़ आंवडया)’ और ‘भारत का राज्य क्षेत्र’ (Territory of India)
पदों के मध्य ऄंतर स्पष्ट कीवजए। प्रस्तावित 119िें संविधान संशोधन विधेयक के विशेष
संदभा में ककसी राज्य के प्रादेवशक गठन को पुनगारठत करने की संसद की शवक्तयों पर चचाा
कीवजए।
दृवष्टकोणः
आस प्रश्न के प्रथम भाग का ईत्तर डी. डी. बसु और लक्ष्मीकांत जैसे लेखकों की मानक पुस्तकों
के संदभा के साथ कदया जा सकता है। ईत्तर के दूसरे भाग के वलए हमें संविधान के ऄनुच्छेद
1 से 4 तक का संदभा देने की अिश्यकता है। हमें 119िें संविधान संशोधन विधेयक की
गहन छानबीन करनी होगी जो संविधान की पहली ऄनुसूची को पररिर्जतत करने की मांग
करता है। आसमें बांग्लादेश के साथ एन्क्लेिों (विदेशी ऄंतःक्षेत्रों) (ऄसम, पवश्चमी बंगाल,

मेघालय और वत्रपुरा) का अदान-प्रदान शावमल है और आसवलए यह ऄनुच्छेद 3 द्वारा


शावसत सामान्य संसदीय प्रकक्रया में शावमल नहीं है। बेरुबारी संघ िाद (1960) में प्रदत्त

वनणाय के अलोक में , विधेयक संविधान की पहली ऄनुसूची में ऄसम, पवश्चमी बंगाल,
मेघालय और वत्रपुरा के प्रदेशों से संबंवधत पररच्छेदों में संशोधन करने की मांग करता है।
ईत्तर:
“भारतीय संघ” पद की तुलना में “भारत का राज्य क्षेत्र” पद का ऄवधक व्यापक संकेताथा है।
“जहाँ संघ शब्द में राज्य शावमल होते हैं जो संघीय प्रणाली के सदस्य होने की वस्थवत का
लाभ ईठाते हैं और संघ के साथ शवक्तयों के वितरण को साझा करते हैं, िही “भारत का

राज्य क्षेत्र” पद में िैसे सभी क्षेत्र शावमल हैं वजन तक भारत की संप्रभुता विस्तृत है”।
 आस प्रकार, भारत का राज्य क्षेत्र , राज्यों से ऄलग प्रदेश के दो ऄन्य िगों को सवममवलत

करता है ऄथाात् (i) संघ शावसत प्रदेश और (ii) ऐसे ऄन्य क्षेत्र जो भारत द्वारा
ऄवधग्रवहत ककए जा सकते हैं {ऄनुच्छेद 1(3)}।
 संविधान के भाग 1 में ऄनुच्छेद 1 से 4 संघ और आसके प्रदेशों की चचाा करते हैं।

 ऄनुच्छेद 3 के ऄनुसार संसद, विवध द्वारा:

(i) ककसी राज्य में से ईसका राज्य क्षेत्र ऄलग करके ऄथिा दो या ऄवधक राज्यों को या
राज्यों के भागों को वमलाकर ऄथिा ककसी राज्य क्षेत्र को ककसी राज्य के भाग के साथ
वमलाकर नए राज्य का वनमााण कर सके गी;
(ii) ककसी राज्य के क्षेत्र को बढ़ा सके गी;

(iii) ककसी राज्य का क्षेत्र घटा सके गी;

(iv) ककसी राज्य की सीमाओं में पररितान कर सके गी; और


(v) ककसी राज्य के नाम में पररितान कर सके गी।

 जैसा कक ऄनुच्छेद 4 में ईवल्लवखत है, ऄनुच्छेद 2 और 3 के ऄधीन वनर्जमत ऄवधवनयम,


ऄनुच्छेद 368 के ऄंतगात संिैधावनक संशोधन के समान नहीं होते हैं।

141 www.visionias.in ©Vision IAS

Google it:- https://upscpdf.com


https://upscpdf.com << Download From >> https://upscpdf.com

 कफर भी, जैसा बेरुबारी संघ िाद (1960) के संदभा में ईच्चतम न्यायलय द्वारा

ऄवभवनधााररत ककया गया है कक ऄनुच्छेद 3 के ऄधीन राज्य के क्षेत्र में पररितान करने
की संसद की शवक्त, भारतीय भू-भाग का ककसी ऄन्य देश के साथ ऄपाण या अदान-

प्रदान को सवममवलत नहीं करता है।

 िषा 2011 में भारत एिं बांग्लादेश ने दोनों राष्ट्रों के मध्य िषा 1974 में संपन्न भूवम
सीमा समझौते (Land Boundary Agreement: LBA) के तहत विदेशी ऄंतः क्षेत्रों

(enclaves) के परस्पर अदान-प्रदान के वलए एक प्रोटोकॉल पर हस्ताक्षर ककए। आस

समझौते के तहत 111 भारतीय एन्क्लेि बांग्लादेश को तथा 51 बांग्लादेशी एन्क्लेिों


को भारत को हस्तांतररत करने की पररकल्पना की गइ। हालांकक, आस हेतु संसद के

ऄनुसमथान की अिश्यकता थी। आसी अिश्यकता को ध्यान में रखते हुए 119िां
संविधान संशोधन विधेयक, 2013 पुरःस्थावपत ककया गया था तथा वजसे िषा 2015
में पाररत ककया गया।
2. जहाँ कु छ लोगों का तका है कक संविधान का ऄनुच्छेद 3 राज्यों की कीमत पर कें र को ऄनन्य
शवक्तयां प्रदान करता है, िहीं दूसरों के ऄनुसार, यह संविधान में वनवहत संघिाद को बनाए
रखने तथा संरवक्षत करने के वलए संसद को सक्षम बनाता है। चचाा कीवजए। क्या सहकारी-
संघिाद की भािना के ऄनुरूप ऄनुच्छेद 3 पर पुनः विचार करने का समय अ गया है?
दृवष्टकोण:
 संक्षेप में ऄनुच्छेद 3 के प्रािधानों का ईल्लेख कीवजए।
 दोनों पक्षों के तकों को प्रस्तुत कीवजए।
 व्याख्या करते हुए आस बात पर चचाा कीवजए कक क्या ऄनुच्छेद 3 को पुनःपररभावषत
करने तथा ऐसे पररितानों हेतु कदशा-वनदेश जारी करने की अिश्यकता है।
ईत्तर:
ऄनुच्छेद 3 संसद को प्रावधकृ त करता है कक िह दो या ऄवधक राज्यों या राज्यों के भागों को

वमलाकर ऄथिा ककसी राज्य की सीमा में िृवद्ध, कमी या पररितान कर नए राज्य का
वनमााण कर सकती है ऄथिा ककसी राज्य के नाम में पररितान कर सकती है। ऐसे पररितान
से संबंवधत विधेयक संसद में अरंभ होंगे , परंतु ऐसे विधेयक को संबंवधत राज्य विधान-
मंडलों को भेजे जाने की अिश्यकता होती है। हालांकक ईनके (राज्यों के ) विचार बाध्यकारी
नहीं होते हैं। ककसी कें र शावसत प्रदेश के संदभा में ईनकी विधावयका को आससे संदर्जभत ककए
जाने की भी अिश्यकता नहीं होती है।
आस प्रकार, संसद ऄपनी आच्छानुसार भारत के राजनीवतक मानवचत्र का पुनर्जनधाारण कर

सकती है। दूसरे शब्दों में, ककसी भी राज्य की क्षेत्रीय ऄखंड ता या ऄविभाज्यता की

संिैधावनक गारंटी प्रदान नहीं की गयी है। आस प्रकार, भारत “विनाशी राज्यों का ऄविनाशी
संघ” है। संघ सरकार, ककसी राज्य को समाप्त कर सकती है जबकक राज्य सरकार, संघ को
समाप्त नहीं कर सकती है। स्ितंत्रता के पश्चात् विभाजनकारी प्रिृवत्तयों के संदभा में भारत
की एकता ि ऄखंडता को बनाए रखने के वलए यह अिश्यक था। ितामान में कइ क्षे त्रों द्वारा
संकीणा नृजातीय ि अकदिासी पहचान के अधार पर की जा रही नए राज्यों की मांग (जो
कक व्यािहाररक रूप से ऄसंभि है) को देखते हुए यह कहा जा सकता है कक ऄमुक प्रािधान
अिश्यक थे।

142 www.visionias.in ©Vision IAS

Google it:- https://upscpdf.com


https://upscpdf.com << Download From >> https://upscpdf.com

आसके ऄवतररक्त, संघ सरकार भाषाइ, प्रशासवनक और विकास संबंधी अकांक्षाओं के अधार
पर नए राज्यों के वनमााण संबंधी मांग को स्िीकार करने में ईदारता का पररचय देते अयी
है। यह फॉमूाला भारत की 'विविधता में एकता' बनाए रखने में बहुत सफल रहा है।

लेककन, यह सहकारी संघिाद की भािना के विपरीत भी है, जो कें र और राज्यों के मध्य


शवक्त के समान वितरण का वसद्धांत प्रस्तुत करता है, जबकक ऄनुच्छेद 3 राज्यों को कें र का

एक ईपकरण बना देता है, क्योंकक यहाँ राज्यों के पास ईनकी ऄविभाज्यता के संदभा में न के

बराबर ऄवधकार है। यह स्थानीय लोगों की आच्छा को नजरऄंदाज करता है , साथ ही यह


विविधता और बहुलिाद की ऄिधारणाओं के विरुद्ध भी है। यह लोगों के बीच ऄसंतोष पैदा
कर राजनीवतक नेतृत्ि एिं संिैधावनक मशीनरी से ईन्हें ऄलग-थलग कर देता है। तेलंगाना
के वनमााण के बाद अंध्र प्रदेश क्षेत्र में ऄसंतोष का भाि और ईत्तर-पूिा में ईग्रिाद की समस्या
अकद आसके कु छ ईदाहरण हैं।
आसवलए, आस पर पुनर्जिचार ककया जाना प्रशंसनीय होगा। आस संबंध में वनम्नवलवखत संभि
विकल्प हो सकते हैं:
 विशेष बहुमत के मामले में संसद को राज्य विधान-मण्डल की आच्छा का ऄिश्य ही पालन
करना चावहए।
 विशेष पररवस्थवतयों को छोड़कर यकद ऐसे पररितानों की अिश्यकता हो तो ईसकी शुरुअत
राज्य विधान-मण्डल में होनी चावहए।
परंतु ऄनुच्छेद 3 में ककए जाने िाले ऐसे ककसी भी पररितान का ऄकवल्पत पररणाम हो सकता है।
आसवलए, सभी वहतधारकों के साथ विचार-विमशा ककया जाना चावहए। साथ ही, देश की एकता

को बनाए रखने हेतु तथा स्थानीय अिश्यकताओं एिं अकांक्षाओं के िृहद समािेश के वलए 5िीं

और 6ठी ऄनुसूची के प्रािधानों के साथ ऄनुच्छेद 3 का वििेकपूणा तरीके से प्रयोग ककया जाना
चावहए।

4. विगत िषों में संघ लोक सेिा अयोग द्वारा पू छे गए प्रश्न


(Past Year UPSC Questions)
1. ऄनेक राज्य सरकारें बेहतर प्रशासन के वलए भौगोवलक प्रशासवनक आकाआयों जैसे जनपद
और तालुकों में विभावजत कर दी जाती हैं। ईक्त के अलोक में, क्या यह भी औवचत्यपूणा कहा

जा सकता है कक ऄवधक संख्या में छोटे राज्य, राज्य स्तर पर प्रभािी शासन देगें ? चचाा
कीवजए। (2013)

Copyright © by Vision IAS


All rights are reserved. No part of this document may be reproduced, stored in a retrieval system or transmitted
in any form or by any means, electronic, mechanical, photocopying, recording or otherwise, without prior
permission of Vision IAS.

143 www.visionias.in ©Vision IAS

Google it:- https://upscpdf.com


https://upscpdf.com << Download From >> https://upscpdf.com

नागररकता
विषय सूची
1. नागररक - ऄथथ __________________________________________________________________________ 146

2. पररचय_______________________________________________________________________________ 146

2.1. ऄवनिासी भारतीय (Non-Resident Indian: NRI) ____________________________________________ 147

2.2. भारतीय मूल के व्यवि (Person of Indian Origin: PIO) ________________________________________ 147

2.3. प्रिासी भारतीय नागररक (Overseas Citizen of India: OCI) ____________________________________ 148
2.3.1. PIO तथा OCI का विलय _____________________________________________________________ 148

3. संिध
ै ावनक प्रािधान (Constitutional Provisions)________________________________________________ 149

3.1. ऄनुच्छेद 5 _________________________________________________________________________ 149

3.2. ऄनुच्छेद 6 _________________________________________________________________________ 149

3.3. ऄनुच्छेद 7 _________________________________________________________________________ 149

3.4. ऄनुच्छेद 8 _________________________________________________________________________ 150

3.5. ऄनुच्छेद 9 _________________________________________________________________________ 150

3.6. ऄनुच्छेद 10________________________________________________________________________ 150

3.7. ऄनुच्छेद 11 ________________________________________________________________________ 150

4. नागररकता का ऄजथन और पयथिसान (समावि) (Acquisition & Termination of Citizenship) __________________ 150

4.1. जन्म से नागररकता (धारा 3) _____________________________________________________________ 150

4.2. ईद्भि/िंश के अधार पर नागररकता (धारा 4) __________________________________________________ 151

4.3. पंजीकरण द्वारा नागररकता {धारा 5 (1)} _____________________________________________________ 152

4.4. देशीयकरण द्वारा नागररकता (धारा 6) _______________________________________________________ 152

4.5. राज्यक्षेत्र के सवममवलत ककए जाने पर पर नागररकता ______________________________________________ 153

5. नागररकता का स्िैवच्छक त्यजन (Renunciation of Citizenship) ______________________________________ 153

5.1. नागररकता का पयथिसान (समावि) (Termination of Citizenship) __________________________________ 153

5.2. दूसरे देश की नागररकता का ऄजथन (Acquisition Of Citizenship Of Another Country)__________________ 154

5.3. नागररकता से िंवचत ककया जाना (Deprivation of Citizenship) ___________________________________ 154

6. दोहरी नागररकता की ऄिधारणा (Concept Of Dual Citizenship) ____________________________________ 155

7. भारत में शरणार्थथयों की वस्थवत (Status Of Refugees In India) _____________________________________ 156

7.1. वतब्बती शरणाथी _____________________________________________________________________ 156

145

Google it:- https://upscpdf.com


https://upscpdf.com << Download From >> https://upscpdf.com

7.1.1. विवधक दजाथ _______________________________________________________________________ 156

7.2. श्रीलंकाइ शरणाथी ____________________________________________________________________ 157

7.3. भूटानी शरणाथी _____________________________________________________________________ 157


7.3.1 कानूनी दजाथ ________________________________________________________________________ 158

7.4. हहदू पाककस्तानी शरणाथी _______________________________________________________________ 158

7.5. बमी शरणाथी _______________________________________________________________________ 159


7.5.1. रोहहग्या __________________________________________________________________________ 159
7.5.2. चकमा तथा हाजोंग __________________________________________________________________ 160

7.6. किलीस्तीनी शरणाथी __________________________________________________________________ 160

7.7. ऄिगान शरणाथी ____________________________________________________________________ 160

8. प्रिासी भारतीय कदिस ____________________________________________________________________ 161

8.1. भारतीय प्रिासी कदिस के अयोजन का ईद्देश्य __________________________________________________ 161

8.2. प्रिासी भारतीय कायथ मंत्रालय (Ministry of Overseas Indian Affairs: MOIA) ________________________ 161

9. ितथमान मुद्दे ____________________________________________________________________________ 162

9.1. नागररकता का विचार: एक विश्लेषण ________________________________________________________ 162

9.2. राष्ट्रीय जनसंख्या रवजस्टर (National Population Register) _____________________________________ 162

9.3. राष्ट्रीय नागररक रवजस्टर (National Register Of Citizens: NRC) _________________________________ 166

10. विगत िषों में संघ लोक सेिा अयोग द्वारा पूछे गए प्रश्न (Past Year UPSC Questions) _____________________ 169

145

Google it:- https://upscpdf.com


https://upscpdf.com << Download From >> https://upscpdf.com

1. नागररक - ऄथथ
ऑक्सफोडथ वडक्शनरी में एक नागररक को वनम्नवलवखत रूप में पररभावषत ककया गया है:
 एक राज्य या राष्ट्रमंडल में विवधक मान्यता प्राि व्यवि या वनिासी।
 ककसी कस्बे या शहर का एक वनिासी।
नागररक एक समाज या समुदाय (मूलतः एक कस्बे या शहर, लेककन ितथमान में सामान्यतः एक देश)
का ऐसा सदस्य होता है वजसे राजनीवतक भागीदारी से संबंवधत ऄवधकार प्राि हो। यह ऄवधकार
नागररकता कहलाता है।

2. पररचय
ककसी राज्य का नागररक िह व्यवि है वजसे राज्य या राजनीवतक समुदाय की पूणथ सदस्यता प्राि होती
है। ककसी भी देश में व्यवियों के दो िगथ होते हैं:
 नागररक (citizens); एिं
 विदेशी (aliens)।
विदेशी व्यवि एिं साधारण वनिासी, नागररकों से ऄलग होते हैं क्योंकक िे ईन ऄवधकारों का पूणथ
ईपयोग नहीं कर पाते जो पूणथ सदस्यता धारण करने िाले नागररकों हेतु सुलभ होते हैं।
ईदाहरण के वलए, नागररक मतदान कर सकते हैं और प्रवतवनवध कायाथलयों की पूणथ सेिाएं प्राि कर
सकते हैं (जैसे- संसद सदस्यता धारण करना, वनिाथचन प्रकिया में भाग लेना आत्याकद), जबकक विदेशी
व्यवियों को ये ऄवधकार प्राि नहीं होते हैं।
विदेशी व्यवि दो प्रकार के हो सकते हैं:
 वमत्र देश के व्यवि; तथा
 शत्रु देश के व्यवि।
हालााँकक, शत्रु देश के नागररक कु छ ऐसे ऄवधकारों से िंवचत होते हैं जो वमत्र देश के नागररकों (विदेशी)
के वलए सुलभ होते हैं।
आसके ऄवतररि, भारत के नागररकों को वनम्नवलवखत ऄवधकार प्राि हैं, जो विदेवशयों को प्राि नहीं हैं:
 कु छ मूल ऄवधकार के िल नागररकों के वलए हैं। ईदाहरणाथथ - ऄनुच्छेद 15, 16, 19, 29, तथा 30
 कु छ पदों पर के िल नागररक ही वनयुि हो सकते हैं, जैसे- राष्ट्रपवत, ईपराष्ट्रपवत, ईच्च न्यायालय
या ईच्चतम न्यायालय के न्यायाधीश, महान्यायिादी, ककसी राज्य का राज्यपाल, महावधििा
अकद।
 लोकसभा तथा राज्य विधान-सभाओं के सदस्यों के वनिाथचन हेतु चुनाि में मतदान करने तथा
सांसद/विधायक के रूप में वनिाथवचत होने का ऄवधकार के िल नागररकों को ही प्राि है।
भारतीय मूल के व्यवि (PIO) से तात्पयथ ऐसे व्यवि से है वजसे ककसी ऄन्य राष्ट्र की नागररकता प्राि है।
ईसे ऄवनिासी भारतीय (NRI) के समान नागररक नहीं माना जाता।
नागररकता से संबंवधत प्रािधान भारत के संविधान के भाग-2 में ऄनुच्छेद 5 से 11 में वनवहत हैं।
नागररकता ऄवधवनयम, 1955 भारतीय नागररकता के ऄजथन, वनधाथरण तथा पयथिसान
(termination) अकद विषयों से संबंवधत है।
 यह जन्म, िंश, पंजीकरण (रवजस्रीकरण) तथा देशीयकरण के अधार पर भारतीय नागररकता के
ऄजथन के वलए प्रािधान करता है।

146 www.visionias.in ©Vision IAS

Google it:- https://upscpdf.com


https://upscpdf.com << Download From >> https://upscpdf.com

 ऄवधवनयम की धारा 2(b) के ऄनुसार- "ककसी देश के संदभथ में प्रथम ऄनुसूची में वनर्ददष्ट नागररक
से तात्पयथ तत्समय ईस देश में नागररकता या राष्ट्रीयता से संबंवधत ककसी विवध (जो प्रितथन में है)
के ऄंतगथत नागररक होने से हैI
o यहााँ ईवललवखत प्रथम ऄनुसूची को नागररकता (संशोधन) ऄवधवनयम, 2003, द्वारा हटा
कदया गया है।
 भारतीय कानून के ऄनुसार, नागररकों के ऄवतररि व्यवियों की कु छ ऄन्य श्रेवणयााँ भी हैं, जो
वनम्नवलवखत हैं:

2.1. ऄवनिासी भारतीय (Non-Resident Indian: NRI)

एक NRI, सामान्यतः भारत से बाहर वनिास करने िाला तथा भारतीय पासपोटथ धारक व्यवि होता
है। भारतीय विवध के ऄनुसार, वनिासी िह व्यवि होता है जो देश में कु छ वनवित कदनों तक (विगत
िषथ में कम से कम 182 कदन) वनिास करता है, जबकक NRIs भारतीय नागररक तो होते हैं परंतु देश में
कु छ वनवित कदनों तक वनिास करने संबंधी शतथ को पूरा नहीं करते हैं।

2.2. भारतीय मू ल के व्यवि (Person of Indian Origin: PIO)

िह व्यवि जो स्ियं या ईसके पूिथज भारत के नागररक रहे हैं ककतु, ितथमान में िह ककसी ऄन्य देश की
नागररकता/राष्ट्रीयता धारण कर चुका है ऄथाथत् िह विदेशी पासपोटथ धारक है ऄथिा िह व्यवि जो
कभी भारतीय पासपोटथ धारण करता था; या िह या ईसके माता-वपता या वपतामह-वपतामही में से
ककसी का ऐसे क्षेत्र में जन्म हुअ है या िे ईस क्षेत्र के स्थायी वनिासी रहे हो जो भारत शासन
ऄवधवनयम, 1935 द्वारा पररभावषत हो और ककसी ऐसे क्षेत्र (बशते कक िे ककसी भी समय
ऄिगावनस्तान, भूटान, चीन, नेपाल के नागररक न रहे हो) का वनिासी रहा है जो बाद में भारत का
भाग बन गया हो; ऄथिा भारत के नागररक या भारतीय मूल के व्यवि का जीिनसाथी PIO काडथ के
वलए अिेदन करने हेतु पात्र हैं।
ध्यातव्य है कक जो व्यवि पाककस्तान, श्रीलंका, बांग्लादेश, नेपाल, भूटान ऄफगावनस्तान ि चीन के
नागररक हैं या कभी रहे हैं, िे PIO िगथ में शावमल नहीं हो सकते। भारत सरकार चाहे तो आस
वनषेधात्मक सूची में कु छ और देशों का नाम भी जोड़ सकती है। भारतीय मूल के व्यवियों को िषथ
2002 में एक काडथ जारी करने की प्रकिया शुरु की गइ थी वजसे PIO काडथ या भारतिंशी काडथ कहा
जाता है। PIO काडथ धारक वनम्नवलवखत सुविधाएं प्राि करने के हकदार हैं:
लाभ
 भारत की यात्रा करने के वलए ऄलग से िीजा की अिश्यकता नहीं।
 भारत में एकल यात्रा के दौरान 180 कदनों से ऄनवधक ऄिवध तक यात्रा के वलए पंजीकरण की
अिश्यकता नहीं।
 भारत में वनरंतर 180 कदनों की ऄिवध से ऄवधक के प्रिास की वस्थवत में ईन्हें आस ऄिवध के समाि
होने के 30 कदनों के भीतर संबंवधत विदेशी क्षेत्रीय पंजीकरण ऄवधकारी या विदेशी पंजीकरण
ऄवधकारी (FRRO/FRO) के पास पंजीकरण कराना अिश्यक होगा।
 ऄवनिासी भारतीयों को प्राि लगभग सभी अर्थथक, वित्तीय तथा शैवक्षक सुविधाएं आन्हें भी प्रदान
की गइ हैं। परंतु कु छ ऄपिाद भी हैं, यथा-
o िे कृ वष/बागान संपवत्तयों की खरीद नहीं कर सकते ; तथा,
o राजनीवतक ऄवधकारों के क्षेत्र में आन्हें NRIs के समान समानता से िंवचत ककया गया है।

147 www.visionias.in ©Vision IAS

Google it:- https://upscpdf.com


https://upscpdf.com << Download From >> https://upscpdf.com

 काडथ जारी होने पर PIO काडथ धारक व्यवि पंद्रह िषों के वलए वबना िीजा के भारत की यात्रा कर
सकता है। ईसे ऄपने प्रिास के दौरान स्थानीय पुवलस ऄवधकारी के यहााँ पंजीकरण की
अिश्यकता तब होगी जब िह पहली बार 180 कदनों से ऄवधक की ऄिवध के वलए भारत में
वनिास करता है।
 नागररकता ऄवधवनयम की धारा 5(1)(a) एिं 5(1)(c) के ऄनुसार, एक PIO काडथ धारक को
भारतीय नागररकता प्राि करने के वलए अिेदन करने से पूिथ कम से कम 7 िषथ के वलए भारत में
वनिास करना अिश्यक होगा।
2.3. प्रिासी भारतीय नागररक (Overseas Citizen of India: OCI)

 नागररकता ऄवधवनयम, 1955 की धारा 7(A) के तहत एक व्यवि OCI के रूप में पंजीकृ त हो
सकता है। OCI योजना 02-12-2005 से लागू है।
 कोइ विदेशी नागररक, जो 26 जनिरी 1950 को भारत का नागररक बनने हेतु पात्र था या 26
जनिरी 1950 या आसके पिात् भारत का नागररक था या ककसी भी समय भारत का नागररक था
या 15 ऄगस्त 1947 के पिात् भारत का वहस्सा बने क्षेत्र में वनिावसत था; िह तथा ईसके बच्चे
और पौत्र-पोती (ग्रैंडवचलरेन) OCI के रूप में पंजीकृ त होने के पात्र हैं, बशते संबंवधत देश के
नागररकता संबंधी कानूनों के तहत दोहरी नागररकता की ऄनुमवत प्राि हो।
OCI वनम्नवलवखत लाभ प्राि करने के हकदार हैं:
 भारत की यात्रा हेतु बहु-प्रिेश, बहु-ईद्देशीय अजीिन िीजा।
 ऄसीवमत ऄिवध हेतु भारत में वनिास करने हेतु स्थानीय पुवलस के पास पंजीकृ त होने से छू ट।
 अर्थथक, वितीय ि शैवक्षक क्षेत्र में ऄवनिासी भारतीयों (NRIs) के साथ समता, परंतु कु छ ऄपिाद
भी हैं, यथा-
o िे कृ वष/बागान संपवत्तयों की खरीद नहीं कर सकते; एिं
o राजनीवतक ऄवधकारों के क्षेत्र में आन्हें NRIs के समान समानता से िंवचत ककया गया है।।

2.3.1. PIO तथा OCI का विलय

िषथ 2015 में, PIO और OCI योजनाओं के विलय के वलए नागररकता ऄवधवनयम, 1955 को
संशोवधत ककया गया था।
विलय के लाभ
 एकल प्रािधान के ऄंतगथत वनयमों का सरलीकरण।
 PIO काडथधारक ऄब ईन लाभों के वलए पात्र हैं, वजनका OCI काडथधारकों द्वारा पहले से ही
ईपभोग ककया जा रहा है।
 भारत में िीजा मुि यात्रा, वनिास के ऄवधकार और देश में व्यािसावयक एिं शैवक्षक गवतविवधयों
में भागीदारी की सुविधा।
 PIOs के वलए स्थानीय पुवलस स्टेशन में पंजीकरण कराने और कठोर िीजा मानदंडों की समस्या
को हल करेगा, वजन्होंने ईन्हें भारत में संपवत्त खरीदने से िंवचत ककया हुअ था। OCI काडथ धारकों
को यह सुविधा पहले से ही प्राि है।
 यह भारतीयों से शादी करने िाले विदेवशयों के वलए ईस प्रािधान को भी समाि करेगा वजसके
ऄंतगथत भारतीय नागररकता प्राि करने से पहले ईनको देश में एक िषथ तक लगातार रहना
ऄवनिायथ ककया गया था।
 यह विदेवशयों को एक िषथ में 30 कदन की राहत देगा जब िे देश के बाहर यात्रा कर सकते हैं।

148 www.visionias.in ©Vision IAS

Google it:- https://upscpdf.com


https://upscpdf.com << Download From >> https://upscpdf.com

3. संिैधावनक प्रािधान (Constitutional Provisions)


3.1. ऄनु च्छे द 5

संविधान के प्रारंभ पर नागररकता: आस संविधान के प्रारंभ पर प्रत्येक व्यवि वजसका भारत के


राज्यक्षेत्र में ऄवधिास है और -
(क) जो भारत के राज्य-क्षेत्र में जन्मा था, या
(ख) वजसके माता या वपता में से कोइ भारत के राज्य-क्षेत्र में जन्मा था, या
(ग) जो संविधान के प्रारंभ होने के कम से कम पांच िषथ पूिथ से भारत के राज्य-क्षेत्र का वनिासी रहा

हो, भारत का नागररक होगा।

3.2. ऄनु च्छे द 6

पाककस्तान से भारत को प्रव्रजन करने िाले कु छ व्यवियों के नागररकता के ऄवधकार: ऄनुच्छेद 5 में

ककसी बात के होते हुए भी, कोइ व्यवि वजसने ऐसे राज्यक्षेत्र से जो आस समय पाककस्तान के ऄंतगथत है,

भारत के राज्यक्षेत्र में प्रव्रजन ककया है, आस संविधान के प्रारंभ पर भारत का नागररक माना जाएगा -
(क) यकद िह या ईसके माता-वपता में से कोइ ऄथिा ईसके वपतामह या वपतामही या मातामह या

मातामही में से कोइ (मूल रूप में यथा ऄवधवनयवमत) भारत शासन ऄवधवनयम, 1935 में पररभावषत
भारत में जन्मा था, और
(ख) (i) जबकक िह व्यवि ऐसा है वजसने 19 जुलाइ 1948 से पूिथ आस प्रकार प्रिजन ककया है तब यकद
िह ऄपने प्रिजन की वतवथ से भारत के राज्यक्षेत्र में मामूली तौर से वनिासी रहा है, या

(ii) यकद ईस व्यवि ने 19 जुलाइ 1948 को या ईसके पिात् आस प्रकार प्रिजन ककया है तब यकद
िह नागररकता प्रावि के वलए भारत की डोवमवनयन सरकार द्वारा विवहत प्रारूप में और रीवत से

ईसके द्वारा आस संविधान के प्रारं भ से पहले ऐसे ऄवधकारी को, वजसे ईस सरकार ने आस प्रयोजन

के वलए वनयुि ककया है, अिेदन ककए जाने पर ईस ऄवधकारी द्वारा भारत का नागररक पंजीकृ त
कर वलया गया है:
परन्तु यकद कोइ व्यवि ऄपने अिेदन की वतवथ से ठीक पूिथ कम से कम छः माह भारत के राज्यक्षेत्र में
वनिासी नहीं रहा है तो िह आस प्रकार पंजीकृ त नहीं ककया जाएगा।

3.3. ऄनु च्छे द 7

पाककस्तान को प्रव्रजन करने िाले कु छ व्यवियों के नागररकता के ऄवधकार: ऄनुच्छेद 5 और ऄनुच्छेद

6 में ककसी प्रािधान के होते हुए भी, कोइ व्यवि वजसने 1 माचथ 1947 के पिात् भारत के राज्यक्षेत्र से

ऐसे राज्यक्षेत्र को प्रव्रजन ककया है, जो आस समय पाककस्तान के ऄंतगथत है, भारत का नागररक नहीं
समझा जाएगा।

परन्तु आस ऄनुच्छेद की कोइ बात ऐसे व्यवि पर लागू नहीं होंगे जो ऐसे राज्यक्षेत्र को, जो आस समय

पाककस्तान के ऄंतगथत है, प्रिजन करने के पिात् भारत के राज्यक्षेत्र में ऐसी ऄनुज्ञा के ऄधीन लौट
अया है जो पुनिाथस के वलए या स्थायी रूप से लौटने के वलए ककसी विवध के प्रावधकार द्वारा या ईसके
ऄधीन दी गइ है। साथ ही, प्रत्येक ऐसे व्यवि के बारे में ऄनुच्छेद 6 के खंड (ख) के प्रयोजनों के वलए यह
समझा जाएगा कक ईसने भारत के राज्यक्षेत्र में 19 जुलाइ 1948 के पिात् प्रव्रजन ककया है।

149 www.visionias.in ©Vision IAS

Google it:- https://upscpdf.com


https://upscpdf.com << Download From >> https://upscpdf.com

3.4. ऄनु च्छे द 8

भारत के बाहर रहने िाले भारतीय मूल के कु छ व्यवियों के नागररकता के ऄवधकार: ऄनुच्छेद 5 में
ककसी प्रािधान के होते हुए भी, कोइ व्यवि या वजसके माता या वपता में से कोइ ऄथिा वपतामह या
वपतामही या मातामह या मातामही में से कोइ (मूल रूप से यथा ऄवधवनयवमत) भारत शासन
ऄवधवनयम, 1935 में पररभावषत भारत का जन्मा था और जो आस प्रकार पररभावषत भारत के बाहर
ककसी देश में मामूली तौर से वनिास कर रहा है, भारत का नागररक समझा जाएगा, यकद िह
नागररकता प्रावि के वलए भारत डोवमवनयन की सरकार द्वारा या भारत सरकार द्वारा विवहत प्ररूप में
और रीवत से ऄपने द्वारा ईस देश में , जहााँ िह तत्समय वनिास कर रहा है, भारत के राजनवयक या
कांसुलर प्रवतवनवध को आस संविधान के प्रारंभ से पूिथ या ईसके पिात् अिेदन ककए जाने पर ऐसे
राजनवयक या कांसुलर प्रवतवनवध द्वारा भारत का नागररक पंजीकृ त कर वलया गया हो।

3.5. ऄनु च्छे द 9


विदेशी राज्य की नागररकता स्िेच्छा से ऄर्थजत करने िाले व्यवियों का नागररक न होना: यकद ककसी
व्यवि ने ककसी विदेशी राज्य की नागररकता स्िेच्छा से ऄर्थजत कर ली है तो िह ऄनुच्छेद 5 के अधार
पर भारत का नागररक नहीं होगा ऄथिा ऄनुच्छेद 6 या ऄनुच्छेद 8 के अधार पर भारत का नागररक
नहीं समझा जाएगा।
3.6. ऄनु च्छे द 10

नागररकता के ऄवधकारों का बना रहना: प्रत्येक व्यवि, जो आस भाग के पूिथगामी ईपबंधों में से ककसी
के ऄधीन भारत का नागररक है या समझा जाता है, संसद द्वारा वनर्थमत ऐसी विवध के ईपबंधों के
ऄधीन रहते हुए, भारत का नागररक बना रहेगा।

3.7. ऄनु च्छे द 11


संसद द्वारा नागररकता के ऄवधकार का विवध द्वारा विवनयमन ककया जाना: आस भाग के पूिथगामी
ईपबंधों की कोइ बात नागररकता के ऄजथन एिं समावि तथा नागररकता से संबंवधत ऄन्य सभी विषयों
के संबंध में ईपबंध करने की संसद की शवि का ऄलपीकरण नहीं करेगी।
4. नागररकता का ऄजथ न और पयथि सान (समावि)
(Acquisition & Termination of Citizenship)
भारत में नागररकता के ऄजथन और समावि के विवधक प्रािधान नागररकता ऄवधवनयम, 1955 में
वनवहत हैं। यह भारतीय कानून वनम्नवलवखत तरीकों से नागररकता प्रावि का िणथन करता है:
 जन्म द्वारा (By Birth);
 ईद्भि द्वारा (िंश के अधार पर) (By Descent);
 पंजीकरण द्वारा (By Registration);
 देशीयकरण द्वारा (By Naturalization); तथा
 राज्यक्षेत्र के सवममवलत ककए जाने पर (By acquisition of territory)।

4.1. जन्म से नागररकता (धारा 3)

{Citizenship by Birth (Section 3)}


 भारत में 26 जनिरी 1950 को या ईसके पिात् परन्तु 1 जुलाइ 1987 से पूिथ जन्मा व्यवि जन्म
से भारत का नागररक होगा, भले ही ईसके माता-वपता की राष्ट्रीयता कु छ भी हो।
 भारत में 1 जुलाइ 1987 को या ईसके पिात् परन्तु 3 कदसमबर 2004 {नागररकता (संशोधन)
ऄवधवनयम, 2003 के लागू होने की वतवथ} से पूिथ जन्मा व्यवि के िल तभी भारत का नागररक
माना जाएगा, यकद ईसके जन्म के समय ईसके माता-वपता में से कोइ एक भारत का नागररक हो।

150 www.visionias.in ©Vision IAS

Google it:- https://upscpdf.com


https://upscpdf.com << Download From >> https://upscpdf.com

 यकद ककसी व्यवि का जन्म 3 कदसंबर 2004 को या ईसके पिात् भारत में हुअ है, तो िह ईसी
दशा में जन्म से भारत का नागररक माना जाएगा, यकद ईसके माता-वपता दोनों ईसके जन्म के
समय भारत के नागररक हों ऄथिा माता या वपता में से कोइ एक ईस समय भारत का नागररक
हो तथा दूसरा ऄिैध प्रिासी न हो।
 आस खंड के अधार पर एक व्यवि भारत का नागररक नहीं माना जाएगा, यकद ईसके जन्म के समय-
o ईसके वपता या माता को, िादों या िैध अदेवशका में ऐसी ईन्मुवि प्राि हो जैसी भारत के
राष्ट्रपवत को प्रत्यायोवजत ऄन्य संप्रभु देश के राजनवयकों को प्राि होती है और िह,
यथावस्थवत, भारत का नागररक नहीं है; या
o ईसके वपता या माता कोइ ऄन्य देशीय शत्रु हैं और ईसका जन्म ककसी ऐसे स्थान पर हुअ हो
जो ईस समय शत्रु के कब्जे के ऄंतगथत हो।

 एक “ऄिैध प्रिासी” (illegal migrant), जैसा कक ऄवधवनयम की धारा 2(1)(b) में पररभावषत
ककया गया है, एक विदेशी है, जो भारत में प्रिेश करता है:
o एक िैध पासपोटथ या ऄन्य वनधाथररत यात्रा दस्तािेजों के वबना; या
o एक िैध पासपोटथ या ऄन्य वनधाथररत यात्रा दस्तािेजों के साथ, लेककन भारत में ईस
समयािवध से ऄवतररि समय तक रहता है वजतनी ऄिवध के वलए ऄनुमवत दी गइ थी।

4.2. ईद्भि/िं श के अधार पर नागररकता (धारा 4)

{Citizenship by Descent (Section 4)}


 कोइ व्यवि वजसका जन्म 26 जनिरी 1950 को या ईसके पिात् परन्तु 10 कदसमबर 1992 से
पूिथ भारत के बाहर हुअ हो, िह िंश (ईद्भि) के अधार पर भारत का नागररक माना जाएगा,
यकद ईसके जन्म के समय ईसका वपता जन्म से भारत का नागररक हो।
o ईसका वपता के िल िंश (ईद्भि) के अधार पर भारत का नागररक हो, तो िह व्यवि भारत
का नागररक नहीं माना जाएगा, जब तक कक ईसके जन्म होने के एक िषथ के ऄंदर या आस
ऄवधवनयम के प्रारंभ में जो कोइ भी िाद का हो, या कवथत कालािवध की समावि के पिात्
कें द्र सरकार की ऄनुज्ञा से एक भारतीय िावणवज्यक दूतािास में रवजस्रीकृ त कराया जाता है।
 कोइ व्यवि वजसका जन्म 10 कदसंबर 1992 को या ईसके पिात् परन्तु 3 कदसंबर 2004 के पूिथ
भारत के बाहर हुअ हो, ईसे भारत के नागररक के रूप में माना जाएगा, यकद ईसके जन्म के समय
ईसके माता या वपता में से कोइ जन्म से, भारत का नागररक हो।
o यकद ईसका माता या वपता के िल िंश (ईद्भि) के अधार पर भारत के नागररक हों, तो िह
व्यवि भारत का नागररक नहीं माना जाएगा, जब तक कक ईसके जन्म होने के एक िषथ के
ऄंदर या आस ऄवधवनयम के प्रारंभ में जो कोइ भी िाद का हो, या कवथत कालािवध की
समावि के पिात् कें द्र सरकार की ऄनुज्ञा से एक भारतीय िावणवज्यक दूतािास में
रवजस्रीकृ त नहीं कराया जाता है।
 3 कदसंबर 2004 को या ईसके पिात् भारत से बाहर जन्मा कोइ व्यवि भारत का नागररक नहीं
होगा, जब तक बच्चे के माता-वपता अिेदन पत्र में घोवषत न करें कक बच्चे के पास ककसी ऄन्य देश
का पासपोटथ नहीं है और जन्म होने के एक िषथ के ऄंदर या आस ऄवधवनयम के प्रारंभ में जो कोइ भी
िाद का हो, या कवथत कालािवध की समावि के पिात् कें द्र सरकार की ऄनुज्ञा से एक भारतीय
िावणवज्यक दूतािास में रवजस्रीकृ त नहीं कराया जाता है।

151 www.visionias.in ©Vision IAS

Google it:- https://upscpdf.com


https://upscpdf.com << Download From >> https://upscpdf.com

4.3. पं जीकरण द्वारा नागररकता {धारा 5 (1)}

[Citizenship by Registration {Section 5(1)}]


पंजीकरण (रवजस्रीकरण) द्वारा ककसी व्यवि (ऄिैध प्रिासी न हो) को भारत की नागररकता
वनम्नवलवखत तरीकों से प्राि हो सकती है:
 भारतीय मूल का व्यवि, जो नागररकता प्रावि करने के वलए अिेदन करने से ठीक पूिथ 7 िषों से
भारत में रह रहा हो। (अिेदन करने से ठीक पूिथ 12 माह की ऄिवध तक लगातार और आस 12
माह की ऄिवध से पूिथ के 8 िषों में से 6 िषथ भारत में वनिास ककया हो।)
 भारतीय मूल का िह व्यवि जो आस धारा के तहत िर्थणत ऄविभावजत भारत के बाहर या ककसी
ऄन्य देश में ऄन्यत्र वनिास कर रहा हो।
 िह व्यवि वजसने भारतीय नागररक से वििाह ककया हो और आस धारा के तहत पंजीकरण के वलए
अिेदन करने से पूिथ सात िषों से भारत िषथ में साधारण तौर पर वनिास कर रहा हो।
 नाबावलग बच्चे, वजनके माता-वपता दोनों आस धारा के तहत भारत के नागररक हों।
 पूणथ अयु एिं सामर्थयथ का एक व्यवि वजसके माता-वपता दोनों भारत के नागररक के रूप में
पंजीकृ त हों, भारत की नागररकता प्राि कर सकता है।
 पूणथ अयु एिं सामर्थयथ का एक व्यवि जो, या वजसके माता-वपता स्ितंत्र भारत के नागररक के रूप
में पंजीकृ त थे या पंजीकरण के वलए अिेदन करने से ठीक पूिथ एक िषथ से भारत में साधारण तौर
पर वनिास कर रहा हो।
 पूणथ अयु एिं सामर्थयथ का एक व्यवि जो OCI के रूप में पांच िषथ से पंजीकृ त हो तथा पंजीकरण
हेतु आस प्रकार का अिेदन करने से एक िषथ पूिथ से भारत में वनिास कर रहा हो।

एक व्यवि को भारतीय मूल का व्यवि (PIO) माना जाएगा, यकद ईसका या ईसके माता-वपता
में से ककसी एक का जन्म ऄविभावजत भारत में या 15 ऄगस्त 1947 के पिात् भारत का ऄंग बनने
िाले ककसी भू-क्षेत्र में हुअ हो।

4.4. दे शीयकरण द्वारा नागररकता (धारा 6)

{Citizenship by Naturalization (Section 6)}


देशीयकरण द्वारा भारत की नागररकता एक विदेशी व्यवि (ऄिैध प्रिासी न हो) के द्वारा प्राि की जा
सकती है, जो सामान्यतः 12 िषों (अिेदन की वतवथ के ठीक पूिथ 12 महीने की ऄिवध से और आस 12
महीने की ऄिवध के पूिथ 14 िषों में से 11 िषों तक) से भारत का वनिासी है और ऄवधवनयम की
तीसरी ऄनुसूची में वनर्ददष्ट ऄन्य योग्यताओं को पूरा करता हो।
 जहााँ ककसी ियस्क और पूणथ सामर्थयथ िाले व्यवि, जो प्रथम ऄनुसूची में वनर्ददष्ट ककसी देश का
नागररक नहीं है, द्वारा वनधाथररत रीवत से देशीयकरण द्वारा नागररकता के वलए अिेदन ककया
जाता है, िहााँ यकद कें द्र सरकार संतुष्ट हो जाती है कक अिेदक तीसरी ऄनुसूची के प्रािधानों के
तहत देशीयकरण द्वारा नागररकता के वलए ऄर्थहत है, तो ईसे देशीयकरण द्वारा नागररकता का
प्रमाण-पत्र प्रदान ककया जा सकता है।
 हालांकक, यकद कें द्र सरकार की राय में अिेदक एक ऐसा व्यवि हो, वजसके द्वारा विज्ञान, दशथन,
कला, सावहत्य, विश्व शांवत या मानि प्रगवत के वलए प्रवतवित सेिाएं प्रदान की गइ हैं, तो सरकार
द्वारा नागररकता ऄवधवनयम, 1955 की तीसरी ऄनुसूची में वनर्ददष्ट सभी या ककसी भी शतथ को
हटाया जा सकता है।

152 www.visionias.in ©Vision IAS

Google it:- https://upscpdf.com


https://upscpdf.com << Download From >> https://upscpdf.com

 वजस व्यवि को देशीयकरण द्वारा नागररकता का प्रमाण-पत्र प्रदान ककया जाता है, ईसे
नागररकता ऄवधवनयम की दूसरी ऄनुसूची में वनर्ददष्ट िॉमथ में भारत के संविधान के प्रवत वनिा की
शपथ लेनी होगी और प्रमाण-पत्र की स्िीकृ वत की वतवथ से देशीयकरण द्वारा भारत का नागररक
माना जाएगा।

4.5. राज्यक्षे त्र के सवममवलत ककए जाने पर पर नागररकता

(Citizenship by Incorporation of Territory)

यकद कोइ भी क्षेत्र भारत का वहस्सा बन जाता है, तो कें द्र सरकार, अवधकाररक राजपत्र में ऄवधसूवचत
अदेश द्वारा ईन व्यवियों को ऄवधसूवचत कर सकती है वजन्हें ईस क्षेत्र से संबंवधत होने के कारण भारत
का नागररक माना जाएगा; और आस प्रकार अदेश में वनर्ददष्ट वतवथ से ही ईन लोगों को भारत का

नागररक माना जाएगा।

5. नागररकता का स्िै वच्छक त्यजन (Renunciation of


Citizenship)
नागररकता ऄवधवनयम, 1955 की धारा 8 में नागररकता के स्िैवच्छक त्यजन से संबंवधत प्रािधान
ऄन्तर्थनवहत हैं:
 यकद भारत का कोइ भी ियस्क और पूणथ सामर्थयथ िाला नागररक, जो ककसी ऄन्य देश का भी

नागररक या रावष्ट्रक है; और जो ऄपनी भारतीय नागररकता के त्यजन की घोषणा विवहत रीवत से

करता है, तो िह घोषणा विवहत प्रावधकारी द्वारा पंजीकृ त की जाएगी; और ऐसे पंजीकरण पर िह

व्यवि भारतीय नागररक नहीं रह जाएगा।


o परंतु यकद ऐसी कोइ घोषणा ककसी ऐसे युद्ध के दौरान की जाती है वजसमें भारत भी युद्ध में
संलग्न है, तो ऐसे पंजीकरण पर रोक लगा दी जाएगी, जब तक कक कें द्र सरकार ऄन्यथा कोइ

वनदेश न दे।
 यकद ककसी व्यवि की भारत की नागररकता समाि हो जाती है तो ईस व्यवि के प्रत्येक नाबावलग
बच्चे की नागररकता भी समाि हो जाएगी। हालांकक, ऐसा कोइ बच्चा ियस्क (18 िषथ की अयु)
होने के पिात् एक िषथ के भीतर यह घोषणा करे कक िह भारत की नागररकता पुनःग्रहण करना
चाहता है, तो ऐसी घोषणा के पिात् िह किर से भारत का नागररक बन जाएगा।

 आस खंड के प्रयोजन के वलए, कोइ मवहला जो शादीशुदा है, या रही है, ईसे पूणथ अयु (ियस्क) का
माना जाएगा।

5.1. नागररकता का पयथ ि सान (समावि) (Termination of Citizenship)

नागररकता िस्तुतः त्यजन (renunciation) या ऄन्य देश की नागररकता ग्रहण करने से समाि हो

सकती है। नागररकता के पयथिसान (समावि) संबंधी प्रािधान, नागररकता ऄवधवनयम, 1955 की धारा

9 में ऄन्तर्थनवहत हैं।

 ऄवधवनयम की धारा 9(1) में प्रािधान है कक भारत का कोइ भी नागररक जो देशीयकरण या

पंजीकरण द्वारा स्िेच्छा से ककसी ऄन्य देश की नागररकता ऄर्थजत कर ले , िह भारत का नागररक

नहीं रह जाएगा।

153 www.visionias.in ©Vision IAS

Google it:- https://upscpdf.com


https://upscpdf.com << Download From >> https://upscpdf.com

5.2. दू स रे दे श की नागररकता का ऄजथ न ( Acquisition Of Citizenship Of Another


Country)

 भारत का कोइ भी नागररक वजसने देशीयकरण, पंजीकरण द्वारा या ऄन्यथा स्िेच्छा से, या 26
जनिरी 1950 से आस ऄवधवनयम के लागू होने के बीच ककसी भी समय स्िैवच्छक रूप से ककसी
दूसरे देश की नागररकता ऄर्थजत की है, िह भारत का नागररक नहीं रहेगा।
 हालांकक, यह प्रािधान ककसी ऐसे युद्ध, वजसमें भारत भी संलग्न है, के दौरान स्िैवच्छक रूप से
ककसी दूसरे देश की नागररकता ऄर्थजत करने िाले भारतीय नागररक पर लागू नहीं होता है, जब
तक कक कें द्र सरकार ऄन्यथा कोइ वनदेश न दे।
 यकद यह प्रश्न ईठता है कक क्या, कब या कै से ककसी व्यवि द्वारा दूसरे देश की नागररकता ऄर्थजत
की गयी है, तो आसका वनधाथरण ईवललवखत वनयमों को ध्यान में रखते हुए, आसके वलए वनर्ददष्ट
प्रावधकरण द्वारा वनधाथररत रीवत से और समुवचत प्रमाणन के ईपरांत ककया जाएगा।
 ककसी दूसरे देश का पासपोटथ ग्रहण करना भी नागररकता वनयम, 1956 के तहत ककसी दूसरे देश
की राष्ट्रीयता का स्िैवच्छक ग्रहण माना जाता है।
 नागररकता वनयम, 1956 की ऄनुसूची III के वनयम 3 में वनर्ददष्ट है कक “यह तर्थय कक भारत के
ककसी नागररक द्वारा ककसी भी ऄन्य देश की सरकार की ओर से ककसी भी वतवथ को पासपोटथ प्राि
ककया गया है, ईसके पास ईस वतवथ से पूिथ ईस देश की नागररकता ग्रहण करने का वनणाथयक
प्रमाण होगा”।

5.3. नागररकता से िं वचत ककया जाना (Deprivation of Citizenship)

भारतीय नागररकता ऄवधवनयम, 1955 की धारा 10 के तहत कें द्र सरकार ककसी भी नागररक को
भारतीय नागररकता से िंवचत कर सकती है, यकद िह संतुष्ट है कक:
 पंजीकरण या देशीयकरण द्वारा नागररकता का प्रमाण-पत्र धोखाधड़ी, िजी वनरूपण या ककसी
तर्थय को वछपाकर प्राि ककया गया हो; या
 ककसी नागररक ने ऄपने कृ त्य या ििव्य द्वारा या विवध द्वारा यथा स्थावपत भारत के संविधान के
प्रवत ऄनादर या ऄसंतुवष्ट प्रदर्थशत की है; या
 यकद ककसी नागररक ने युद्ध के दौरान, वजसमें भारत संलग्न है, शत्रु देश के साथ गैर-क़ानूनी रूप से
संबंध स्थावपत ककया है या संचार ककया है या िह ककसी ऐसे कृ त में संलग्न रहा है वजसके बारे में
ईसे यह ज्ञान था कक ईससे शत्रु देश को सहायता वमल सकती है; या
 िह नागररक पंजीकरण या देशीयकरण के पिात् 5 िषथ के भीतर ककसी ऄन्य देश में 2 िषथ से
ऄवधक ऄिवध के वलए कारािास की सजा प्राि करता है; या
 िह नागररक सामान्य रूप से 7 िषथ की वनरंतर ऄिवध से भारत के बाहर वनिास कर रहा हो और
आस ऄिवध के दौरान भारत के बाहर िह कभी भी ककसी शैक्षवणक संस्थान का छात्र नहीं रहा हो
या भारत सरकार की सेिा में नहीं रहा हो या भारत की सदस्यता िाले ककसी ऄंतराथष्ट्रीय संगठन
में सेिारत नहीं रहा हो और न ही भारत की नागररकता बनाए रखने के ऄपने मंतव्य के संदभथ में
वनधाथररत तरीके से भारतीय कौन्सलेट में प्रवतिषथ विवहत रीवत से पंजीकरण कराया हो; या
o कें द्र सरकार द्वारा ककसी नागररक को नागररकता से तब तक िंवचत नहीं ककया जा सकता है
जब तक कक यह समाधान नहीं हो जाता कक ऄमुक व्यवि की भारतीय नागररकता जारी
रखना सािथजवनक वहत के प्रवतकू ल है।

154 www.visionias.in ©Vision IAS

Google it:- https://upscpdf.com


https://upscpdf.com << Download From >> https://upscpdf.com

6. दोहरी नागररकता की ऄिधारणा (Concept Of Dual


Citizenship)
सामान्यतः नागररकता को कु छ सामान्य कारकों के अधार पर पररभावषत ककया जाता है। कोइ व्यवि
ककसी देश का नागररक वनम्नवलवखत एक या एक से ऄवधक तरीकों से बन सकता है:
 "जन्म के अधार पर (Right of the Soil)" - यकद ईसका जन्म ईस देश के राज्य-क्षेत्र (सीमाओं के
भीतर) में हुअ हो।
 "िंश के अधार पर (Right of Blood)”- यकद ईसके माता-वपता में से कोइ एक या दोनों देश के
नागररक हों।
 वििाह के अधार पर (By Marriage) - यकद ईसने ऐसे व्यवि से वििाह ककया है जो कक ईस देश
का नागररक है। (यह एक स्ितः प्रकिया नहीं है ऄथाथत् व्यवि को नागररकता के वलए अिेदन करने
की अिश्यकता होती है।)
 देशीयकरण के अधार पर (Naturalization) - यकद िह देशीयकरण की विवधक प्रकिया के
माध्यम से देश की नागररकता प्राि करता है।
ककसी देश की नागररकता प्राि करने के विवभन्न तरीके हैं। ककसी भी व्यवि के वलए यह संभि है कक ईसे
एक ही समय में दो या दो से ऄवधक देशों के कानूनों के तहत एक नागररक माना जा सकता है। यह
दोहरी नागररकता है।
दोहरी नागररकता का ऄथथ है - एक ही समय में दो देशों का नागररक होना। दोहरी नागररकता िाले
व्यवि के दो पासपोटथ हो सकते हैं तथा िे ऄपने मूल (native) और देशीयकृ त राष्ट्रों में स्ितंत्र रूप से
वनिास कर सकते है, कायथ एिं यात्रा कर सकते हैं। कु छ देशों में दोहरी नागररकता की ऄनुमवत नहीं है।
पुनः दवक्षण कोररया और ऄमेररका की नागररकता एक साथ धारण नहीं की जा सकती।
कदसंबर 2003 में भारतीय संसद ने भारतीय मूल के लोगों को प्रिासी भारतीय नागररकता (OCI)
प्रदान करने के वलए एक विधेयक पाररत ककया। OCI योजना 2 कदसंबर 2005 से लागू हुइ थी।
भारतीय संविधान में एक साथ भारतीय नागररकता एिं ककसी ऄन्य देश की नागररकता धारण करने
(ऄथाथत् दोहरी नागररकता) की ऄनुमवत प्राि नहीं है।
भारतीय डायसपोरा पर ईच्च-स्तरीय सवमवत की ररपोटथ की वसिाररश के अधार पर भारत सरकार ने
प्रिासी भारतीय नागररकता (OCI) प्रदान करने का वनणथय वलया, वजसे सामान्यतया ‘दोहरी
नागररकता' के रूप में जाना जाता है।
वजस विवध के तहत भारतीय मूल के व्यवि (PIO) {जो भारत से प्रिास कर विदेश में बस गए हैं और
ककसी ऄन्य देश (पाककस्तान और बांग्लादेश को छोड़कर) की नागररकता प्राि कर ली है} कु छ लाभों के
वलए पात्र हैं, ईसी विवध के तहत भारत सरकार, प्रिासी भारतीय नागररक (OCI) का दजाथ प्रदान
करती है ऄथाथत् "यकद ईनके देश में ककसी भी ऄन्य रूप में या ईनके स्थानीय कानूनों के तहत दोहरी
नागररकता की ऄनुमवत प्राि हैं"।
यह दजाथ/पात्रता भारत के नागररक होने के समान नहीं है - भारत सरकार द्वारा जारी ककए गए वनदेशों
के ऄनुसार, ऐसे व्यवि को:
 भारतीय पासपोटथ प्राि नहीं होता है;
 कोइ मतावधकार प्राि नहीं होते हैं;
 चुनाि लड़ने या लोकसभा, राज्यसभा, विधानसभा या विधान-पररषद के वलए नामवनर्ददष्ट ककए
जाने का ऄवधकार नहीं होता है;
 राष्ट्रपवत, ईपराष्ट्रपवत, ईच्चतम न्यायालय या ईच्च न्यायालय के न्यायाधीश जैसे संिैधावनक पद
ग्रहण करने का ऄवधकार नहीं होता है।

155 www.visionias.in ©Vision IAS

Google it:- https://upscpdf.com


https://upscpdf.com << Download From >> https://upscpdf.com

7. भारत में शरणार्थथयों की वस्थवत (Status Of Refugees


In India)
 भारत, न तो िषथ 1951 के शरणाथी ऄवभसमय और न ही िषथ 1967 के आसके प्रोटोकॉल का
पक्षकार देश है। भारत में शरणार्थथयों से संबंवधत विवशष्ट कानून के ऄभाि में सरकार को विविध
प्रकार से शरणार्थथयों के होने िाले प्रिेश के संदभथ में तदथथ दृवष्टकोण ऄपनाने पर बाध्य होना पड़ा है।
 भारत में शरणार्थथयों की वस्थवत को संवहताबद्ध मॉडल अचरण सहहता के स्थान पर मुख्य रूप से
राजनीवतक और प्रशासवनक वनणथयों द्वारा वनयंवत्रत ककया जाता है।
 सरकार के दृवष्टकोण की आस तदथथ प्रकृ वत ने विवभन्न शरणाथी समूहों के संदभथ में वभन्न-वभन्न
दृवष्टकोण ऄपनाने की प्रिृवत्त को प्रेररत ककया है।
 कु छ समूहों को िैध वनिास और िैध रूप से वनयोवजत होने की योग्यता सवहत ऄन्य कइ लाभ
प्रदान ककए गए हैं जबकक ऄन्य समूहों को गैर-कानूनी घोवषत ककया गया है और ईन्हें मूलभूत
सामावजक संसाधनों का ईपयोग करने से िंवचत ककया गया है।
 भारत में शरणार्थथयों की िैधावनक वस्थवत मुख्य रूप से विदेशी विषयक ऄवधवनयम, 1946
(Foreigners Act, 1946) और नागररकता ऄवधवनयम, 1955 द्वारा विवनयवमत एिं वनयंवत्रत
होती है। ये ऄवधवनयम शरणार्थथयों में भेद नहीं करते हैं और सभी गैर-नागररकों के वलए समान
रूप से लागू होते हैं।
 आन ऄवधवनयमों के तहत वबना िैध यात्रा या वनिास दस्तािेजों के भारत में ईपवस्थवत एक दंडनीय
ऄपराध है। ये प्रािधान, शरणार्थथयों को वनिाथसन और वनरोध के वलए ईत्तरदायी बनाते हैं।
 द UNHCR चीि ऑफ वमशन आन आंवडया का कायाथलय नइ कदलली में ऄिवस्थत है। एक बार
मान्यता प्राि करने के पिात्, ऄिगानी, बमी, किवलस्तीनी और सोमाली शरणाथी UNHCR से
सुरक्षा प्राि करते हैं।
 कइ शरणाथी, सामान्य मावसक गुजारा भत्ता प्राि करते हैं और सभी शरणार्थथयों को कदलली में
UNHCR के कायाथन्ियन भागीदारों, यथा- YMCA, डॉन बोस्कोि एिं सोशल-लीगल सेंटर
(SLIC) द्वारा ईपलब्ध कराइ जाने िाली सेिाओं तक पहुाँच प्राि होती है।
 भारत की ऄवधकांश शरणाथी जनसंख्या UNHCR के ऄवधदेश के तहत शावमल नहीं है, परन्तु
किर भी सरकार द्वारा आन्हें शरणाथी माना जाता है। ज्ञातव्य है कक 1,50,000 से ऄवधक वतब्बती
और 90,000 से ऄवधक श्रीलंकाइ लोगों ने हहसा और ईत्पीड़न से व्यवथत होकर पलायन ककया
और भारत में शरण की मांग की है।
 आन समूहों को विविध प्रकार से वशक्षा, स्िास्र्थय, रोजगार और वनिास जैसी सुविधाओं तक पहुाँच
प्राि करने में सहायता एिं सुविधा प्रदान की जाती है।
7.1. वतब्बती शरणाथी

िषथ 1951 में वतब्बत पर चीन के ऄचानक अिमण के पिात्, िषथ 1959 और अगामी कइ िषों में
ऄनेक वतब्बवतयों का भारत में वनरंतर अगमन हुअ तथा ईन्होंने भारत में शरण वलया। ज्ञातव्य है कक
भारत में लगभग 1,50,000 वतब्बती शरणाथी हैं।

7.1.1. विवधक दजाथ


शरणार्थथयों की वस्थवत से संबंवधत संयुि राष्ट्र ऄवभसमय (1951) और 1967 के आसके प्रोटोकॉल का
पक्षकार देश न होने के बािजूद भारत सरकार द्वारा वतब्बवतयों को शरणाथी का दजाथ प्रदान ककया गया
है, जो 1950 के दशक के ऄंत और 1960 के दशक के अरमभ में भारत में अए थे।
 आन वतब्बती शरणार्थथयों के वलए पंजीकरण प्रमाण-पत्र जारी ककए गए हैं, वजसे िषथ में एक या दो
बार निीनीकृ त करिाना ऄवनिायथ है। भारत में जन्म लेने िाले वतब्बती भी 18 िषथ की अयु प्राि
करने के पिात् पंजीकरण प्रमाण-पत्र प्राि करने के पात्र हैं।

156 www.visionias.in ©Vision IAS

Google it:- https://upscpdf.com


https://upscpdf.com << Download From >> https://upscpdf.com

 यद्यवप, ितथमान में भी भारत सरकार द्वारा वतब्बवतयों को देश में प्रिेश करने की ऄनुमवत प्रदान
की जा रही है, तथावप सरकार ने निीन वतब्बती शरणार्थथयों को सिथप्रथम (first wave) अए
वतब्बवतयों के समान विवधक दजाथ प्रदान नहीं ककया है।
 वतब्बवतयों को भारत में ऄन्य शरणाथी समूहों की तुलना में ऄवधक ऄवधकार प्रदान ककए गए हैं:
o आन्हें वनिास परवमट (residence permits) जारी ककए गए हैं, जो ईन्हें औपचाररक
रोजगार की मांग करने का ऄवधकार प्रदान करता है।
o ये एकमात्र शरणाथी समूह हैं वजन्हें भारत सरकार द्वारा यात्रा परवमट प्रदान ककया गया है।
7.2. श्रीलं काइ शरणाथी

 भारत में श्रीलंकाइ शरणार्थथयों की विवधक वस्थवत, अवधकाररक तौर पर विदेशी विषयक
ऄवधवनयम, 1946 और नागररकता ऄवधवनयम, 1955 द्वारा वनयंवत्रत होती है।
 ईपयुथि ऄवधवनयमों के तहत बगैर िीजा के भारत में प्रिेश करने िाले ऄिैध प्रिासी, शरणाथी या
शरण चाहने िाले सभी गैर-नागररकों को पररभावषत ककया गया है।
 िैसे श्रीलंकाइ वजन्हें राष्ट्रीय सुरक्षा के वलए खतरा माना जाता है, ईन्हें ईग्रिादी (militants)
घोवषत ककया जाता है और आन्हें चेंगलापेट या िेललोर में ऄिवस्थत 'विशेष वशविरों' में बंदी बनाया
जाता है।
 सामान्यतया भारत सरकार द्वारा हहसा के कारण ऄपना देश छोड़कर अए श्रीलंकाइ लोगों को
शरणाथी के रूप में मान्यता प्रदान की गयी है और तदनुसार ईन्हें सुरक्षा प्रदान भी प्रदान की गयी है।
7.3. भू टानी शरणाथी

 20िीं सदी के अरमभ में बड़ी संख्या में नेपाली लोगों ने भूटान में प्रिास करना अरंभ कर कदया
था। 1980 के दशक तक आसकी संख्या भूटान की कु ल जनसंख्या का लगभग एक चौथाइ हो गइ
थी। 1980 के दशक के ईत्तराधथ में, भूटानी ऄवधकाररयों ने भूटान में हहदू नेपाली लोगों की बढ़ती
संख्या को भूटान की नृजातीय पहचान के समक्ष एक स्पष्ट खतरा माना। आसके पिात्, नेपावलयों
को सरकारी सेिा से सबंवधत नौकररयों, पदोन्नवत प्राि करने और पासपोटथ प्राि करने से रोकने के
वलए कइ भेदभािपूणथ ईपाय ककए गए।
 आन ईपायों के साथ-साथ, सरकार ने पारंपररक संस्कृ वत को पुनजीवित करने के वलए एक राष्ट्रीय
ऄवभयान की शुरुअत की। विद्यालयों में ऄन्य भाषा के रूप में नेपाली के वशक्षण पर प्रवतबंध लगा
कदया गया तथा विद्यालयों एिं सरकारी ऄिसरों पर भूटानी राष्ट्रीय पोशाक पहनना ऄवनिायथ
बना कदया गया। भूटान में नेपाली लोगों की संख्या वनधाथररत करने के वलए 1980 के दशक में
जनगणना की गयी।
 जनगणना के पररणामस्िरूप, नागररकता ऄवधवनयम, 1985 ऄवधवनयवमत ककया गया, वजसके
तहत भूटान की नागररकता के वलए नइ शतें लागू की गयी। आसके पररणामस्िरूप रातोंरात हहदू
नेपाली, बड़ी संख्या में ऄिैध वनिासी बन गए। आनके वलए नागररकता को पुनः प्राि करने का
एकमात्र तरीका यह था कक आनके द्वारा भूटान में विगत िषों में ऄपने वनिास स्थान को वसद्ध
करना था।
 पररणामतः, कइ देशीयकृ त नागररकों का दजाथ समाि हो गया। ऄवधवनयम के तहत ककसी भी
देशीयकृ त नागररकों की नागररकता समाि करने की ऄनुमवत भी प्रदान की गइ है, यकद ईनके
द्वारा राजा, देश या भूटान के लोगों के प्रवत 'विश्वासघात (disloyal)' ककया जाता है। ईललेखनीय
है कक हहदू नेपाली लोगों की नागररकता को समाि करने हेतु विश्वासघात शब्द का बार-बार
ईपयोग ककया गया।

157 www.visionias.in ©Vision IAS

Google it:- https://upscpdf.com


https://upscpdf.com << Download From >> https://upscpdf.com

 विवभन्न प्रकार के प्रताड़नाओं के ईपरांत ऄनेक नृजातीय नेपाली भूटान से वनिाथवसत होने पर
वििश हुए और ईनमें से कइ लोगों ने ऄसम तथा पविम बंगाल में शरण वलया। ईललेखनीय है कक
भारत सरकार आन वनिाथवसत नेपाली लोगों को शरण देने हेतु आच्छु क नहीं थी वजसके कारण ईनमें
से कइ लोगों ने पुनः नेपाल में प्रिेश ककया तथा िहााँ से भारत अए, जबकक ऄनके लोग अज भी
नेपाल के UNHCR के वशविर में रह रहे हैं।
 भारत में लगभग 30,000 नेपाली लोग वनिास कर रहे हैं। आन लोगों के वलए, शरणार्थथयों के रूप
में मान्यता प्राि करना एक ऄसंभि कायथ है।
7.3.1 कानू नी दजाथ

 िषथ 1949 से भूटानी नागररकों को वबना प्रवतबंध भारतीय सीमा में प्रिेश करने की ऄनुमवत
प्रदान की गइ है। एक संवध के माध्यम से भारत और नेपाल तथा भारत और भूटान के मध्य एक
खुली सीमा का प्रािधान ककया गया है। िषथ 1949 में भारत एिं भूटान के मध्य हस्ताक्षररत रीटी
ऑफ फ्रेंडवशप एंड को-ऑपरेशन नामक संवध को ऄंवतम बार िरिरी 2007 में ऄद्यवतत ककया
गया। भारत एिं भूटान के मध्य एक पारस्पररक व्यिस्था के ऄंतगथत भूटान के नागररकों के साथ
समान व्यिहार ककया जाता है और ईन्हें कु छ विशेषावधकार प्रदान ककए गए हैं।
 पहचान पत्रों की अिश्यकता के बगैर वनिास, ऄध्ययन और अजीविका के ऄवधकारों की गारंटी
प्रदान की गइ है। आस कारण से , भारत सरकार ने ईन नेपाली-भूटानी शरणार्थथयों को जो पलायन
करने एिं शरणाथी बनने के वलए बाध्य हैं, ईन्हें मान्यता प्रदान नहीं की है तथा न ही ककसी भी
प्रकार की सहायता प्रदान की गइ है। ज्ञातव्य है कक UNHCR द्वारा भूटावनयों की वस्थवत का
वनधाथरण नहीं ककया जाता है। संभितः आसका कारण दोनों देशों के मध्य मैत्री संवध का होना है।
7.4. हहदू पाककस्तानी शरणाथी

 िषथ 1965 के पिात् पाककस्तान से बड़ी संख्या में विस्थावपत लोग भारत में अए हैं। भारत
सरकार द्वारा आस समूह को शरणाथी समूह के रूप में मान्यता प्रदान नहीं की गइ है। आसके
पररणामस्िरूप, िे वनिास परवमट प्राि करने में ऄसमथथ हैं तथा ईनके वलए रोजगार प्राि करना
करठन होता है।
 हालांकक, भारतीय संविधान एिं नागररकता ऄवधवनयम, 1955 ईन व्यवियों को भारतीय
नागररकता के ऄजथन हेतु अिेदन करने के वलए विशेष प्रािधान करते हैं, वजनका या वजनके माता-
वपता का जन्म ऄविभावजत भारत में हुअ था।
 नागररकता (संशोधन) वनयम, 2004 विशेष रूप से पाककस्तानी ऄलपसंख्यक नागररकों को
गुजरात और राजस्थान में नागररकता हेतु अिेदन करने की व्यिस्था प्रदान करता है। आस हेतु शतथ
यह है कक ऐसे व्यवि को भारत में लगातार 5 िषथ से वनिासी होना चावहए और ईसके स्थायी रूप
से भारत में बसने की मंशा होनी चावहए, जबकक ऄन्य विदेवशयों को नागररकता के वलए अिेदन
करने के मामले में 12 िषथ की वनिास ऄवनिायथता संबंधी प्रािधान है।
 आस कानून के पररणामस्िरूप, िषथ 2005 एिं 2006 के मध्य भारत सरकार द्वारा लगभग
13,000 हहदू पाककस्तावनयों को भारतीय नागररकता प्रदान की गइ है। पाककस्तानी शरणार्थथयों
को भारतीय नागररकता प्राि होने के पिात् ईन्हें भारतीय नागररकों के समान ऄवधकार प्राि हो
जाते हैं।
 हालांकक, िषथ 2005 में नागररकता ऄवधवनयम में हुए संशोधन के माध्यम से नागररकता के वलए
अिेदन संबंधी शुलक व्यिस्था में ऄत्यवधक िृवद्ध की गयी है।

158 www.visionias.in ©Vision IAS

Google it:- https://upscpdf.com


https://upscpdf.com << Download From >> https://upscpdf.com

7.5. बमी शरणाथी

 ऄवधकांश बमी पूिोत्तर क्षेत्र से भारत में प्रिेश करते हैं, लेककन वसिथ ईन्हें UNHCR द्वारा
शरणाथी के रूप में मान्यता प्रदान की जाती है वजनके द्वारा कदलली पहुाँच कर UNHCR में
शरणाथी दजे के वलए अिेदन ककया जाता है। आस संगठन द्वारा ऄत्यवधक सुभेद्य लोगों को मावसक
भत्ता प्रदान ककया जाता है। UNHCR द्वारा मान्यता प्राि शरणार्थथयों के ऄवतररि बड़ी संख्या में
भारत में शरण चाहने िाले बमी लोग देश में वनिास कर रहे हैं। कु छ ऄन्य शरणाथी समूहों के
विपरीत, बमी शरणार्थथयों को भारत में ठहरने के वलए वनिास परवमट प्रदान ककए गए हैं।

7.5.1. रोहहग्या

 रोहहग्या िस्तुतः मयांमार के रखाआन प्रांत के वनिासी हैं। एक ऄनुमान के ऄनुसार िषथ 2016-17
के संकट से पूिथ मयांमार में लगभग 1 वमवलयन रोहहग्या वनिास करते थे। िषथ 2013 में संयुि राष्ट्र
द्वारा आन्हें विश्व में सिाथवधक ईत्पीवड़त ऄलपसंख्यकों में से एक के रूप में िर्थणत ककया गया था।
ईललेखनीय है कक आन्हें िषथ 1982 के मयांमार राष्ट्रीयता कानून के तहत नागररकता से िंवचत कर
कदया गया है। ह्यूमन राआट्स िॉच के ऄनुसार, िषथ 1982 का कानून "रोहहग्याओ को मयांमार की
राष्ट्रीयता प्राि करने की संभािनाओ से प्रभािी रूप से िंवचत करता हैं"।
 रोहहग्याओं के 8िीं शताब्दी के आवतहास संबंधी साक्ष्य प्राि होने के बािजूद, मयांमार का कानून
आन्हें "राष्ट्रीय स्िदेशी प्रजावत" के अठ मानदंडों में से एक ‘नृजातीय ऄलपसंख्यक’ के रूप में भी
मान्यता प्रदान नहीं करता है। आन्हें अिाजाही की स्ितंत्रता तथा राष्ट्रीय वशक्षा एिं वसविल सेिा
की नौकररयों से भी प्रवतबंवधत कर कदया गया है।
 मयांमार में रोहहग्या द्वारा सामना की जा रही कानूनी वस्थवतयों की तुलना व्यापक रूप से रंगभेद
(apartheid) से की गइ है और आनके विरुद्ध की जाने िाली कठोर करथिाइ को नस्लीय
पवित्रीकरण के ईदाहरण के रूप में पाठ्यपुस्तकों के ऄंतगथत शावमल ककया गया है।
 िषथ 2015 के पिात से, हजारों रोहहग्याओं का व्यापक पैमाने पर एवशयाइ देशों, यथा-
बांग्लादेश, भारत, मलेवशया, आंडोनेवशया एिं थाइलैंड में प्रिास के साथ शरणाथी संकट में ओर
ऄवधक िृवद्ध हुइ है।
 ऄगस्त 2017 में, भारत सरकार ने घोषणा की थी कक िह देश में वनिास करने िाले सभी
40,000 रोहहग्या शरणार्थथयों को वनिाथवसत करने की योजना बना रही है। ईच्चतम न्यायालय के
समक्ष प्रस्तुत एक यावचका में , यह अरोप लगाया गया है कक भारत में प्रिेश करने का प्रयास करने
िाले रोहहग्याओ को बलपूिथक मयांमार िापस वनिाथवसत ककया जा रहा है , जहााँ ईनके
मानिावधकारों का हनन हो रहा है। भारत के ईच्चतम न्यायालय के मुख्य न्यायधीश की ऄध्यक्षता
िाली एक पीठ द्वारा रोहहग्याओं के वनिाथसन (जहााँ ईनकी मृत्यु का खतरा बना हुअ है) से संबंवधत
यावचका की सुनिाइ की गइ।
 सरकार के ऄनुसार, भारत में रोहहग्याओं के ऄिैध अव्रजन के "राष्ट्रीय सुरक्षा पर गंभीर प्रभाि
एिं खतरे" हो सकते हैं। आसके ऄवतररि, यह भी अशंका व्यि की गइ कक रोहहग्या लोग ऄत्यवधक
जनसंख्या िाले हमारे देश के संसाधनों पर ऄत्यवधक संकट ईत्पन्न करेंगे। पूिोत्तर में पहले से ही
रोहहग्या अप्रिास के कारण नृजातीय जनसांवख्यकी के प्रारूप में पररितथन के पररणामस्िरूप
ऄसंतोष की वस्थवत ईत्पन्न हुइ है। हालााँकक, चकमा समुदाय को नागररकता प्रदान करने के मामले
में ईच्चतम न्यायालय ने यह पुवष्ट की कक ऄनुच्छेद 21 के ऄंतगथत ‘जीिन का ऄवधकार’ सभी

159 www.visionias.in ©Vision IAS

Google it:- https://upscpdf.com


https://upscpdf.com << Download From >> https://upscpdf.com

"व्यवियों" को प्राि है न कक के िल देश के नागररकों को। ऄत: भारत, नागररकों के समान ही


विदेवशयों के जीिन की रक्षा के वलए भी समान रूप से बाध्य है। आस प्रकार गैर -वनिाथवसत या गैर-
िापसी के वसद्धांत की व्याख्या ऄनुच्छेद 21 में वनवहत है। यह ककसी देश को ककसी शरणाथी को
ककसी भी देश में वनिाथवसत करने से प्रवतबंवधत करता है, जहां ईन शरणार्थथयों के साथ बलात्कार,
हत्या और ऄत्याचार जैसे ईत्पीड़न की संभािना है।
 ितथमान में आन यावचकाओं पर सुनिाइ जारी है तथा भारत में रोहहग्याओं का भविष्य ऄभी भी
ऄवनवित बना हुअ है।
7.5.2. चकमा तथा हाजोंग

 चकमा एिं हाजोंग, चटगांि पहाड़ी क्षेत्रों में वनिास करने िाले नृजातीय लोग थे। ईललेखनीय है
कक आस पहाड़ी क्षेत्र का ऄवधकांश भाग बांग्लादेश में वस्थत है। 1960 के दशक में कपाइ बांध
पररयोजना के कारण ईन्होंने भारत एिं मयांमार सीमा पर वस्थत ऄपनी जन्मभूवम से विस्थावपत
होना पड़ा था। ईन्हें कवथत रूप से धार्थमक ईत्पीड़न का सामना करना पड़ा और आसके
पररणामस्िरूप आन्होंने ऄसम (ितथमान में वमजोरम) के लुशाइ पहाड़ी वजले के माध्यम से भारत
में प्रिेश ककया। कें द्र सरकार ने आनमें से ऄवधकांश को नॉथथ -इस्ट फ्रंरटयर एजेंसी (ितथमान में
ऄरुणाचल प्रदेश) में स्थानांतररत कर कदया था। ईच्चतम न्यायालय के िषथ 2015 के अदेश के
ऄनुसार ितथमान में कें द्र सरकार द्वारा चकमा (बौद्ध) और हाजोंग (हहदू) शरणार्थथयों को
नागररकता प्रदान करने का वनणथय वलया गया है।
 हालााँकक, ऄरुणाचल प्रदेश के मूल वनिावसयों द्वारा आसका विरोध ककया जा रहा है। आन विरोधों के
अलोक में, कें द्र सरकार ने ईन्हें “सीवमत नागररकता” प्रदान करने का वनणथय वलया है। ईन्हें यहााँ
भूवम संबंधी ऄवधकार या ऄरुणाचल प्रदेश में ऄनुसूवचत जनजावत के रूप में मान्यता प्रदान नहीं
की जाएगी ताकक मूल वनिावसयों के ऄवधकार का हनन न हो।
7.6. किलीस्तीनी शरणाथी

 ितथमान में 160 कफवलस्तीनी शरणाथी के दजे की मांग कर रहे हैं। ये भारत में सबसे हाल में अने
िाले शरणाथी समूह हैं। संयुि राष्ट्र शरणाथी ईच्चायुि (UNHCR) के कदलली में ऄिवस्थत वमशन
द्वारा कु छ किवलस्तीवनयों को शरणाथी के रूप में मान्यता प्रदान की गइ है जबकक कु छ ऄन्य
अिेदन विचाराधीन हैं। आन शरणार्थथयों को भारत सरकार द्वारा वनिास परवमट नहीं जारी ककया
गया है।

7.7. ऄिगान शरणाथी

 भारत सरकार द्वारा अवधकाररक तौर पर ऄिगान समुदाय को शरणाथी के रूप में मान्यता प्रदान
नहीं की जाती है तथा ईन्हें UNHCR के ऄवधदेश के तहत, मान्यता और संरक्षण प्रदान ककया
जाता है।
 भारत सरकार ने ऄवधकांश ऄिगान शरणार्थथयों को िैध वनिास परवमट जारी ककया है। यह ईन्हें
एक सीमा तक विवधक संरक्षण प्रदान करता है जो ईन्हें िैध पासपोटथ के वबना भी भारत में रहने
की ऄनुमवत देता है।
 भारत में 2004-07 के मध्य निीन अगंतुकों के वलए वनिास परवमट प्राि करना ऄवधक करठन
रहा है। ऄिगान शरणार्थथयों को पहले 6 महीने तक, मुख्य अिेदक और ईन पर अवश्रत प्रत्येक
व्यवि हेत,ु न्यूनतम वनिाथह भत्ता प्रदान ककया जाता है।
आन समूहों के ऄवतररि, भारत सूडान, आराक, इरान, आररररया और आवथयोवपया के कु छ शरणार्थथयों
को भी अश्रय प्रदान करता है।

160 www.visionias.in ©Vision IAS

Google it:- https://upscpdf.com


https://upscpdf.com << Download From >> https://upscpdf.com

8. प्रिासी भारतीय कदिस


 भारत के विकास में प्रिासी भारतीय समुदाय के योगदान को वचवन्हत करने के वलए प्रवतिषथ 9
जनिरी को प्रिासी भारतीय कदिस (PBD) का अयोजन ककया जाता है। ईललेखनीय है कक 9
जनिरी 1915 के कदन ही महात्मा गााँधी दवक्षण ऄफ्रीका से भारत िापस लौटे थे और भारतीय
स्ितंत्रता अंदोलन का नेतृत्ि करते हुए भारतीयों के जीिन में अमूलचूल पररितथन ककया। आसी के
ईपलक्ष्य में प्रवतिषथ 9 जनिरी को प्रिासी भारतीय कदिस मनाया जाता है।
 प्रिासी भारतीय कदिस सममेलन का अयोजन िषथ 2003 से प्रवतिषथ ककया जा रहा है। यह
सममेलन प्रिासी भारतीय समुदायों को, ऄपने पूिथजों की मातृभूवम के लोगों और सरकार के साथ
पारस्पररक लाभ हेतु संलग्न होने के वलए एक मंच प्रदान करता है। यह सममेलन विश्व के विवभन्न
भागों में वनिास करने िाले प्रिासी भारतीय समुदाय के मध्य नेटिर्ककग और विविध क्षेत्रों में ईनके
ऄनुभिों को साझा करने में ईन्हें संलग्न करने हेतु ईपयोगी है। आस सममेलन के दौरान, विशेष
योग्यता के व्यवियों को भारत के विकास में ईनकी भूवमका की सराहना करने के वलए प्रवतवित
प्रिासी भारतीय सममान पुरस्कार से सममावनत ककया जाता है। यह सममेलन, विदेशों में बसे
भारतीयों के संबंध में महत्िपूणथ मुद्दों पर चचाथ के वलए भी एक मंच प्रदान करता है।

8.1. भारतीय प्रिासी कदिस के अयोजन का ईद्दे श्य


 प्रिासी भारतीय कदिस अयोवजत करने का प्रमुख ईद्देश्य प्रिासी भारतीय समुदाय की
ईपलवब्धयों को एक मंच प्रदान कर ईनको विश्व के समक्ष प्रस्तुत करना है।
 प्रिासी भारतीयों की भारत के प्रवत सोच, भािना की ऄवभव्यवि तथा देशिावसयों के साथ
सकारात्मक चचाथ के वलए एक मंच ईपलब्ध कराना।
 विश्व के सभी देशों में प्रिासी भारतीयों का एक नेटिकथ बनाना।
 युिा पीढ़ी को प्रिावसयों के साथ जोड़ना।
 विदेशों में वनिास कर रहे भारतीय श्रवमकों और लोगों की समस्याओं को पहचानना तथा ईनका
समाधान करने का प्रयास करना।
 भारतीय ऄवनिावसयों को अकर्थषत करना।
 वनिेश के ऄिसरों में िृवद्ध करना।
8.2. प्रिासी भारतीय कायथ मं त्रालय (Ministry of Overseas Indian Affairs:
MOIA)
 प्रिासी भारतीय कायथ मंत्रालय, विदेशों में बसे भारतीय नागररकों की सेिा के वलए समर्थपत है,
वजसकी स्थपना मइ 2004 में हुइ थी। आसे वसतंबर 2004 में प्रिासी भारतीय मामलों के मंत्रालय
के रूप में पुननाथवमत ककया गया था। सीमा रवहत िैवश्वक गठबंधन के माध्यम से विकास के लक्ष्यों
से प्रेररत MOIA का ईद्देश्य, प्रिासी भारतीय समुदाय को ऄपनी मातृभूवम के साथ जोड़ना है।
 एक 'सेिाप्रदाता' के रूप में पररकवलपत आस मंत्रालय द्वारा, प्रिासी भारतीयों {भारतीय मूल के
व्यवि (PIOs) तथा ऄवनिासी भारतीय (NRI)} से संबंवधत सभी मामलों पर सूचना, भागीदारी
एिं सुविधा प्रदान की है। प्रिासी भारतीयों से संबंवधत सभी मामलों से वनपटने के ऄवतररि, यह
मंत्रालय व्यापार एिं वनिेश, ईत्प्रिास, वशक्षा, संस्कृ वत, स्िास्र्थय एिं विज्ञान और प्रौद्योवगकी को
प्रोत्साहन करने हेतु ईनके साथ कइ पहलों में संलग्न है।
 िषथ 2016 में, आस मंत्रालय का विदेश मंत्रालय के साथ विलय कर कदया गया था क्योंकक आसकी
ऄवधकांश गवतविवधयां के िल विदेश मंत्रालय के माध्यम से ही संचावलत की जाती थीं। यह महसूस
ककया गया था कक एक पृथक मंत्रालय की अिश्यकता नहीं हैं और आसे विदेश मंत्रालय के तहत
एक विभाग द्वारा ही संचावलत ककया जा सकता है। आसवलए, ितथमान में विदेश मंत्रालय के तहत
एक विभाग (प्रिासी भारतीय विभाग) के द्वारा प्रिासी भारतीयों के मामलों का वनस्तारण ककया
जा रहा है।

161 www.visionias.in ©Vision IAS

Google it:- https://upscpdf.com


https://upscpdf.com << Download From >> https://upscpdf.com

9. ितथमान मुद्दे
9.1. नागररकता का विचार: एक विश्ले ष ण

 भारत में नागररकता की समझ के संदभथ में ईललेखनीय पररितथन अया है। लगभग 40 िषों तक,
भारत में नागररकता का एक दाशथवनक और िैचाररक अधार रहा है। स्ितंत्रता के पिात् भारत के
राज्यक्षेत्र में जन्मे प्रत्येक व्यवि को यहााँ का नागररक बनने का ऄवधकार था।
o आस ऄवधकार का अधार संबद्धता है: संविधान के संस्थापक नागररकता की एक ऐसी
ऄिधारणा को ऄपनाना चाहते थे जो भारतीय भूवम पर जन्मे प्रत्येक व्यवि (वबना ककसी
भेदभाि के ) को समायोवजत करने के वलए पयाथि रूप से विस्तृत हो। िस्तुतः, कइ गणराज्यीय
देशों के स्ितंत्र होने के पिात् िहां के राज्यक्षेत्र में जन्म लेने के अधार पर ही नागररकता
प्राि हो जाती है।
 आसके ऄवतररि, जो लोग भारतीय नागररकता के वलए दािा पेश करते हैं, िे ऐसा या तो ऄपने
माता-वपता के भारत की भूवम पर जन्म के अधार पर या िषथ 1950 में संविधान के ऄंगीकरण से
ठीक पूिथ भारत में कम से कम 5 िषों तक साधारण तौर पर वनिास के अधार पर करते हैं।
 भारत ने नागररकता के आस अदशथ ऄथथ के प्रवत ऄपनी प्रवतबद्धता से िमशः ऄलग होना अरमभ
ककया। िषथ 1955 के नागररकता ऄवधवनयम के ऄनुसार, 26 जनिरी 1950 को या ईसके पिात्
भारत में जन्मा हर व्यवि, जन्म से भारत का नागररक हो गया था।
 ककतु, नागररकता (संशोधन) ऄवधवनयम, 1986 के वनयमानुसार भारत में जन्मा व्यवि तभी
भारतीय होगा जब ईसके जन्म के समय ईसके माता-वपता में से कोइ एक भारत का नागररक रहा
हो। आस प्रकार भारतीय ईद्भि को आसमें प्राथवमकता दी गयी।
 िषथ 2003 के नागररकता संशोधन ऄवधवनयम में जन्म के अधार पर प्राि नागररकता को सशतथ
वनधाथररत ककया गया। आसे भारत में जन्मे व्यवि तक सीवमत कर कदया गया, वजसके माता-वपता
दोनों ही भारत के नागररक हों या ईसके जन्म के समय दोनों में से कोइ एक भारत का नागररक हो
तथा दूसरा व्यवि भारत में ऄिैध प्रिासी न हो।

9.2. राष्ट्रीय जनसं ख्या रवजस्टर (National Population Register )

 हाल ही में, सरकार द्वारा देश भर में नागररकों के पंजीकरण हेतु एक रवजस्टर के प्रितथन का
अधार तैयार करने के वलए वसतमबर 2020 तक एक राष्ट्रीय जनसंख्या रवजस्टर (NPR) वनर्थमत
करने का वनणथय वलया गया है।
पृिभूवम
 कारवगल युद्ध के पिात् एक मंवत्रयों का समूह (Group of Ministers: GoMs) गरठत ककया
गया था, वजसने नागररकों के एक राष्ट्रीय रवजस्टर के सृजन को सुविधाजनक बनाने तथा ऄिैध
प्रिास को वनयंवत्रत करने हेतु भारत के सभी वनिावसयों के ऄवनिायथ पंजीकरण की ऄनुशंसा की थी।
o आसके द्वारा यह ऄनुशंसा की गयी थी कक सभी भारतीय नागररकों को एक बहुईद्देशीय राष्ट्रीय
पहचान-पत्र (Multi-Purpose National Identity Card: MPNIC) प्रदान ककया जाना
चावहए तथा गैर-नागररकों हेतु विवभन्न रंगों एिं वडजाइन के पहचान-पत्र जारी ककए जाने
चावहए।
 िषथ 2011 की जनगणना के दौरान गणना हेतु िषथ 2010 में रवजस्रार जनरल ऑफ आंवडया
(RGI) ने एक राष्ट्रीय जनसंख्या रवजस्री के वलए अंकड़ों का संग्रहण ककया था।
o िषथ 2015 में आन अंकड़ों को घर-घर जाकर एक सिेक्षण के माध्यम से ऄद्यवतत ककया गया था।

162 www.visionias.in ©Vision IAS

Google it:- https://upscpdf.com


https://upscpdf.com << Download From >> https://upscpdf.com

 हालांकक, िषथ 2016 में सरकारी लाभों के ऄंतरण हेतु प्रमुख साधन के रूप में सरकार द्वारा अधार
(Aadhaar) का चयन ककया गया था, जबकक NPR की धीमी प्रगवत के कारण NPR के प्रितथन
को रोक कदया गया था।
 RGI द्वारा ऄगस्त 2019 में जारी ऄवधसूचना के माध्यम से आस योजना को ऄब पुनजीवित ककया
गया है। साथ ही, ऄवतररि अंकड़ों के साथ NPR-2015 को ऄद्यवतत करने का कायथ अरमभ
ककया गया है, वजसे िषथ 2020 तक पूणथ कर वलया जाएगा।

NPR में संग्रवहत अंकड़े


 NPR में जनसांवख्यकीय तथा बायोमेररक दोनों प्रकार के अंकड़ों का संग्रहण ककया
जाएगा।
 जनसांवख्यकीय अकड़ों की 15 विवभन्न श्रेवणयां होंगी, जो नाम, जन्मस्थान, वशक्षा ि
व्यिसाय अकद ऄनेक अधारों पर वभन्न होंगी।
 बायोमेररक अंकड़ों हेतु यह ‘अधार’ पर वनभथर होगा, वजसके वलए वनिावसयों के ‘अधार
वििरण’ का ईपयोग ककया जाएगा।
 यह जन्म एिं मृत्यु प्रमाण-पत्रों के वसविल रवजस्रेशन वसस्टम को ऄद्यवतत करने का
कायथ कर रहा है।
 यद्यवप, NPR में पंजीकरण कराना ऄवनिायथ है, तथावप PAN, अधार, राआहिग लाआसेंस
और मतदाता पहचान-पत्र जैसे ऄवतररि अंकड़ों का समािेशन स्िैवच्छक है।

राष्ट्रीय जनसंख्या रवजस्टर (NPR) के बारे में


 NPR “देश के सामान्य वनिावसयों” की एक सूची है।
o गृह मंत्रालय के ऄनुसार “देश का सामान्य वनिासी” िह व्यवि है, जो कम से कम विगत छह
माह से एक स्थानीय क्षेत्र में वनिास कर रहा है तथा अगामी छह माह हेतु एक विशेष स्थान
पर रहने का आच्छु क है।
 NPR को नागररकता ऄवधवनयम, 1955 तथा नागररकता (नागररकों का पंजीकरण और राष्ट्रीय
पहचान-पत्र जारी करना) वनयमािली, 2003 के प्रािधानों के तहत तैयार ककया जा रहा है।
o नागररकता ऄवधवनयम, 1955 को िषथ 2004 में संशोवधत करते हुए आसमें धारा 14A को
समाविष्ट ककया गया था, जो वनम्नवलवखत हेतु प्रािधान करती है:
 कें द्र सरकार ऄवनिायथत: भारत के प्रत्येक नागररक को पंजीकृ त कर सकती है तथा राष्ट्रीय
पहचान-पत्र जारी कर सकती है।
 कें द्र सरकार भारतीय नागररकों का एक राष्ट्रीय रवजस्टर (National Register of
Indian Citizens: NRIC) बना सकती है तथा आस प्रयोजनाथथ राष्ट्रीय पंजीकरण
प्रावधकरण (National Registration Authority) स्थावपत कर सकती है।
 वनिावसयों के सािथभौवमक अंकड़ों को संग्रवहत करने के पिात् नागररकता का ईवचत
सत्यापन ककया जाएगा, तत्पिात ईसमें से नागररकों के ईप-समुच्चय को वनधाथररत ककया
जाएगा। आसवलए, सभी सामान्य वनिावसयों हेतु NPR में पंजीकरण करिाना ऄवनिायथ है।
 NPR का स्थानीय, ईप-वजला, वजला, राज्य और राष्ट्रीय स्तरों पर संचालन ककया जाएगा।
 आसे गृह मंत्रालय के ऄंतगथत RGI के कायाथलय द्वारा जनगणना 2021 के प्रथम चरण के साथ
संयोजन में संचावलत ककया जाएगा।

163 www.visionias.in ©Vision IAS

Google it:- https://upscpdf.com


https://upscpdf.com << Download From >> https://upscpdf.com

o हाल ही में पूणथ हुए NRC (राष्ट्रीय नागररक रवजस्टर) को ध्यान में रखते हुए के िल ऄसम को
NPR में सवममवलत नहीं ककया जाएगा।
 NPR में पंजीकृ त 18 िषथ के अयु िगथ के सभी सामान्य वनिावसयों हेतु वनिास पहचान-पत्र जारी
ककए जाने का प्रािधान भी ककया गया है।

NPR, जनगणना और NRC से ककस प्रकार वभन्न है?


 यह कायथ प्रत्येक दस िषों में अयोवजत होने िाली जनगणना से वभन्न है तथा यह NRC से
संबद्ध नहीं है।
 हालांकक, जनगणना एक िृहद् कायथ है, ककतु आसमें व्यविगत पहचान संबंधी वििरणों को
शावमल नहीं ककया जाता है। दूसरी ओर, NPR, प्रत्येक व्यवि के पहचान संबंधी वििरण को
संग्रवहत करने हेतु ऄवभकवलपत है।
o जनगणना संबंधी अंकड़े गोपनीयता खंड द्वारा संरवक्षत होते हैं। सरकार ने यह प्रवतबद्धता
व्यि की है कक िह व्यवियों की कु ल गणना (headcount) हेतु एक व्यवि से प्राि सूचना
को प्रकट नहीं करेगी।
 NRC के विपरीत NPR, नागररकता प्रमाणन का ऄवभयान नहीं है, क्योंकक यह छह माह से
ऄवधक ऄिवध से एक स्थान पर वनिावसत एक विदेशी व्यवि का भी ररकॉडथ रखेगा।
o NPR के एक बार पूणथ ि प्रकावशत हो जाने के ईपरांत, यह भारतीय नागररकों के राष्ट्रीय
रवजस्टर (National Register of Indian Citizens: NRIC) को वनर्थमत करने का अधार
तैयार करेगा। आस प्रकार NRIC, ऄसम के NRC का ही एक ऄवखल भारतीय संस्करण
होगा।

NPR के लाभ
 वनिावसयों का डेटाबेस: यह प्रासंवगक जनांकककीय वििरणों के साथ देश के वनिावसयों का एक
व्यापक पहचान डेटाबेस वनर्थमत करने में सहायता प्रदान करेगा तथा विवभन्न मंचों पर वनिावसयों
के अंकड़ों को सुव्यिवस्थत करेगा।
 बेहतर कियान्ियन: यह सरकार को ऄपनी नीवतयों को बेहतर तरीके से सूत्रबद्ध करने में सहायता
प्रदान करेगा तथा राष्ट्रीय सुरक्षा में भी योगदान देगा।
o यह न के िल सरकारी लाभार्थथयों को बेहतर तरीके से लवक्षत करने में सहायता प्रदान करेगा,
ऄवपतु समान रीवत से कागजी कायथिाही ि लालिीताशाही में ईसी प्रकार कटौती करेगा जैसे
कक अधार काडथ योजना में की गइ है।
o गृह मंत्रालय ने यह तकथ कदया है कक अधार की तुलना में NPR सवब्सवडयों के वितरण हेतु
ऄवधक ईपयुि होगा, क्योंकक NPR में प्रत्येक व्यवि को एक पररिार से संबद्ध करने हेतु
अंकड़े ईपलब्ध हैं।
 ककसी भी प्रकार के दोषों का वनिारण: ईदाहरणाथथ- एक व्यवि की विवभन्न सरकारी दस्तािेजों में
वभन्न-वभन्न जन्म-वतवथयााँ दजथ होना सामान्य है। NPR आस प्रकार की ऄसंगवत के ईन्मूलन में
सहायता प्रदान करेगा।
 दोहराि (duplication) की समावि: NPR अंकड़ों के चलते, वनिावसयों को अवधकाररक कायों
हेतु अयु, पता और ऄन्य वििरण के विवभन्न साक्ष्यों को प्रस्तुत करने की अिश्यकता नहीं होगी।
सरकार ने दृढ़ता से कहा है कक यह मतदाता सूची में दोहराि का ईन्मूलन करेगा।

164 www.visionias.in ©Vision IAS

Google it:- https://upscpdf.com


https://upscpdf.com << Download From >> https://upscpdf.com

NPR से संबवं धत मुद्दे

 वनजता का मुद्दा: ज्ञातव्य है कक देश में अधार (Aadhaar) से संबंवधत वनजता के मुद्दे पर िाद-

वििाद ऄभी भी जारी है, तथावप NPR में भारत के वनिावसयों के व्यापक अंकड़ों को संग्रवहत

करने का प्रयास ककया जा रहा है। साथ ही, आतने व्यापक मात्रा में अंकड़ों के संरक्षण हेतु ककसी

प्रकार के तंत्र पर स्पष्टता ऄभी तक पररलवक्षत नही हुइ है।

o आससे पूिथ, विवभन्न ररपोटों में यह िर्थणत ककया गया है कक विविध ऄिसरों पर अधार डेटा के

दुरुप्रयोग के मामले प्रकट हुए हैं, परन्तु UIDAI (भारतीय विवशष्ट पहचान प्रावधकरण)

वनरंतर आस प्रकार के ईललंघनों को पूणथतः ऄस्िीकृ त करता रहा है।

 अंकड़ों के सहभाजन की िैधता: UIDAI और NPR, दोनों द्वारा एकवत्रत ककए जाने िाले सामान्य
एिं बायोमेररक अंकड़ों के संग्रहण की िैधता पर प्रश्नवचन्ह अरोवपत ककए गए हैं। ईदाहरणाथथ -
यह तकथ कदया गया है कक NPR के माध्यम से बायोमेररक सूचना का संग्रहण ऄधीनस्थ विधान

(subordinate legislation) के विषय-क्षेत्र के ऄंतगथत शावमल नहीं है।

 राष्ट्रीय सुरक्षा: सृवजत ककए जाने िाले डेटाबेस के अकार, डेटाबेस की कें द्रीकृ त प्रकृ वत, डेटाबेस में
संग्रवहत सूचना की संिेदनशील प्रकृ वत तथा ऄंतराथष्ट्रीय ऄवभकरणों की संवलिता को देखते हुए
आससे राष्ट्रीय सुरक्षा के समक्ष खतरा ईत्पन्न हो सकता है।
 NRC के समान मुद्दे: NPR िस्तुतः राष्ट्रव्यापी NRC के समान होगा। NRC के वनमाथण के दौरान
ऐसे ऄनेक दृष्टांत दृवष्टगोचर हुए थे जहां एक पररिार के कु छ सदस्यों को प्रारूप सूची में सूचीबद्ध
ककया गया था जबकक ऄन्यों को सूचीबद्ध नहीं ककया गया।
 पररयोजनाओं का दोहराि: यह ऄस्पष्ट है कक सरकार के वलए भारतीय नागररकता के संबंध में एक
ऄन्य पहचान ऄवभयान संचावलत करने की क्या अिश्यकता है, जबकक लगभग 90% नागररक
अधार योजना के ऄंतगथत सवममवलत ककए जा चुके हैं।
o अधार, NRC, NPR, जनगणना आत्याकद जैसी बहुविध पररयोजनाओं के चलते देश में
नागररकता के विचार के संदभथ में भ्ांवत ईत्पन्न हुइ है।
 गैर-सूचीबद्ध जनसंख्या (Uncounted people): जनगणना के तहत संपूणथ जनसंख्या को शावमल

नहीं ककया जाता है, वजसके कारण ईन नागररकों की वस्थवत से संबंवधत प्रश्न का समाधान नहीं हो

सका है, वजन्हें जनगणना ऄवधकारी द्वारा सूचीबद्ध नहीं ककया गया है।

o यह प्रिासी श्रवमकों की वस्थवत को भी स्पष्ट नहीं करता है, जो नागररक तो हो सकते हैं, परन्तु

“सामान्य वनिासी” के रूप में पात्र नहीं होंगे।

NPR बनाम अधार


NPR में संग्रवहत अंकड़ों को दोहराि से संरवक्षत करने तथा अधार संख्या जारी करने हेतु UIDAI

को प्रेवषत ककया जाएगा।

 स्िैवच्छक बनाम ऄवनिायथ: सभी भारतीय वनिावसयों को NPR में पंजीकरण करिाना ऄवनिायथ
है, जबकक UIDAI में पंजीकरण करिाना स्िैवच्छक है।
 संख्या बनाम रवजस्टर: UIDAI एक संख्या जारी करता है जबकक NPR नागररकों के राष्ट्रीय
रवजस्टर का सूचक है। आस प्रकार यह के िल एक रवजस्टर है।

165 www.visionias.in ©Vision IAS

Google it:- https://upscpdf.com


https://upscpdf.com << Download From >> https://upscpdf.com

 प्रमाणीकरण बनाम पहचान वनधाथरण: अधार संख्या, आस कायथ-कलाप के दौरान एक प्रमाणकत्ताथ

(authenticator) के रूप में कायथ करेगा। आसे ककसी भी मंच द्वारा स्िीकृ त ककया जा सकता है
तथा ऄवनिायथ बनाया जा सकता है। राष्ट्रीय वनिासी काडथ (National Resident Card) िस्तुत:

वनिासी की वस्थवत और नागररकता का द्योतक होगा। यह ऄस्पष्ट है कक ककन पररवस्थवतयों में आस


काडथ का प्रयोग ककए जाने की अिश्यकता होगी।
 UIDAI बनाम RGI: UIDAI विवशष्ट पहचान योजना में व्यवियों को नामांककत करने

हेतु ईत्तरदायी है तथा RGI व्यवियों को NPR में सूचीबद्ध करने हेतु ऄवधदेवशत है।
 घर-घर जाकर नामांकन करना बनाम ककसी कें द्र पर नामांकन करिाना (Door to

door canvassing vs. center enrollment): UID में पंजीकरण हेतु व्यवियों को एक

नामांकन कें द्र में जाना होता है जबकक NPR के तहत घर-घर जाकर वनिावसयों का पंजीकरण
ककया जाएगा।
 ऄवग्रम दस्तािेजीकरण बनाम जनगणना सामग्री: UID दस्तािेजीकरण और पहचान-वनधाथरण के
ऄवग्रम रूपों पर अधाररत है जबकक NPR जनगणना द्वारा प्रदत्त सूचना पर अधाररत होगा।

वनष्कषथ
NPR में संग्रवहत ककए जाने िाले अंकड़ों से संबद्ध वनजता संबध
ं ी सरोकारों को स्पष्ट ककया जाना

ऄत्यािश्यक है तथा ऄसम में समपाकदत ऐसे समान कायों (ईदाहरणाथथ- NRC) से ईत्पन्न समस्याओं को
भी ध्यान में रखना अिश्यक है। यह नागररकता के सत्यापन हेतु एक मूलभूत डेटाबेस के रूप में कायथ
करने में तभी सक्षम होगा जब एक राष्ट्रव्यापी NRC को पिातिती चरणों में संपन्न ककया जाएगा।

9.3. राष्ट्रीय नागररक रवजस्टर (National Register Of Citizens: NRC)

NRC क्या है?


 राष्ट्रीय नागररक रवजस्टर ऄसम के सभी िैध (प्रमावणत) नागररकों की एक सूची है तथा यह
रवजस्टर के िल ऄसम राज्य से ही संबंवधत है।
 यह नागररकता ऄवधवनयम, 1955 ि नागररकता (नागररकों का पंजीकरण और राष्ट्रीय पहचान-

पत्र जारी करना) वनयमािली, 2003 (िषथ 2009 में संशोवधत) तथा भारत के राजपत्र में िषथ
2010 में प्रकावशत गृह मंत्रालय के एक अदेश द्वारा शावसत होता है।
 आसमें, 24 माचथ 1971 की मध्यरावत्र से पूिथ ककसी वनिाथचक नामािाली या राष्ट्रीय नागररक
रवजस्टर, 1951 में शावमल व्यवियों और ईनके िंशजों को सवममवलत ककया जाएगा।

 NRC बनाम जनगणना: जनगणना कायथ राष्ट्रीय स्तर पर प्रत्येक दशक में संपाकदत ककया जाता है

तथा यह भारत की जनसंख्या के अकार और प्रकृ वत का वििरण प्रदान करता है। परन्तु NRC
नागररकों की नागररकता को प्रमावणत करने के दावयत्ि हेतु एक विवशष्ट ऄभ्यास है। आसके तहत
नागररकों के वलए एक दस्तािेजी साक्ष्य के माध्यम से यह दशाथना ऄवनिायथ है कक िे कै से ऄसम में
रहने िाले भारत के नागररक बन गए हैं।

हाल ही में, ऄसम के प्रामावणक भारतीय नागररकों की पुवष्ट करने िाले राष्ट्रीय नागररक रवजस्टर
(National Register of Citizens: NRC) के ऄपडेटेड (ऄद्यवतत) और ऄंवतम प्रारूप को प्रकावशत ककया
गया। ज्ञातव्य है कक 19 लाख से ऄवधक अिेदक आस सूची में ऄपना नाम दजथ करिाने में वििल हुए हैं।

166 www.visionias.in ©Vision IAS

Google it:- https://upscpdf.com


https://upscpdf.com << Download From >> https://upscpdf.com

पृिभूवम

 NRC का मूल ऄसम स्टेट स्टूडेंट्स यूवनयन और भारत सरकार के मध्य िषथ 1985 में हस्ताक्षररत
समझौता ज्ञापन (Memorandum of Settlement) ऄथिा ऄसम समझौते (Assam Accord)

में वनवहत है। यह समझौता 1980 के दशक के प्रिासी विरोधी हहसक अंदोलन का पररणाम था

तथा आसमें ऄिैध प्रिास (illegal migration) को वनयंवत्रत करने हेतु विवभन्न खंडों का समािेश
ककया गया था।

 ईललेखनीय है कक ऄसम समझौते के पिात् 1 जनिरी 1966 से पूिथ बांग्लादेश से अए सभी


भारतीय मूल के लोगों को भारतीय नागररक के रूप में मान्यता प्रदान करने हेतु नागररकता
ऄवधवनयम, 1955 में संशोधन ककया गया था।

o िे व्यवि जो 1 जनिरी 1966 और 25 माचथ 1971 के मध्य भारत अए थे, ईन्हें पंजीकरण
कराने और 10 िषों तक राज्य में वनिास करने के पिात् नागररकता हेतु पात्र समझा गया

था, जबकक िे वजन्होंने 25 माचथ 1971 के पिात् राज्य में प्रिेश ककया था ईन्हें वनिाथवसत

ककया जाना था। हालांकक, विगत दशकों आस कदशा में कोइ ठोस कायथिाही नहीं की गइ है।
 िषथ 2014 में ईच्चतम न्यायालय ने राज्य सरकार को एक समयबद्ध रीवत में NRC, 1951 को
ऄद्यवतत करने का वनदेश जारी ककया था। ितथमान प्रकिया को ईच्चतम न्यायालय के वनरीक्षण में
संचावलत ककया जा रहा है।

ऄसम समझौता (The Assam Accord)


 यह भारत सरकार के प्रवतवनवधयों और ऄसम अन्दोलन के नेताओं के मध्य 15 ऄगस्त 1985 को

नइ कदलली में हस्ताक्षररत एक समझौता ज्ञापन (Memorandum of Settlement) था।

 आसमें यह प्रािधान शावमल था कक ईन सभी “विदेवशयों” की पहचान की जाएगी, वजन्होंने 25


माचथ 1971 के पिात् ऄसम में प्रिेश ककया है तथा तत्पिात ईन्हें ऄिैध प्रिासी (ऄवधकरण द्वारा
वनधाथरण) ऄवधवनयम, 1983 {Illegal Migrants (Determination by Tribunals) Act,

1983 (IMDT)} के तहत पता लगाकर वनिाथवसत ककया जाएगा। आसमें वनिाथचक नामािली से
विदेवशयों के नामों को हटाने का भी ईपबंध ककया गया है।

वनर्ददष्ट वतवथ (Cut of Date) का महत्ि

 बांग्लादेश से भारत में प्रिेश करने िाले प्रिावसयों की वस्थवत को वनधाथररत करने के संदभथ में कट
ऑफ डेट (वनर्ददष्ट वतवथ) का मुद्दा वििाद का एक प्रमुख विषय था। ईललेखनीय है कक 25 माचथ

1971 को ही पाककस्तान की सैन्य सरकार ने पूिी पाककस्तान के स्ितंत्रता सेनावनयों और


नागररकों पर कठोर कायथिाही की शुरुअत की थी।
 कठोर दमन-चि और िू रता के कारण 10 वमवलयन से ऄवधक लोगों को भारत में शरण लेना पड़ा

तथा अरंभ में ये शरणाथी मुख्यत: ऄसम, पविम बंगाल और वत्रपुरा में बस गए।

NRC का महत्ि
 दीघथकावलक समाधान: यह बांग्लादेश से ऄिैध प्रिास को वनयंवत्रत करने हेतु ईपायों की व्यिस्था
करता है, क्योंकक कू टनीवतक और सीमा प्रबंधन प्रयास आस मुद्दे का पूणथत: समाधान करने में वििल

167 www.visionias.in ©Vision IAS

Google it:- https://upscpdf.com


https://upscpdf.com << Download From >> https://upscpdf.com

रहे हैं। आसके दो प्रमुख कारण वनम्नवलवखत हैं: i) बांग्लादेश ऄपने क्षेत्र से भारत में ककसी भी प्रकार

की घुसपैठ को स्िीकार नहीं करता है तथा ii) भारत और बांग्लादेश के मध्य वस्थत वछकद्रल सीमा,

प्रभािी सीमा प्रबंधन में बाधा ईत्पन्न करती है। आस प्रकार, ऄिैध प्रिास के संकट से वनपटने हेतु

NRC को एक िैकवलपक साधन और एक दूरदशी प्रशासवनक समाधान स्िीकार ककया गया है।

 पहचान वनधाथरण: NRC ऄिैध प्रिावसयों द्वारा राज्य की जनसांवख्यकी को पररिर्थतत करने तथा

राज्य की राजनीवत को प्रभावित ककए जाने की अशंका का भी वनिारण करेगा।

 एक वनिारक साधन: यह ऄपेक्षा की गइ है कक NRC भविष्य में बांग्लादेश से ऄिैध प्रिावसयों को

ऄसम में प्रिेश करने से वनरुद्ध करेगा, क्योंकक आसके प्रारूप (राफ्ट) के प्रकाशन ने आस संभािना का
सृजन ककया है कक िैध दस्तािेजों के वबना ऄसम में वनिास करना वनरोध/कारािास और वनिाथसन
का कारण बनेगा।
NRC से संबद्ध मुद्दे

 वनष्कासन का भय: आस संबंध में हचताएं विद्यमान हैं कक NRC से सूची में लोगों का ऄनुवचत

रीवत से समािेशन ऄथिा वनष्कासन हो सकता है।

o D-मतदाताओं से संबवं धत मुद्दे भी मौजूद हैं। D-मतदाता िे मतदाता होते हैं वजन्हें सरकार
द्वारा ईनके ईवचत नागररकता प्रमाण-पत्रों के कवथत ऄभाि के अधार पर मतावधकार से
िंवचत कर कदया जाता है तथा ईनका समािेशन विदेशी विषयक ऄवधकरण (Foreigners

Tribunal) के वनणथय पर वनभथर करे गा।


 ऄलपसंख्यकों की हचताएं: यह अशंका प्रकट की गइ है कक आस प्रकार की कायथिावहयां देश के
ऄलपसंख्यकों को लवक्षत कर सकती हैं।

o नागररकता (संशोधन) विधेयक, 2016 ऄिगावनस्तान, बांग्लादेश और पाककस्तान से अए


हहदू और कु छ ऄन्य ऄलपसंख्यक समुदायों से संबद्ध ऄिैध प्रिावसयों को भारतीय नागररकता
हेतु पात्रता प्रदान करता है, जबकक यह कदम आस प्रकिया से ऄलपसंख्यकों को हटाने

(alienation) के संबंध में अशंकाओं का सृजन कर सकता है।

 लोगों के भाग्य को ऄवभवनवश्चत करने हेतु ककसी विवशष्ट नीवत का न होना: सरकार ने NRC

प्रकिया के पूणथ होने के पिात् (पोस्ट NRC) कोइ कियान्ियन योजना वनर्थमत नहीं की है, क्योंकक-

o ऄिैध प्रिावसयों का बांग्लादेश में वनिाथसन की संभािना ऄत्यलप है, क्योंकक सूची से बवहष्कृ त
लोगों को बांग्लादेश के प्रामावणक नागररक के रूप में वसद्ध करना होगा तथा आस हेतु
बांग्लादेश से सहयोग की भी अिश्यकता होगी।

o वजनकी पहचान “ऄिैध प्रिावसयों” के रूप में की जाएगी, ईन्हें ईनके वनिाथसन तक वनरोध

वशविरों (detention camps) में रखा जाएगा। मीवडया ररपोट्सथ में यह भी ईललेख ककया
गया है कक ये वनरोध वशविर ऄपनी ऄमानिीय वनिास पररवस्थवतयों के वलए प्रवसद्ध हैं।
 सुरक्षा संबध
ं ी हचताएं: ज्ञातव्य है कक पहले से ही सुरक्षा संबंधी हचताएं प्रकट की जा रही हैं।
 राज्यविहीनता का मुद्दा: ऐसी अशंकाएं विद्यमान हैं कक भारत राज्यविहीन लोगों के निीनतम
समूह का सृजन करेगा, वजससे एक स्िदेशी संकट का भय ईत्पन्न होगा तथा यह मयांमार से
बांग्लादेश पलायन कर गए रोहहग्या लोगों की कहानी दोहराएगा।

168 www.visionias.in ©Vision IAS

Google it:- https://upscpdf.com


https://upscpdf.com << Download From >> https://upscpdf.com

NRC सूची में नाम दजथ करिाने में वििल लोगों हेतु प्रािधान

 ऄसम सरकार ने NRC सूची में नाम दजथ करिाने से चूक गए लोगों को यह अश्वासन कदया

है कक ईन्हें तत्काल प्रभाि से “विदेशी” ऄथिा “ऄिैध प्रिासी” घोवषत नहीं ककया जाएगा।

 ऐसे लोगों को विदेशी विषयक ऄवधकरण के समक्ष ऄपने पक्ष को रखने (ऄथाथत् विरोध दजथ कराने)

की ऄनुमवत प्रदान की जाएगी। आसके ऄवतररि, िे आस मामले में ईच्च न्यायालय और यहााँ तक कक
ईच्चतम न्यायालय में भी ऄपील दायर कर सकते हैं।
 राज्य सरकार NRC सूची से िंवचत वनधथन लोगों को विवधक सहायता भी प्रदान करेगी।

वनष्कषथ
ईपयुथि सभी अशंकाओं के मध्य भारतीय नागररकों के दस्तािेजीकरण और घुसपैठ की वनगरानी एिं
रोकथाम के संदभथ में NRC एक दूरदशी ईपाय है। ितथमान NRC प्रकिया न्यावयक ऄवभप्रेरण और

राजनीवतक शवि संतुलन दोनों का ही एक पररणाम है, क्योंकक NRC का संचालन ईच्चतम न्यायालय
के तत्िािधान में ककया जा रहा है तथा यह समपूणथ प्रकिया की सूक्ष्मता से वनगरानी कर रहा है। ज्ञातव्य
है कक राज्यों का सहयोग NRC की सिलता हेतु महत्िपूणथ वसद्ध होगा।

10. विगत िषों में संघ लोक सेिा अयोग द्वारा पू छे गए प्रश्न
(Past Year UPSC Questions)
1. सरकार की दो समानांतर चलाइ जा रही योजनाओं, यथा ‘अधार काडथ’ और ‘राष्ट्रीय जनसंख्या

रवजस्टर’, एक स्िैवच्छक और दूसरी ऄवनिायथ, ने राष्ट्रीय स्तर पर िाद-वििाद और मुकदमों को ईत्पन्न


ककया है। सकारात्मक एिं नकारात्मक पक्षों के अधार पर चचाथ कीवजए कक क्या दोनों योजनाओं को
साथ-साथ चलाना अिश्यक है या नहीं। आन योजनाओं का विकासात्मक लाभों और न्यायोवचत संिृवद्ध
को प्राि करने की संभाव्यता का विश्लेषण कीवजए। (2014)

Copyright © by Vision IAS


All rights are reserved. No part of this document may be reproduced, stored in a retrieval system or transmitted
in any form or by any means, electronic, mechanical, photocopying, recording or otherwise, without prior
permission of Vision IAS.

169 www.visionias.in ©Vision IAS

Google it:- https://upscpdf.com


https://upscpdf.com << Download From >> https://upscpdf.com

मूल ऄधधकार
धिषय सूची
1. ऄधधकारों की संकल्पना ____________________________________________________________________ 173

2. भारतीय संधिधान में मूल ऄधधकारों की संकल्पना___________________________________________________ 174

3. मूल ऄधधकारों का क्रधमक धिकास _____________________________________________________________ 174

3.1. मूल ऄधधकारों की सीमाएँ _______________________________________________________________ 175

4. मूल ऄधधकारों की धिशेषताएं ________________________________________________________________ 175

5. मूल ऄधधकारों का धििरण __________________________________________________________________ 176

5.1. ऄनुच्छेद 12 : राज्य की पररभाषा (DEFINITION OF STATE) _______________________________________ 176


5.1.1. क्या राज्य की पररभाषा के तहत धनजी धनकाय भी शाधमल हैं?_____________________________________ 177

5.2. ऄनुच्छेद 13 : मूल ऄधधकारों से ऄसंगत या ईनका ऄल्पीकरण करने िाली धिधधयां __________________________ 177
5.2.1. न्याधयक पुनर्विलोकन (Judicial Review) __________________________________________________ 178
5.2.1.1. संधिधान के पूिव की धिधधयों की धिधधमान्यता _____________________________________________ 178
5.2.1.2. पृथक्करणीयता का धसद्ांत (Doctrine of Severability) ____________________________________ 179
5.2.1.3. ग्रहण का धसद्ांत (Doctrine of Eclipse) ______________________________________________ 179
5.2.1.4. ऄधधत्यजन का धसद्ांत (Doctrine of Waiver) __________________________________________ 179
5.2.1.5. भधिष्यलक्षी धिधनणवय का धसद्ांत (Doctrine of Prospective Overruling) _____________________ 179
5.2.1.6. मोटर जनरल ट्रेड्स बनाम स्टेट ऑफ अन्र प्रदेश (1984)_____________________________________ 179
5.2.2. मूल ऄधधकारों की संशोधनीयता और मूल ढांचे का धसद्ांत ________________________________________ 180

5.3. ऄनुच्छेद 14 - धिधध के समक्ष समता_________________________________________________________ 181

5.4. ऄनुच्छेद 15 - भेदभाि के धिरुद् ऄधधकार (RIGHT AGAINST DISCRIMINATION) _________________________ 182
5.4.1. संबंधधत न्याधयक िाद _________________________________________________________________ 183
5.4.2. ऄनुच्छेद 15 एिं संिैधाधनक संशोधन_______________________________________________________ 184
5.4.3. ऄनुच्छेद 15 और सामाधजक प्रगधत ________________________________________________________ 184

5.5. ऄनुच्छेद 16 - लोक धनयोजन के धिषय में ऄिसर की समता का ऄधधकार _________________________________ 185
5.5.1. मंडल अयोग और ईसके पश्चात ___________________________________________________________ 186

5.6. ऄनुच्छेद 17 - ऄस्पृश्यता का ऄंत __________________________________________________________ 187


5.6.1. ऄस्पृश्यता की समाधि के धलए धिधभन्न ऄधधधनयम ______________________________________________ 188

5.7. ऄनुच्छेद 18 - ईपाधधयों का ऄंत ___________________________________________________________ 189

5.8. ऄनुच्छेद 19 - स्ितंत्रता का ऄधधकार ________________________________________________________ 189


5.8.1. सूचना का ऄधधकार - सूचना का ऄधधकार ऄधधधनयम, 2005 के तहत एक कानूनी ऄधधकार ________________ 191
5.8.2. प्रेस की स्ितंत्रता की धस्थधत _____________________________________________________________ 192
5.8.3. ऄनुच्छेद 19 के तहत ऄधधकारों से संबंधधत मुद्दे _______________________________________________ 194
5.8.3.1. िाक् स्िातंत्र्य और धसधिल सेिक: 19 (1) (a) ____________________________________________ 194

170

Google it:- https://upscpdf.com


https://upscpdf.com << Download From >> https://upscpdf.com

5.8.3.2. 19 (1) (a) और 19 (2) - हेट स्पीच __________________________________________________ 194


5.8.3.3. 19 (1) (a) और 19 (2) - मानहाधन ___________________________________________________ 194
5.8.3.4. फफल्मों और पुस्तकों पर प्रधतबंध: 19 (1) (a) ____________________________________________ 195
5.8.3.5. 19 (1) (a) आंटरनेट तक पहंच का ऄधधकार ______________________________________________ 195
5.8.3.6. 19 (1) (c) - सहकारी सधमधतयों के गठन का ऄधधकार ______________________________________ 195
5.8.3.7. राष्ट्रीय अपातकाल का ऄनुच्छेद 19 के तहत प्रद्त ऄधधकारों पर प्रभाि ___________________________ 195

5.9. ऄनुच्छेद 20- ऄपराधों के धलए दोषधसधद् के संबध


ं में संरक्षण ________________________________________ 196

5.10. ऄनुच्छेद 21- प्राण और दैधहक स्ितंत्रता का ऄधधकार ____________________________________________ 196


5.10.1. न्याधयक व्याख्या द्वारा ऄनुच्छेद 21 के क्षेत्र का धिस्तार__________________________________________ 197
5.10.2. धनजता का ऄधधकार (Right to Privacy) _________________________________________________ 198
5.10.3. जीिन का ऄधधकार और अत्महत्या (IPC की धारा 309) _______________________________________ 200
5.10.4. जीिन का ऄधधकार एिं मृत्युदड
ं _________________________________________________________ 201

5.11. ऄनुच्छेद 21-A : धशक्षा का ऄधधकार _______________________________________________________ 202

5.12. ऄनुच्छेद 22: कु छ दशाओं में धगरफ्तारी और धनरोध से संरक्षण ______________________________________ 203

5.13 ऄनुच्छेद 23 – मानि के दुव्यावपार एिं बलात् श्रम का प्रधतषेध _______________________________________ 205

5.14. ऄनुच्छेद 24 – कारखानों अफद में बालकों के धनयोजन का प्रधतषेध ____________________________________ 206
5.14.1. बाल श्रम से संबंधधत धिधेयक ___________________________________________________________ 206

5.15. ऄनुच्छेद 25: ऄंतःकरण की और धमव को ऄबाध रूप से मानने, अचरण और प्रचार करने की स्ितंत्रता _____________ 207

5.16. ऄनुच्छेद 26: धार्वमक कायों के प्रबंध की स्ितंत्रता _______________________________________________ 209

5.17. ऄनुच्छेद 27: फकसी धिधशष्ट धमव की ऄधभिृधद् के धलए करों के संदाय के बारे में स्ितंत्रता ______________________ 210

5.18. ऄनुच्छेद 28: कु छ धशक्षा संस्थाओं में धार्वमक धशक्षा या धार्वमक ईपासना में ईपधस्थत होने के बारे में स्ितंत्रता ________ 211

5.19. ऄनुच्छेद 29: ऄल्पसंख्यक-िगों के धहतों का संरक्षण ______________________________________________ 211

5.20. ऄनुच्छेद 30: धशक्षा संस्थाओं की स्थापना और प्रशासन करने का ऄल्पसंख्यक िगों का ऄधधकार ________________ 212
5.20.1. ऄनुच्छेद 29 तथा 30 के मध्य संबंध_______________________________________________________ 212

5.21. ऄनुच्छेद 31: संपध्त का ऄधनिायव ऄजवन (धनरधसत) ______________________________________________ 213

5.22. ऄनुच्छेद 32 : संिध


ै ाधनक ईपचारों का ऄधधकार ________________________________________________ 215

5.23. ऄनुच्छेद 33 - मूल ऄधधकारों के , सुरक्षा बलों अफद पर लागू होने में, ईपांतरण करने की संसद की शधि ____________ 218

5.24. ऄनुच्छेद 34 - जब फकसी क्षेत्र में सेना धिधध प्रिृ्त है तब आस भाग द्वारा प्रद्त ऄधधकारों पर धनबंधन ______________ 219

5.25. ऄनुच्छेद 35: भाग 3 के ईपबंधों को प्रभािी करने के धलए धिधान _____________________________________ 220

6. संपध्त के ऄधधकार की प्रधस्थधत _______________________________________________________________ 221

7. क्या मूल ऄधधकार अत्यंधतक हैं? ______________________________________________________________ 221

8. मूल ऄधधकारों पर अपातकाल का प्रभाि ________________________________________________________ 221

9. मूल ऄधधकारों की अलोचना ________________________________________________________________ 222

171

Google it:- https://upscpdf.com


https://upscpdf.com << Download From >> https://upscpdf.com

10. मूल ऄधधकारों का महत्ि __________________________________________________________________ 222

11. धिगत िषों में Vision IAS GS मेंस टेस्ट सीरीज में पूछे गए प्रश्न (Previous Year Vision IAS GS Mains Test
Series Questions) _______________________________________________________________________ 222

12. धिगत िषों में संघ लोक सेिा अयोग (UPSC) द्वारा पूछे गए प्रश्न (Past Year UPSC Questions)______________ 231

172

Google it:- https://upscpdf.com


https://upscpdf.com << Download From >> https://upscpdf.com

1. ऄधधकारों की संक ल्पना


 ऄधधकार िस्तुतः फकसी व्यधि को संधिधान ऄथिा धिधध द्वारा प्रद्त दािे , सुधिधाएं या

धिशेषाधधकार होते हैं। धिधध द्वारा प्रद्त सुधिधाएँ ऄधधकारों की रक्षा करती हैं। दोनों का ऄधस्तत्ि
परस्पर धनभवर है। जहां धिधध एक ओर ऄधधकारों को मान्यता प्रदान करती है, िहीं दूसरी ओर

आन्हें लागू करने ऄथिा आनकी ऄिहेलना पर धनयंत्रण स्थाधपत करने की व्यिस्था भी करती है।

 ऄधधकार, लोकतंत्र का सार है। ये व्यधि को स्ियं का धिकास करने के धलए सक्षम और सशि

बनाते हैं। ऄधधकारों की धारणा समय-समय पर तथा एक समाज से दूसरे समाज में पररिर्वतत
होती रहती है।
 जब कु छ दािों को संधिधान ऄथिा धिधध द्वारा मान्यता प्रदान की जाती है तो िे प्रितवनीय हो
जाते हैं। आस धस्थधत में नागररक (कु छ मामलों में गैर-नागररक) आन्हें लागू करने की मांग कर सकते
हैं। आनके ईल्लंघन की धस्थधत में नागररक ऄपने ऄधधकारों के संरक्षण हेतु न्यायालय में ऄपील कर
सकते हैं। आस प्रकार, ऄधधकारों को समाज द्वारा मान्यता प्राि और राज्य द्वारा स्िीकृ त, तकव संगत

दािों के रूप में पररभाधषत फकया जा सकता है। ऄधधकारों के धिधभन्न प्रकार धनम्नधलधखत हैं:

o प्राकृ धतक ऄधधकार (Natural Rights): ये ऄधधकार प्रत्येक व्यधि को ईनके मनुष्य होने के

कारण जन्म से ईपलब्ध होते हैं। ये ऄहस्तांतरणीय नैसर्वगक ऄधधकार होते हैं। ये धिधध द्वारा
प्रद्त नहीं हैं बधल्क धिधध द्वारा के िल प्रितवनीय होते हैं, जैसे- जीिन का ऄधधकार।

o मानिाधधकार (Human Rights): आन ऄधधकारों को प्राकृ धतक ऄधधकारों का व्यािहाररक


संस्करण माना जाता है। ये मानि के रूप में जन्म के साथ ही सभी मनुष्यों के धलए ईपलब्ध
होते हैं। आस संदभव में, ये राष्ट्रीयता, जाधत, धमव, ललग अफद के अधार पर धिभेद फकए धबना

सािवभौम प्रकृ धत के होते हैं। आन ऄधधकारों को सुधनधश्चत करने के धलए संयुि राष्ट्र संघ द्वारा
िषव 1948 में मानिाधधकारों की सािवभौम घोषणा की गइ।

o नागररक ऄधधकार (Civil Rights): ये ऄधधकार फकसी देश की धिधध ऄथिा संधिधान द्वारा

के िल नागररकों को प्रदान फकए जाते हैं। ईदाहरण के धलए, स्ितंत्रता का ऄधधकार।

o धिधधक ऄधधकार (Legal rights): ये िे नागररक ऄधधकार होते हैं जो धिधाधयका द्वारा

पाररत धिधधयों के माध्यम से प्रदान फकए जाते हैं। ईदाहरण के धलए, संपध्त का ऄधधकार।

(पूिव में यह भारतीय संधिधान में मूल ऄधधकार के रूप में शाधमल था परन्तु, ितवमान में आसे

ऄनुच्छेद 300(A) के ऄंतगवत धिधधक ऄधधकार के रूप में मान्यता प्राि हैं।)

o संिध
ै ाधनक ऄधधकार (Constitutional Rights): ये ऄधधकार संधिधान में ईधल्लधखत होते

हैं। कु छ ऄधधकारों को धिशेष दजाव प्रदान फकया जाता है, जैसे- मूल ऄधधकार; जबफक ऄन्य

ऄधधकारों को के िल साधारण दजाव प्रदान फकया जाता है।


o मूल ऄधधकार (Fundamental Rights): यह संिैधाधनक ऄधधकारों की एक शाखा है तथा

आनके महत्ि के अधार पर आन ऄधधकारों को धिशेष दजाव प्रदान फकया गया है और ये सीधे
ईच्चतम एिं ईच्च न्यायालय द्वारा प्रितवनीय हैं।

173 www.visionias.in ©Vision IAS

Google it:- https://upscpdf.com


https://upscpdf.com << Download From >> https://upscpdf.com

2. भारतीय संधिधान में मूल ऄधधकारों की संक ल्पना


 आन्हें मूल आसधलए कहा जाता है, क्योंफक:

o ये एक व्यधि के सिांगीण धिकास के धलए अिश्यक होते हैं; तथा


o आन्हें देश की मूल धिधध ऄथावत् संधिधान के द्वारा गारंटी प्रदान की गइ है।
 मूल ऄधधकार, भारतीय संधिधान के भाग 3 के ऄंतगवत ऄनुच्छेद 12 से 35 तक समाधिष्ट फकए गए

हैं। ये न के िल देश में राजनीधतक स्ितंत्रता की गारंटी प्रदान करते हैं, बधल्क राज्य की मनमानी

कारविाइ के धिरूद् धनयंत्रक की भूधमका का धनिवहन भी करते हैं। आसके ऄधतररि, ये ‘व्यधि के

शासन’ के स्थान पर ‘धिधध के शासन’ की स्थाधपत करने में सहायता करते हैं, धजसका ऄधभप्राय है
फक राज्य मनमाने तरीके से कायव नहीं कर सकता है।
 आसके ऄधतररि, न्याधयक पुनर्विलोकन की शधि के साथ एक स्ितंत्र न्यायपाधलका, मूल ऄधधकारों

के संरक्षक की भूधमका धनभाने के साथ-साथ धिधध के शासन की स्थापना हेतु ‘ऄधभभािक’ एिं

‘गारंटर’ के रूप में कायव करती है।

3. मूल ऄधधकारों का क्रधमक धिकास


संधिधान में मूल ऄधधकारों को सधममधलत करने की प्रेरणा, स्ितंत्रता के धलए एक लंबे संघषव और धिश्व

के प्रमुख लोकतंत्रों के ऄनुभिों से प्राि हइ।

 िषव 1928 में मोतीलाल नेहरू की ऄध्यक्षता में सिवदलीय सममेलन की सधमधत द्वारा एक

संिैधाधनक योजना का मसौदा तैयार फकया गया था, धजसे नेहरू ररपोटव कहा गया। आसके ऄंतगवत,
भारत में संसदीय लोकतंत्र की स्थापना और ऄल्पसंख्यकों को संरक्षण प्रदान करने का अह्िान
फकया गया।
 िषव 1931 के कराची ऄधधिेशन के प्रस्ताि में व्यधिगत ऄधधकारों और स्ितंत्रताओं के मुद्दों के
प्रधत प्रधतबद्ता व्यि की गइ। आसमें मूल नागररक ऄधधकार तथा न्यूनतम मजदूरी सुधनधश्चत करने
और ऄस्पृश्यता एिं दासता के ईन्मूलन जैसे सामाधजक-अर्वथक ऄधधकार भी शाधमल थे।
 हालाँफक, साआमन कमीशन और संयुि संसदीय सधमधत, जो भारत सरकार ऄधधधनयम, 1935 के

धलए ई्त रदायी थे, ने आस अधार पर मूल ऄधधकारों की घोषणाओं के धिचार को ऄस्िीकार कर

फदया था फक “ये ऄमूतव घोषणाएँ तब तक गैर-ईपयोगी हैं, जब तक फक आनके धलए आच्छाशधि और

ईन्हें प्रभािी बनाने के साधन ईपलब्ध न हो"। नेहरू ररपोटव के पश्चात्, राष्ट्रिादी मत ‘धबल ऑफ़

राआट्स’ के पक्ष में था, क्योंफक धिरटश शासन के ऄधीन प्राि ऄनुभि यह था फक एक ऄधीनस्थ
धिधाधयका व्यधिगत स्ितंत्रता का ऄधतक्रमण करने में कायवकाररणी के धलए कठपुतली के रूप में
कायव कर सकती है।
 धिरटश मत पर ध्यान फदए धबना, संधिधान धनमावताओं ने व्यधिगत स्ितंत्रता के संरक्षण तथा

समुदाय के प्रत्येक सदस्य हेतु सामाधजक, अर्वथक और राजनीधतक न्याय (राज्य की नीधत के

धनदेशक तत्िों के साथ) सुधनधश्चत करने के धलए मूल ऄधधकारों को ऄपनाया। संधिधान सभा,
संयुि राज्य ऄमेररका एिं धिटेन के धबल ऑफ़ राआट्स के साथ-साथ फ्ांस के मानिाधधकारों की
घोषणा से भी प्रेररत थी।

174 www.visionias.in ©Vision IAS

Google it:- https://upscpdf.com


https://upscpdf.com << Download From >> https://upscpdf.com

3.1. मू ल ऄधधकारों की सीमाएँ

 धिश्व भर में, यहां तक फक ऐधतहाधसक रूप से सुदढ़ृ लोकतंत्रों में भी मूल ऄधधकारों के ईपयोग पर
युधियुि सीमाएं अरोधपत की गइ हैं। भारतीय संधिधान में भी आस प्रकार की सीमाओं का ईल्लेख
फकया गया है। ईदाहरण के धलए, ऄनुच्छेद 19 के ऄंतगवत प्रद्त मूल ऄधधकारों को सशतव प्रदान
फकया है और ये युधियुि धनबंधनों के ऄधीन हैं। जहाँ एक ओर ऄनुच्छेद 19 के ऄंतगवत कु छ
युधियुि प्रधतबंधों का ईल्लेख फकया गया है, िहीं दूसरी ओर ऄन्य मूल ऄधधकारों के संबंध में आस
प्रकार की सीमाएं, धिधभन्न संशोधन ऄधधधनयमों या ईच्चतम न्यायालय के धनणवयों के माध्यम से
जोड़े गए हैं। सामाधजक न्याय की स्थापना के ईद्देश्य से ऄनुच्छेद 15 और 16 को संसद द्वारा
समय-समय पर संशोधधत फकया गया है।

4. मूल ऄधधकारों की धिशे ष ताएं


मूल ऄधधकारों की कु छ महत्िपूणव धिशेषताएं धनम्नधलधखत हैं:
 मूल ऄधधकारों के दो िगव हैं। कु छ ऄधधकार के िल भारतीय नागररकों को ही प्राि हैं, जबफक कु छ
ऄधधकार भारतीय नागररकों और धिदेशी व्यधियों, दोनों को समान रूप से प्राि हैं। ऄनुच्छेद 15,
16, 19, 29 और 30 द्वारा प्रद्त ऄधधकार के िल भारतीय नागररकों के धलए ही ईपलब्ध हैं।
जबफक शेष सभी ऄधधकार, धिदेशी नागररकों (शत्रु राष्ट्र के नागररकों के ऄधतररि) के धलए भी
ईपलब्ध हैं।
 ये ऄधधकार ऄसीधमत नहीं हैं, ऄधपतु आन पर कु छ युधियुि धनबंधन अरोधपत फकए जा सकते हैं।
"युधियुिता” (reasonableness) का ऄथव न्यायपाधलका द्वारा ‘समाज के समग्र कल्याण और
व्यधि के ऄधधकारों के मध्य संतुलन की ईधचत ऄिस्था’ के रूप में धनधावररत फकया गया है।
 संसद को धिधध द्वारा यह धनधावररत करने का ऄधधकार है फक ये ऄधधकार सैन्य बलों तथा खुफफ़या
धिभागों के संबंध में फकस सीमा तक प्रभािी होंगे।
 ये स्थायी नहीं हैं। संसद, संिैधाधनक संशोधन के माध्यम से आनमें कटौती ऄथिा कमी कर सकती
है, बशते फक आससे संधिधान का मूल ढांचा प्रधतकू ल रूप से प्रभाधित न हो।
 आन ऄधधकारों का दायरा ऄनुच्छेद 31A, 31B, 31C, 33, 34 और 35 द्वारा सीधमत होता है।
 ऄनुच्छेद 20 एिं 21 के ऄधतररि, सभी ऄधधकारों को राष्ट्रीय अपातकाल के दौरान धनलंधबत
फकया जा सकता है। ऄनुच्छेद 19 के तहत प्रद्त 6 स्ितंत्रताओं को के िल तभी धनलंधबत फकया जा
सकता है जब अपातकाल युद् और बाह्य अक्रमण के अधार पर घोधषत फकया गया हो, न फक
सशस्त्र धिद्रोह के अधार पर।
 मूल ऄधधकारों के ऄंतगवत , कु छ ऄधधकार राज्य को यह धनदेश देते हैं फक िह कु छ धिधशष्ट कायव न
करे। आनका स्िरूप नकारात्मक होता है। ईदाहरण के धलए ऄनुच्छेद 14, 15(1), 16(2), 18(1),
20, 21, 22(1), 27 तथा 28(1).
o ऄनुच्छेद 14: राज्य, भारत के राज्यक्षेत्र में फकसी व्यधि को धिधध के समक्ष समता से या
धिधधयों के समान संरक्षण से िंधचत नहीं करेगा।
 कु छ ऄनुच्छेद ऄधभव्यि रूप से, एक ऄथिा ऄधधक ऄधधकारों का सृजन करते हैं तथा ईन्हें प्रदान
करते हैं। आनका स्िरूप सकारात्मक होता है। ईदाहरण के धलए ऄनुच्छेद 19(1), 21क, 29, 30
तथा 32 की भाषा भी सकारात्मक है।

175 www.visionias.in ©Vision IAS

Google it:- https://upscpdf.com


https://upscpdf.com << Download From >> https://upscpdf.com

मूल ऄधधकारों का िगीकरण


संधिधान के भाग 3 में ईधल्लधखत मूल ऄधधकारों को धनम्नधलधखत छह श्रेधणयों में िगीकृ त फकया गया
है:
 समता का ऄधधकार (ऄनुच्छेद 14-18)
 स्ितंत्रता का ऄधधकार (ऄनुच्छेद 19-22)
 शोषण के धिरुद् ऄधधकार (ऄनुच्छेद 23-24)
 धमव की स्ितंत्रता का ऄधधकार (ऄनुच्छेद 25-28)
 संस्कृ धत एिं धशक्षा संबंधी ऄधधकार (ऄनुच्छेद 21क, 29-30)
 संिैधाधनक ईपचारों का ऄधधकार (ऄनुच्छेद 32)

5. मूल ऄधधकारों का धििरण


5.1. ऄनु च्छे द 12 : राज्य की पररभाषा (Definition of State)
मूलपाठ
आस भाग में, जब तक फक संदभव से ऄन्यथा ऄपेधक्षत न हो, "राज्य” के ऄंतगवत भारत की सरकार और
संसद तथा राज्यों में से प्रत्येक राज्य की सरकार और धिधान-मंडल तथा भारत के राज्यक्षेत्र के भीतर
या भारत सरकार के धनयंत्रण के ऄधीन सभी स्थानीय और ऄन्य प्राधधकारी शाधमल हैं।
धििरण
ऄनुच्छेद 12 के ऄंतगवत संधिधान के भाग 3 के प्रयोजन के धलए "राज्य" को पररभाधषत फकया गया है।
कोइ नागररक, राज्य की पररभाषा में सधममधलत फकसी भी धनकाय द्वारा मूल ऄधधकारों के ईल्लंघन
फकए जाने पर सीधे ईच्चतम और ईच्च न्यायालय में ऄपील कर सकता है। न्याधयक घोषणाओं में "ऄन्य
प्राधधकाररयों" (other authorities) को धिस्तृत रूप से िर्वणत फकया गया है। राज्य की यह पररभाषा
धन:शेषकारी नहीं है, यह समािेशक है तथा आसका समय-समय पर धिस्तार फकया जाता रहा है। आसके
ऄंतगवत संसद, संघीय सरकार, राज्य धिधाधयका, राज्य कायवपाधलका, स्थानीय ऄधधकारी अफद शाधमल हैं।
"ऄन्य प्राधधकाररयों" के ऄंतगवत धनम्नधलधखत धनकायों को शाधमल फकया गया है:
 धिधध के तहत गरठत िैधाधनक शधियों का प्रयोग करने िाले धनकाय;
 सरकार द्वारा पयावि धि्त ीय सहायता प्राि करने िाले धनकाय;
 सरकारी कायों का धनष्पादन करने िाले धनकाय; तथा
 सरकार के धनयंत्रण के ऄधीन धनकाय।
धनम्नधलधखत को न्यायालय के धनणवयों द्वारा “राज्य” ऄधभधनधावररत फकया गया है:
 राज्य ऄधधधनयम के ऄधीन पंजीकृ त सोसाआटी द्वारा स्थाधपत प्रादेधशक आंजीधनयररग
महाधिद्यालय,
 भारतीय सांधख्यकी संस्थान,
 भारतीय कृ धष ऄनुसंधान पररषद,
 भारतीय आस्पात प्राधधकरण,
 राजस्थान धिद्युत बोडव,
 ऄंतरावष्ट्रीय धिमानप्त न प्राधधकरण,
 तेल एिं प्राकृ धतक गैस धनगम,

176 www.visionias.in ©Vision IAS

Google it:- https://upscpdf.com


https://upscpdf.com << Download From >> https://upscpdf.com

 सभी राष्ट्रीयकृ त बैंक….अफद।


o ककतु, भारतीय फक्रके ट कं ट्रोल बोडव (BCCI) एिं राष्ट्रीय शैधक्षक ऄनुसध
ं ान और प्रधशक्षण
पररषद (NCERT) राज्य की पररभाषा में शाधमल नहीं हैं।
 रामन दयाराम शेट्टी बनाम भारतीय धिमानप्त न प्राधधकरण िाद में, ईच्चतम न्यायालय द्वारा
“ऄन्य प्राधधकारी” के तहत शाधमल फकए जाने िाले धनकाय हेतु धनम्नधलधखत पांच अधार
धनधावररत फकए गए थे:
o यफद संपूणव शेयर पूंजी का स्िाधमत्ि या प्रबंधन राज्य के ऄधीन हो;
o एकाधधकार का दजाव प्राि धनकाय;
o यफद सरकार का कोइ धिभाग फकसी धनगम के तत्िािधान में हो;
o यफद फकसी धनकाय की कायावत्मक प्रकृ धत मूलरूप में सरकारी हो; तथा
o पूणव और व्यापक रूप से राज्य के धनयंत्रण िाले धनकाय।
5.1.1. क्या राज्य की पररभाषा के तहत धनजी धनकाय भी शाधमल हैं ?
 सािवजधनक संसाधनों के प्रबंधन सधहत धनजी धनकायों की सािवजधनक कायों में बढ़ती भागीदारी ने
धनरंतर यह प्रश्नधचन्ह ईत्पन्न फकए हैं फक क्या ऐसे कायों का प्रदशवन करने िाली धनजी कं पधनयां ,
धनगम, सहकारी सधमधतयों अफद जैसे धनकाय ऄनुच्छेद 12 के तहत राज्य की पररभाषा के तहत
शाधमल हैं।
 आस तरह के प्रश्न न्यायालय के समक्ष प्रस्तुत हए हैं, धजन्हें न्यायालय द्वारा के स-दर-के स अधार पर
धनधावररत फकया गया है फक राज्य की पररभाषा के तहत फकसी धनकाय को शाधमल फकया जाना
चाधहए या नहीं। आन प्रश्नों पर धिचार करने के पश्चात् फदल्ली ईच्च न्यायालय और भारतीय ईच्चतम
न्यायालय दोनों ने धनणवय फदया है फक CAG ईन धनजी दूरसंचार कं पधनयों का ऑधडट कर सकती
है जो सरकार के साथ ऄपना राजस्ि साझा करती हैं। न्यायालय के ऄनुसार आस तरह की जांच यह
सुधनधश्चत करने के धलए महत्िपूणव है फक धनजी दूरसंचार ऑपरेटरों को मूल्यिान प्राकृ धतक संसाधन
का ईपयोग करने की ऄनुमधत देने के पश्चात् सरकार को ऄपना "िैध धहस्सा" प्राि हअ है या नहीं।
न्यायालय का यह मानना है फक प्राकृ धतक संसाधन से संबंधधत फकसी मामले की सुनिाइ के दौरान,
ईसे ऄनुच्छेद 149 (जो CAG के कतवव्यों और शधियों से संबंधधत है) के तहत एक ईद्देश्यपूणव
व्याख्या प्रस्तुत करनी होगी। आस धनणवय ने CAG की पहँच को सरकार और सािवजधनक क्षेत्र की
कं पधनयों से लेकर फकसी भी आकाइ तक धिस्ताररत कर फदया गया जो ऄपने व्यिसाय में
सािवजधनक संसाधन का ईपयोग करते हैं तथा सरकार के साथ राजस्ि साझा करते हैं।
 हालांफक, आस पर रटप्पणी नहीं की गइ है फक फकसी धनजी संस्था को फकन पररधस्थधतयों में राज्य
का दजाव फदया जाएगा। दुभावग्यिश, ऄभी तक कोइ धनणावयक अधार धनधावररत नहीं फकया गया है।

5.2. ऄनु च्छे द 13 : मू ल ऄधधकारों से ऄसं ग त या ईनका ऄल्पीकरण करने िाली धिधधयां

मूल पाठ
मूल ऄधधकारों से ऄसंगत या ईनका ऄल्पीकरण करने िाली धिधधयाँ:
(1) संधिधान के प्रारंभ से ठीक पूि,व भारत के राज्यक्षेत्र में प्रिृ्त सभी धिधधयां ईस सीमा तक शून्य
होंगी, जहाँ तक िे आस भाग के ईपबंधों से ऄसंगत हैं।
(2) राज्य द्वारा ऐसी फकसी धिधध का धनमावण नहीं फकया जाएगा जो आस भाग द्वारा प्रद्त ऄधधकारों
को समाि ऄथिा न्यून करती है तथा आस खंड के ईल्लंघन में बनाइ गइ प्रत्येक धिधध, ईल्लंघन की
सीमा तक शून्य होगी।
(3) आस ऄनुच्छेद में, जब तक फक संदभव से ऄन्यथा ऄपेधक्षत न हो,

177 www.visionias.in ©Vision IAS

Google it:- https://upscpdf.com


https://upscpdf.com << Download From >> https://upscpdf.com

 "धिधध" के ऄंतगवत भारत के राज्यक्षेत्र में धिधध का बल रखने िाला कोइ ऄध्यादेश , अदेश,
ईपधिधध, धनयम, धिधनयम, ऄधधसूचना, रूफढ़ या प्रथा शाधमल है;
 "प्रिृ्त धिधध" के ऄंतगवत भारत के राज्यक्षेत्र में फकसी धिधान-मंडल या ऄन्य सक्षम प्राधधकारी
द्वारा आस संधिधान के प्रारंभ से पूिव पाररत या धनर्वमत धिधध है जो पहले ही धनरधसत नहीं कर दी
गइ है, चाहे ऐसी कोइ धिधध या ईसका कोइ भाग ईस समय पूणवतया या धिधशष्ट क्षेत्रों में प्रितवन में
नहीं है।
(4) आस ऄनुच्छेद की कोइ बात ऄनुच्छेद 368 के ऄधीन फकए गए आस संधिधान के फकसी संशोधन को
लागू नहीं होगी।
धििरण

ऄनुच्छेद 13, न्याधयक पुनर्विलोकन (judicial review) की शधि से संबंधधत है। यह देश में ईच्चतर
न्यायपाधलका को मूल ऄधधकारों के संरक्षक के रूप में स्थाधपत करता है। आसका ईद्देश्य मूल ऄधधकारों
के धिषय में संधिधान की सिोपरर धस्थधत को सुरधक्षत करना है। न्याधयक पुनर्विलोकन संधिधान का
अधारभूत लक्षण (मूल ढांचा) है।

5.2.1. न्याधयक पु न र्विलोकन (Judicial Review)


 न्याधयक पुनर्विलोकन का तात्पयव न्यायपाधलका की ईस शधि से है धजसके ऄंतगवत िह धिधाधयका
द्वारा धनर्वमत फकसी भी धिधध (ऄथिा ईसके फकसी भाग) को संधिधान के प्रािधानों का ईल्लंघन
होने के अधार पर ‘ऄिैध एिं शून्य’ (null & void) ऄथिा कायवपाधलका द्वारा जारी फकए गए

फकसी भी अदेश को ईसके क्षेत्राधधकार से बाहर होने पर ऄधधकारातीत (ultra-vires) घोधषत

कर सकती है तथा साथ ही, पूिव में फदए गए ऄपने धनणवयों की भी समीक्षा कर सकता है।
o यह धसद्ांत संयुि राज्य ऄमेररका की न्याधयक व्यिस्था की देन है, जहाँ मारबरी बनाम

मेधडसन िाद (1803) में न्यायपाधलका की ऄन्तर्वनधहत शधि के रूप में न्याधयक पुनर्विलोकन
की स्थापना की गइ।
 भारतीय संधिधान, मूल ऄधधकारों की सुरक्षा के धलए कायवपाधलका एिं धिधाधयका की ऄनुधचत

कायविाधहयों के धिरुद् स्पष्ट रूप से न्याधयक पुनर्विलोकन की शधि प्रदान करता है। ‘पुनर्विलोकन’
शब्द का प्रयोग के िल ऄनुच्छेद 137 के ऄंतगवत फकया गया है, धजसमें ईच्चतम न्यायालय को ऄपने
द्वारा फदए गए धनणवयों ऄथिा अदेशों की न्याधयक पुनर्विलोकन (समीक्षा) की शधि प्रदान की गइ
है। लेफकन, संधिधान के ऄंतगवत कु छ धिशेष प्रािधान है धजनके संबंध में न्यायालय को पुनर्विलोकन

की शधि प्राि होती है।


o ऄनुच्छेद 13, 32, 131-136, 143, 226, 245, 246, 251, 254 एिं 372 के ऄंतगवत
ईच्चतर न्यायपाधलका को कु छ ऐसी शधियाँ एिं ई्त रदाधयत्ि प्रद्त हैं, धजससे न्यायालय को

पुनर्विलोकन की शधि प्राि होती है।

5.2.1.1. सं धिधान के पू िव की धिधधयों की धिधधमान्यता

 ऄनुच्छेद 13 (1) ईन धिधधयों के संबंध में है जो संधिधान के प्रारंभ के पूिव प्रिृ्त थीं। ऄनुच्छेद 13

(2) संधिधान के प्रारंभ के पश्चात् की धिधधयों को लागू होता है। संधिधान के प्रारंभ के पूिव की
धिधधयों को शुरुअत से ही ऄधिधधमान्य घोधषत नहीं फकया गया है। संधिधान के प्रारंभ के पूिव की
कोइ धिधध यफद भाग 3 के ईपबंधों के ऄनुरूप नहीं है तो िह धिधधमान्य नहीं रहती है।

178 www.visionias.in ©Vision IAS

Google it:- https://upscpdf.com


https://upscpdf.com << Download From >> https://upscpdf.com

5.2.1.2. पृ थ क्करणीयता का धसद्ां त (Doctrine of Severability)


 ऄनुच्छेद 13 (1) और ऄनुच्छेद 13 (2) में यह कहा गया है फक धिधधयाँ ऄसंगत होने की ईल्लंघन
की मात्रा तक शून्य होंगी। यफद धिधाधयका द्वारा ऐसी धिधध पाररत की जाती है जो मूल ऄधधकारों
के फकसी प्रािधान का ईल्लंघन करती है तो िह धिधध न्यायालय द्वारा ऄसंगतता की सीमा तक
शून्य घोधषत कर दी जाती है। संपूणव धिधध को ऄमान्य घोधषत करने के स्थान पर धिधध के के िल
ईस भाग को हटाया जा सकता है, जो मूल ऄधधकारों से ऄसंगत है। यह पृथक्करण का धसद्ांत है।
o धमनिाव धमल्स बनाम भारत संघ िाद, 1980 में ईच्चतम न्यायालय ने ऄनुच्छेद 368 के खंड 4
और 5 को ऄिैध घोधषत कर फदया था, धजसे िषव 1976 में 42िें संधिधान संशोधन
ऄधधधनयम द्वारा जोड़ा गया था।

5.2.1.3. ग्रहण का धसद्ां त (Doctrine of Eclipse)


 ऄनुच्छेद 13 (1) के ऄनुसार, संधिधान के लागू होने से पूिव प्रिृ्त धिधधयां ईस सीमा तक शून्य
होंगी, धजस सीमा तक िे फकसी मूल ऄधधकार का ईल्लंघन करती हैं ऄथिा मूल ऄधधकारों से
ऄसंगत हैं। ग्रहण का धसद्ांत संधिधान के पूिव की ऐसी धिधध को लागू होता है (ऄथावत् ऐसी धिधध
जो 26 जनिरी 1950 के पहले ऄधधधनयधमत की गइ हो) जो ऄधधधनयधमत फकए जाने के समय
धिधधमान्य थी। यह धसद्ांत संधिधान के प्रारंभ के बाद की धिधधयों को लागू नहीं होता है। यफद
संधिधान के प्रारंभ के पश्चात् बनायी गयी धिधध संधिधान से ऄसंगत है तो िह प्रारंभ से ही शून्य
होगी।
 हालाँफक, यफद आस प्रकार के मूल ऄधधकार (जो फक संधिधान पूिव धनर्वमत धिधधयों को अच्छाफदत
करता है) को आस सीमा तक संशोधधत फकया गया है फक संधिधान पूिव धनर्वमत धिधधयां संशोधधत
मूल ऄधधकार के साथ ऄसंगत नहीं हैं तो ऐसी धिधधयां पुनः िैध हो जाएँगी। आस प्रकार आन
धिधधयों पर ग्रहण का धसद्ांत लागू नहीं होता है। यह धनणवय भीखाजी नारायण िाद (1955) में
फदया गया था।

5.2.1.4. ऄधधत्यजन का धसद्ां त (Doctrine of Waiver)


 यह संयुि राज्य ऄमेररका में लागू है। आसके ऄनुसार कोइ व्यधि ऄपने ऄधधकारों का स्िेच्छा से
पररत्याग कर सकता है।
o हालांफक, भारत में ऄधधत्यजन का धसद्ांत मूल ऄधधकारों पर लागू नहीं होता है। राज्य फकसी
व्यधि के मूल ऄधधकार को आस अधार पर भंग नहीं कर सकता फक व्यधि ने ईसका
ऄधधत्यजन कर फदया था।

5.2.1.5. भधिष्यलक्षी धिधनणव य का धसद्ां त ( Doctrine of Prospective Overruling)


 मूल ऄधधकार का प्रभाि भूतलक्षी (Retrospective) नहीं है बधल्क आसका भधिष्यलक्षी
(Prospective) प्रभाि है। भारतीय संधिधान के प्रितवन के पूिव प्रिृ्त धिधधयों पर मूल ऄधधकारों
का प्रभाि ईस धतधथ से लागू होगा, धजस धतधथ से आन्हें लागू फकया गया है। संधिधान के लागू होने
के पूिव फकए गए कायों के संबंध में संधिधान पूिव धिधधयाँ लागू होंगी। आसधलए, संधिधान पूिव प्रिृ्त
धिधधयों के ऄधीन संधिधान के लागू होने के पूिव ईत्पन्न ऄधधकार एिं दाधयत्ि का मूल ऄधधकारों
के ईल्लंघन के बािजूद भी प्रितवन कराया जा सकता है।

5.2.1.6. मोटर जनरल ट्रे ड्स बनाम स्टे ट ऑफ अन्र प्रदेश (1984)
 यफद फकसी धिधध के िैध खंड को ऄिैध खंड से ऄलग फकया जा सकता है और िैध खंड को एक
स्ितंत्र धिधध बनाने के धलए धिचार फकया जा सकता है, तो ईि ऄनुभाग िैध रहता है।
 यफद िैध और ऄिैध खंड अपस में आतने धमधश्रत हैं फक ईन्हें पृथक नहीं फकया जा सकता है तो
संपूणव धिधध को शून्य घोधषत फकया जा सकता है।

179 www.visionias.in ©Vision IAS

Google it:- https://upscpdf.com


https://upscpdf.com << Download From >> https://upscpdf.com

5.2.2. मू ल ऄधधकारों की सं शोधनीयता और मू ल ढां चे का धसद्ां त

 ऄनुच्छेद 13 (2) में कहा गया है फक राज्य, ऐसी कोइ धिधध (law) ऄधधधनयधमत नहीं करेगा, जो
मूल ऄधधकारों में कटौती ऄथिा कमी करती है। आस संबंध में धििाद तब ईत्पन्न हअ जब यह प्रश्न
सामने अया फक क्या ऐसी कोइ ‘धिधध’ शब्द में ऄनुच्छेद 368 के तहत ‘संधिधान संशोधन
ऄधधधनयम’ भी सधममधलत हैं?”
o संधिधान के भाग 4 में ईधल्लधखत नीधत-धनदेशक तत्िों का ईद्देश्य देश में सामाधजक-अर्वथक
लोकतंत्र की स्थापना करना है। आन तत्िों को राज्य की नीधतयों द्वारा लागू करने के धलए
संसद ने अरंभ से ही संपध्त के ऄधधकार को सीधमत करने हेतु संधिधान संशोधन की नीधत
ऄपनाइ, धजसे मूल ऄधधकार में हस्तक्षेप के अधार पर चुनौती दी गइ।
 मूल ऄधधकारों की संशोधनीयता से व्युत्पन्न कु छ प्रमुख िाद (एिं संधिधान संशोधन ऄधधधनयम)
धनम्नधलधखत हैं:
o शंकरी प्रसाद िाद (1951): आसमें ईच्चतम न्यायालय ने धनणवय फदया फक संधिधान संशोधन,
साधारण धिधध नहीं है। ऄत: ऄनुच्छेद 13 में ‘धिधध’ शब्द के ऄंतगवत के िल साधारण धिधधयां
शाधमल हैं, न फक ‘संधिधान संशोधन ऄधधधनयम’। आसधलए संसद के पास मूल ऄधधकारों में
संशोधन करने की शधि है।
o गोलकनाथ िाद (1967): ईच्चतम न्यायालय ने ऄपने पूिविती धनणवय को ईलट फदया और
धनणवय फदया फक संधिधान में मूल ऄधधकारों को एक सिोच्च धस्थधत प्राि है। ऄत: संसद द्वारा
मूल ऄधधकारों में संशोधन नहीं फकया जा सकता है।
 24िां संधिधान संशोधन ऄधधधनयम, 1971: संसद ने आस ऄधधधनयम द्वारा ऄनुच्छेद 13 में एक
नया खंड जोड़कर यह स्पष्ट कर फदया गया फक ऄनुच्छेद 13 के ऄथांतगवत ऄनुच्छेद 368 के ऄधीन
पाररत संिैधाधनक संशोधन, साधारण धिधध नहीं है। आस प्रकार ऄनुच्छेद 13, संधिधान संशोधन
ऄधधधनयमों पर लागू नहीं होगा। आस प्रकार, राज्य के पास एक बार पुनः मूल ऄधधकारों में
om

संशोधन करने की शधि धनधहत हो गइ।


l.c
ai
gm

 के शिानंद भारती िाद (1973): न्यायपाधलका ने संधिधान के मूल ढांचे की ऄिधारणा प्रस्तुत की,
@
80

धजसके ऄनुसार संधिधान की कु छ ऐसी मूल धिशेषताएँ हैं, धजनका संशोधन नहीं फकया जा सकता
h1

है। मूल ऄधधकारों का संशोधन के िल ईस सीमा तक फकया जा सकता है धजस सीमा तक िे मूल
ng
si

ढांचे का भाग नहीं हैं।


sh
ar
ad

 42िां संधिधान संशोधन ऄधधधनयम (1976): आस ऄधधधनयम द्वारा मूल ढांचे के धसद्ांत को शून्य
घोधषत करने हेतु ऄनुच्छेद 368(4) और 368(5) समाधिष्ट फकए गए। आस ऄधधधनयम ने संसद को
संधिधान में संशोधन करने की ऄसीधमत शधियां प्रदान की, धजन्हें न्यायालय में चुनौती नहीं दी
जा सकती थी।
o ऄनुच्छेद 368 (4) - कोइ संसोधन फकसी न्यायालय में फकसी भी अधार पर प्रश्नगत नहीं
फकया जाएगा।
o ऄनुच्छेद 368 (5) - संसद की संधिधायी शधियों पर फकसी प्रकार का धनबंधन नहीं होगा।
 धमनिाव धमल्स िाद (1980): आस धनणवय में ईच्चतम न्यायालय ने ऄनुच्छेद 368(4) एिं 368(5)
को धनरस्त कर फदया, क्योंफक आनके द्वारा न्याधयक पुनर्विलोकन संबंधी शधियों में कटौती की गइ
थी, जो संधिधान का मूल ढांचा है। ऄत: ितवमान में मूल ऄधधकारों में ईस सीमा तक संशोधन
फकया जा सकता है, जहाँ तक फक िह संशोधन के मूल ढांचे को प्रधतकू ल रूप से प्रभाधित नही
करता हैं।

180 www.visionias.in ©Vision IAS

Google it:- https://upscpdf.com


https://upscpdf.com << Download From >> https://upscpdf.com

o ईच्चतम न्यायालय द्वारा 368 (4) और 368 (5) को धनरधसत कर फदया गया।
o ईच्चतम न्यायालय ने कहा फक न्याधयक पुनर्विलोकन संधिधान की एक अधारभूत धिशेषता है,
धजसे पररिर्वतत नहीं फकया जा सकता है।
o आस प्रकार, ितवमान में मूल ऄधधकारों में संशोधन फकया जा सकता है तथा ये न्याधयक
पुनर्विलोकन के ऄधीन भी हैं।
 अइ. अर. कोएल्हो िाद (2007): यह 9िीं ऄनुसच
ू ी में सधममधलत ऄधधधनयमों की िैधता से
संबंधधत है। मूल ढांचे के धसद्ांत को के शिानंद भारती के धनणवय की धतधथ से ऄनुसूची में शाधमल
फकए गए ऄधधधनयमों पर लागू फकया जाएगा क्योंफक न्याधयक पुनर्विलोकन संधिधान की मूल
धिशेषता है।

मूल ढांचे का धसद्ांत


आस धसद्ांत के ऄनुसार संधिधान के मूल ढांचे या अधारभूत धिशेषताओं को संिैधाधनक संशोधन के
माध्यम से संसद द्वारा समाि नहीं फकया जा सकता है। धिधभन्न धनणवयों के माध्यम से , न्यायपाधलका
द्वारा संधिधान के मूल ढांचे के रूप में धनम्नधलधखत धिशेषताओं का ईल्लेख फकया गया है:
 संधिधान की सिोच्चता;
 सरकार का गणतंत्रात्मक और लोकतांधत्रक स्िरूप;
 पंथधनरपेक्ष प्रकृ धत;
 शधियों का पृथक्करण;
 न्याधयक पुनर्विलोकन;
 संप्रभुता आत्याफद।

5.3. ऄनु च्छे द 14 - धिधध के समक्ष समता

मूलपाठ
राज्य, भारत के राज्यक्षेत्र में फकसी व्यधि को धिधध के समक्ष समता से या धिधधयों के समान संरक्षण से
िंधचत नहीं करेगा।
धििरण
धिधध के समक्ष समता (Equality before Law)
 आसका ऄथव यह है फक धिधध के समक्ष सभी नागररक समान है। यह धिरटश परंपरा से ग्रहण फकया
गया है। आसकी प्रिृध्त नकारात्मक है क्योंफक आसके तहत फकसी भी धिशेषाधधकार को स्िीकार
नहीं फकया गया है।
धिधध का समान संरक्षण (Equal Protection of Law)
 आसे संयिु राज्य ऄमेररका के संधिधान से ग्रहण फकया गया है। आसका ऄथव है फक राज्य को यह
सुधनधश्चत करना चाधहए फक समान के साथ समान और ऄसमान के साथ ऄसमान व्यिहार फकया
जाए। समान संरक्षण का धसद्ांत ऄसमान व्यधियों को धिशेष सुधिधाएँ और ऄिसर प्रदान करके
राज्य द्वारा सकारात्मक कायव फकए जाने की ऄपेक्षा करता है। ऄतः यह एक सकारात्मक
ऄिधारणा है।
 पधश्चम बंगाल राज्य बनाम ऄनिर ऄली सरकार िाद (1951) में ईच्चतम न्यायालय ने यह स्पष्ट
फकया फक ‘धिधध के समक्ष समता’ तथा ‘धिधधयों का समान संरक्षण’ दोनों िाक्यांशों का ऄथव एक
ही है। सैद्ाधन्तक रूप से आसमें ऄंतर हो सकता है फकन्तु, व्यािहाररक स्तर पर दोनों में कोइ धिभेद
नहीं है।

181 www.visionias.in ©Vision IAS

Google it:- https://upscpdf.com


https://upscpdf.com << Download From >> https://upscpdf.com

धिधध का शासन (Rule of Law)


धिधध के समक्ष समता का धिचार ए. िी. डायसी द्वारा प्रधतपाफदत ‘धिधध का शासन’ के धसद्ांत का मूल
तत्ि है। आसमें धनम्नधलधखत तीन तत्ि सधममधलत हैं:
 स्िेच्छाचारी शधियों की ऄनुपधस्थधत, ऄथावत फकसी भी व्यधि को धिधध के ईल्लंघन के ऄधतररि
दधडडत नहीं फकया जा सकता है।
 धिधध के समक्ष समता ऄत्यािश्यक है, ऄथावत सभी नागररक धिधध के समक्ष समान हैं।
 संधिधान व्यधिगत ऄधधकारों का स्रोत नहीं बधल्क व्यधिगत ऄधधकारों का पररणाम है, क्योंफक
संधिधान के ईद्भि के पूिव भी व्यधिगत ऄधधकारों का ऄधस्तत्ि था।
भारत में आसके प्रथम एिं धद्वतीय तत्ि ही लागू होते हैं। चूँफक भारतीय व्यिस्था में, संधिधान व्यधिगत
ऄधधकारों का स्रोत है, ऄत: तृतीय तत्ि लागू नहीं होता है।
ईच्चतम न्यायालय ने ऄपने धिधभन्न धनणवयों में ऄनुच्छेद 14 के ऄंतगवत धिधध के शासन को संधिधान के
मूल ढांचे का एक घटक माना है।

धिधध के समक्ष समता के ऄपिाद


भारतीय संधिधान में धनधहत धिधध के समक्ष समता का धसद्ांत पूणवतया धनरपे क्ष नहीं हैं। आसके कु छ
ऄपिाद धनम्नधलधखत हैं:
 राष्ट्रपधत और राज्यपाल को भारतीय संधिधान के ऄनुच्छेद 361 और 361A के ऄंतगवत
ऄधभयोजन के धिरुद् ईन्मुधि प्रदान की गइ है।
 सांसदों और धिधायकों को धिधाधयका में धिशेषाधधकार (ऄनुच्छेद 105 और 194) प्रदान फकए
गए हैं।
 धिदेशी राजनधयकों को भी ऄधभयोजन के धिरुद् ईन्मुधि प्रदान की गइ है।
प्रयोजनीयता
 ऄनुच्छेद 14, चाहे िह भारतीय नागररक, धिदेशी ऄथिा यहां तक फक कं पनी जैसी कोइ धिधधक
आकाइ ही क्यों न हो, सभी को समानता का ऄधधकार प्रदान करता है। साथ ही, यह के िल राज्य
की कायविाइ के धिरुद् ही ईपलब्ध है।
5.4. ऄनु च्छे द 15 - भे द भाि के धिरुद् ऄधधकार (Right against Discrimination)

मूलपाठ
धमव, मूलिंश, जाधत, ललग या जन्म स्थान के अधार पर धिभेद का प्रधतषेध।
1. राज्य फकसी भी नागररक के धिरुद् के िल धमव, मूलिंश, जाधत, ललग, जन्मस्थान या आनमें से फकसी
के अधार पर धिभेद नहीं करेगा।
2. कोइ नागररक, के िल धमव, मूलिंश, जाधत, ललग, जन्म स्थान या आनमें से फकसी के अधार पर –
(क) दुकानों, सािवजधनक भोजनालयों, होटलों और सािवजधनक मनोरंजन के स्थानों में प्रिेश; या
(ख) पूणवतः या ऄंशतः राज्य-धनधध से पोधषत या साधारण जनता के प्रयोग के धलए समर्वपत कु ओं,
तालाबों, स्नान घाटों, सड़कों और सािवजधनक समागम स्थानों के ईपयोग के संबंध में; फकसी
भी धनयोग्यता, दाधयत्ि, धनबवन्धन या शतव के ऄधीन नहीं होगा।
3. आस ऄनुच्छेद की कोइ बात राज्य को धस्त्रयों और बालकों के धलए कोइ धिशे ष ईपबंध करने से
धनिाररत नहीं करेगी।
4. आस ऄनुच्छेद की या ऄनुच्छेद 29 के खंड (2) की कोइ बात राज्य को सामाधजक ि शैधक्षक दृधष्ट से
धपछड़े हए नागररकों के फकन्ही िगों की ईन्नधत के धलए या ऄनुसूधचत जाधतयों और ऄनुसूधचत
जनजाधतयों के धलए कोइ धिशेष ईपबंध करने से धनिाररत नहीं करेगी।

182 www.visionias.in ©Vision IAS

Google it:- https://upscpdf.com


https://upscpdf.com << Download From >> https://upscpdf.com

5. आस ऄनुच्छेद या ऄनुच्छेद 19 के खंड (1) के ईपखंड (g) की कोइ बात राज्य को, सामाधजक और
शैधक्षक रूप से धपछड़े हए नागररकों के फकन्हीं िगों की ईन्नधत के धलए ऄथिा ऄनुसूधचत जाधतयों
या ऄनुसूधचत जनजाधतयों के धलए, धिधध द्वारा, कोइ धिशेष ईपबंध करने से धनिाररत नहीं
करेगी, जहाँ तक ऐसे धिशेष ईपबंध, ऄनुच्छेद 30 के खंड (1) में धनर्ददष्ट ऄल्पसंख्यक धशक्षा
संस्थाओं से धभन्न, धशक्षा संस्थाओं में, धजनके ऄंतगवत धनजी धशक्षा संस्थाएँ भी शाधमल हैं, चाहे िे
राज्य से सहायता प्राि हों या नहीं, प्रिेश से संबंधधत हैं।
धििरण
 ऄनुच्छेद 15(1) के िल धमव, मूलिंश, ललग, जाधत और जन्म स्थान के अधार पर राज्य द्वारा
धिभेद का प्रधतषेध करता है। धिभेद के िल ईपयुवि अधारों पर धनधषद् है। राज्य ऄन्य अधारों पर
धिभेद कर सकता है।
 आस संदभव में ‘के िल’ का ऄथव - आसका ऄथव है फक के िल ईपयुवि अधार पर भेदभाि की ऄनुमधत
नहीं दी गयी है। राज्य ईपयुवि में से फकसी भी अधार के साथ संयोधजत कु छ ऄन्य अधारों पर
धिभेद कर सकता है।
 ईदाहरणाथव, के िल जाधत के अधार पर कोइ धिभेद नहीं फकया जा सकता। लेफकन, जाधत और
धपछड़ेपन के अधार पर सकारात्मक धिभेद ऄथिा सकारात्मक कारविाइ की ऄनुमधत प्रदान की
गइ है।
 ऄनुच्छेद 15 (2) धिधशष्ट प्रकार की धद्वयांगता के मामले में भेदभाि पर रोक लगाने से संबंधधत है,
जबफक 15 (1) राज्य के धलए के िल फदशा-धनदेश है। ज्ञातव्य हैफक 15 (2) धनजी व्यधियों के
धिरूद् भी ईपलब्ध है
 ऄनुच्छेद 15(3) मधहलाओं एिं बच्चों के धलए सकारात्मक कायविाइ का प्रािधान करता है। यह
राज्य को सामाधजक समानता की स्थापना के धलए सकारात्मक एिं तकव पूणव धिभेद करने की
ऄनुमधत प्रदान करता है। ईदाहरण के धलए, स्थानीय धनकायों में मधहलाओं के धलए अरक्षण की
व्यिस्था एिं बच्चों के धलए धनःशुल्क धशक्षा की व्यिस्था।
5.4.1. सं बं धधत न्याधयक िाद
 धपछड़ी जाधतयों के धलए अरक्षण के मुद्दे पर कायवपाधलका एिं न्यायपाधलका के मध्य टकराि की
धस्थधत ईत्पन्न हो गइ। ईच्चतम न्यायालय ने चंपकम दोराइराजन बनाम मद्रास राज्य िाद (1951)
में राज्य सरकार के ईस धनणवय को धनरस्त कर फदया धजसके द्वारा मेधडकल एिं आंजीधनयररग
कॉलेजों में गैर-िाह्मण छात्रों के धलए सीटें अरधक्षत की गइ। यह ऄनुच्छेद 15(1) के तहत धमव,
जाधत या ललग के अधार पर धिभेद का प्रधतषेध के अधार पर फकया गया था।
 लेफकन, आस धनणवय को बाद में संसद के द्वारा एक संधिधान संशोधन के माध्यम से धनरस्त कर
फदया गया एिं एक निीन प्रािधान ऄनुच्छेद 15(4) को जोड़ा गया धजसने कायवपाधलका को यह
शधि प्रदान की है फक सामाधजक एिं शैक्षधणक रूप से धपछड़े िगों के नागररकों के संबंध में
सकारात्मक कारविाइ (affirmative action) के प्रािधान करने में सरकार सक्षम है।
 लेफकन, संशोधधत प्रािधानों के ऄंतगवत राज्य द्वारा ईठाए गए कदमों को भी चुनौती प्रदान की गइ
एिं पुन: ईच्चतम न्यायालय ने एम. अर. बालाजी बनाम मैसरू राज्य िाद (1963) में यह धनणवय
फदया फक एक धिशेष िगव धपछड़ा है ऄथिा नहीं आसे ज्ञात करने हेतु लोगों का एक समूह ऄथिा
ईसकी जाधत एकमात्र या प्रमुख कारण नहीं हो सकता है। साथ ही, ईच्चतम न्यायालय ने अरक्षण
के मामले में एक ऄन्य धनदेश यह फदया फक अरक्षण एक ईधचत सीमा से ऄधधक नहीं हो सकता हैं।
 ईच्चतम न्यायालय ने मंडल िाद के धनणवय (1993) में जाधत अधाररत अरक्षण को िैध माना और
क्रीमीलेयर की ऄिधारणा प्रदान की। आसके ऄनुसार, धपछड़े िगों में संपन्न लोगों को समाज के
ऄग्रणी समुदायों के समकक्ष मानकर फकसी भी प्रकार का अरक्षण नहीं फदया जाता है।

183 www.visionias.in ©Vision IAS

Google it:- https://upscpdf.com


https://upscpdf.com << Download From >> https://upscpdf.com

5.4.2. ऄनु च्छे द 15 एिं सं िै धाधनक सं शोधन

 ऄनुच्छेद 15(4) को प्रथम संधिधान संशोधन ऄधधधनयम द्वारा जोड़ा गया। यह सामाधजक और
शैक्षधणक रूप से धपछड़े िगों या ऄनुसूधचत जाधत/ऄनुसूधचत जनजाधतयों के धलए सकारात्मक
कारविाइ (अरक्षण) का प्रािधान करता है।
 ऄनुच्छेद 15(5), सरकार द्वारा सहायता-प्राि या गैर-सहायता प्राि धशक्षण संस्थानों में समाज के
सामाधजक और शैक्षधणक रूप से कमजोर िगों के धलए सकारात्मक कारविाइ (अरक्षण) का
प्रािधान करता है।

 जहाँ ऄनुच्छेद 15(4) की प्रकृ धत साधारण है, िहीं ऄनुच्छेद 15(5) धिधशष्ट रूप से धशक्षा से

संबंधधत है। ऄनुच्छेद 15(5) को 93िें संधिधान संशोधन ऄधधधनयम (2005) द्वारा जोड़ा गया।

ऄल्पसंख्यक धशक्षण संस्थान, आस प्रािधान के ऄपिाद हैं। आस संशोधन के द्वारा SC, ST एिं

OBC िगव के छात्रों हेतु धशक्षण संस्थाओं में अरक्षण की सुधिधा प्रदान की गइ है।
धनजी धशक्षण संस्थानों में अरक्षण से संबधं धत धििाद
 TMA पाइ फाईं डेशन बनाम कनावटक राज्य (2002) और आनामदार बनाम महाराष्ट्र राज्य

(2005) िाद में ईच्चतम न्यायालय ने कहा है फक सरकार द्वारा गैर-सहायता प्राि धनजी धशक्षण

संस्थानों में कोटा (अरक्षण) प्रणाली लागू नहीं कर सकती है, क्योंफक यह ऄनुच्छेद 19(1)(g) के
तहत प्रद्त मूल ऄधधकारों का ईल्लंघन है। आसके माध्यम से फकसी भी िृध्त (पेशा) के चयन की
स्ितंत्रता प्रदान की गयी है।
 93िाँ संधिधान संशोधन (2005), आस धनणवय को प्रधतस्थाधपत करने के धलए ऄधधधनयधमत फकया
गया।
 ईच्चतम न्यायालय ने ईपयुवि संधिधान संशोधन की िैधता को बनाए रखा। तदुपरांत, कें द्र सरकार

द्वारा संधिधान के ईपबंधों को प्रभािी बनाने के धलए कें द्रीय शैधक्षक संस्थान ऄधधधनयम, 2006

पाररत फकया गया धजसके माध्यम से IIT, IIM एिं ऄन्य शैक्षधणक संस्थानों में ऄन्य धपछड़े िगो

(OBC) के धलए 27% सीटें अरधक्षत की गइ।

 ए. के . ठाकु र बनाम भारतीय संघ (2008) िाद में न्यायालय ने यह धनणवय फदया फक राज्य एिं

धनजी धशक्षण संस्थाओं में ऄनुच्छेद 15(5) के तहत अरक्षण का प्रािधान मान्य हैं।

 राजस्थान के गैर-सहायता प्राि धनजी स्कू लों की सोसाआटी बनाम भारत संघ िाद, 2013: धनजी

गैर-सहायता प्राि संस्थानों में भी धशक्षा के ऄधधकार ऄधधधनयम, 2009 के तहत 25 % सीटों पर
अरक्षण की व्यिस्था को अरंभ करने की िैधता को ईच्चतम न्यायालय द्वारा बनाए रखा गया। आस
धनणवय में ईच्चतम न्यायालय द्वारा धनम्नधलधखत तकव फदए गए:
 धशक्षा को धिशुद् रूप से व्यािसाधयक ईद्यम के रूप में नहीं माना जा सकता।
 ऄनुच्छेद 21A राज्य पर एक दाधयत्ि है।

 धशक्षा का ऄधधकार, संस्था कें फद्रत ऄधधधनयम की ऄपेक्षा एक बाल कें फद्रत ऄधधधनयम है।

5.4.3. ऄनु च्छे द 15 और सामाधजक प्रगधत

 ऄनुच्छेद 14 धिधध के समक्ष समता को स्थाधपत करता है, फकन्तु ऄसमानता के ऐधतहाधसक तथ्य

िंधचत समूहों के धलए धिशेष ईपचार की अिश्यकता पर बल प्रदान करते हैं। ऄतः ऄनुच्छेद 15 में
समाज में हाधशए पर धस्थत िगों के पक्ष में प्रािधान पहले से ही मौजूद हैं।

184 www.visionias.in ©Vision IAS

Google it:- https://upscpdf.com


https://upscpdf.com << Download From >> https://upscpdf.com

 शैधक्षक और ऄन्य सुधिधाओं के संदभव में ऄनुसूधचत जाधत/ऄनुसूधचत जनजाधत और ऄन्य धपछड़ा
िगव के ईममीदिारों के पक्ष में ऄधधमानी (Preferantial) व्यिहार, ऄनुच्छेद 15 पर अधाररत
सामाधजक सुधार का ही एक रूप है। दूसरी ओर, ईच्चतम न्यायालय ने सामान्य सामाधजक कल्याण
को ध्यान में रखते हए, मंडल िाद में, अरक्षण को 50% तक सीधमत करते हए संतुलन स्थाधपत
फकया है। आसके ऄधतररि, सामाधजक-अर्वथक रूप से धपछड़े िगों से संबंधधत िाद में , ईच्चतम
न्यायालय ने क्रीमीलेयर का प्रािधान प्रस्तुत फकया है।
 ऄनुच्छेद 15 के अधार पर मधहलाओं और ईनकी सामाधजक प्रगधत के संदभव में, धनम्नधलधखत
अिश्यकताओं पर ध्यान फदया जाना चाधहए:
o ईच्चतम न्यायालय के ऄनुसार, धिशेष रूप से मधहलाओं के धलए पदों का अरक्षण ऄनुच्छेद
15 (3) के तहत िैध है क्योंफक यह राज्य कारविाइ के सकारात्मक अयाम को सममधलत करता
है।
o न्यायालय ने अपराधधक कानून (Criminal Law) और प्रफक्रयात्मक कानून (Procedural
Law) में मधहलाओं के पक्ष में फकए गए प्रािधानों को ईनकी सामाधजक धनबवलता के अलोक
में स्िीकार फकया गया।
o ऄनुच्छेद 15 में स्थानीय धनकायों और शैक्षधणक संस्थानों में मधहलाओं के धलए अरक्षण के
धिशेष ईपबंध फकए गए हैं।
o धिशाखा बनाम राजस्थान राज्य िाद (1997) में ईच्चतम न्यायालय द्वारा कायव-स्थल पर
यौन ईत्पीड़न को समाि करने हेतु ईपाय प्रस्तुत फकए सुझाएँ गए क्योंफक मधहलाओं के प्रधत
ईत्पीड़न ऄनुच्छेद 14, 15, और 21 का ईल्लंघन करता है।

5.5. ऄनु च्छे द 16 - लोक धनयोजन के धिषय में ऄिसर की समता का ऄधधकार

मूल पाठ
लोक धनयोजन के धिषय में ऄिसर की समता-
1. राज्य के ऄधीन फकसी पद पर धनयोजन या धनयुधि से संबंधधत धिषयों में सभी नागररकों के धलए
ऄिसर की समता होगी।
2. राज्य के ऄधीन फकसी धनयोजन या पद के संबंध में के िल धमव, मूलिंश, जाधत, ललग, ईद्भि,
जन्मस्थान, धनिास या आनमें से फकसी के अधार पर न तो कोइ नागररक ऄपात्र होगा और न ही
ईससे धिभेद फकया जाएगा।
3. आस ऄनुच्छेद की कोइ बात संसद को कोइ ऐसी धिधध बनाने से धनिाररत नहीं करेगी, जो फकसी
राज्य या संघ राज्य क्षेत्र की सरकार के या ईसमें के फकसी स्थानीय या ऄन्य प्राधधकारी के ऄधीन
िाले, फकसी िगव या िगों के पद पर धनयोजन या धनयुधि के संबंध में, ऐसे धनयोजन या धनयुधि से
पहले, ईस राज्य या संघ राज्य क्षेत्र के भीतर धनिास धिषयक कोइ ऄपेक्षा धिधहत करती है।
4. आस ऄनुच्छेद की कोइ बात राज्य को धपछड़े हए नागररकों के फकसी िगव के पक्ष में , धजनका
प्रधतधनधधत्ि राज्य की राय में राज्य के ऄधीन सेिाओं में पयावि नहीं है, धनयुधियों या पदों के
अरक्षण के धलए ईपबंध करने से धनिाररत नहीं करेगी।
(4-A) आस ऄनुच्छेद की कोइ बात राज्य को ऄनुसूधचत जाधतयों और जनजाधतयों के पक्ष में,
धजनका प्रधतधनधधत्ि राज्य की राय में राज्य के ऄधीन सेिाओं में पयाव ि नहीं है, राज्य के
ऄधीन सेिाओं में, फकसी िगव के ऄनुिती िररष्ठता के साथ प्रोन्नधत के मामलों में, अरक्षण के
धलए ईपबंध करने से धनिाररत नहीं करेगी।

185 www.visionias.in ©Vision IAS

Google it:- https://upscpdf.com


https://upscpdf.com << Download From >> https://upscpdf.com

(4-B) आस ऄनुच्छेद की कोइ बात राज्य को फकसी िषव में फकन्हीं न भरी गयी ऐसी ररधियों को,
जो खंड 4 या खंड 4(A) के ऄधीन फकए गए अरक्षण के धलए फकसी ईपबंध के ऄनुसार ईस
िषव में भरी जाने के धलए अरधक्षत हैं, फकसी ई्त रिती िषव या िषों में भरे जाने के धलए पृथक
िगव की ररधियों के रूप में धिचार करने से धनिाररत नहीं करेगी और ऐसे िगव की ररधियों
पर ईस िषव की ररधियों के साथ, धजसमें िे भरी जा रही हैं, ईस िषव की ररधियों की कु ल
संख्या के समबन्ध में पचास प्रधतशत अरक्षण की ऄधधकतम सीमा का ऄिधारण करने के धलए
धिचार नहीं फकया जाएगा।
5. आस ऄनुच्छेद की कोइ बात फकसी ऐसी धिधध के प्रितवन पर प्रभाि नहीं डालेगी जो यह ईपबंध
करती है फक फकसी धार्वमक या सांप्रदाधयक संस्था के कायवकलाप से संबंधधत कोइ पदधारी या ईसके
शासी धनकाय का कोइ सदस्य फकसी धिधशष्ट धमव का मानने िाला या धिधशष्ट संप्रदाय का ही हो।
धििरण
 ऄनुच्छेद 16(2) में ईधल्लधखत है फक राज्य धमव, मूलिंश, जाधत, ललग, ईद्भि, जन्मस्थान और
धनिास के अधार पर लोक धनयोजन के धिषय में धिभेद नहीं कर सकता है। ककतु राज्य ऄन्य
अधारों पर धिभेद करने के धलए स्ितंत्र है। ध्यान देने योग्य है फक यह ईपखडड, ऄनुच्छेद 15 में
धनर्ददष्ट अधारों में दो और अधारों को समाधिष्ट करता है- ईद्ि और धनिास।
 धनिास अरक्षण का अधार नहीं हो सकता है। हालांफक, ऄनुच्छेद 16(3) एक ऄपिाद है। राज्य या
संघ राज्य क्षेत्र के िल ऄपने धनिाधसयों के धलए कु छ पदों को अरधक्षत कर सकते है , हालांफक के िल
संसद ही आस संबंध में कानून बनाने में सक्षम है।
 ऄनुच्छेद 16(4) राज्य को फकन्हीं धपछड़े िगों के पक्ष में लोक धनयोजन में अरक्षण प्रदान करने के
धलए सक्षम बनाता है। धपछड़ा िगव में ऄनुसूधचत जाधत/ऄनुसूधचत जनजाधतयों और सामाधजक और
शैधक्षक रूप से धपछड़े िगव या ऐसा कोइ भी िगव शाधमल फकया जा सकता है, धजसे राज्य धपछड़ा
मानता हो। आस प्रकार, सकारात्मक कारविाइ की संभािना ऄनुच्छेद 15 की ऄपेक्षा ऄनुच्छेद 16 में
व्यापक है।
 ऄनुच्छेद 16(5) फकसी धार्वमक व्यिस्था के कु छ पदों के संबंध में धमव के अधार पर धिभेद का
प्रािधान करता है।
5.5.1. मं ड ल अयोग और ईसके पश्चात

मंडल अयोग को िषव 1979 में ऄनुच्छेद 340 के तहत, जनसंख्या के सामाधजक एिं शैधक्षक रूप से
धपछड़े िगों की जांच करने और ईनकी ईन्नधत के धलए ईपाय सुझाने के धलए गरठत फकया गया था।
अयोग ने जनसंख्या के लगभग 52% भाग को धपछड़ा हअ माना। िषव 1990 में िी.पी. लसह सरकार
द्वारा ‘ऄन्य धपछड़े िगों (OBC)’ के धलए सरकारी नौकररयों में 27% अरक्षण की व्यिस्था की गइ।
आंफदरा साहनी बनाम भारत संघ िाद (1992) में ईच्चतम न्यायालय ने ऄन्य धपछड़े िगों के धलए
अरक्षण प्रदान करने की सरकार की नीधत को सही ठहराया। यह मंडल िाद के नाम से प्रधसद् हअ।
आस धनणवय के तहत:
 न्यायालय ने धनणवय फदया फक ऄनु.16(4), ऄनु.16(1) का धिरोधी नहीं है। दोनों एक ही क्षेत्र में
कायवरत है। आसी अधार पर न्यायालय ने 27 प्रधतशत अरक्षण को िैध माना।
 अरक्षण के िल अरंधभक धनयुधि के समय पर फदया जाएगा, पदोन्नधत के धलए नहीं।
 आसमें क्रीमीलेयर से संबंधधत लोगों को अरक्षण से बाहर रखने का धसद्ांत फदया गया।
 आसने अरक्षण में फकसी जाधत को शाधमल करने ऄथिा ईसे बाहर करने के मापदंडों के धिषय में
धनणवय लेने के धलए सरकार को एक सांधिधधक धनकाय (राष्ट्रीय धपछड़ा िगव अयोग) के गठन का
धनदेश फदया। ितवमान में 102िें संधिधान संशोधन ऄधधधनयम के द्वारा राष्ट्रीय धपछड़ा िगव अयोग
को संिैधाधनक धनकाय का दजाव प्रदान कर फदया गया है।

186 www.visionias.in ©Vision IAS

Google it:- https://upscpdf.com


https://upscpdf.com << Download From >> https://upscpdf.com

प्रोन्नधत में अरक्षण का मुद्दा

 77िां संधिधान संशोधन के द्वारा ऄनुच्छेद 16 में एक नया खंड 4(a) ऄंतःस्थाधपत फकया गया।
आसके द्वारा पदोन्नधत में अरक्षण को िैध बनाया गया।
 िीरपाल लसह बनाम भारत संघ िाद (1995) में न्यायालय ने कहा था फक ऄनुसूधचत जाधत एिं
जनजाधत को प्रोन्नधत में अरक्षण नहीं फदया जाएगा।
 85िां संधिधान संशोधन द्वारा न्यायालय के आस धनणवय को धनष्प्रभािी कर फदया गया।

 नागराज िाद में, 85िें संधिधान संशोधन की िैधता पर प्रश्न फकया गया। ईच्चतम न्यायालय ने
माना फक संशोधन संधिधान के मूल ढांचे का ईल्लंघन नहीं करता है। ईच्चतम न्यायालय ने तीन
और फदशा-धनदेश फदए:
o यह ध्यान रखना होगा फक धजस िगव के धलए पदोन्नधत में अरक्षण की मांग की है, ईस िगव का
पयावि प्रधतधनधधत्ि नहीं हो।
o ऄनुच्छेद 335 के ऄंतगवत, आससे प्रशासन की दक्षता प्रभाधित नहीं होनी चाधहए।

 ऄधखल भारतीय अयुर्विज्ञान संस्थान फै कल्टी एसोधसएशन बनाम भारत संघ (2013) िाद में
ईच्चतम न्यायालय के पांच न्यायाधीशों की पीठ ने धनणवय फदया फक कु छ ऐसे कायव हैं धजसके धलए
के िल योग्यता एकमात्र मापदंड (सुपर स्पेशधलटी पदों हेत)ु होनी चाधहए। कें द्र सरकार ने आस
धनणवय के धिरुद् याधचका दायर की और समीक्षा याधचका की सुनिाइ के दौरान एक पांच
सदस्यीय पीठ ने यह धनणवय फदया फक सरकार, फै कल्टी के सुपर स्पेशधलटी पदों में अरक्षण हेतु

संधिधान में संशोधन के धलए स्ितंत्र है। आस संदभव में न्यायालय का पूिव धनणवय, ऐसे पदों पर
अरक्षण के धलए सरकार के धनणवय लेने की स्ितंत्रता पर कोइ प्रधतबन्ध अरोधपत नहीं करता है।
 न्यायमूर्वत टीएस ठाकु र एिं न्यायमूर्वत अर भानुमधत की खंडपीठ ने एस. पन्नीरसेल्िम एिं ऄन्य
बनाम तधमलनाडु (धसधिल ऄपील) तथा ऄन्य याधचकाओं को स्िीकार करते हए SC/STs को
प्रोन्नधत में अरक्षण और पररणामी िररष्ठता के मामले में मद्रास हाइकोटव के धनणवय को धनरस्त कर
फदया। खंडपीठ ने कहा फक धनयमों में पररणामी िररष्ठता का कोइ प्रािधान न होने के कारण ‘कै च

ऄप रूल्स’ लागू होगा।


प्रयोजनीयता
 लोक धनयोजन के धिषय में ऄिसर की समानता का ऄधधकार के िल भारतीय नागररकों के धलए
ईपलब्ध है।

5.6. ऄनु च्छे द 17 - ऄस्पृ श्यता का ऄं त

मूल पाठ

ऄस्पृश्यता का ऄंत .- "ऄस्पृश्यता" का ऄंत फकया जाता है और ईसका फकसी भी रूप में अचरण

धनधषद् फकया जाता है। "ऄस्पृश्यता" से ईपजी फकसी धनयोग्यता को लागू करना ऄपराध होगा जो
धिधध के ऄनुसार दंडनीय होगा।
धििरण
ऄस्पृश्यता देश में सभी रूपों में धनधषद् है। ईल्लेखनीय है फक संधिधान में 'ऄस्पृश्यता' शब्द को

पररभाधषत नहीं फकया गया है। ऄनुच्छेद 35 के तहत, संसद द्वारा आस ईपबंध को लागू करने के धलए
कइ ऄधधधनयम बनाए गए हैं:

187 www.visionias.in ©Vision IAS

Google it:- https://upscpdf.com


https://upscpdf.com << Download From >> https://upscpdf.com

5.6.1. ऄस्पृ श्यता की समाधि के धलए धिधभन्न ऄधधधनयम

नागररक ऄधधकार संरक्षण ऄधधधनयम, 1955


ऄस्पृश्यता (ऄपराध) ऄधधधनयम, 1955 का िषव 1976 में संशोधन करके नागररक ऄधधकार संरक्षण
ऄधधधनयम द्वारा आसका नाम पररिर्वतत कर फदया गया। यह ऄस्पृश्यता धिरोधी प्रािधानों को और
सशि करता है। ऄस्पृश्यता एक संज्ञेय ऄपराध (cognizable offence) (ऄथावत पुधलस ऄधधकारी,
मधजस्ट्रेट िारंट के धबना अरोपी को धगरफ्तार कर सकते हैं) तथा गैर-प्रशमय (non-compoundable)
(ऄथावत ऐसे िाद धजन्हें िापस नहीं धलया जा सकता, चाहे दोनों पक्षों में समझौता हो चुका हो; आन
मामलों में राज्य एक पक्ष बन जाता है) ऄपराध है। यह क़ानून त्िररत सुनिाइ के धलए एक धिशेष
न्यायालय का प्रािधान करता है।
ऄनुसूधचत जाधत और ऄनुसूधचत जनजाधत समुदाय को न्याय प्रदान करने और ईत्पीड़न के धशकार
लोगों को राहत, पुनिावस ईपलब्ध कराने के साथ ही, ईनको गररमापूणव जीिन जीने का ऄधधकार प्रदान
करने के धलए ऄनुसधू चत जाधत और ऄनुसधू चत जनजाधत (ऄत्याचार धनिारण) ऄधधधनयम, 1989
पाररत फकया गया।
ऄनुसधू चत जाधत और ऄनुसधू चत जनजाधत (ऄत्याचार धनिारण) संशोधन ऄधधधनयम, 2015 का
ईद्देश्य ऄनुसूधचत जाधत और ऄनुसूधचत जनजाधत के धिरुद् ऄत्याचार की रोकथाम तथा आसके धलए
और ऄधधक कठोर प्रािधानों को सुधनधश्चत करना है। यह ऄधधधनयम मूल ऄधधधनयम में एक संशोधन है
और ऄनुसूधचत जाधत और ऄनुसूधचत जनजाधत (ऄत्याचार धनिारण) ऄधधधनयम, 1989 के साथ
संशोधन प्रभािों के साथ लागू फकया गया है। आसके ऄंतगवत धनम्नधलधखत प्रािधान फकए गए है:
1. ऄपराधों की धिस्तृत सीमा: ऄनुसूधचत जाधतयों और ऄनुसूधचत जनजाधतयों के धिरुद् फकए जाने
िाले नए ऄपराधों में ऄनुसूधचत जाधत तथा ऄनुसूधचत जनजाधत के लोगों के धसर और मूंछ की
बालों का मुंडन कराने और आसी तरह ऄनुसूधचत जाधतयों और जनजाधतयों के लोगों के सममान के
धिरुद् फकए गए कृ त हैं। ऄत्याचारों में समुदाय के लोगों को जूते की माला पहनाना, ईन्हें लसचाइ
om

सुधिधाओं तक जाने से रोकना या िन ऄधधकारों से िंधचत करने रखना, मानि और पशु नरकं काल
l.c
ai

को धनपटाने और लाने-ले जाने के धलए तथा बाध्य करना, कि खोदने के धलए बाध्य करना, धसर
gm
@

पर मैला ढोने की प्रथा का ईपयोग और ऄनुमधत देना, ऄनुसूधचत जाधतयों और ऄनुसूधचत


80
h1

जनजाधतयों की मधहलाओं को देिदासी के रूप में समर्वपत करना, जाधत सूचक गाली देना, जादू-
ng
si
sh

टोना ऄत्याचार को बढ़ािा देना, सामाधजक और अर्वथक बधहष्कार करना, चुनाि लड़ने में
ar
ad

ऄनुसूधचत जाधतयों और ऄनुसूधचत जनजाधतयों के ईममीदिारों को नामांकन दाधखल करने से


रोकना, ऄनुसूधचत जाधतयों और ऄनुसूधचत जनजाधतयों की मधहलाओं को िस्त्र हरण कर अहत
करना, ऄनुसूधचत जाधतयों और ऄनुसूधचत जनजाधतयों के फकसी सदस्य को घर, गांि और अिास
छोड़ने के धलए बाध्य करना, ऄनुसूधचत जाधतयों और ऄनुसूधचत जनजाधतयों के पूजनीय िस्तुओं
को धिरुधपत करना, ऄनुसूधचत जाधतयों और ऄनुसूधचत जनजाधतयों के सदस्य के धिरुद् यौन
दुव्यविहार करना, यौन दुव्यविहार भाि से ईन्हें छू ना और भाषा का ईपयोग करना है।
2. अहत करना और धमकाना: ऄनुसूधचत जाधतयों और ऄनुसूधचत जनजाधतयों के सदस्य को अहत
करने, ईन्हें दुखद रूप से अहत करने, धमकाने और ऄपहरण करने जैसे ऄपराधों को, धजनमें 10
िषव से कम की सजा का प्रािधान है, ईन्हें ऄत्याचार धनिारण ऄधधधनयम में ऄपराध के रूप में
शाधमल करना। ऄभी ऄत्याचार धनिारण ऄधधधनयम के तहत ऄनुसूधचत जाधत और जनजाधत के
लोगों पर फकए गए ऄत्याचार मामलों में 10 िषव और ईससे ऄधधक की सजा िाले ऄपराधों को ही
ऄपराध माना जाता है।

188 www.visionias.in ©Vision IAS

Google it:- https://upscpdf.com


https://upscpdf.com << Download From >> https://upscpdf.com

3. मामलों का तेजी से धनपटान: मामलों का तीव्र धनपटान के धलए ऄत्याचार धनिारण ऄधधधनयम के
ऄंतगवत अने िाले ऄपराधों में धिशेष रूप से मुकदमा चलाने के धलए धिशेष न्यायालय बनाना
और धिशेष लोक ऄधभयोजक को धनर्ददष्ट करना।
4. प्रत्यक्ष संज्ञान लेने की शधि: धिशेष ऄदालतों को ऄपराध का प्रत्यक्ष संज्ञान लेने की शधि प्रदान
करना और जहां तक संभि हो, अरोप पत्र दाधखल करने की धतधथ से दो महीने के ऄंदर सुनिाइ
पूरी करना।
5. ऄधतररि ऄध्याय: पीधड़तों तथा गिाहों के ऄधधकारों पर ऄधतररि ऄध्याय शाधमल करना अफद।
6. जानबूझकर की गइ फढलाइ की स्पष्ट पररभाषा: धशकायत दजव होने से लेकर एिं ऄधधधनयम के
ऄंतगवत कायव की ईपेक्षा के अयामों को लेते हए हर स्तर के सरकारी कमवचाररयों के धलए
'जानबूझकर की गइ फढलाइ' पद की स्पष्ट पररभाषा धनधावररत करना।
7. ऄपराध की ऄन्य प्रकल्पनाएं: ऄपराधों में प्रकल्पनाओं का शाधमल फकया जाना - यफद ऄधभयुि
पीधड़त या ईसके पररिार से पररधचत है, तो जब तक आसके धिपरीत धसद् न फकया जाए न्यायालय
यह मानेगा फक ऄधभयुि पीधड़त की जाधत ऄथिा जनजातीय पहचान के बारे में जानता था।
5.7. ऄनु च्छे द 18 - ईपाधधयों का ऄं त

मूल पाठ
1. राज्य सेना या धिद्या संबंधी सममान के धसिाय और कोइ ईपाधध प्रदान नहीं करेगा।
2. भारत का कोइ नागररक फकसी धिदेशी राज्य से कोइ ईपाधध स्िीकार नहीं करेगा।
3. कोइ व्यधि जो भारत का नागररक नहीं है, राज्य के ऄधीन लाभ या धिश्वास के फकसी पद को
धारण करते हए फकसी धिदेशी राज्य से कोइ ईपाधध राष्ट्रपधत की सहमधत के धबना स्िीकार नहीं
करेगा ।
4. राज्य के ऄधीन लाभ या धिश्वास का पद धारण करने िाला कोइ व्यधि फकसी धिदेशी राज्य से या ईसके
ऄधीन फकसी रूप में कोइ भेंट, ईपलधब्ध या पद राष्ट्रपधत की सहमधत के धबना स्िीकार नहीं करेगा।
धििरण
 यह राज्य, नागररकों और गैर-नागररकों के ऄधधकारों पर एक प्रधतबंध अरोधपत करता है। राज्य
सैन्य या शैक्षधणक ईपाधध को छोड़कर कोइ ऄन्य ईपाधध प्रदान नहीं करेगा। भारत के फकसी
नागररक को फकसी धिदेशी राज्य से कोइ ईपाधध स्िीकार करने की ऄनुमधत नहीं है। भारतीय
राज्य के ऄधीन लाभ या धिश्वास का पद धारण करने िाले धिदेशी नागररक को राष्ट्रपधत की
ऄनुमधत के धबना फकसी धिदेशी राज्य से फकसी भी प्रकार की ईपाधध, भेंट, ईपलधब्ध या पद
स्िीकार करने की ऄनुमधत नहीं है।
भारत रत्न और पद्म पुरस्कारों से संबधं धत िाद
 बालाजी राघिन िाद में, ईच्चतम न्यायालय ने राज्य को भारत रत्न और पद्म पुरस्कार देने की
ऄनुमधत प्रदान की है, फकन्तु साथ ही यह स्पष्ट फकया है फक आन्हें नाम के प्रत्यय या ईपसगव के तौर
पर ईपयोग नहीं फकया जा सकता।
5.8. ऄनु च्छे द 19 - स्ितं त्र ता का ऄधधकार

मूल पाठ
िाक् -स्िातंत्र्य अफद धिषयक कु छ ऄधधकारों का संरक्षण-
1. सभी नागररकों को--
 िाक् एिं ऄधभव्यधि की स्ितंत्रता का,
 शांधतपूिवक और धनरायुध सममेलन का,
 संगम या संघ बनाने का,

189 www.visionias.in ©Vision IAS

Google it:- https://upscpdf.com


https://upscpdf.com << Download From >> https://upscpdf.com

 भारत के राज्यक्षेत्र में सिवत्र ऄबाध संचरण का,


 भारत के राज्यक्षेत्र के फकसी भाग में धनिास करने और बस जाने का,
 कोइ िृध्त , ईपजीधिका, व्यापार या कारोबार करने का, ऄधधकार होगा।
2. खंड (1) के ईपखंड (a) की कोइ बात ईि ईपखंड द्वारा फदए गए ऄधधकार के प्रयोग पर भारत की

प्रभुता और ऄखंडता, राज्य की सुरक्षा, धिदेशी राज्यों के साथ मैत्रीपूणव संबंधों, लोक व्यिस्था,
धशष्टाचार या सदाचार के धहतों में ऄथिा न्यायालय-ऄिमान, मानहाधन या ऄपराध-ईद्दीपन के
संबंध में युधियुि धनबंधन जहाँ तक कोइ धिद्यमान धिधध ऄधधरोधपत करती है िहाँ तक ईसके
प्रितवन पर प्रभाि नहीं डालेगी या िैसे धनबंधन ऄधधरोधपत करने िाली कोइ धिधध बनाने से राज्य
को धनिाररत नहीं करेगी।
3. ईि खंड के ईपखंड (b) की कोइ बात ईि ईपखंड द्वारा फदए गए ऄधधकार के प्रयोग पर, भारत
की प्रभुता और ऄखंडता या लोक व्यिस्था के धहतों में युधियुि धनबंधन जहाँ तक कोइ धिद्यमान
धिधध ऄधधरोधपत करती है िहाँ तक ईसके प्रितवन पर प्रभाि नहीं डालेगी या िैसे धनबंधन
ऄधधरोधपत करने िाली कोइ धिधध बनाने से राज्य को धनिाररत नहीं करे गी।
4. ईि खंड के ईपखंड (c) की कोइ बात ईि ईपखंड द्वारा फदए गए ऄधधकार के प्रयोग पर भारत की

प्रभुता और ऄखंडता या लोक व्यिस्था या सदाचार के धहतों में युधियुि धनबवन्धन, जहाँ तक कोइ

धिद्यमान धिधध ऄधधरोधपत करती है, िहाँ तक ईसके प्रितवन पर प्रभाि नहीं डालेगी या िैसे
धनबवन्धन ऄधधरोधपत करने िाली कोइ धिधध बनाने से राज्य को धनिाररत नहीं करेगी।
5. ईि खंड के ईपखंड (d) और (e) की कोइ बात ईि ईपखंडों द्वारा फदए गए ऄधधकारों के प्रयोग
पर साधारण जनता के धहतों में या फकसी ऄनुसूधचत जनजाधत के धहतों के संरक्षण के धलए
युधियुि धनबवन्धन जहाँ तक कोइ धिद्यमान धिधध ऄधधरोधपत करती है िहाँ तक ईसके प्रितवन पर
प्रभाि नहीं डालेगी या िैसे धनबवन्धन ऄधधरोधपत करने िाली कोइ धिधध बनाने से राज्य को
धनिाररत नहीं करेगी।
6. ईि खंड के ईपखंड (g) की कोइ बात ईि ईपखंड द्वारा फदए गए ऄधधकार के प्रयोग पर साधारण
जनता के धहतों में युधियुि धनबवन्धन जहाँ तक कोइ धिद्यमान धिधध ऄधधरोधपत करती है िहाँ
तक ईसके प्रितवन पर प्रभाि नहीं डालेगी या िैसे धनबवन्धन ऄधधरोधपत करने िाली कोइ धिधध
बनाने से राज्य को धनिाररत नहीं करेगी और धिधशष्टतया ईि ईपखंड की कोइ बात--
o कोइ िृध्त , ईपजीधिका, व्यापार या कारोबार करने के धलए अिश्यक िृध्त क या तकनीकी
ऄहवताओं से, या
o राज्य द्वारा या राज्य के स्िाधमत्ि या धनयंत्रण में फकसी धनगम द्वारा कोइ व्यापार, कारोबार,
ईद्योग या सेिा, नागररकों का पूणवतः या भागतः ऄपिजवन करके या ऄन्यथा, चलाए जाने से,
जहाँ तक कोइ धिद्यमान धिधध संबंध रखती है िहाँ तक ईसके प्रितवन पर प्रभाि नहीं डालेगी या आस
प्रकार संबंध रखने िाली कोइ धिधध बनाने से राज्य को धनिाररत नहीं करेगी।
धििरण
 संधिधान के ऄनुच्छेद 19 के तहत प्रदत ऄधधकारों की प्रकृ धत सकारात्मक हैं तथा यह के िल भारत
के नागररक को प्राि हैं। प्रारंभ में आस ऄनुच्छेद के ऄंतगवत नागररकों को सात स्ितंत्रताएं प्राि थीं।
लेफकन, 44िाँ संधिधान संशोधन ऄधधधनयम, 1978 द्वारा ऄनुच्छेद 19(1)(f) (संपधत के ऄजवन ,
धारण और व्ययन का ऄधधकार) समाि कर फदया गया है। ितवमान में यह ऄनुच्छेद नागररकों को
छः प्रकार की स्ितंत्रताएं प्रदान करता हैं।

190 www.visionias.in ©Vision IAS

Google it:- https://upscpdf.com


https://upscpdf.com << Download From >> https://upscpdf.com

 ऄनुच्छेद 19 को भारतीय लोकतांधत्रक शासन का अधार है। हालांफक, कोइ भी स्ितंत्रता


ऄसीधमत (अत्यंधतक) नहीं है और आन पर ‘युधियुि’ प्रधतबंध अरोधपत करके कटौती की जा
सकती है।
 युधियुि प्रधतबंध के िल धिधध के प्राधधकार द्वारा अरोधपत फकया जा सकता है , न फक
कायवपाधलका की कारविाइ से। यह न्याधयक पुनर्विलोकन के ऄधीन है। प्रधतबंध के िल संधिधान में
ईधल्लधखत अधारों पर लगाया जा सकता है।
ऄनुच्छेद 19(1) के द्वारा धनम्नधलधखत स्ितंत्रताएं प्रदान की गइ हैं:
ऄनुच्छेद 19(1)(a) - िाक् एिं ऄधभव्यधि की स्ितंत्रता
 िाक् एिं ऄधभव्यधि की स्ितंत्रता का ऄथव है प्रत्येक नागररक ऄपने धिचारों और धिश्वासों को
धनबावध रूप से मौधखक, धलधखत ऄथिा मुद्रण और धचत्रण के द्वारा ऄधभव्यि करने के धलए स्ितंत्र
है। सिोच्च न्यायालय ने एक मामले की सुनिाइ के दौरान ‘मौन’ (Silence) को भी ऄधभव्यधि का
रूप माना है।

िाक् एिं ऄधभव्यधि की स्ितंत्रता

प्रधतबंध के अधार व्युत्पन्न महत्िपूणव ऄधधकार

 भारत की संप्रभुता और ऄखंडता  सूचना का ऄधधकार


 प्रेस की स्ितंत्रता
 राज्य की सुरक्षा,
 धनजता का ऄधधकार
 धिदेशी राष्ट्रों के साथ मैत्रीपूणव संबंध,  राष्ट्रीय ध्िज फहराने का ऄधधकार
 लोक व्यिस्था,  प्रदशवन एिं धिरोध का ऄधधकार, लेफकन हड़ताल का
 धशष्टाचार या सदाचार ऄधधकार नहीं

 न्यायालय की ऄिमानना,  मौन रहने का ऄधधकार

 मानहाधन; तथा
 ऄपराध-ईद्दीपन।

5.8.1. सू च ना का ऄधधकार - सू च ना का ऄधधकार ऄधधधनयम, 2005 के तहत एक कानू नी


ऄधधकार
 यह एक नागररक कें फद्रत कानून है जो सरकार से सूचना प्राि करने की गारंटी प्रदान करता है, जब
तक फक ऐसी सूचना देश की सुरक्षा के धलए खतरा न हो। संबंधधत धिभाग द्वारा समयबद् तरीके से
सूचना प्रदान की जाती है। यह ऄधधधनयम सूचना अयुिों िाली एक संस्थागत तंत्र का भी
प्रािधान करता है जो फकसी नागररक द्वारा मांगी गइ सूचना प्राि न होने पर ईसकी धशकायत से
संबंधधत मामलों की जांच कर सकता है।
 धद्वतीय प्रशासधनक सुधार अयोग (ARC) ने आस ऄधधधनयम को "सुशासन के धलए एक प्रमुख
अधार" माना है। धिशेषज्ञों का मानना है फक यह भारतीय लोकतंत्र के धद्वतीय चरण की शुरुअत
को प्रदर्वशत करता है।
 यह शासकीय गुि बात ऄधधधनयम, 1923 से व्यापक पररितवनों को प्रदर्वशत करता है जहां ऄब
के िल सीधमत सूचना को गोपनीयता के ऄधीन रखा जाएगा। आसके ऄधतररि, सरकारी धिभागों
को स्ितः संज्ञान लेते हए और ऄग्रसफक्रय रूप से सूचना प्रदान करने हेतु प्रोत्साधहत फकया गया है।

191 www.visionias.in ©Vision IAS

Google it:- https://upscpdf.com


https://upscpdf.com << Download From >> https://upscpdf.com

 कु छ संगठनों को संप्रभुता, क्षेत्रीय ऄखंडता और ऄन्य कारणों के सन्दभव में ऄधधधनयम के दायरे से
बाहर रखा गया है।
 क्षेत्रीय भाषाओं में भी सूचना प्राि की जा सकती है और सूचना प्राि करने का कोइ कारण बताने
की अिश्यकता नहीं है।

 सािवजधनक प्राधधकरण से सूचना प्राि की जा सकती है न फक फकसी धनजी संस्था से। हालाँफक, यफद
फकसी धनजी धनकाय की कायव की प्रकृ धत मुख्यतः सािवजधनक है ऄथिा िह सरकार द्वारा पयावि
तौर पर धि्त पोधषत है तो ईससे भी सूचनाएं प्राि की जा सकती हैं।
 सािवजधनक प्राधधकरण का ऄथव संिैधाधनक धनकाय, िैधाधनक धनकाय और िैसे गैर-सरकारी
संगठन हैं धजन्हें पयावि रूप से सरकारी धि्त पोषण प्राि हो रहा हो।

5.8.2. प्रे स की स्ितं त्र ता की धस्थधत

 संयुि राज्य ऄमेररका जैसे कइ देशों के धिपरीत, जहां प्रेस की स्ितंत्रता की गारंटी का ऄलग से
कोइ प्रािधान नहीं है,
 भारत में साकल पेपर धमल्स बनाम भारत संघ िाद (1962) में ईच्चतम न्यायालय ने कहा फक प्रेस

की स्ितंत्रता, ऄनुच्छेद 19(1)(a) के तहत ‘ऄधभव्यधि की स्ितंत्रता’ में शाधमल है। ककतु आस
स्ितंत्रता पर भी ऄनुच्छेद 19(2) के तहत युधियुि धनबंधन लगाए जा सकते हैं।
 बृज भूषण िाद में ईच्चतम न्यायालय ने स्पष्ट फकया फक मीधडया पर कोइ पूिव सेंसरधशप अरोधपत
नहीं है, ऄथावत्, फकसी पूिावनुमधत की अिश्यकता नहीं है।
 44िें संधिधान संशोधन ऄधधधनयम, 1976 द्वारा ऄनुच्छेद 361A को जोड़ा गया है; जो एक
व्यधि को संसद और राज्य धिधानमंडलों की कायविाधहयों को प्रकाधशत करने के संबंध में संरक्षण
प्रदान करता है।
 आंधडयन एक्सप्रेस िाद (1985) में, यह स्पष्ट फकया गया फक प्रेस की स्ितंत्रता में धनम्नधलधखत
सधममधलत हैं:
o सूचना का ऄधधकार
o प्रकाधशत करने का ऄधधकार
o प्रसाररत करने का ऄधधकार
संधिधान की कायवप्रणाली की समीक्षा हेतु गरठत राष्ट्रीय अयोग (NCRWC) ने ऄनुशंसा की है फक प्रेस
की स्ितंत्रता को स्पष्ट स्िरूप प्रदान फकया जाना चाधहए तथा आसे ऄधभव्यधि की स्ितंत्रता में धनधहत
मानकर, आसकी ऄिहेलना नहीं करनी चाधहए।

ऄनुच्छेद: 19(1)(b) शांधतपूणव सममेलन की स्ितंत्रता


 ऄनुच्छेद 19(1)(b) के ऄंतगवत नागररकों को शांधतपूणव और धनरायुध सममेलन करने का ऄधधकार

प्राि है। आस ऄधधकार के कारण नागररकों को सािवजधनक सभा करने, प्रदशवन करने एिं जुलूस
धनकालने से प्रधतबंधधत नहीं फकया जा सकता है।
 आस पर भारत की संप्रभुता और ऄखंडता या लोक व्यिस्था के धहत में युधियुि प्रधतबंध अरोधपत
फकए जा सकते हैं।
ऄनुच्छेद: 19(1)(c) संगम या संघ बनाने का ऄधधकार
 आसके तहत भारत के सभी नागररकों को संगम या संघ बनाने का ऄधधकार फदया गया है। यह संघ
सांस्कृ धतक, राजनीधतक, अर्वथक एिं शैक्षधणक हो सकता है। आस ऄधधकार के तहत राजनीधतक

दल, कं पधनयाँ, सहकारी संघ, मजदूर संघ, क्लब अफद बनाने की शधि प्राि होती है।

192 www.visionias.in ©Vision IAS

Google it:- https://upscpdf.com


https://upscpdf.com << Download From >> https://upscpdf.com

 सशस्त्र सेनाओं, पुधलस, अफद को भी संघ बनाने का ऄधधकार है, हालाँफक आन्हें के िल सांस्कृ धतक

संघ बनाने की ऄनुमधत है न की राजनीधतक संघ बनाने की।


 सरकारी ऄधधकाररयों के िाद में हड़ताल का ऄधधकार: औद्योधगक धििाद ऄधधधनयम के तहत ट्रेड
यूधनयनों को कु छ धनधश्चत पररधस्थधतयों में, हड़ताल करने का ऄधधकार प्राि है। हालांफक, ईच्चतम

न्यायालय ने सरकारी ऄधधकाररयों के धलए कहा है फक, जब संचार के ऄन्य सभी चैनल ऄसफल

हो जाएं, के िल तभी हड़ताल का ऄधधकार एक ऄंधतम ईपाय के रूप में ईपलब्ध है। हालांफक, आसे
मूल ऄधधकारों के ऄंतगवत सधममधलत नहीं समझा जा सकता है। ऄत: सरकार ऐसी पररधस्थधतयों में
अिश्यक सेिा ऄनुरक्षण कानून (Essential Services Maintenance Act) लागू कर हड़ताल

िापस लेने के धलए बाध्य कर सकती है।

 ईच्चतम न्यायालय ने टी.के . रंगराजन बनाम तधमलनाडु राज्य िाद (2003) में कहा है फक
सरकारी ऄधधकाररयों को हड़ताल का मूल ऄधधकार प्राि नहीं है।
ऄनुच्छेद 19(1)(d) भारत के राज्यक्षेत्र में सिवत्र ऄबाध संचरण का ऄधधकार

ऄनुच्छेद 19(1)(d) के द्वारा भारत के नागररक को भारत के राज्यक्षेत्र में सिवत्र ऄबाध संचरण का

ऄधधकार प्राि है। हालाँफक, जन-साधारण के धहत में या ऄनुसूधचत जनजाधत के धहत में ऄबाध भ्रमण
(संचरण) पर युधियुि प्रधतबंध अरोधपत फकए जा सकते हैं। यह प्रधतबंध ऄनुसूधचत जनजाधत की
संस्कृ धत, भाषा, रीधत-ररिाज अफद को सुरधक्षत रखने के ईद्देश्य से लगाया गया है।

ऄनुच्छेद: 19(1)(e) भारत के राज्यक्षेत्र के फकसी भाग में धनिास करने और बस जाने का ऄधधकार
 यह भारतीय नागररकों को भारत के राज्यक्षेत्र के फकसी भाग में धनिास करने और बस जाने का
ऄधधकार प्रदान करता है। धिधशष्ट पररधस्थधतयों में, जन-साधारण या ऄनुसूधचत जाधत के धहत मे,
आस पर युधियुि प्रधतबंध अरोधपत फकए जा सकते हैं। यह देश के अंतररक ऄिरोधों को समाि
कर राष्ट्रिाद को प्रोत्साधहत करता है।
19(1)(d) और 19(1)(e) पर अरोधपत प्रधतबन्ध

 19(1)(d) भारतीय नागररकों को देश के राज्यक्षेत्र में ऄबाध संचरण का और 19(1)(e) भारत के
राज्यक्षेत्र के फकसी भाग में धनिास करने और बसने का ऄधधकार प्रदान करता है। आन ऄधधकारों के
प्रदान करने का अधार भारत का ‘एकल क्षेत्र’ होने की संकल्पना पर अधाररत है।
 दोनों ऄनुच्छेद परस्पर संबद् हैं तथा िास्ति में एक-दूसरे का ऄनुसरण करते हैं। धनम्नधलधखत कु छ
प्रधतबंध हैं, जो आन पर अरोधपत हैं:
o लोक व्यिस्था बनाए रखने के धलए: यफद ऄधधकाररयों को संदह
े है फक फकसी व्यधि के संचरण
से एक क्षेत्र में लोक व्यिस्था में खतरा ईत्पन्न होने की संभािना है, तो ईसके आस ऄधधकार

पर प्रधतबंध लगाया जा सकता है। हालांफक, यह प्रधतबंध दमनकारी या मनमाना नहीं हो


सकता।
o सुरक्षा कारणों के धलए: दो पधहया िाहन सिारों के धलए हेलमेट धनधावररत फकया जा सकता
है।
 फकसी राज्यक्षेत्र से नागररक के धिरुद् धनष्कासन का अदेश , यफद ईसे एक ऄसामाधजक तत्ि माना

जाता है। ईदाहरण के धलए, एक व्यधि को राज्य से धनष्काधसत फकया जा सकता है यफद िह फकसी
िाद में गिाह को डरा-धमका रहा हो।
 ऄनुसूधचत जनजाधतयों के धहतों के संरक्षण हेतु।

193 www.visionias.in ©Vision IAS

Google it:- https://upscpdf.com


https://upscpdf.com << Download From >> https://upscpdf.com

ऄनुच्छेद 19(1)(f) के तहत भारत के नागररक को संपध्त के ऄजवन , धारण और व्ययन का ऄधधकार
प्राि था। सामाधजक और अर्वथक समानता लाने के ईद्देश्य से आस ऄनुच्छेद को 44िाँ संधिधान
ऄधधधनयम, 1978 द्वारा मूल ऄधधकार से हटा फदया गया है। आसे ऄनुच्छेद 300A में सामान्य कानूनी
ऄधधकार के तहत रखा गया है।
ऄनुच्छेद: 19(1)(g) कोइ िृध्त , ईपजीधिका, व्यापार या कारोबार करने का ऄधधकार
 ऄनुच्छेद 19(1)(g) भारत के सभी नागररकों को कोइ भी िृध्त , ईपजीधिका, व्यापार या
कारोबार करने का ऄधधकार प्रदान करता है। हालाँफक राज्य जनधहत में आस ऄधधकार पर
युधियुि धनबंधन अरोधपत लगा सकती है, जैसे- नशीली दिाओं या शराब जैसे ऄधहतकर,
जोधखमपूणव और खतरनाक िस्तुओं का व्यापार करने पर कु छ प्रधतबंध है।
 राज्य फकसी िृध्त या कारोबार के धलए अिश्यक िृध्त क (Professional) या तकनीकी ऄहवता
(Technical qualifications) धनधावररत कर सकता है।

5.8.3. ऄनु च्छे द 19 के तहत ऄधधकारों से सं बं धधत मु द्दे

5.8.3.1. िाक् स्िातं त्र्य और धसधिल से ि क: 19 (1) (a)


 ईच्चतम न्यायालय के ऄनुसार, धसधिल सेिकों के िाक् स्िातंत्र्य को ऄनुशासन के सन्दभव में सीधमत
या ईनमें कमी की जा सकती है, हालांफक ऄनुच्छेद 19 (2) में आस तरह के प्रधतबंध का ईल्लेख नहीं
फकया गया है। सेिाओं के भीतर ऄनुशासन हेतु सेिा धनयम अिश्यक हैं। यहां ईद्देश्य धसधिल
सेिकों की िाक् स्िातंत्र्य पर ऄंकुश अरोधपत करना नहीं है , बधल्क यह सुधनधश्चत करना है फक िे
प्रभािी ढंग से ऄपने कतवव्यों का धनिवहन करने में सक्षम हो सकें । आसधलए, संगठनात्मक
कायवप्रणाली और िाक् स्िातंत्र्य के मध्य संतुलन को बनाए रखना अिश्यक है। हालाँफक, ऐसे
प्रधतबंध फकसी संगठन में धनिावधचत प्रधतधनधधयों पर लागू नहीं होते हैं क्योंफक िे लोगों का
प्रधतधनधधत्ि करते हैं।

5.8.3.2. 19 (1) (a) और 19 (2) - हेट स्पीच


 हेट स्पीच, िाक् एिं ऄधभव्यधि की स्ितंत्रता के धलए एक जरटल चुनौती है। हालांफक, यह प्रश्न
बना हअ है फक िाक् एिं ऄधभव्यधि की स्ितंत्रता पर ऄंकुश अरोधपत फकए धबना फकसे हेट स्पीच
के रूप में स्िीकार फकया जाना चाधहए। यद्यधप “धिधध अयोग” द्वारा आसे "नस्ल, नृजाधतयता,
ललग, यौन ऄधभधिन्यास, धार्वमक धिश्वास और पसंद" के रूप में पररभाधषत व्यधियों के एक समूह
के धिरुद् मुख्य रूप से घृणा के धलए ईकसाने के रूप में पररभाधषत फकया गया है। आस प्रकार, "हेट
स्पीच के तहत कोइ भी शब्द धलधखत या मौधखक, संकेत, श्रिण या दृश्य रूप में फकसी व्यधि को
भय या धमकाने ऄथिा लहसा के धलए ईकसाने की मंशा से फकए गए कृ त शाधमल हैं।
 कें द्र द्वारा गरठत टी. के . धिश्वनाथन सधमधत ने हेट स्पीच के धलए कठोर प्रािधान लागू फकए जाने
की धसफाररश की है।

5.8.3.3. 19 (1) (a) और 19 (2) - मानहाधन


 िषव 2016 में, ईच्चतम न्यायालय द्वारा देश के औपधनिेधशक युग के अपराधधक मानहाधन कानूनों
की संिैधाधनक िैधता को यथाित रखते हए यह धनणवय फदया फक िाक् स्िातंत्र्य के ऄधधकार एिं
मानहाधन के मध्य फकसी प्रकार का टकराि नहीं हैं। भारतीय दंड संधहता की धारा 499 और 500
के तहत मानहाधन एक दंडनीय ऄपराध है। मानहाधन िाले कृ त्यों के ऄंतगवत "पढ़े या बोले जाने
िाले शब्द" संकेत या दृश्य धनरूपण/साक्ष्य शाधमल हो सकते हैं, जो पधब्लक डोमेन में प्रकाधशत या
सािवजधनक फकए गए हों। आस ऄपराध के धलए दो िषव तक के कारािास, जुमावना या दोनों की सजा
का प्रािधान है।

194 www.visionias.in ©Vision IAS

Google it:- https://upscpdf.com


https://upscpdf.com << Download From >> https://upscpdf.com

 यह कहा गया है फक ऄसहमधत व्यि करना अिश्यक है, लेफकन यह फकसी व्यधि की प्रधतष्ठा को
क्षधत पहंचाने का ऄधधकार प्रदान नहीं करता है। ऄधभव्यधि की स्ितं त्रता के व्यापक और धिस्तृत
दायरे के बािजूद, सभी ऄधधकारों के समान, िाक् एिं ऄधभव्यधि की स्ितंत्रता का ऄधधकार

धनरपेक्ष नहीं है। यह युधियुि धनबंधनों के ऄधीन है। न्यायालय ने कहा फक फकसी व्यधि की

प्रधतष्ठा भारतीय संधिधान के ऄनुच्छेद 21 के तहत प्रद्त जीिन के ऄधधकार का एक ऄधभन्न ऄंग
है। फकसी की प्रधतष्ठा को ऄन्य की िाक् स्िातंत्र्य द्वारा सीधमत फकए जाने की ऄनुमधत नहीं दी जा
सकती है।

5.8.3.4. फफल्मों और पु स्तकों पर प्रधतबं ध : 19 (1) (a)

 कु छ िगों द्वारा ऄपने धलए ऄपमानजनक समझे जाने के कारण भारत में फकसी पुस्तक, काटूवन या

फफल्म के प्रकाशन को लेकर धनयधमत धिरोध और लहसा होती रहती है। आनके द्वारा लहसा, धिरोध-
प्रदशवन और बंद का सहारा धलया जाता है धजसके पररणामस्िरूप जीिन और संपध्त को नुकसान
पहँचता है। पररणामतः सरकारों द्वारा प्राय: कानून और व्यिस्था की समस्या का हिाला देते हए
कला की आस प्रकार की ऄधभव्यधि पर प्रधतबंध अरोधपत फकया जाता है। यह प्रत्यक्ष रूप से
कलाकारों की िाक् एिं ऄधभव्यधि की स्ितंत्रता पर ऄंकुश लगाने और देश में िाक् -स्िातंत्र्य के
ऄधधकार का हनन है।

 आस संदभव में, कइ धनणवयों में ईच्चतम न्यायालय ने ऐसे प्रधतबंधों को ऄिैध माना है। कानून व्यिस्था
बनाए रखना सरकार का दाधयत्ि है। फकसी भी लोकतांधत्रक समाज में धिधभन्न धिचारों का होना
ऄधनिायव है। के िल आस अधार पर फक समाज के एक छोटे िगव का धभन्न दृधष्टकोण है और गैर -
कानूनी तरीकों से ऄपने धिचारों को व्यि फकया गया है आत्याफद के अधार पर प्रधतबंधों को
अरोधपत नहीं फकया जा सकता है। कानून और व्यिस्था सुधनधश्चत करना सरकार का कतवव्य है।

5.8.3.5. 19 (1) (a) आं ट रने ट तक पहंच का ऄधधकार


 ईच्चतम न्यायालय द्वारा एक धनणवय में कहा गया है फक आंटरनेट तक पहंच का ऄधधकार ऄधभव्यधि
के मूल ऄधधकार के तहत शधमल है और आसे फकसी भी अधार पर बाधधत नहीं फकया जा सकता है।
पूिव गभावधान और प्रसि पूिव धनदान तकनीक (PCPNDT) ऄधधधनयम, 1994 की धारा 22 के

दृढ़ ऄनुपालन के संदभव में सचव आंजनों (Yahoo, Google और Microsoft) के धिरुद् दायर
जनधहत याधचका की सुनिाइ के दौरान आसे पाररत फकया गया था। न्यायालय ने कहा फक जब तक
आंटरनेट का ईपयोग िैध तरीके से फकया जाता है तब तक आंटरनेट तक पहंच का ऄधधकार ऄनुमत
है।

5.8.3.6. 19 (1) (c) - सहकारी सधमधतयों के गठन का ऄधधकार

97िें संधिधान संशोधन ऄधधधनयम, 2011 के द्वारा सहकारी सधमधतयों के गठन के ऄधधकार को जोड़ा
गया।

5.8.3.7. राष्ट्रीय अपातकाल का ऄनु च्छे द 19 के तहत प्रद्त ऄधधकारों पर प्रभाि

 यफद अपातकाल की घोषणा बाह्य अक्रमण के अधार पर की गइ हो तो ऄनुच्छेद 19 स्ितः

धनलंधबत हो जाता है और अपातकाल जारी रहने तक धनलंधबत रहता है। हालांफक, 44िें संधिधान

संशोधन के पश्चात्, यफद अपातकाल सशस्त्र धिद्रोह के अधार पर घोधषत फकया गया है, तो आसे
धनलंधबत नहीं फकया जा सकता है।
नोट: ऄनुच्छेद 19 के तहत प्रद्त ऄधधकार के िल भारतीय नागररकों को ईपलब्ध हैं।

195 www.visionias.in ©Vision IAS

Google it:- https://upscpdf.com


https://upscpdf.com << Download From >> https://upscpdf.com

5.9. ऄनु च्छे द 20- ऄपराधों के धलए दोषधसधद् के सं बं ध में सं र क्षण


मूल पाठ
1. कोइ व्यधि फकसी ऄपराध के धलए तब तक दोषधसद् नहीं ठहराया जाएगा, जब तक फक ईसने
ऐसा कोइ कायव करने के समय, जो ऄपराध के रूप में अरोधपत है, फकसी प्रिृ्त धिधध का
ऄधतक्रमण नहीं फकया है या ईससे ऄधधक शाधस्त का भागी नहीं होगा जो ईस ऄपराध के फकए
जाने के समय प्रिृ्त धिधध के ऄधीन ऄधधरोधपत की जा सकती थी।
2. फकसी व्यधि को एक ही ऄपराध के धलए एक बार से ऄधधक ऄधभयोधजत और दंधडत नहीं फकया
जाएगा।
3. फकसी ऄपराध के धलए ऄधभयुि फकसी व्यधि को स्ियं ऄपने धिरुद् साक्षी होने के धलए बाध्य नहीं
फकया जाएगा।
धििरण
 ऄनुच्छेद 20(1) यह प्रािधान करता है फक कोइ भी अपराधधक धिधध भूतलक्षी प्रभाि से
ऄधधधनयधमत नहीं की जा सकती। फकसी ऄपराधी को के िल ऄपराध फकए जाने के समय प्रभािी
कानून द्वारा ही दंधडत फकया जा सकता है। के दारनाथ बनाम पधश्चम बंगाल िाद (1954) में
ईच्चतम न्यायालय ने धनणवय फदया फक जब धिधानमंडल फकसी कायव को ऄपराध घोधषत करता है
ऄथिा फकसी ऄपराध के दंड का ईपबंध करता है तो िह धिधध को भूतलक्षी बनाकर ईन व्यधियों
पर प्रधतकू ल रूप से प्रभाधित नहीं कर सकता है धजन्होंने ईस धिधध के ऄधधधनयधमत होने के पूिव
ऄपराध फकए थे। हालांफक, आस तरह के संरक्षण के िल अपराधधक कानूनों के संदभव में फदए गए हैं ,
धसधिल कानूनों के संदभव में नहीं।
 ऄनुच्छेद 20(2) दोहरे जोधखम से सुरक्षा प्रदान करता है। आसके ऄनुसार एक व्यधि को एक ही
ऄपराध के धलए एक बार से ऄधधक ऄधभयोधजत और दंधडत नहीं फकया जाएगा। हालांफक, यह
धसद्ांत के िल न्यायालय ऄथिा न्याधयक ऄधधकरण के समक्ष कायविाधहयों पर लागू होता है।
धिभागीय जाँच ऄथिा प्रशासधनक कायविाधहयाँ आस धसद्ांत का ईल्लंघन नहीं मानी जाती है।
ईल्लेखनीय है फक संधिधान के िल एक ही ऄपराध के धलए दोहरा दंड िर्वजत करता है। एक
ऄपराध के धलए दोषधसद् ठहराए जाने के बाद भी फकसी ऄन्य ऄपराध के संबंध में दंड फदया जा
सकता है। यफद एक ही ऄधधधनयम की धिधभन्न धाराओं के तहत धिधभन्न ऄपराध फकए गए हों तो
संधिधान ईनके धलए पृथक धिचारण पर रोक नहीं लगाता है।
 ऄनुच्छेद 20(3), फकसी ऄपराध के संबंध में ऄधभयुि को स्ियं ऄपने धिरुद् गिाही देने के धिरुद्
संरक्षण प्रदान करता है। प्रत्येक व्यधि को स्ियं का बचाि करने का ऄधधकार है। यह ऄधधकार
प्राकृ धतक व्यधियों को और धनगमों को भी प्राि है। यह संरक्षण फकसी न्यायालय या न्याधयक
प्राधधकरण के समक्ष दांधडक कायविाधहयों तक सीधमत है तथा धसधिल कायविाधहयों एिं न्याधयक
प्रकृ धत फक कायविाधहयों पर लागू नहीं होता है। सेल्िी बनाम कनावटक राज्य िाद में ईच्चतम
न्यायालय ने नाको एनाधलधसस और िेन मैलपग पर प्रधतबंध लगा फदया। हालांफक, डीएनए
परीक्षण और ऄन्य सैंपल एकत्र फकए जा सकते हैं।
o यह ऄनुच्छेद भारतीय नागररक ऄथिा धिदेशी दोनों पर लागू होता है।
5.10. ऄनु च्छे द 21- प्राण और दै धहक स्ितं त्र ता का ऄधधकार

मूल पाठ
फकसी व्यधि को ईसके प्राण या दैधहक स्ितंत्रता से धिधध द्वारा स्थाधपत प्रफक्रया के ऄनुसार ही िंधचत
फकया जाएगा, ऄन्यथा नहीं।
धििरण:
 आसके ऄंतगवत प्रत्येक भारतीय को न के िल जीिन बधल्क गररमापूिवक जीिन का ऄधधकार प्रदान
फकया गया है। ऄतः जीिन के ऄधधकार में िे सभी अयाम सधममधलत हैं धजनसे मनुष्य का जीिन
साथवक, संपूणव और जीने योग्य बनता है।

196 www.visionias.in ©Vision IAS

Google it:- https://upscpdf.com


https://upscpdf.com << Download From >> https://upscpdf.com

 ऄनुच्छेद 21 के ऄंतगवत व्यधि को ईसके प्राण या दैधहक स्ितंत्रता से धिधध द्वारा स्थाधपत प्रफक्रया
के ऄनुसार ही िंधचत फकया जाएगा ऄन्यथा नहीं ऄथावत यह ऄनुच्छेद राज्य की मनमानी शधियों
पर एक प्रधतबंध अरोधपत करता है। राज्य एक सुधनधश्चत प्रफक्रया के ऄनुसार ही फकसी व्यधि को
ईसके जीिन की स्ितंत्रता से िंधचत कर सकता है।
 धिधध द्वारा स्थाधपत प्रफक्रया (Procedure established by law) धिरटश परंपरा से ग्रहण की
गइ है। आसके ऄंतगवत यह जाँच की जाती है फक क्या कानून प्रफक्रयागत रूप से सही है। हालांफक,
न्यायपाधलका को आस कानून के ईद्देश्यों को चुनौती देने की ऄनुमधत नहीं है। िहीं दूसरी ओर धिधध
की समयक प्रफक्रया (Due process of law) ऄमेररकी न्यायपाधलका की एक ऄधभव्यधि है।
धजसके ऄंतगवत न्यायपाधलका कानून को न के िल प्रफक्रयात्मक अधार पर बधल्क आसके औधचत्य के
अधार पर भी चुनौती दे सकती है।
ए. के . गोपालन िाद (1950) एिं मेनका गांधी िाद (1978)
 गोपालन िाद में ईच्चतम न्यायालय ने यह मत व्यि फकया था फक धिधध द्वारा स्थाधपत प्रफक्रया के
प्रािधान को ऄपनाकर हमारे संधिधान के ऄनुच्छेद 21 ने व्यधिगत स्ितंत्रता की धिरटश
संकल्पना को ऄपनाया है तथा ऄमेररका की धिधध की समयक प्रफक्रया की संकल्पना को नहीं
ऄपनाया है।
 ईच्चतम न्यायालय ने संधिधान में धिधध द्वारा स्थाधपत प्रफक्रया की, गोपालन िाद (1950) में सूक्ष्म
व्याख्या की तथा घोधषत फकया फक आस ऄधधकार के तहत धसफव मनमानी कायवकारी प्रफक्रया के
धिरुद् सुरक्षा प्राि है, न फक धिधानमंडलीय प्रफक्रया के धिरुद्। आसका ऄथव यह है फक राज्य प्राण
एिं दैधहक स्ितंत्रता के ऄधधकार को कानूनी अधार पर प्रधतबंधधत कर सकता है।
 मेनका गांधी िाद (1978) में ईच्चतम न्यायालय ने स्िीकार फकया है फक, धिधध द्वारा स्थाधपत
प्रफक्रया धिधध की धिधधित प्रफक्रया में ही धनधहत है। ईच्चतम न्यायालय ने आस िाद में पहली बार
ऄमेररका की धिधध की समयक प्रफक्रया की संकल्पना को लागू फकया। यह धनम्नधलधखत तकों को
लागू करता है:-
o ऄनुच्छेद 19 और 21 को धनरपेक्ष नहीं समझा जा सकता और युधियुिता के समान मापदंड
ऄनुच्छेद 21 के धलए भी लागू फकया जाना चाधहए। ईच्चतम न्यायालय ने संधिधान के
ऄनुच्छेद 21 को ऄनुच्छेद 19 के साथ जोड़कर व्याख्या की तथा पहली बार प्रफक्रया की
युधियुिता (ऄथावत् प्रफक्रया मनमानी-पूणव या ऄनौधचत्यपूणव न हो) पर बल प्रदान करते हए
प्राकृ धतक न्याय के धसद्ांत को ऄपनाया। साथ ही, यह भी स्पष्ट फकया फक ऄनुच्छेद 19 और
21 परस्पर ऄपिजवनकारी नहीं हैं, बधल्क िे एक-दूसरे को बल प्रदान करते हैं।
o के िल धिधध द्वारा स्थाधपत प्रफक्रया का पालन करना ही पयावि नहीं है। न्यायालय को समीक्षा
करने और कानून की युधियुिता पर प्रश्न ईठाने का ऄधधकार भी प्राि होता है।
o प्रधतबंध युधियुि, न्यायसंगत और धनष्पक्ष होने चाधहए, मनमाने ढंग का नहीं।
 भारत में परंपरागत रूप से धिटेन में प्रचधलत धिधध द्वारा स्थाधपत प्रफक्रया के धसद्ांत का
ऄनुपालन फकया जाता रहा है। हालांफक िषव 1978 के पश्चात से, भारत में दोनों प्रफक्रयाओं का
ऄनुपालन फकया जा रहा हैं।
5.10.1. न्याधयक व्याख्या द्वारा ऄनु च्छे द 21 के क्षे त्र का धिस्तार
 मेनका गांधी िाद में ईच्चतम न्यायालय ने धनणवय फदया फक ऄनुच्छेद 21 के तहत प्राण का ऄथव
के िल ‘पशुित ऄधस्तत्ि’ या जीधित रहना मात्र नहीं है। आसके ऄंतगवत मानिीय गररमा के साथ
जीने का ऄधधकार भी सधममधलत है।
 आसमें जीिन के िे सभी अयाम सधममधलत हैं धजनसे मनुष्य का जीिन साथवक, संपूणव और जीने
योग्य बनता है। मेनका गांधी िाद के पश्चात् भी कइ न्याधयक व्याख्याओं ने ऄनुच्छेद 21 के दायरे
को धिस्तृत करते हए आसके ऄंतगवत कइ ऄधधकारों को समाधहत फकया है।

197 www.visionias.in ©Vision IAS

Google it:- https://upscpdf.com


https://upscpdf.com << Download From >> https://upscpdf.com

 ईन्नीकृ ष्णन बनाम अंरप्रदेश राज्य िाद (1993) में धनधावररत फकया गया फक जीिन के ऄधधकार

में धशक्षा का ऄधधकार शाधमल है। ऄनुच्छेद 21 का धिस्तृत क्षेत्राधधकार ईच्चतम न्यायालय द्वारा
िर्वणत फकया गया है और ईच्चतम न्यायालय ने स्ियं ही पूिवन्यायधनणवयों के अधार पर ऄनुच्छेद
21 के तहत अने िाले ऄधधकारों की सूची ईपलब्ध कराइ है। आनमें से कु छ धनम्नधलधखत हैं:

o धिदेश यात्रा का ऄधधकार;

o धनजता का ऄधधकार;

o अश्रय का ऄधधकार;

o सामाधजक न्याय और अर्वथक सशधिकरण का ऄधधकार;

o एकांत कारािास के धिरुद् ऄधधकार;

o हथकड़ी लगाने के धिरुद् ऄधधकार;

o मृत्युदड
ं में देरी के धिरुद् ऄधधकार;

o बंदीगृह में मृत्यु के धिरुद् ऄधधकार;

o सािवजधनक रूप से फांसी के धिरुद् ऄधधकार;

o धचफकत्सकीय सहायता का ऄधधकार;

o सांस्कृ धतक धिरासत की सुरक्षा का ऄधधकार;

o प्रत्येक धशशु के पूणव धिकास का ऄधधकार;


o प्रदूषणमुि िायु एिं जल का ऄधधकार अफद।
आस प्रकार यह स्पष्ट है फक प्रारंभ में ऄनुच्छेद 21 के प्रािधानों की संकीणव व्याख्या की गइ परन्तु क्रमशः
दैधहक और व्यधिगत स्ितंत्रता के संदभव में कानून का धिस्तार हअ और आसकी ईदार व्याख्या की गइ।
ऄनुच्छेद 21 के क्षेत्राधधकार में समय के साथ नये अयाम जोड़े गये। आसने फकसी व्यधि को प्राण एिं

दैधहक स्ितंत्रता से िंधचत फकए जाने की प्रफक्रया पर यह कहते हए सीमाएँ अरोधपत की, फक प्रफक्रया को

न्यायोधचत तथा धििेकपूणव होना चाधहए तथा कानून को मनमाना, दमनकारी ऄथिा काल्पधनक नहीं
होना चाधहए।
{नोट: हसैन अरा खातून िाद (1979) धजसे भारत में जनधहत याधचका (PIL) के ईद्भि के रूप में देखा

जाता है; में धनधावररत फकया गया फक त्िररत सुनिाइ का ऄधधकार न्याय प्राधि का सार है। आस िाद में
ईच्चतम न्यायालय ने पहली बार िंधचतों को मुफ्त कानूनी सहायता की अिश्यकता पर बल फदया
ताफक ईनके ऄधधकारों को साथवक संरक्षण प्रदान करने िाली न्याय प्रणाली को स्थाधपत फकया जा सके ।
साथ ही आस िाद में ईच्चतम न्यायालय ने ऄनुच्छेद 21 में प्रद्त धिधध द्वारा स्थाधपत प्रफक्रया में

संतुधलत, ईधचत एिं न्याय पर अधाररत प्रयोजनों को लागू करने पर बल प्रदान फकया।)}

5.10.2. धनजता का ऄधधकार (Right to Privacy)

 हाल ही में, ईच्चतम न्यायालय की नौ सदस्यीय संिैधाधनक खंडपीठ ने के . एस. पुट्टास्िामी बनाम

भारत संघ िाद में सिवसममधत से धनणवय देते हए ‘धनजता के ऄधधकार’ को ऄनुच्छेद-21 के जीिन
और स्ितंत्रता के ऄधधकार के तहत मूल ऄधधकार का ऄधभन्न भाग माना है। यह धनणवय धिधभन्न
जन-कल्याण कायवक्रमों का लाभ ईठाने के धलए कें द्र सरकार द्वारा अधार काडव को ऄधनिायव करने
को चुनौती देने िाली याधचकाओं से संबंधधत है।

198 www.visionias.in ©Vision IAS

Google it:- https://upscpdf.com


https://upscpdf.com << Download From >> https://upscpdf.com

 हालाँफक, न्यायालय ने माना फक धनजता कोइ पूणव (धनरपेक्ष) ऄधधकार नहीं है। सरकार द्वारा एक
कानून प्रस्तुत फकया का सकता है धजसके माध्यम से सािवजधनक और िैध राष्ट्रीय कारणों के अधार
पर गोपनीयता को "सीधमत" फकया जा सकता है। राज्य के िैध ईद्देश्यों के ऄंतगवत राष्ट्रीय सुरक्षा,
ऄपराध को रोकना और जांच करना, निाचार को प्रोत्साधहत एिं ज्ञान का प्रसार करना तथा
सामाधजक कल्याण से समबधन्धत लाभों के ऄपव्यय को रोकना जैसे ईदाहरण शाधमल होंगे। लेफकन
फकसी व्यधि द्वारा धनजता के मूल ऄधधकार के ईल्लंघन के संदभव में देश के संिैधाधनक न्यायालय में
आस कानून को चुनौती दी जा सकती है।
धनणवय की मुख्य धिशेषताएं
 व्यधि के मूल ऄधधकारों का धिस्तार करता है: धजसकी ऄनुच्छेद 21 में गारंटी प्रदान की गयी है
और आसके ऄंतगवत फकसी के घर में ऄिांधछत प्रिेश, भोजन के चयन का ऄधधकार, संघ बनाने की
स्ितंत्रता अफद शाधमल हैं।।
 गररमा सुधनधश्चत करता है: क्योंफक नागररकों के धलए गोपनीयता के धबना स्ितंत्रता और गररमा
का ईपयोग करना संभि नहीं है।
 स्ितंत्र बाह्य धनयामक: ऄब नागररक ऄनुच्छेद 32 और 226 के तहत ऄपने मूल ऄधधकारों के
ईल्लंघन के धलए सीधे ईच्चतम न्यायालय या ईच्च न्यायालयों में याधचका दायर कर सकते हैं। आस
प्रकार यह सुधनधश्चत करता फक ऄधधकार के िल लोक स्िास्थ्य, सदाचार और लोक व्यिस्था के
युधियुि धनबंधनों के ऄधीन हैं।
 सूचनात्मक गोपनीयता आस ऄधधकार का एक पहलू है: सरकार को डेटा सुरक्षा के धलए एक सुदढ़ृ
व्यिस्था की स्थापना करनी चाधहए।
मेनका गांधी बनाम भारत संघ (1978) िाद में न्यायालय ने यह माना फक व्यधिगत स्ितंत्रता और
धनजता के ऄधधकार में ऄल्पीकरण या दखल देने िाला कोइ भी कानून ऄधनयंधत्रत या मनमाना नहीं
होना चाधहए।
 धनजता संबंधी कानूनों का ऄध्ययन करने के धलए न्यायमूर्वत ए. पी. शाह की ऄध्यक्षता में
धिशेषज्ञों की एक सधमधत गरठत की गइ। आस सधमधत को धनजता पर प्रस्ताधित मसौदा धिधेयक,
2011 से संबंधधत सुझाि प्रदान करना था। आसके द्वारा कें द्र और राज्यों में धनजता अयुिों
(privacy commissioners) की धनयुधि, डेटा संग्रहकतावओं द्वारा पालन फकए जाने िाले धनजता
संबंधी नौ धसद्ांत तथा ईद्योगों द्वारा स्ि-धिधनयामक संगठन की स्थापना, अफद ऄनुशंसाएँ की
गईं।

डेटा (गोपनीयता और संरक्षण) धिधेयक, 2017


 हाल ही में, डेटा (गोपनीयता और संरक्षण) धिधेयक, 2017 को लोकसभा में पेश फकया गया।
 आसमें डेटा के संकलन, प्रसंस्करण, भंडारण और हटाने के धलए व्यधिगत सहमधत तथा आस प्रफक्रया
में मामलों की धिधशष्टता के अधार पर बहत सीधमत ऄपिादों की स्िीकृ धत को ऄधनिायव बनाने का
प्रािधान है।
 आसके साथ ही आसमें डेटा गोपनीयता और संरक्षण प्राधधकरण को ऄपील के प्रािधान के साथ
ऄंधतम ईपयोगकतावओं के धशकायत धनिारण हेतु डेटा संरक्षण ऄधधकारी के पद का सृजन करना,
जैसे प्रािधान फकए गए हैं।

धनजता के ऄधधकार से संबधं धत ऄंतरावष्ट्रीय कानून


 िषव 1981 में यूरोपीय कॉईधन्सल द्वारा हस्ताक्षररत कन्िेंशन फॉर फद आंधडधिजुऄल धिद ररगाडव टू
ऑटोमेरटक प्रोसेलसग ऑफ़ पसवनल डेटा (कन्िेंशन 108) धनजता के ऄधधकार का संरक्षण प्रदान
करने िाली प्रथम कानूनी रूप से बाध्यकारी ऄंतरावष्ट्रीय संधध है।

199 www.visionias.in ©Vision IAS

Google it:- https://upscpdf.com


https://upscpdf.com << Download From >> https://upscpdf.com

 मानिाधधकारों की सािवभौम घोषणा, 1948 का ऄनुच्छेद 12 और आंटरनेशनल कोिेनेंट ऑन


धसधिल एंड पोधलरटकल राआट (ICCPR) 1966 का ऄनुच्छेद 17, व्यधियों की धनजता, पररिार,
घर, पत्राचार, सममान और प्रधतष्ठा के साथ "मनमाने ढंग से हस्तक्षेप" के धिरुद् कानूनी संरक्षण
प्रदान करता है।
धनजता के ऄधधकार का महत्ि
 आस धनणवय के पररणामस्िरूप व्यधि के मूल ऄधधकारों का धिस्तार तथा ईनकी गररमा सुधनधश्चत
होगी। आसके ऄनुसार, राष्ट्रीय डेटा संग्रह में धनधहत फकसी भी व्यधिगत अंकड़ों के दुरुपयोग की
धस्थधत में राज्य को क्षधतपूर्वत प्रदान करनी होगी। ऄब नागररक ईच्चतम न्यायालय या ईच्च
न्यायालयों में ऄनुच्छेद 32 और 226 के तहत ऄपनी धनजता के ऄधधकार के ईल्लंघन के धलए
सीधे ऄपील दायर कर सकते हैं। आस प्रकार यह सुधनधश्चत फकया गया है फक ऄधधकार सािवजधनक
स्िास्थ्य, नैधतकता और व्यिस्था के ईधचत प्रधतबंधों के ऄधीन है। ध्यातव्य है फक धनजता को
ऄंतरराष्ट्रीय स्तर पर एक सुदढ़ृ कानूनी ढाँचे का समथवन प्राि है। िषव 1979 में भारत ने ICCPR
पर हस्ताक्षर तथा ऄधभपुधष्ट भी की थी।
 परन्तु आस धनणवय से कइ लचताएँ भी ईत्पन्न हइ हैं। यथा सरकार की कल्याणकारी योजनाओं और
ऄन्य मामलों जैसे फक अधार, धारा 377, व्हाट्सएप प्राआिेसी पॉधलसी, खानपान की अदतों पर
प्रधतबंध अफद पर प्रभाि पड़ेगा। धनजता की रूपरेखा को पररभाधषत नहीं फकया जा सकता क्योंफक
यह ऄन्य सभी मूल ऄधधकारों में भी समाधहत है। यह धनगरानी, खोज और जब्ती, कॉल टैलपग,
ट्रांसजेंडर के ऄधधकार अफद सधहत धिधभन्न ऄधधकारों का एक क्लस्टर (समूह) है।
 आस ऄधधकार के सफल कायावन्ियन के धलए अिश्यक है फक भारत में धनजता के संरक्षण की
भािना को बढ़ािा फदया जाए। यह पधश्चमी देशों की तुलना में भारत में कम है। आस हेतु एक
राष्ट्रीय डेटा संरक्षण प्रारूप का धिकास फकया जाना चाधहए जो व्यधियों के धलए के िल डेटा
संरक्षण से परे एक व्यापक संदभव में व्यधिगत धनजता की रूपरेखा को पररभाधषत करे। एक
धिधधक ढांचे के धनमावण के माध्यम से डेटा माआलनग और धबग डेटा के लाभों के साथ फकसी व्यधि
के धनजता के ऄधधकार को संतुधलत करना चाधहए।
5.10.3. जीिन का ऄधधकार और अत्महत्या (IPC की धारा 309)

 भारत में अत्महत्या संबध


ं ी कानून: मानधसक स्िास्थ्य देखभाल धिधेयक - 2016 के पाररत होने से
पूिव अत्महत्या, या अत्महत्या का प्रयास करना भारतीय दंड संधहता की धारा 309 के ऄंतगवत
ऄपराध था। आस नये कानून के ऄंतगवत भारतीय दंड संधहता की धारा 309 का धनरपराधीकरण
कर फदया गया है तथा मानधसक रूप से बीमार व्यधि के अत्महत्या के प्रयासों को दडडनीय नहीं
माना गया है।
 पी. रत्नम िाद में ईच्चतम न्यायालय ने िषव 1994 में धारा 309 को यह कह कर ऄसंिैधाधनक
करार फदया था फक ‘न जीने का ऄधधकार’ संधिधान के ऄनुच्छेद 21 में प्रद्त ‘जीने के ऄधधकार’ में
ही धनधहत है।
 साथ ही आस िाद में कहा गया फक सभी मूल ऄधधकारों के सकारात्मक के साथ नकारात्मक
धनधहताथव भी हैं। जैसे फक भाषण एिं ऄधभव्यधि की स्ितंत्रता के मूल ऄधधकार में ‘नहीं बोलने के
ऄधधकार’ भी समाधिष्ट है। आस व्याख्या से धारा 309 का ऄधस्तत्ि समाि हो गया था और िषव
1996 तक यही धस्थधत रही।
 हालाँफक, ईच्चतम न्यायालय ने ज्ञान कौर िाद (1996) में िषव 1994 के धनणवय को ईलट फदया धजसके
कारण धारा 309 पुनः ऄधस्तत्ि में अ गयी और ‘न जीने का ऄधधकार’ ऄसंिैधाधनक बन गया।

200 www.visionias.in ©Vision IAS

Google it:- https://upscpdf.com


https://upscpdf.com << Download From >> https://upscpdf.com

 िषव 2008 में धिधध अयोग ने ऄपनी 210िींररपोटव में अत्महत्या के प्रयास को ऄपराध की श्रेणी
से बाहर धनकालने की ऄनुशंसा की।
 मानधसक स्िास्थ्य देखभाल ऄधधधनयम, 2017 के ऄनुसार अत्महत्या का प्रयास मानधसक तनाि
की धस्थधत को दशावता है तथा आसे ऄपराध की श्रेणी में नहीं रखा जा सकता। अत्महत्या का प्रयास
करने िाले व्यधि को दंड की नहीं िरन देखभाल और ईपचार की अिश्यकता है।
आच्छामृत्यु/दया मृत्यु (Euthanasia/Mercy killing)
ऄरुणा शानबाग िाद में ईच्चतम न्यायालय द्वारा धनधष्क्रय आच्छामृत्यु की ऄनुमधत प्रदान की गइ थी
लेफकन सफक्रय आच्छामृत्यु की नहीं।
सफक्रय आच्छामृत्यु (Active Euthanasia): फकसी व्यधि के जीिन का ऄंत ईसे सफक्रय साधनों जैसे
प्राणघाती आंजेक्शन, ड्रग्स अफद देकर फकया जाता है।
धनधष्क्रय आच्छामृत्यु (Passive Euthanasia): आसमें फकसी व्यधि या मरीज के लाआफ सपोटव धसस्टम
को हटा फदया जाता है। आसकी ऄनुमधत ईच्चतम न्यायालय द्वारा संरधक्षत पररधस्थधतयों में ही प्रदान की
जाती है।
5.10.4. जीिन का ऄधधकार एिं मृ त्यु दं ड
 ितवमान में िैधश्वक रुख मृत्युदड
ं को समाि करने के पक्ष में है, भारत ने स्ियं को ऐसे देशों की श्रेणी
में बनाए रखा है जहां मृत्युदड
ं को पूणवतया समाि नहीं फकया गया है, जैसे- चीन, इरान,
पाफकस्तान, ऄमेररका।
 मृत्युदड
ं के समथवकों ने आसको धनिारक क्षमता के धलए ऄपनाया है। आसके ऄधतररि, कु छ ऄपराध
आतने जघन्य होते हैं धजनकी सजा यफद मृत्युदड
ं से कम हो तो िे न्याय के लक्ष्य को पूरा नहीं कर
पाते हैं। अतंकिाद जैसे मामलों में, यफद अतंकिाफदयों को मौत की सजा नहीं दी जाए तो िे
राष्ट्रीय सुरक्षा के धलए एक गंभीर खतरा बने रह सकते हैं।
हालांफक मृत्युदड ं को समाि करने के पक्ष में धनम्नधलधखत तकव फदए जाते हैं: -
 धनिारक तकव (deterrent logic) का समथवन करने के धलए कोइ पयावि अंकड़े नहीं हैं।
 संयुि राज्य ऄमेररका में फकए गए एक ऄध्ययन से पता चलता है फक धजन राज्यों ने मौत की सजा
को समाि कर फदया है िहाँ पर हत्या की घटना में धगरािट देखने को धमली।
 फकसी भी सभ्य समाज में प्रधतशोध का धसद्ांत (The principle of revenge) (जैसे-अंख के
बदले अंख) न्याय का अधार नहीं हो सकता।
सजा का ईद्देश्य दंड के बजाए सुधार होना चाधहए। मृत्युदड
ं की सजा के िल रेयरेस्ट ऑफ द रेयर
मामलों में दी जाती है।
बच्चन लसह िाद (1980) में ईच्चतम न्यायालय ने अजीिन कारािास को धनयम और मृत्युदड
ं का
ऄपिाद मानते हए मृत्युदड
ं की संभािना को बरकरार रखा और आसे न्यायसंगत बनाने के धलए
ऄनन्यतम (रेयरेस्ट ऑफ फद रेयर) का धसद्ांत धनष्पाफदत फकया।
मच्छी लसह बनाम पंजाब सरकार (1983) िाद में ईच्चतम न्यायालय के तीन सदस्यीय खंडपीठ ने
मृत्युदड
ं देने के धलए धनम्नधलधखत मापदंडों को रखा।
 ऄत्यन्त क्रूर, कठोर और भयानक तरीके से हत्या के मामले में;
 हत्या का ईद्देश्य धन होने पर;
 ऄनुसूधचत जाधत या ऄल्पसंख्यक समुदाय के लोगों की हत्या करने पर;
 फकसी धनदोष बच्चे, ऄसहाय मधहला या गणमान्य व्यधि की हत्या करने पर।
प्रयोजनीयता
ऄनुच्छेद 20 और 21 के तहत प्रद्त ऄधधकारों को कभी भी (राष्ट्रीय अपातकाल में भी) धनलंधबत नहीं
फकया जा सकता और ये दोनों ऄधधकार सभी व्यधियों, चाहे िह भारतीय नागररक हो या धिदेशी सभी
को ईपलब्ध हैं।

201 www.visionias.in ©Vision IAS

Google it:- https://upscpdf.com


https://upscpdf.com << Download From >> https://upscpdf.com

5.11. ऄनु च्छे द 21-A : धशक्षा का ऄधधकार

मूल पाठ
धशक्षा का ऄधधकार (Right to education)
राज्य छह से चौदह िषव की अयु के सभी बच्चों को धनःशुल्क और ऄधनिायव धशक्षा आस प्रकार प्रदान
करेगा धजस प्रकार से राज्य धिधध के ऄधीन धनधावररत करे।
धििरण एिं ऐधतहाधसक धिकास
 धशक्षा के ऄधधकार की मांग सिवप्रथम गोपाल कृ ष्ण गोखले ने धिरटश काल के दौरान की थी।
हालांफक, 100 िषव पश्चात धशक्षा का ऄधधकार ऄधस्तत्ि में अया।
 िषव 1992 में मोधहनी जैन िाद में ईच्चतम न्यायालय ने कहा फक धशक्षा का ऄधधकार जीिन के
ऄधधकार का भाग है और आसधलए यह संधिधान के भाग 3 के तहत एक मूल ऄधधकार है।
 ईन्नीकृ ष्णन िाद (1993) में ईच्चतम न्यायालय के धनणवय ने आसको और सुदढ़ृ फकया धजसमें यह
पुधष्ट की गइ फक धशक्षा का ऄधधकार ऄनुच्छेद 21 के तहत प्रत्याभूत जीिन के ऄधधकार से संबंधधत है।
 ईल्लेखनीय है फक ईच्चतम न्यायालय ने कहा फक यह एक धनरपेक्ष एिं ऄनन्य ऄधधकार नहीं है। यह
ऄधधकार फकस तरह लागू फकया जाए आसका धनधावरण राज्य पर धनभवर करेगा।
 86िें संधिधान संशोधन ऄधधधनयम, 2002 द्वारा संधिधान के भाग 3 में ऄनुच्छेद 21(A) को
जोड़कर 6-14 िषव तक की अयु के सभी बच्चों के धलए धनःशुल्क एिं ऄधनिायव प्रारंधभक/प्राथधमक
धशक्षा की संिैधाधनक गारंटी प्रदान फक गइ है। 86िें संशोधन ऄधधधनयम द्वारा संधिधान में
धनम्नधलधखत पररितवन फकए गए:
 राज्य की नीधत के धनदेशक तत्ि के रूप में संधिधान के ऄनुच्छेद 45 में कहा गया है फक, राज्य
सभी बच्चों को 0-6 िषव की अयु पूरी हो जाने तक धनःशुल्क और ऄधनिायव धशक्षा प्रदान करने के
धलए ईपबंध करने का प्रयास करेगा।
 ऄनुच्छेद 51A(k) के तहत एक नया मूल क्त वव्य जोड़ा गया धजसके ऄनुसार - यह माता-धपता या
संरक्षक का क्त वव्य होगा फक िह 6 से 14 िषव की मध्य की अयु के , यथाधस्थधत, ऄपने बच्चे ऄथिा
प्रधतपाल्य को धशक्षा प्राि करने के ऄिसर प्रदान करेगा।
 यह ईपयुवि अयु समूह के सभी बच्चों के धलए लागू होता है चाहे िह भारतीय नागररक हैं या नहीं हैं।
RTE की मुख्य धिशेषताएं
 6 से 14 िषव की अयु के सभी बच्चों को ऄपने धनकट धस्थत स्कू ल में प्रारंधभक और ऄधनिायव धशक्षा
प्राि करने का ऄधधकार होगा।
 यहां तक फक जो आस ऄिसर से िंधचत रह गए हैं, ईनके धलए आस ऄधधधनयम के तहत 8 िषव की
स्कू ली धशक्षा का प्रािधान फकया गया है। साथ ही, गैर-नामांफकत बच्चों को ईनकी अयु के ऄनुसार
कक्षा में प्रिेश फदया जाएगा।
 कें द्रीय धिद्यालय, निोदय धिद्यालय, सैधनक स्कू ल और गैर-सहायता प्राि स्कू लों द्वारा कम से कम
25% िंधचत और अर्वथक रूप से कमजोर समूहों के छात्रों का नामांकन फकया जाना ऄधनिायव है।
 प्रारंधभक धशक्षा पूरी होने तक फकसी भी बच्चे को बोडव परीक्षा में ऄनु्त ीणव या धनष्काधसत नहीं
फकया जाएगा।
 धिद्यालय में प्रिेश के दौरान अिेदकों की स्क्रीलनग तथा ईनसे कै धपटेशन फ़ीस प्राि नहीं कर सकते हैं।
 सभी धिधालयो को 5 िषव के भीतर धशक्षक योग्यता के मानदंडों का ऄनुपालन फकया जाना
ऄधनिायव होगा। 1:30 के धशक्षक-छात्र ऄनुपात को बनाए रखने हेतु धनधश्चत संख्या में धशक्षकों की
भती करनी होगी।

202 www.visionias.in ©Vision IAS

Google it:- https://upscpdf.com


https://upscpdf.com << Download From >> https://upscpdf.com

 ईन गैर-सहायता प्राि धनजी धिद्यालयों के ऄधतररि सभी धिद्यालयों द्वारा धिद्यालय प्रबंधन
सधमधतयों का गठन फकया जायेगा। आसके 75% सदस्य धिद्यार्वथयों के माता-धपता या संरक्षक होंगे।
 राष्ट्रीय बाल ऄधधकार संरक्षण अयोग द्वारा आसकी धनगरानी की जाएगी। राज्य ऄपने स्तर पर भी
आसके समान राज्य धनकायों का गठन करेगा I
 आसके धलए कें द्र और राज्य सरकारों द्वारा परस्पर धितपोषण को साझा फकया जाएगा:
o कें द्र सरकार धि्त अयोग से राज्यों को ऄधतररि संसाधन अिंरटत करने हेतु ऄनुशंसा कर
सकती है।
o नागररक समाज की साझेदारी के साथ धि्त ीयन ऄंतराल को कम फकया जाएगा।
o संिैधाधनक ऄधधकारों के ऄनुसार पाठ्यचयाव का धिकास फकया जाएगा।
RTE का मूल्यांकन
 यफद प्राधधकरण प्रारंधभक धशक्षा का ऄधधकार लागू करने में धिफल रहते हैं तो आसके धलए आस
ऄधधधनयम में कोइ धिधशष्ट दंड का प्रािधान नहीं फकया गया है।
 राज्य सरकार और स्थानीय प्राधधकरण दोनों का कतवव्य है फक िे धन:शुल्क और ऄधनिायव प्राथधमक
धशक्षा प्रदान करें। ज्ञातव्य है फक दोनों द्वारा आस कतवव्य का साझाकरण फकए जाने के कारण दोनों
में से फकसी को भी जिाबदेह नहीं ठहराया जा सकता है।
 आस ऄधधधनयम में स्कू ली धशक्षा और भौधतक ऄिसंरचना के ऄधधकार का प्रािधान फकया गया है,
लेफकन यह सुधनधश्चत नहीं है फक बच्चे सीख पा रहे या नहीं। यफद सरकारी स्कू लों द्वारा धनर्ददष्ट
मानदंडों को पूरा नहीं फकया जाता है तो आसके धलए ईन्हें फकसी भी प्रकार से दंधडत नहीं फकया जा
सकता है।
 आस धिधेयक में मल्टी-ग्रेड (बह-कोरट) धशक्षण की प्रथा को िैधता प्रदान की गइ है। धशक्षकों की
संख्या ग्रेड के स्थान पर छात्रों की संख्या के अधाररत होगी।
 नामांकन सािवभौधमक स्तर पर पहंच गया है लेफकन छात्रों की स्कू ल छोड़ने (ड्रॉपअईट) और
ऄनुपधस्थधत की समस्या ऄभी भी बनी हइ है।
 आसके ऄधतररि, आस ऄधधधनयम में ईन लोगों के के संबंध में कोइ प्रािधान नहीं फकए गए हैं जो
स्कू ल जाने में ऄसमथव हैं।
 आसधलए, यह कहा जाता है फक यह धशक्षा के ऄधधकार के बजाय एक स्कू ली धशक्षा का ऄधधकार है।
 ऄिसंरचना में सुधार के लक्ष्यों को पूरा करने में ऄनेक धिद्यालय धिफल रहे।
 देश में प्रधशधक्षत धशक्षकों की की ऄत्यधधक कमी है।
 कु छ लोगों का मानना है फक बच्चों को फे ल न करना एक बेहतर धिकल्प नहीं है क्योंफक यह धशक्षकों
को ईसके ई्त रदाधयत्िों से मुि करता है।
प्रयोज्यता
 यह संबंधधत अयु िगव के सभी बच्चों (चाहे भारतीय नागररक हैं या नहीं) पर लागू होता है।

5.12. ऄनु च्छे द 22: कु छ दशाओं में धगरफ्तारी और धनरोध से सं र क्षण


मूल पाठ
कु छ दशाओं में धगरफ्तारी और धनरोध से संरक्षण।
1. फकसी व्यधि को धजसे धगरफ्तार फकया गया है, ऐसी धगरफ्तारी के कारणों से यथाशीर अ ऄिगत
कराए धबना ऄधभरक्षा में धनरुद् नहीं रखा जाएगा या ऄपनी रुधच के धिधध व्यिसायी से परामशव
करने और प्रधतरक्षा कराने के ऄधधकार से िंधचत नहीं रखा जाएगा।
2. प्रत्येक व्यधि को, धजसे धगरफ्तार फकया गया है और ऄधभरक्षा में धनरुद् रखा गया है , धगरफ्तारी
के स्थान से मधजस्ट्रेट के न्यायालय तक यात्रा के धलए अिश्यक समय को छोड़कर, ऐसी
धगरफ्तारी से 24 घंटे की ऄिधध में धनकटतम मधजस्ट्रेट के समक्ष पेश फकया जाएगा और ऐसे फकसी
व्यधि को मधजस्ट्रेट के प्राधधकार के धबना ईि ऄिधध से ऄधधक ऄिधध के धलए ऄधभरक्षा में धनरुद्
नहीं रखा जाएगा।
3. खडड (1) और खडड (2) की कोइ बात फकसी ऐसे व्यधि को लागू नहीं होगी जो-

203 www.visionias.in ©Vision IAS

Google it:- https://upscpdf.com


https://upscpdf.com << Download From >> https://upscpdf.com

(a) तत्समय शत्रु ऄन्यदेशीय है; या

(b) धनिारक धनरोध का ईपबंध करने िाली फकसी धिधध के ऄधीन धगरफ्तार या धनरुद् फकया गया है।
4. धनिारक धनरोध का ईपबंध करने िाली कोइ धिधध फकसी व्यधि को 3 माह से ऄधधक की
ऄिधध के धलए तब तक धनरुद् फकया जाना प्राधधकृ त नहीं करेगी जब तक फक-
(a) ऐसे व्यधियों से, जो ईच्च न्यायालय के न्यायाधीश हैं या न्यायाधीश रहे हैं या न्यायाधीश धनयुि

होने के धलए ऄर्वहत हैं, से धमलकर बने सलाहकार बोडव ने तीन मास की ईि ऄिधध की समाधि से
पूिव यह प्रधतिेदन नहीं फदया है फक ईसकी राय में ऐसे धनरोध के धलए पयावि कारण है:
परन्तु आस ईपबंध की कोइ भी बात फकसी व्यधि का ईस ऄधधकतम ऄिधध से ऄधधक ऄिधध के धलए
धनरुद् फकया जाना प्राधधकृ त नहीं करेगी जो खडड (7) के ईपखडड (b) के ऄधीन संसद द्वारा बनाइ गइ
धिधध द्वारा धिधहत की गइ है; या
(b) ऐसे व्यधि को खडड (7) ईपखडड (a) और ईपखडड (b) के ऄधीन संसद द्वारा धनर्वमत धिधध के
ईपबंधों के ऄनुसार धनरुद् नहीं फकया जाता है।
5. धनिारक धनरोध का ईपबंध करने िाली फकसी धिधध के ऄधीन फकए गए अदेश के ऄनुसरण में
जब फकसी व्यधि को धनरुद् फकया जाता है तब अदेश करने िाला प्राधधकारी यथाशीर अ ईस व्यधि
को यह संसूधचत करेगा फक िह अदेश फकन अधारों पर फकया गया है और ईस अदेश के धिरुद्
ऄभ्यािेदन करने के धलए ईसे शीर अाधतशीर अ ऄिसर देगा।
6. खडड (5) की फकसी बात से ऐसा अदेश , जो ईस खडड में धनर्ददष्ट है, करने िाले प्राधधकारी के
धलए ऐसे तथ्यों को प्रकट करना अिश्यक नहीं होगा धजन्हें प्रकट करना ऐसा प्राधधकारी लोकधहत
के धिरुद् समझता है।
7. संसद धिधध द्वारा यह धिधहत कर सके गी फक-
(a) फकन पररधस्थधतयों के ऄधीन और फकस िगव या िगों के िाद में फकसी व्यधि को धनिारक धनरोध का

ईपबंध करने िाली फकसी धिधध के ऄधीन तीन मास से ऄधधक ऄिधध के धलए खडड (4) के ईपबंध
(a) के ईपबंधों के ऄनुसार सलाहकार बोडव की राय प्राि फकए धबना धनरुद् फकया जा सके गा;
(b) फकसी िगव या िगों के िाद में फकतनी ऄधधकतम ऄिधध के धलए फकसी व्यधि को धनिारक धनरोध

का ईपबंध करने िाली फकसी धिधध के ऄधीन धनरुद् फकया जा सके गा; और
(c) खडड (4) के ईपखडड (a) के ऄधीन की जाने िाली जांच में सलाहकार बोडव द्वारा ऄनुसरण की
जाने िाली प्रफक्रया क्या होगी।
धििरण
ऄनुच्छेद 22 में दो प्रकार की धगरफ्तारी या धनरोध का धििरण है:

 दंडात्मक धनरोध (Punitive detention); तथा


 धनिारक धनरोध (Preventive detention)

दंडात्मक धगरफ्तारी से सुरक्षा नागररकों और गैर-नागररकों के धलए ईपलब्ध है, परन्तु शत्रु देश के
धनिाधसयों के धलए ईपलब्ध नहीं है। एक व्यधि को ईसकी धगरफ्तारी के अधार के बारे में ऄिश्य
सूधचत फकया जाना चाधहए ताफक िह ऄपने बचाि की तैयारी कर सके । व्यधि को ऄपनी पसंद के
धिधधिे्त ा से परामशव करने और ईसके द्वारा बचाि करने का भी ऄधधकार है। आस तरह के फकसी व्यधि
को 24 घंटे के भीतर फकसी मधजस्ट्रेट के समक्ष ऄिश्य प्रस्तुत फकया जाना चाधहए ताफक कायवपाधलका
की फकसी भी गलत कारविाइ को सुधारा जा सके ।

204 www.visionias.in ©Vision IAS

Google it:- https://upscpdf.com


https://upscpdf.com << Download From >> https://upscpdf.com

धनिारक धनरोध का ईद्देश्य फकसी व्यधि को ऄपराध करने से रोकना है। ऐसे व्यधि के धलए भी कु छ
ऄधधकार ईपलब्ध हैं। ईसे, ईसकी धगरफ्तारी के अधार के बारे में सूधचत फकया जाना चाधहए। पुधलस
फकसी व्यधि को तीन माह से ऄधधक समय के धलए धनरुद् नहीं कर सकती जब तक फक पुधलस के पास
सलाहकार बोडव की ऄनुमधत न हो। आस तरह के सलाहकार बोडव में तीन न्यायाधीश शाधमल होंगे (आस
तरह का सलाहकार बोडव ऐसे तीन व्यधियों से धमलकर बनेगा, जो ईच्च न्यायालय के न्यायाधीश हैं या
न्यायाधीश रहे हैं ऄथिा न्यायाधीश धनयुि होने के धलए ऄर्वहत हैं)। संसद, 3 माह से ऄधधक धनरोध
हेतु कानून बना सकती है।
44िें संधिधान संशोधन ऄधधधनयम, 1978 द्वारा धनरोध की ऄिधध को धबना सलाहकार बोडव की राय
के तीन से दो माह कर फदया गया। हालांफक यह व्यिस्था ऄब तक प्रयोग में नहीं अइ, जबफक धनरोध
की मूल ऄिधध तीन माह ऄब भी जारी है।
धनिारक धनरोध की अलोचना
भारत में, आस तरह के कानूनों का दुरुपयोग फकया गया है और आसीधलए यह मानिाधधकारों से संबंधधत
लचता का धिषय बन गया है। यह राज्य की पुधलस शधि का प्रधतधनधधत्ि करता है। फकसी ऄन्य
लोकतांधत्रक देश ने ऄपने संधिधान में धनिारक धनरोध का ईल्लेख नहीं फकया है और िहाँ आस तरह के
कानून के िल अपात धस्थधत के तहत प्रभाि में अते हैं।
धनिारक धनरोध के पक्ष में फदए गए तकव
रक्षा, धिदेश और देश की सुरक्षा से संबंधधत संघ सूची की प्रधिधष्ट 9, और सािवजधनक व्यिस्था बनाए
रखने, राज्य की सुरक्षा और अिश्यक अपूर्वत और सेिाएं बनाए रखने से संबंधधत समिती सूची की
प्रधिधष्ट 3 में ऐसे धिषय संधिधान में पहले से ही दजव हैं धजन पर धनिारक कानून बनाया जा सकता है।
आस प्रकार, यह राज्य द्वारा मनमाने ढंग से की गइ कारविाइ को रोकता है।
धिधायन/कानून
धनम्नधलधखत कानूनों में फकसी व्यधि को तीन महीने से ऄधधक समय तक धनरोध फकए जाने के संबंध में
प्रािधान हैं:
 राष्ट्रीय सुरक्षा ऄधधधनयम (National Security Act: NSA), 1980
 धिदेशी मुद्रा का संरक्षण एिं व्यसन धनिारण ऄधधधनयम (COFEPOSA),1974 तथा
 अतंकिाद धनिारक कानून (Prevention of Terrorism Act: POTA), 2002
आसी प्रकार, कइ राज्यों ने भी आसी तरह के कानून बनाए हैं। कें द्रीय और राज्य स्तर के कानूनों को धमला
दें तो धनिारक धनरोध से संबंधधत ऐसे लगभग चालीस कानून ऄधस्तत्ि में हैं।
5.13 ऄनु च्छे द 23 – मानि के दुव्याव पार एिं बलात् श्रम का प्रधतषे ध

मूल पाठ
मानि के दुव्यावपार और बलात् श्रम का प्रधतषेध।
1. मानि का दुव्यावपार और बेगार तथा आसी प्रकार का ऄन्य बलात् श्रम प्रधतधषद् फकया जाता है और
आस ईपबंध का कोइ भी ईल्लंघन ऄपराध होगा जो धिधध के ऄनुसार दंडनीय होगा।
2. आस ऄनुच्छेद की कोइ बात राज्य को सािवजधनक प्रयोजनों के धलए ऄधनिायव सेिा ऄधधरोधपत
करने से धनिाररत नहीं करेगी। ऐसी सेिा ऄधधरोधपत करने में राज्य के िल धमव, मूलिंश, जाधत या
िगव या आनमें से फकसी के अधार पर कोइ धिभेद नहीं करेगा।
धििरण
 ऄनुच्छेद 23 (1) मानि के दुव्यावपार, बेगार और सभी प्रकार के बलात् श्रम को प्रधतधषद् करता
है। आसका ऄनुसूधचत जाधत/ऄनुसूधचत जनजाधत और मधहलाओं के धलए धिशेष महत्ि है। "बेगार"
ऐसी श्रम या सेिा के रूप में िर्वणत है, धजसमें एक व्यधि को आसके धलए कोइ पाररश्रधमक फदए
धबना आसे करने को बाध्य फकया जाता है।

205 www.visionias.in ©Vision IAS

Google it:- https://upscpdf.com


https://upscpdf.com << Download From >> https://upscpdf.com

 ऄनुच्छेद 23 (2) में कहा गया है फक राज्य ऄधनिायव सेिा लागू कर सकता है, यफद आसकी
अिश्यकता है। धिशेषतः, देिदासी प्रथा को ईपयुवि ऄनुच्छेद के धनषेध के कारण समाि कर फदया
गया है।
धिधान
मानि दुव्यावपार को रोकने के धलए पाररत फकए गए कु छ ऄधधधनयम धनम्नधलधखत है:
 ऄनैधतक व्यापार (धनिारण) ऄधधधनयम, 1956 (Immoral Traffic Prevention Act: ITPA,
1956)
 बंधधत श्रम पद्धत (ईत्सादन) ऄधधधनयम, 1976 {Bonded Labor System (Abolition) Act,
1976}
 फकशोर न्याय (देखभाल और संरक्षण) ऄधधधनयम, 2000 {Juvenile Justice (Care and
Protection) Act, 2000} को धनरस्त करके फकशोर न्याय (देखभाल एिं संरक्षण) ऄधधधनयम,
2015 पाररत फकया गया है, आसके कु छ प्रमुख प्रािधान धनम्न हैं:
o ऄधधधनयम में 'फकशोर' शब्द से जुड़े कइ नकारात्मक संकेताथव को समाि करने के धलए
'फकशोर' शब्द से 'बच्चे' शब्द की नामािली में पररितवन। ऄनाथ, पररत्यि और अत्मसमर्वपत
बच्चों की नइ पररभाषाओं को सधममधलत फकया गया है।
o बच्चों के छोटे, गंभीर और जघन्य ऄपराध, फकशोर न्याय बोडव (JJB) और बाल कल्याण
सधमधत (CWC) के ऄधधकारों, कायों और धजममेदाररयों में स्पष्टीकरण, फकशोर न्याय बोडव
द्वारा जांच में स्पष्ट ऄिधध, 16 िषव से उपर के बच्चों द्वारा फकए गए जघन्य ऄपराध की धस्थधत
में धिशेष प्रािधान, ऄनाथ, पररत्यि और अत्मसमर्वपत बच्चों को गोद लेने संबंधी धनयमों से
संबंधधत पृथक नया ऄध्याय, बच्चों के धिरुद् फकए गए नए ऄपराधों को शाधमल फकया गया है,
बाल कल्याण ि देखभाल संस्थानों के पंजीकरण को ऄधनिायव बनाया गया है।
o धारा 15 के ऄंतगवत 16-18 िषव की अयु के बाल ऄपराधधयों द्वारा फकए गए जघन्य ऄपराधों
के संबंध में धिशेष प्रािधान फकए गए हैं। फकशोर न्याय बोडव के पास बच्चों द्वारा फकए गए
जघन्य ऄपराधों के मामलों को प्रारंधभक अकलन के बाद ईन्हें बाल न्यायालय (कोटव ऑफ
सेशन) को स्थानांतररत करने का धिकल्प होगा।
 यह नागररकों और गैर-नागररकों दोनों के धलए ईपलब्ध है।
5.14. ऄनु च्छे द 24 – कारखानों अफद में बालकों के धनयोजन का प्रधतषे ध

मूल पाठ
कारखानों अफद में बालकों के धनयोजन का प्रधतषेध - चौदह िषव से कम अयु के फकसी बालक को फकसी
कारखाने या खान में काम करने के धलए धनयोधजत नहीं फकया जाएगा या फकसी ऄन्य पररसंकटमय
धनयोजन में नहीं लगाया जाएगा।
धििरण
 ऄनुच्छेद 24 खतरनाक व्यिसायों में बच्चों के धनयोजन का प्रधतषेध करता है। हालांफक, यह
हाधनरधहत कायों में ईनके धनयोजन का प्रधतषेध नहीं करता है।
नोट: ऄनुच्छेद 23 और 24 ऄनुच्छेद 39 (a) और 39 (f) द्वारा पूररत होते हैं।

5.14.1. बाल श्रम से सं बं धधत धिधे य क

 बाल श्रम (प्रधतषेध एिं धिधनयमन) ऄधधधनयम, 1986 बाल श्रम को रोकने के धलए एक धिधान
है। आसको बाल श्रम (धनषेध एिं धिधनयमन) संशोधन धिधेयक, 2016 के द्वारा संशोधधत फकया
गया है। आसमें फकए गए प्रमुख संशोधन धनम्नधलधखत है:

206 www.visionias.in ©Vision IAS

Google it:- https://upscpdf.com


https://upscpdf.com << Download From >> https://upscpdf.com

o धिधेयक में 14 िषव से कम अयु के बालकों के रोजगार में धनयोजन पर अरोधपत प्रधतबंध का
सभी क्षेत्रों में धिस्तार फकया गया है।
o धनशुल्क और ऄधनिायव बाल धशक्षा ऄधधधनयम, 2009 के अलोक में यह ऄधधधनयम सभी
व्यिसायों में 14 िषव से कम अयु के बच्चों के रोजगार पर प्रधतबंध अरोधपत करता है। ज्ञातव्य
है फक आसके ऄंतगवत स्कू ल के घंटों के पश्चात् ऄपने पररिार की सहायता करने िाले बच्चों या
मनोरंजन ईद्योग में कायव कर रहा बच्चों को शाधमल नहीं फकया गया है।
o यह खतरनाक व्यिसायों में 18 िषव से कम अयु के फकसी भी व्यधि को धनयोधजत करने से
प्रधतबंधधत करता है।
o आस ऄधधधनयम में "फकशोर" व्यधियों की एक नइ श्रेणी को जोड़ा गया है। एक फकशोर का
ऄथव है 14 से 18 िषव अयु का व्यधि। यह ऄधधधनयम धनर्ददष्ट खतरनाक व्यिसायों में (खानों,
ज्िलनशील पदाथव और खतरनाक प्रफक्रयाओं) में फकशोरों के रोजगार पर प्रधतबंध अरोधपत
करता है।
o कें द्र सरकार द्वारा ऄधधधनयम में शाधमल सूची में से फकसी भी खतरनाक व्यिसाय को जोड़ या
हटा सकती है।
o 14-18 िषव के फकशोरों के खतरनाक व्यिसायों में धनयोजन पर प्रधतबंध लगाया गया है और
o आन प्रािधानों के ईल्लंघन पर ऄधधक कठोर सजा; छह माह से दो िषव तक की कै द और
50,000 रुपए तक जुमावने का प्रािधान।
o धिधेयक पहले धनधावररत फकए गए 83 खतरनाक व्यिसायों की सूची को के िल तीन तक
सीधमत करता है। आनके तहत खनन, ज्िलनशील पदाथव तथा कारखाना ऄधधधनयम के ऄंतगवत
धनधावररत खतरनाक प्रफक्रयाएँ शाधमल होंगी धजन्हें कें द्र द्वारा धचधन्हत फकया जाएगा।
o धिधेयक में बच्चों के पुनिावस के धलए पुनिावस कोष धनर्वमत फकए जाने का प्रस्ताि है, धजसे
पहले ही स्थाधपत फकया जा चुका है।
 यह भी नागररकों और गैर-नागररकों दोनों के धलए ईपलब्ध है।
5.15. ऄनु च्छे द 25: ऄं तःकरण की और धमव को ऄबाध रूप से मानने , अचरण और प्रचार
करने की स्ितं त्र ता
मूल पाठ
1. लोक व्यिस्था, सदाचार और स्िास्थ्य तथा आस भाग के ऄन्य ईपबंधों के ऄधीन रहते हए, सभी
व्यधियों को ऄंतःकरण की स्ितंत्रता का और धमव के ऄबाध रूप से मानने, अचरण करने तथा
प्रचार करने का समान ऄधधकार होगा।
2. आस ऄनुच्छेद की कोइ बात फकसी ऐसी धिद्यमान धिधध के प्रितवन पर प्रभाि नहीं डालेगी या राज्य
को कोइ ऐसी धिधध बनाने से धनिाररत नहीं करेगी जो-
o धार्वमक अचरण से समबद् फकसी अर्वथक, धि्त ीय, राजनैधतक या ऄन्य लौफकक फक्रयाकलाप
का धिधनयमन या धनबवन्धन करती है;
o सामाधजक कल्याण और सुधार के धलए या सािवजधनक प्रकार की लहदुओं की धार्वमक संस्थाओं
को लहदुओं के सभी िगों और ऄनुभागों के धलए खोलने का ईपबंध करती है।
स्पष्टीकरण 1 – कृ पाण धारण करना और लेकर चलना धसख धमव के ऄनुकरण का ऄंग समझा जाएगा।
स्पष्टीकरण 2- खंड (2) के ईपखंड (b) में लहदुओं के प्रधत धनदेश का यह ऄथव लगाया जाएगा फक ईसके
ऄंतगवत धसख, जैन या बौद् धमव के मानने िाले व्यधियों के प्रधत धनदेश है और लहदुओं की धार्वमक
संस्थाओं के प्रधत धनदेश का ऄथव तदनुसार लगाया जायेगा।

207 www.visionias.in ©Vision IAS

Google it:- https://upscpdf.com


https://upscpdf.com << Download From >> https://upscpdf.com

धििरण
ऄनुच्छेद 25 के ऄनुसार लोगों को,
 ऄंतःकरण की;
 धमव को मानने (ऄपने धार्वमक धिश्वासों को खुले तौर पर घोधषत करने) की;
 धमव के अचरण (धार्वमक पूजा का प्रदशवन) की; और
 धमव के प्रसार (ऄपनी धार्वमक मान्यताओं के प्रचार-प्रसार) की स्ितंत्रता है।
यह ऄनुच्छेद भारत में पंथधनरपेक्षता का अधार है।
 ऄंतःकरण की स्ितंत्रता से अशय फकसी व्यधि के ऄपने धार्वमक धिश्वास और अस्था को अकार
देने की अतंररक स्ितंत्रता से है। राज्य व्यधि की आस अंतररक स्ितंत्रता में हस्तक्षेप नहीं कर
सकता। सािवजधनक ऄधभव्यधि में यह अतंररक स्ितंत्रता; धार्वमक पूजा, परंपरा एिं धार्वमक
प्रदशवन की स्ितंत्रता का रूप धारण करती है। धमव को मानने के ऄधधकार से अशय फकसी व्यधि के
ईसके धार्वमक धिश्वास और अस्था को खुले तौर पर व्यि करने के ऄधधकार से है। ईदाहरण के
धलए, धसखों के कृ पाण रखने के ऄधधकार को ईनके धमव को ऄबाध रूप से मानने के ऄधधकार के
ऄंतगवत माना गया है।
 अचरण करने के ऄधधकार का ऄथव धार्वमक पूजा, परंपरा, समारोह अयोधजत करने और ऄपनी
अस्था और धिचारों के प्रदशवन की स्ितंत्रता है।
 ‘प्रसार’ से तात्पयव, ऄपने धार्वमक धिश्वास का ऄन्य लोगो के जीिन को ईधचत फदशा प्रदान करने के
दृधष्टकोण से, धमव के पररष्कृ त रूप का प्रसार करना है धजसकी तार्दकक पररणधत फकसी ऄन्य को
ऄपने धमव में धमावन्तररत करने में होती है।
 ऄत: प्रसार का ऄथव ऄनुनय और धबना फकसी धोखाधड़ी, बलपूिवक ऄथिा धमावन्तरण के धलए
प्रलोभन के धबना धिचारों का प्रसार करना। ध्यातव्य है फक, फकसी व्यधि को ऄपने धमव में
धमावन्तररत करने का ऄधधकार, ईसके ऄपनी रूधच के मत में धमावन्तररत होने के व्यधिगत
ऄधधकार से धभन्न है।
 जहाँ स्िेच्छा से, ऄपने ऄंतःकरण के ऄनुसार फकसी भी मत या संप्रदाय में धमावन्तररत होना,
धनधश्चत रूप से संधिधान प्रद्त धार्वमक तथा ऄंतःकरण की स्ितंत्रता के ऄधधकार के ऄनुरूप है,
िहीं ऄपने द्वारा प्रसाररत धमव में फकसी व्यधि को धमावन्तररत करना राजनीधतक और सामाधजक
क्षेत्र में धििाद का कें द्र बन रहा है|
 आस प्रकार, ऄनुच्छेद 25 के िल धार्वमक धिश्वास को ही नहीं, ऄधपतु धार्वमक अचरण को भी
समाधहत करता है।
 हालाँफक भारतीय संधिधान में धार्वमक स्ितंत्रता का ऄधधकार धनरपेक्ष ऄधधकार नही है। आसे
सदाचार, स्िास्थ्य और लोक व्यिस्था बनाए रखने के धलए प्रधतबंधधत फकया जा सकता है। धार्वमक
मुद्दे से जुड़े लौफकक धिषयों के प्रबंध में भी राज्य का हस्तक्षेप हो सकता है, जैसे मंफदरों एिं
मधस्जदों को सभी लोगों को प्रिेश प्रदान करने के संदभव में धिधध का धनमावण फकया जा सकता है।
यह धार्वमक स्ितंत्रता के ऄंतगवत सधममधलत नहीं होगा। ऄतः भारतीय संधिधान धनमावताओं ने एक
ओर ऄन्तःकरण और धमव की स्ितंत्रता का पूणव समथवन फकया, तो दूसरी ओर सामाधजक सुधार
और लोक व्यिस्था पर भी ऄत्यधधक बल फदया है।
महत्िपूणव िाद
रधतलाल पानाचंद गाँधी बनाम बॉमबे राज्य, 1954
 आस िाद में ईच्चतम न्यायालय ने घोधषत फकया फक, “ऄन्तःकरण की स्ितंत्रता (ऄपने धमव में
धिश्वास की स्ितंत्रता) फकसी एक धमव के ऄनुयाधययों के धलए नहीं है िरन् सभी के धलए समान
रूप से ईपलब्ध होती है”।

208 www.visionias.in ©Vision IAS

Google it:- https://upscpdf.com


https://upscpdf.com << Download From >> https://upscpdf.com

स्टेधनस्लास बनाम मध्य प्रदेश राज्य, 1977

 ईच्चतम न्यायालय की संधिधान पीठ ने धनणवय फदया फक, ऄनुच्छेद 25 (1) धमांतरण का ऄधधकार

प्रदान नहीं करता है बधल्क, के िल ऄपने धमव के धसद्ांतों के प्रसार का ऄधधकार प्रदान करता है।

 आस प्रकार, भारत में के िल स्िैधच्छक धमांतरण ही मान्य है। कु छ राज्यों ने बलात् धमां तरण पर

रोक लगाने के धलए धमांतरण धिरोधी कानून भी पाररत फकए हैं।

जगदीश्वरानन्द िाद, 1984


 आस िाद में ईच्चतम न्यायालय द्वारा फदए गए धनणवय में यह कहा गया फक अनंदमार्वगयों द्वारा
कपाल लेकर नृत्य करते हए जुलूस धनकालना धमव का मूलभूत तत्ि नहीं है तथा आसे यथोधचत रूप
से प्रधतबंधधत फकया जा सकता है।

 आसी तरह, बक़रीद के ऄिसर पर गोहत्या आस्लाम के धलए अिश्यक तत्ि नहीं माना जा सकता है।

आस प्रकार, राज्य यह धिधनयधमत कर सकता है फक अिश्यक धार्वमक प्रथाएं क्या हैं और क्या नहीं

तथा जो नहीं हैं ईन्हें, ऄसामाधजक होने की धस्थधत में गैर-कानूनी घोधषत कर सकता है।
व्यिहायवता
 यह ऄधधकार नागररकों और गैर-नागररकों दोनों को प्रदान फकया गया है।
 पंथधनरपेक्षता के ऄथव से संबंधधत धिधिध व्याख्याओं और आससे संबंधधत धिरोधाभासों को दूर करने
के धलए ईच्चतम न्यायालय के नौ जजों की बेंच ने एस. अर.बोममइ िाद (1994) में ऄपने एक
धनणवय में आससे संबंधधत अशंकाओं के धनराकरण का प्रयास फकया। न्यायालय के ऄनुसार
पंथधनरपेक्षता से संबंधधत धनम्नधलधखत तथ्य हैं, जो भारतीय संदभव में प्रासंधगक हैं:
o पंथधनरपेक्षता का यह ऄथव नही है फक राज्य का धमव के प्रधत शत्रुभाि है। आसका ऄथव यह है फक
राज्य को धिधभन्न धमों के मध्य तटस्थ रहना चाधहए।
o प्रत्येक व्यधि को ऄपने धमव को मानने और ईसके ऄनुसार अचरण करने की स्ितंत्रता है। यह
तकव मान्य नही है फक यफद कोइ व्यधि धनष्ठािान धहन्दू या धनष्ठािान मुधस्लम है तो िह
पंथधनरपेक्ष नहीं रह जाता।
o यफद धमव का ईपयोग राजनीधतक प्रयोजनों के धलए फकया जाता है और राजनीधतक दल ऄपने
राजनीधतक प्रयोजनों के धलए ईसका अश्रय लेते हैं तो आससे राज्य की तटस्थता का ईल्लंघन
होगा। धमव के अधार पर धनिावचकों से ऄपील करना पंथधनरपेक्षीय लोकतंत्र के धिरुद् है।
राजनीधत और धमव को परस्पर तटस्थ होना चाधहए और ईन्हें परस्पर संबद् नहीं फकया जाना
चाधहए। यफद कोइ राज्य सरकार ऐसा करती है तो ईसके धिरुद् संधिधान के ऄनुच्छेद 356 के
ऄधीन कारविाइ ईधचत होगी। ऄतः आस ऄथव में पंथधनरपेक्षता संधिधान का मूलभूत लक्षण होगी।

5.16. ऄनु च्छे द 26: धार्वमक कायों के प्रबं ध की स्ितं त्र ता

मूलपाठ

लोक व्यिस्था, सदाचार और स्िास्थ्य के ऄधीन रहते हए, प्रत्येक धार्वमक संप्रदाय या ईसके फकसी
ऄनुभाग को
 धार्वमक और धमावथव प्रयोजनों के धलए संस्थाओं की स्थापना और पोषण का;
 ऄपने धमव धिषयक कायों का प्रबंध करने का;
 जंगम और स्थािर संपध्त के ऄजवन और स्िाधमत्ि का; और
 ऐसी संपध्त का धिधध के ऄनुसार प्रशासन करने का ऄधधकार होगा।

209 www.visionias.in ©Vision IAS

Google it:- https://upscpdf.com


https://upscpdf.com << Download From >> https://upscpdf.com

धििरण
ऄनुच्छेद 26 के ऄनुसार प्रत्येक धार्वमक संप्रदाय को ऄधधकार है:

 धार्वमक एिं धमावथव प्रयोजनों के धलए संस्थाओं की स्थापना और धनिावह करने का,
 ऄपने स्ियं के मामलों का प्रबंधन और
 आसके धलए संपध्त का ऄधधग्रहण।
नोट: यद्यधप ितवमान में व्यधि के धलए संपध्त का ऄधधकार मूल ऄधधकार नहीं है, तथाधप यह धार्वमक
संप्रदाय के संबंध में ईनका मूल ऄधधकार है।
 राज्य ऐसी संपध्त के प्रशासन को धिधनयधमत करने के धलए कानून बना सकता है, लेफकन
प्रशाधसत करने के ऄधधकार को समग्र रूप से समाि नहीं कर सकता है।
हालांफक, यह स्ितंत्रता सािवजधनक व्यिस्था, नैधतकता और स्िास्थ्य के ऄधीन है।

ऄनुच्छेद 25 के साथ संबध


 ऄनुच्छेद 25, जहाँ फकसी व्यधि को धमव की स्ितंत्रता प्रदान करता है, िहीं ऄनुच्छेद 26 एक

धार्वमक संप्रदाय या ईसके फकसी ऄनुभाग से संबंधधत है।

ऄनुच्छेद 26 के ऄंतगवत, प्रद्त ऄधधकार नागररकों एिं धिदेशी व्यधियों दोनों को ईपलब्ध हैं।

5.17. ऄनु च्छे द 27: फकसी धिधशष्ट धमव की ऄधभिृ धद् के धलए करों के सं दाय के बारे में
स्ितं त्र ता
मूल पाठ
फकसी भी धिधशष्ट धमव की ऄधभिृधद् के धलए करों के संदाय के बारे में स्ितंत्रता - फकसी भी व्यधि को
ऐसे करों का संदाय करने के धलए बाध्य नहीं फकया जाएगा धजनके अगम फकसी धिधशष्ट धमव या धार्वमक
संप्रदाय की ऄधभिृधद् या पोषण में व्यय करने के धलए धिधनर्ददष्ट रूप से धिधनयोधजत फकए जाते हैं।
धििरण
 ऄनुच्छेद 27, राज्य द्वारा करों के माध्यम से एकत्र सािवजधनक धन को फकसी भी धमव की ऄधभिृधद्
के धलए व्यय करने पर प्रधतबंध अरोधपत करता है। यह पंथधनरपेक्षता की मूल ऄिधारणा के
ऄनुरूप है। राज्य फकसी धिशेष धमव या धार्वमक संप्रदाय का संरक्षण नहीं कर सकता है। दूसरे शब्दों
में, कर के माध्यम से एकत्र लोगों के धन को राज्य द्वारा फकसी धिशेष धमव के रखरखाि या प्रसार

पर व्यय नहीं फकया जाना चाधहए। यह प्रािधान, राज्य द्वारा ऄन्य धमों की तुलना में फकसी एक
धमव के पक्ष में संरक्षण और समथवन फकए जाने पर प्रधतबंध अरोधपत करता है। आसका ऄथव यह है
फक करों का ईपयोग सभी धमों के समान ऄनुरक्षण या प्रसार के धलए फकया जा सकता है, फकसी
धमव धिशेष के धलए नहीं।
 यह प्रािधान के िल कर िसूलने पर प्रधतबंध लगाता है; शुल्क िसूलने पर नहीं। आसका कारण यह

है फक शुल्क का ईद्देश्य धार्वमक संस्थाओं के प्रशासन का पंथधनरपेक्ष स्िरूप को बनाए रखना है, न

फक धमव का ऄनुरक्षण या प्रसार करना। आस प्रकार, कु छ धिशेष सेिा या सुरक्षा ईपाय प्रदान करने
के धलए तीथवयाधत्रयों से शुल्क िसूला जा सकता है तथा धिधनयमन व्यय को पूरा करने के धलए
धार्वमक धनधध पर शुल्क लगाया जा सकता है।
ऄनुच्छेद 27 के ऄंतगवत, प्रद्त ऄधधकार नागररकों एिं धिदेशी व्यधियों दोनों को ईपलब्ध हैं।

210 www.visionias.in ©Vision IAS

Google it:- https://upscpdf.com


https://upscpdf.com << Download From >> https://upscpdf.com

5.18. ऄनु च्छे द 28: कु छ धशक्षा सं स्थाओं में धार्वमक धशक्षा या धार्वमक ईपासना में ईपधस्थत
होने के बारे में स्ितं त्र ता
मूल पाठ
1. राज्य-धनधध से पूणवतः पोधषत फकसी धशक्षा संस्था में कोइ धार्वमक धशक्षा नहीं दी जाएगी।
2. खंड (1) की कोइ बात ऐसी धशक्षा संस्था को लागू नहीं होगी धजसका प्रशासन राज्य करता है
फकन्तु जो फकसी ऐसे धिन्यास या न्यास के ऄधीन स्थाधपत हइ है धजसके ऄनुसार ईस संस्था में
धार्वमक धशक्षा देना अिश्यक है।
3. राज्य से मान्यता प्राि या राज्य-धनधध से सहायता प्राि संस्था में ईपधस्थत होने िाले फकसी व्यधि
को ऐसी संस्था में दी जाने िाली धार्वमक धशक्षा में भाग लेने के धलए या ऐसी संस्था में या ईससे
संलग्न स्थान में की जाने िाली धार्वमक ईपासना में ईपधस्थत होने के धलए तब तक बाध्य नहीं
फकया जाएगा जब तक फक ईस व्यधि ने, या यफद िह ऄियस्क है तो ईसके संरक्षक ने , आसके धलए
ऄपनी स्िीकृ धत प्रदान न की हो।
धििरण
ऄनुच्छेद 28 के ऄनुसार,
 राज्य धनधध से पूणवतः पोधषत फकसी धशक्षा संस्था में कोइ धार्वमक धशक्षा नहीं दी जाएगी।
 हालांफक, यह प्रािधान फकसी धमवस्थ संस्था या ट्रस्ट द्वारा स्थाधपत एिं राज्य द्वारा प्रशाधसत
संस्थानों पर लागू नहीं होगा।

 आसके ऄधतररि, राज्य द्वारा मान्यता प्राि या अर्वथक सहायता प्राि धशक्षण संस्था में फकसी व्यधि
को फकसी धमव धिशेष की धशक्षा ग्रहण करने के धलए बाध्य नहीं फकया जा सके गा। हालाँफक, फकसी
व्यधि को ईसकी सहमधत पर धार्वमक धनदेश प्रदान फकया जा सकता है। ध्यातव्य है फक व्यधि के
ऄल्पियस्क होने की धस्थधत में ईसके ऄधभभािक की सहमधत अिश्यक है।

5.19. ऄनु च्छे द 29: ऄल्पसं ख्यक-िगों के धहतों का सं र क्षण

मूल पाठ
(1) भारत के राज्यक्षेत्र या ईसके फकसी भाग के धनिासी नागररकों के फकसी ऄनुभाग को, धजसकी
ऄपनी धिशेष भाषा, धलधप या संस्कृ धत है, ईसे बनाए रखने का ऄधधकार होगा।
(2) राज्य द्वारा पोधषत या राज्य-धनधध से सहायता प्राि फकसी धशक्षा संस्था में प्रिेश से फकसी भी
नागररक को के िल धमव , मूलिंश, जाधत, भाषा या आनमें से फकसी भी अधार पर िंधचत नहीं फकया
जाएगा।
धििरण
 ऄनुच्छेद 29(1) फकसी नागररक को ईसकी भाषा, धलधप एिं संस्कृ धत के संरक्षण का ऄधधकार

प्रदान करता है। ऄनुच्छेद 29(2) राज्य द्वारा शैक्षधणक संस्थाओं में प्रिेश की ऄनुमधत देने में फकए

जाने िाले धिभेद का प्रधतषेध करता है।


 धिशेष: ऄनुच्छेद 15 भाषा को धिभेद के अधार के रूप में ईधल्लधखत नहीं करता है, जबफक

ऄनुच्छेद 29 में भाषा को शाधमल फकया गया है।

 ऄनुच्छेद 29 भाषायी और धार्वमक ऄल्पसंख्यकों, दोनों को संरक्षण प्रदान करता है। परंतु , ईच्चतम
न्यायालय ने स्पष्ट फकया है फक आस ऄनुच्छेद का कायवक्षेत्र के िल ऄल्पसंख्यकों तक सीधमत नहीं है।
ऄधपतु, यह जनसंख्या के “सभी िगों’’ को सधममधलत करता है धजसमें बहसंख्यक भी शाधमल हैं।

211 www.visionias.in ©Vision IAS

Google it:- https://upscpdf.com


https://upscpdf.com << Download From >> https://upscpdf.com

 चमपकम दोराइराजन के िाद (1951) में धपछड़े िगों को प्रदान फकए गए अरक्षण को आस अधार
पर चुनौती दी गइ फक यह ऄनुच्छेद 29(2) का ईल्लंघन करता है। आसकी प्रधतफक्रया में संधिधान
का प्रथम संशोधन ऄधधधनयम पाररत फकया गया, धजसके तहत अरक्षण के प्रािधान सुधनधश्चत
करने के धलए ऄनुच्छेद 15 (4) को प्रधिष्ट फकया गया।
 ऄनुच्छेद 29 और 30 दोनों के िल भारतीय नागररकों पर लागू होते हैं।

5.20. ऄनु च्छे द 30: धशक्षा सं स्थाओं की स्थापना और प्रशासन करने का ऄल्पसं ख्यक िगों का
ऄधधकार
मूलपाठ
1. धमव या भाषा पर अधाररत सभी ऄल्पसंख्यक-िगों को ऄपनी रुधच की धशक्षा संस्थाओं की
स्थापना और प्रशासन का ऄधधकार होगा।
खंड (1) में धनर्ददष्ट फकसी ऄल्पसंख्यक-िगव द्वारा स्थाधपत और प्रशाधसत धशक्षा संस्था की संपध्त के
ऄधनिायव ऄजवन के धलए ईपबंध करने िाली धिधध बनाते समय, राज्य यह सुधनधश्चत करेगा फक
ऐसी संपध्त के ऄजवन के धलए ऐसी धिधध द्वारा धनयत या ईसके ऄधीन ऄिधाररत राधश आतनी हो
फक ईस खंड के ऄधीन प्रत्याभूत ऄधधकार धनबवधन्धत या धनराकृ त न हो जाए।
2. धशक्षा संस्थाओं को सहायता देने में राज्य फकसी धशक्षा संस्था के धिरुद् आस अधार पर धिभेद नहीं
करेगा फक िह धमव या भाषा पर अधाररत फकसी ऄल्पसंख्यक-िगव के प्रबंध में संलग्न है।
धििरण
 ऄनुच्छेद 30 के ऄंतगवत ऄल्पसंख्यकों (भाषायी या धार्वमक) को ऄपनी रूधच के शैधक्षक संस्थानों
की स्थापना एिं प्रशासन का ऄधधकार प्रदान फकया गया है। राज्य धशक्षा के क्षेत्र में ईत्कृ ष्टता को
बढ़ािा देने िाले धनयमों के धनमावण के ऄधतररि, ऄल्पसंख्यकों के आस ऄधधकार पर कोइ प्रधतबंध
नहीं लगा सकता।
 फकसी ऄल्पसंख्यक संस्था की संपध्त का राज्य द्वारा ऄधधग्रहण कर धलए जाने की धस्थधत में यह
ईसके धलए पयावि क्षधतपूर्वत का प्रािधान करता है।
 आस तरह के संस्थानों को सहायता ईपलब्ध कराने में राज्य कोइ भेदभाि नहीं करेगा।
 हालाँफक, संधिधान में ‘ऄल्पसंख्यक’ शब्द का ऄथव व्याख्याधयत नहीं फकया गया है। िस्तुतः आसका
अशय ‘गैर-प्रभािी’ समूह (non-dominant group) से है।
 के रल धशक्षा धिधेयक तथा तत्पश्चात गुरु नानक देि धिश्वधिद्यालय के िाद में राष्ट्रपधत को दी गयी
ऄपनी सलाह में न्यायालय ने ऄल्पसंख्यकों की धस्थधत के धनधावरण हेतु कु छ मानक धनधावररत फकए
हैं। कें द्रीय स्तर पर, आसका अशय ईन समूहों से है धजनकी जनसंख्या, ऄधखल भारतीय स्तर पर
50 प्रधतशत से कम है। आसी प्रकार, राज्य स्तर पर भी राज्य की जनसंख्या के 50 प्रधतशत से कम
जनसंख्या िाले समूह ऄल्पसंख्यक समूह माने जाते हैं।

5.20.1. ऄनु च्छे द 29 तथा 30 के मध्य सं बं ध


ऄनुच्छेद 29 जनसंख्या के सभी िगों के धलए ईपलब्ध एक साधारण संरक्षण प्रािधान है। जबफक,
ऄनुच्छेद 30 के तहत के िल भाषाइ या धार्वमक ऄल्पसंख्यकों के धलए संरक्षण ईपलब्ध है।
ऄनुच्छेद 29 और 30 दोनों, के िल भारतीय नागररकों के धलए ईपलब्ध हैं।
महत्त्िपूणव िाद
सेंट स्टीफें स बनाम फदल्ली धिश्वधिद्यालय, 1992
 आस िाद में ईच्चतम न्यायालय ने अदेश फदया फक ऄल्पसंख्यक संस्थानों को ऄपनी िार्वषक प्रिेश
प्रफक्रया के दौरान कम से कम 50 प्रधतशत स्थान ऄन्य िगों के धलए ईपलब्ध करना चाधहए तथा
ऄन्य िगों के प्रिेश के धलए के िल योग्यता को ही अधार माना जाना चाधहए।

212 www.visionias.in ©Vision IAS

Google it:- https://upscpdf.com


https://upscpdf.com << Download From >> https://upscpdf.com

TMA पाइ फाईं डेशन और ऄन्य बनाम कनावटक राज्य, 2002


आस ऐधतहाधसक धनणवय की प्रमुख धिशेषताएँ धनम्नधलधखत हैं:
 सभी नागररकों को शैधक्षक संस्थान स्थाधपत करने और ईनके प्रशासन का ऄधधकार है।

 ऄल्पसंख्यक धशक्षा संस्थानों के प्रशासन का ऄधधकार अत्यंधतक नहीं है।


 राज्य द्वारा गैर-सहायता प्राि ऄल्पसंख्यक धशक्षा संस्थानों की शैधक्षक ईत्कृ ष्टता सुधनधश्चत करने के
धलए धनयम बनाए जा सकते हैं।
 धि्त पोधषत ऄल्पसंख्यक धशक्षा संस्थानों में प्रिेश हेतु ऄहव गैर-ऄल्पसंख्यक छात्रों का प्रधतशत
राज्य ऄथिा धिश्वधिद्यालय द्वारा धनधावररत फकया जाएगा।

 गैर-सहायता प्राि ऄल्पसंख्यक धशक्षा संस्थानों के शुल्क को धिधनयधमत नहीं फकया जाएगा, परंतु

कोइ भी संस्थान कै धपटेशन फीस नहीं िसूल सकता।

आस्लाधमक एके डमी ऑफ़ एजुकेशन बनाम कनावटक राज्य, 2003

 आस िाद में, ईच्चतम न्यायालय ने TMA पाइ िाद में फदए गए ऄपने धनणवय को और ऄधधक स्पष्ट

फकया। धनणवय के ऄनुसार, ऄनुच्छेद 30 भाषायी एिं धार्वमक ऄल्पसंख्यकों को शैधक्षक संस्थानों की

स्थापना के धलए संपूणव ऄधधकार प्रदान करता है, परंतु सरकार ईच्च मानकों को सुधनधश्चत करने के

धलए धनयम बना सकती है तथा ईन्हें धनयंधत्रत कर सकती है।

5.21. ऄनु च्छे द 31: सं प ध्त का ऄधनिायव ऄजव न (धनरधसत)

 आस ऄनुच्छेद के तहत संपध्त का ऄधधकार प्रदान फकया गया था। परंत,ु आसे 1978 में 44िें
संधिधान संशोधन ऄधधधनयम द्वारा धनरस्त कर फदया गया तथा ितवमान में यह एक मूल ऄधधकार
नहीं िरन् ऄनुच्छेद 300A के तहत एक साधारण धिधधक ऄधधकार है।

 19(1)(f) के द्वारा प्रद्त आस मूल ऄधधकार को ऄनुच्छेद 31 द्वारा पूणवता प्रदान की गइ। िषव

1978 में 44िें संधिधान संशोधन के माध्यम से आन दोनों प्रािधानों को धनरधसत कर संधिधान में

ऄनुच्छेद 300A समाधिष्ट फकया गया। आसके ऄनुसार “फकसी भी व्यधि को धबना ईधचत धिधधक

प्राधधकरण के ईसकी धनजी संपध्त से िंधचत नहीं फकया जा सके गा।” आसका ऄथव है-

 आस प्रकार, ितवमान में संपध्त का ऄधधकार मूल ऄधधकार नहीं है ऄधपतु यह एक संिैधाधनक एिं

धिधधक ऄधधकार है। आसके ईल्लंघन के धलए कोइ सीधे ईच्च या ईच्चतम न्यायालय नहीं जा सकता है।

 आसके ऄधतररि, यह व्यधि को के िल एकपक्षीय कायवपाधलका की कायविाही से सुरक्षा प्रदान करता

है, एकपक्षीय धिधायी प्रफक्रयाओं से नहीं।


 ऄधधग्रहण की धस्थधत में राज्य ऄधनिायव रूप से क्षधतपूर्वत प्रदान करने के धलए बाध्य नहीं है।
ऄनुच्छेद 31A: संपदाओं अफद के ऄजवन के धलए ईपबंध करने िाली धिधधयों की व्यािृध्त

 ऄनुच्छेद 31A को मूलत: संपध्त का ऄधधकार (ऄनुच्छेद 31), ऄनुच्छेद 14 और ऄनुच्छेद

19(1)(f) के ऄपिाद के रूप में ऄधधधनयधमत फकया गया था, संपध्त के ऄधधकार के ईत्सादन के
पश्चात् भी आस ऄनुच्छेद को बनाए रखा गया है।
 यह कानूनों की 5 श्रेधणयों को ऄनुच्छेद 14 और ऄनुच्छेद 19 द्वारा प्रद्त मूल ऄधधकारों के
ईल्लंघन के अधार पर चुनौती फदए जाने और ऄिैध ठहराए जाने से सुरधक्षत करता है। ये श्रेधणयाँ
कृ धष, भूधम सुधार, ईद्योग और िाधणज्य अफद से संबंधधत हैं।

213 www.visionias.in ©Vision IAS

Google it:- https://upscpdf.com


https://upscpdf.com << Download From >> https://upscpdf.com

 यह ऄनुच्छेद प्रथम संधिधान संशोधन ऄधधधनयम द्वारा जोड़ा गया था। यह राज्य को व्यधिगत
संपध्त यों के ऄजवन की ऄनुमधत प्रदान करता है। संसद और राज्य धिधानमंडल दोनों आस अशय के
कानूनों का धनमावण कर सकते हैं। हालाँफक यह ऄनुच्छेद राज्य द्वारा धनर्वमत फकसी कानून को तब
तक ईन्मुधि नहीं प्रदान करता, जब तक फक ईसे राष्ट्रपधत के धिचार के धलए सुरधक्षत न कर धलया
जाए और राष्ट्रपधत द्वारा आस अशय की सहमधत प्राि न हो जाए।
 यफद राज्य फकसी ऐसे व्यधि की भूधम का ऄधधग्रहण करे , जो ईसकी कृ धष भूधम के ऄधीन है तथा
यह भूधम कानूनी रूप से लागू हदबंदी सीमा के भीतर है तो यह ऄनुच्छेद ईसे आसके एिज में
बाजार कीमतों अधार पर मुअिजे का प्रािधान करता है।
ऄनुच्छेद 31B: कु छ ऄधधधनयमों और धिधनयमों का धिधधमान्यीकरण
ऄनुच्छेद 31B, निीं ऄनुसूची में सधममधलत कानूनों को ऄनुच्छेद 14 और 19 के ईल्लंघन के अधार पर
ऄिैध ठहराए जाने से सुरक्षा प्रदान करता है।
9िीं ऄनुसच
ू ी से संबधं धत धििाद
 भूधम सुधारों को बढ़ािा देने के धलए संधिधान संशोधन के माध्यम से िषव 1951 में संधिधान में
9िीं ऄनुसूची शाधमल की गइ। आस ऄनुसूची का मूलभूत ईद्देश्य जमींदारी प्रथा को समाि करना
था। हालाँफक बाद में, आसका दुरुपयोग भी फकया जाने लगा और ितवमान में 9िीं ऄनुसूची में भूधम
सुधार धनयमों के ऄधतररि ऄन्य ऄनेक धििाफदत धनयम भी सधममधलत कर धलए गए हैं यथा:
तधमलनाडु का 69 प्रधतशत अरक्षण का धनयम जो अरक्षण हेतु ईच्चतम न्यायालय द्वारा धनधावररत
50 प्रधतशत की ईच्चतम सीमा का ईल्लंघन करता है।
 9िीं ऄनुसूची में सधममधलत सभी धिषयों को पूणव सुरक्षा प्रदान करने िाला ऄनुच्छेद 31B,
भूतलक्षी प्रभाि भी रखता है। ऄतः यफद फकसी धिधान को फकसी न्यायालय द्वारा ऄसंिैधाधनक भी
घोधषत कर फदया गया हो तो आस ऄनुसूची में शाधमल होने की धस्थधत में आसे लागू होने की धतधथ
से संिैधाधनक माना जाएगा।
 हालाँफक, िषव 2007 के अइ. अर. कोएल्हो िाद में ईच्चतम न्यायालय ने धनणवय फदया फक 24
ऄप्रैल 1973 (के शिानंद भारती िाद के धनणवय की धतधथ) के पश्चात 9िीं ऄनुसूची में शाधमल
फकसी भी कानून को ऄनुच्छेद 14, 19, 20 और 21 द्वारा प्रद्त मूल ऄधधकारों के ईल्लंघन के
अधार पर चुनौती दी जा सकती है। ईच्चतम न्यायालय ने यह भी कहा है फक यफद 9िीं ऄनुसूची में
सधममधलत कोइ कानून संधिधान द्वारा प्रद्त मूल ऄधधकारों को आस प्रकार संधक्षि या धनषेधधत
करते हैं फक संधिधान के मूल ढांचे को हाधन पहँचता है तो ऐसे कानून न्याधयक पुनर्विलोकन के
ऄधीन होंगे तथा ईन्हें शून्य माना जाएगा।

ऄनुच्छेद 31C: कु छ नीधत धनदेशक तत्िों को प्रभािी बनाने िाले कानूनों का संरक्षण
 ऄनुच्छेद 31C को 25िें संधिधान संशोधन द्वारा िषव 1971 में शाधमल फकया गया था। यह
ऄनुच्छेद 39(b) और 39(c) में धनधहत नीधत धनदेशक तत्िों के को लागू करने हेतु धनर्वमत फकसी
कानून को ऄनुच्छेद 14, 19 तथा 31 के ईल्लंघन के अधार पर ऄिैध ठहराए जाने से सुरक्षा
प्रदान करता है।
 मूलतः ऄनुच्छेद 31(C) के दो भाग थे। प्रथम, भाग ऄनुच्छेद 39(b) और 39(c) में ईधल्लधखत
धसद्ांतों पर धनर्वमत राज्य की फकसी नीधत को ऄनुच्छेद 14, 19 तथा 31 द्वारा प्रद्त मूल
ऄधधकारों के ईल्लंघन के अधार पर शून्य घोधषत न फकए जाने का प्रािधान करता है।

214 www.visionias.in ©Vision IAS

Google it:- https://upscpdf.com


https://upscpdf.com << Download From >> https://upscpdf.com

 जबफक, आसका दूसरा भाग ऄनुच्छेद 39(b) और 39(c) को प्रभािी बनाने के ईद्देश्य से धनर्वमत
धिधधयों को न्याधयक पुनर्विलोकन के दायरे से बाहर करता है और यह प्रािधान करता है फक ऐसी
धिधध न्यायालय में आस अधार पर प्रश्नगत नहीं की जाएगी फक ईसने ऄनुच्छेद 39(b) और 39(c)
को प्रभािी बनाया है ऄथिा नहीं।
 हालाँफक, के शिानंद भारती िाद में ईच्चतम न्यायालय ने आसके दूसरे भाग को ऄसंिैधाधनक एिं
शून्य घोधषत कर फदया क्योंफक यह न्यायालय द्वारा न्याधयक पुनर्विलोकन की शधि को सीधमत
करता था, जो फक संधिधान के मूल ढांचे का भाग है।

 बाद में, 42िें संधिधान संशोधन के माध्यम से आस ऄनुच्छेद के दायरे को धिस्तृत कर सभी नीधत
धनदेशक तत्िों को फकसी भी मूल ऄधधकार पर प्राथधमकता प्रदान कर दी गयी। आसके ऄनुसार कोइ
भी ऐसा कानून जो फकसी भी नीधत धनदेशक तत्ि {न फक के िल ऄनुच्छेद 39(b) और 39(c)} को
प्रभािी बनाता है, ईसे ऄनुच्छेद 14 तथा 19 के ईल्लंघन के अधार पर शून्य घोधषत नहीं फकया

जा सकता।

 हालाँफक, िषव 1980 में धमनिाव धमल िाद में ईच्चतम न्यायालय द्वारा ईपयुवि प्रािधान को समाि
कर फदया गया एिं मूल ऄधधकारों एिं नीधत धनदेशक तत्िों के मध्य पुनः संतल
ु न स्थाधपत कर
फदया गया।

5.22. ऄनु च्छे द 32 : सं िै धाधनक ईपचारों का ऄधधकार

मूल पाठ

भाग 3 द्वारा प्रद्त ऄधधकारों को प्रिर्वतत कराने के धलए ईपचार -


 आस भाग द्वारा प्रद्त ऄधधकारों को प्रिर्वतत कराने के धलए समुधचत कायविाधहयों द्वारा ईच्चतम
न्यायालय में समािेदन करने का ऄधधकार प्रत्याभूत फकया जाता है।
 आस भाग द्वारा प्रद्त ऄधधकारों में से फकसी को प्रिर्वतत कराने के धलए ईच्चतम न्यायालय को ऐसे
धनदेश या अदेश या ररट (बंदी प्रत्यक्षीकरण, परमादेश, प्रधतषेध, ऄधधकार-पृच्छा और ईत्प्रेषण
ररट), जो भी समुधचत हो, जारी करने की शधि होगी।
 ईच्चतम न्यायालय को खंड (1) और खंड (2) द्वारा प्रद्त शधियों पर प्रधतकू ल रूप से प्रभाधित
फकए धबना, संसद, ईच्चतम न्यायालय द्वारा खंड (2) के ऄधीन प्रयोिव्य फकन्हीं या सभी शधियों
का फकसी ऄन्य न्यायालय को ऄपनी ऄधधकाररता की स्थानीय सीमाओं के भीतर प्रयोग करने के
धलए धिधध द्वारा सशि कर सके गी ऄथावत् संसद को यह शधि प्राि है फक िह फकसी ऄन्य
न्यायालय को सभी प्रकार के धनदेश, अदेश और ररट जारी करने की शधि प्रदान करे।
 आस संधिधान द्वारा ऄन्यथा ईपबंधधत के ऄधतररि, आस ऄनुच्छेद द्वारा प्रत्याभूत ऄधधकार
धनलंधबत नहीं फकया जाएगा।
धििरण
 ऄनुच्छेद 32 (1) मूल ऄधधकारों के प्रितवन के धलए ईच्चतम न्यायालय में समािेदन एिं ईपचार का

ऄधधकार प्रदान करता है। हालांफक, आसमें समुधचत कायविाही के माध्यम से ही ईच्चतम न्यायालय
में याधचका करने का ईल्लेख है। यह ईच्चतम न्यायालय का क्त वव्य और व्यधियों का ऄधधकार है।
 ईच्चतम न्यायालय समुधचत कायविाही को धनधावररत कर सकता है। परंपरागत दृधष्टकोण यह है फक
न्यायालय में समािेदन करने िाले व्यधि द्वारा िाद ईधचत प्रफक्रया (ऄथिा स्ियं ईपधस्थत होकर)
के तहत फकया जाना चाधहए। हालांफक, ईच्चतम न्यायालय ने आस दृधष्टकोण को जनधहत याधचका,
स्ितः संज्ञान (Suo Moto), पत्र व्यिहार अफद प्रफक्रयाओं के द्वारा ईदार बनाया है।

215 www.visionias.in ©Vision IAS

Google it:- https://upscpdf.com


https://upscpdf.com << Download From >> https://upscpdf.com

जनधहत याधचका (PIL)

 USA के सामाधजक धहत याधचका (SIL) से ऄपनाया गया।


 यह न तो धनजी धहत याधचका है और न ही राजनीधतक धहत याधचका है।
 यह कोइ मूल ऄधधकार नहीं है।
आधपस्टलेरी जूररधस्डक्शन (Epistolary jurisdiction)

 पोस्ट काडव, पत्र के अधार पर कारविाइ करना।

स्ित: संज्ञान लेना (Suo moto)


 ईच्चतम न्यायालय स्ित: संज्ञान के अधार पर कारविाइ कर सकता है।
लेश का धसद्ांत (Doctrine of Laches)

 ईच्चतम न्यायालय ईन लोगों के ऄधधकारों की रक्षा करता है जो ऄपने ऄधधकारों के बारे


में सतकव हैं।
 ऄधधकारों को लागू करने के धलए न्यायालयों से संपकव करने में ऄनािश्यक देरी के

मामले में, ईच्चतम न्यायालय ररट जारी करने से ऄस्िीकार कर सकता है।

ऄनुच्छेद 32 का महत्ि

 सभी मूल ऄधधकारों में संिैधाधनक ईपचारों का ऄधधकार ऄत्यंत महत्िपूणव ऄधधकार है। साथ ही,
मूल ऄधधकारों के प्रितवन के धलए एक प्रभािी तंत्र का धिद्यमान होना ऄधनिायव हैं एिं ईपचारों के
धबना ऄधधकारों की घोषणा मूल्यहीन है।

 ऄनुच्छेद 32 की ऄनुपधस्थधत में ऄन्य मूल ऄधधकारों की ईपादेयता संफदग्ध हो जाती है, क्योंफक
यह मूल ऄधधकार ही नागररकों को फकसी ऄन्य मूल ऄधधकारों की ईल्लंघन की दशा में न्यायालय
जाने का ऄधधकार प्रदान करता है और धिधाधयका या कायवपाधलका द्वारा फकसी भी मूल ऄधधकारों
के ईल्लंघन करने िाली धिधध को शून्य घोधषत करता है ऄथावत यह ईच्चतम न्यायालय की न्याधयक
पुनर्विलोकन शधियों को भी दशावता है। ईच्चतम न्यायालय ने अइ.अर. कोएल्हो बनाम

तधमलनाडु राज्य (2007) िाद में कहा है फक ऄनुच्छेद 32 संधिधान के मूल ढाँचे का ऄधभन्न ऄंग
है। आस ऄनुच्छेद के द्वारा ही मूल ऄधधकारों के संरक्षक के रूप में ईच्चतम न्यायालय की स्थापना
की गइ है। मूल ऄधधकार के महत्ि को देखते हए ही भीमराि ऄंबड
े कर ने आस ऄनुच्छेद को
भारतीय संधिधान की अत्मा कहा था।
 आस ऄनुच्छेद के ऄंतगवत ईच्चतम न्यायालय को संधिधान द्वारा प्रद्त मूल ऄधधकारों को लागू करने
के धलए अिश्यक धनदेश, अदेश, लेख या ररट जारी करने का ऄधधकार प्राि है। यह ऄनुच्छेद
धिशेष रूप से धनम्नधलधखत लेखों का ईल्लेख करता है :
(i) बंदी प्रत्यक्षीकरण (Habeas Corpus): बंदी प्रत्यक्षीकरण ररट ईच्चतम न्यायालय या ईच्च

न्यायालय द्वारा ईस व्यधि के संबंध में जारी एक अदेश है, धजसे धहरासत में धलया गया है या
धनरुद् फकया गया हो (आसका ऄथव ईसकी स्ितंत्रता के मूल ऄधधकार का ईल्लंघन है)। आसके तहत
न्यायालय, धहरासत में धलए गए व्यधि को सशरीर न्यायालय के समक्ष प्रस्तुत करने का अदेश

जारी करता है। तत्पश्चात, न्यायालय धहरासत में धलए जाने के कारणों की जाँच करता है और

यफद ईसकी धहरासत से संबंधधत कोइ कानूनी औधचत्य धसद् नहीं होता है, तो ईसे ररहा फकया जा
सकता है।

216 www.visionias.in ©Vision IAS

Google it:- https://upscpdf.com


https://upscpdf.com << Download From >> https://upscpdf.com

(ii) परमादेश (Mandamus): आसका शाधब्दक ऄथव है ‘हम अदेश देते हैं’। आसे ईच्चतम न्यायालय

या ईच्च न्यायालय द्वारा फकसी व्यधि, धनगम, ऄधीनस्थ न्यायालय, सािवजधनक प्राधधकरण या

राज्य प्राधधकरण को जारी फकया जाता है, ताफक आनसे ईनके कायों और ईसे नकारने के संबंध में

पुचा जा सके ।

(iii) ईत्प्रेषण (Certiorari): आसका शाधब्दक ऄथव है: “प्रमाधणत होना” या “सूचना देना”। ईत्प्रेषण

ररट को पहले से ही एक ऄिर न्यायालय, ऄधधकरण या ऄधव-न्याधयक प्राधधकारी द्वारा पाररत


अदेश को धनरस्त करने के धलए ईच्चतम न्यायालय या फकसी ईच्च न्यायालय द्वारा जारी फकया जा
सकता है। ईत्प्रेषण ररट जारी करने के धलए कइ पररधस्थधतयों का होना अिश्यक है। जब भी कोइ
धिधधक धनकाय धजसे जनता के ऄधधकार को प्रभाधित करने िाले फकसी प्रश्न को ऄिधाररत करने
का धिधधक प्राधधकार प्राि है और धजसका क्त वव्य है फक िह न्याधयक रीधत से कायव करे , यफद िह
ऄपने धिधधक प्राधधकार से आतर कायव करता है तो ईसकी ऄधधकाररता से आतर धिधनश्चय को रद्द
करने के धलए ईत्प्रेषण ररट जारी की जाती है।

(iv) प्रधतषेध (Prohibition): प्रधतषेध ररट का ऄथव “रोकना या मना करना” होता है और यह
स्थगन अदेश (स्टे अडवर) के रूप में लोकधप्रय है। यह ररट तब जारी की जाती है जब कोइ
ऄधीनस्थ न्यायालय या धनकाय ऄपने प्राधधकार के ऄधतक्रमण का प्रयास करता है। प्रधतषेध ररट
फकसी ऄधीनस्थ न्यायालय या ऄधव-न्याधयक धनकाय को फकसी धिशेष िाद में, जहाँ आन धनकायों

को कारविाइ का ऄधधकार प्राि न हो, कायविाही करने से रोकने के धलए ईच्च न्यायालय या ईच्चतम
न्यायालय द्वारा जारी की जाती है। आस ररट के जारी फकए जाने के पश्चात ऄधीनस्थ न्यायालय में
होने िाली कायविाही स्थधगत हो जाती है।
प्रधतषेध और ईत्प्रेषण में ऄंतर :
जहां प्रधतषेध ररट प्रफक्रया या कायविाही के धनलंबन के दौरान ईपलब्ध होती हैं, िही ईत्प्रेषण ररट
के िल अदेश या धनणवय की ईद्घोषणा के पश्चात ही जारी की जा सकती है। दोनों ही ररट धिधधक
धनकायों के धिरुद् जारी की जा सकती हैं।
(v) ऄधधकार पृच्छा (Quo warranto)

ऄधधकार पृच्छा का ऄथव है "फकस प्राधधकृ त या िारंट के द्वारा?" या "अपका प्राधधकार क्या है?"
आसे फकसी व्यधि को फकसी ऐसे सािवजधनक पद को धारण करने से रोकने की दृधष्ट से जारी फकया
जाता है धजसे धारण करने के िह ऄयोग्य है। ररट जारी करने के पश्चात यह अिश्यक हो जाता है
फक संबद् व्यधि न्यायालय के समक्ष आस तथ्य की व्याख्या करे फक िह फकस प्राधधकार से ईि पद
धारण फकया है।
ऄधधकार पृच्छा जारी करने के धलए शतें:
 कायावलय सािवजधनक होना चाधहए और यह फकसी धिधध या स्ियं संधिधान द्वारा स्थाधपत होना
अिश्यक है।
 आसके धलए एक मूल कायावलय का होना अिश्यक है और आसके प्रकायव के िल ऄधीनस्थ के रूप में
फकसी और के प्रसाद पयंत नहीं होने चाधहए ।
 ऐसे फकसी व्यधि की ईस पद पर धनयुधि में संधिधान या धिधध ऄथिा िैधाधनक ईपकरण का
ईल्लंघन न हअ हो।
ऄनुच्छेद 32 (3)
संसद मूल ऄधधकारों को लागू करने के धलए फकसी भी ऄन्य न्यायालय को ऄधधकृ त कर सकती है।

217 www.visionias.in ©Vision IAS

Google it:- https://upscpdf.com


https://upscpdf.com << Download From >> https://upscpdf.com

शतें:
 ऐसा करने के ईपरांत ईच्चतम न्यायालय की शधियां नकारात्मक रूप से प्रभाधित न हों।
 ऄन्य न्यायालय धजसे ररट जारी करने के धलए प्राधधकृ त फकया गया है; ईसकी शधियां ईसके
ऄधधकार क्षेत्र की स्थानीय सीमाओं के भीतर सीधमत होनी चाधहए।
ऄनुच्छेद 32 (4)

यह ऄनुच्छेद 359 के तहत धनधावररत धिशेष रीधत से ऄनुच्छेद 32 के धनलंबन का प्रािधान करता है।
ऄनुच्छेद 359- राष्ट्रीय अपातकाल की घोषणा के दौरान मूल ऄधधकारों का धनलंबन

 ऄनुच्छेद 19 (जो बाह्य अक्रमण या युद् के अधार पर स्ितः धनलंधबत हो जाता है) के ऄधतररि
ऄन्य ऄधधकारों का धनलंबन स्ितः नहीं होता है।
 ऄनुच्छेद 20 और 21 के तहत प्रद्त ऄधधकारों को कभी भी धनलंधबत नहीं फकया जाता है। शेष

ऄधधकारों को तभी धनलंधबत फकया जा सकता है, जब राष्ट्रपधत फकसी ऄधधकार को धनलंधबत करने
के धलए अदेश जारी करे। ऄपने अदेश में राष्ट्रपधत को धनलंधबत फकए जाने िाले ऄधधकार, ईसकी
धनलंबन की ऄिधध तथा ईसकी भौगोधलक सीमा का स्पष्ट धििरण प्रस्तुत करना होता है।

5.23. ऄनु च्छे द 33 - मू ल ऄधधकारों के , सु र क्षा बलों अफद पर लागू होने में , ईपां त रण करने
की सं स द की शधि

मूल पाठ
आस भाग द्वारा प्रद्त ऄधधकारों का, सुरक्षा बलों अफद पर लागू होने में, ईपांतरण करने की संसद की
शधि- संसद, धिधध द्वारा ऄिधारण कर सके गी फक आस भाग द्वारा प्रद्त ऄधधकारों में से कोइ-
(क) सशस्त्र बलों के सदस्यों को, या
(ख) लोक व्यिस्था बनाए रखने का भारसाधन करने िाले बलों के सदस्यों को, या
(ग) असूचना या प्रधत असूचना के प्रयोजनों के धलए राज्य द्वारा स्थाधपत फकसी ब्यूरो या ऄन्य संगठन
में धनयोधजत व्यधियों को, या
(घ) खंड (a) से खंड (c) में धनर्ददष्ट फकसी बल, ब्यूरो या संगठन के प्रयोजनों के धलए स्थाधपत दूरसंचार
प्रणाली में या ईसके संबंध में धनयोधजत व्यधियों को,
लागू होने में, फकस धिस्तार तक धनबवधन्धत या धनराकृ त फकया जाए धजससे ईनके कतवव्यों का ईधचत
पालन और ईनमें ऄनुशासन बना रहे।
धििरण
 ऄनुच्छेद 33 संसद को यह ऄधधकार देता है फक िह सशस्त्र बलों, ऄद्वसैधनक बलों, पुधलस बलों
आत्याफद के मूल ऄधधकारों को सीधमत या कु छ स्तर तक युधियुि रूप से प्रधतबंधधत कर सके ।
परंतु, आसका ऄथव यह नहीं फक यह ऄनुच्छेद स्ियं फकसी ऄधधकार का प्रधतषेध करेगा।
 आस ऄनुच्छेद का कायव संसदीय धिधानों की प्रकृ धत पर धनभवर करेगा, भले ही िे आस ऄनुच्छेद को
संदर्वभत न करें। संसद द्वारा धनर्वमत आस प्रकार का धिधान समानता, ऄधभव्यधि की स्ितंत्रता, संघ
धनमावण की स्ितंत्रता, व्यधिगत स्ितंत्रता अफद के रूप में फकसी भी मूल ऄधधकार के संचालन को

प्रधतबंधधत कर सकता है। पुधलस बल (धिशेषाधधकार के प्रधतबंध) ऄधधधनयम, 1966 संसद द्वारा
पाररत एक ऐसा ही ऄधधधनयम है। आसे ईच्चतम न्यायालय में चुनौती भी दी गयी थी, परंतु आसे

िैध घोधषत कर फदया गया था।

218 www.visionias.in ©Vision IAS

Google it:- https://upscpdf.com


https://upscpdf.com << Download From >> https://upscpdf.com

5.24. ऄनु च्छे द 34 - जब फकसी क्षे त्र में से ना धिधध प्रिृ ्त है तब आस भाग द्वारा प्रद्त
ऄधधकारों पर धनबं ध न
मूल पाठ
आस भाग के पूिवगामी ईपबंधों में फकसी बात के होते हए भी, संसद धिधध द्वारा संघ या फकसी राज्य की
सेिा में फकसी व्यधि की या फकसी ऄन्य व्यधि की फकसी ऐसे कायव के संबंध में क्षधतपूर्वत कर सके गी जो
ईसने भारत के राज्यक्षेत्र के भीतर फकसी ऐसे क्षेत्र में, जहाँ सेना धिधध प्रिृ्त थी, व्यिस्था के बनाए
रखने या पुनःस्थापन के संबंध में फकया है या ऐसे क्षेत्र में सेना धिधध के ऄधीन पाररत दंडादेश , फदए गए
दंड, अफदष्ट समपहरण या फकए गए ऄन्य कायव को धिधधमान्य कर सके गी।
धििरण
 यह ऄनुच्छेद ‘माशवल लॉ’ के लागू होने की धस्थधत में मूल ऄधधकारों को सीधमत या प्रधतबंधधत
करने से संबंधधत है।
 माशवल लॉ की पररभाषा संधिधान में नहीं दी गइ है। ककतु, आसका सामान्य ऄथव ऐसे सैन्य कानूनों
से है जो फकसी ऄशांत क्षेत्र में सामान्य प्रशासन के संचालन हेतु साधारण कानून को धनलंधबत
करके सेना को प्रशासन के संचालन हेतु प्राधधकृ त करने से हैं।
 माशवल लॉ को लागू करने हेतु ऄसाधारण पररधस्थधतयाँ जैसे युद्, ऄशांधत, दंगे या कानून का
ईल्लंघन अफद की धस्थधत ईत्पन्न होनी चाधहए।
 आसके कायावन्ियन के दौरान एिं कानून व्यिस्था बनाए रखने हेतु फकए गए कृ त्यों हेतु सरकारी
कमवचारी को सुरक्षा प्रदान की गइ है।
 आसके फक्रयान्ियन के दौरान मूल ऄधधकारों पर प्रधतबंध को आस अधार पर चुनौती नहीं दी जा
सकती फक यह मूल ऄधधकार का ईल्लंघन करता है।
 ऄनुच्छेद 34 के तहत घोधषत माशवल लॉ एिं ऄनुच्छेद 352 के घोधषत राष्ट्रीय अपातकाल में
धनम्नधलधखत ऄंतर हैः

माशवल लॉ राष्ट्रीय अपातकाल

यह के िल मूल ऄधधकारों को प्रभाधित आससे के िल मूल ऄधधकार ही प्रभाधित नहीं होते ऄधपतु यह
करता है। कें द्र-राज्य समबन्ध, राजस्ि धितरण एिं कें द्र तथा राज्य की
धिधायी शधियों को भी प्रभाधित करता है और आस दौरान
सरकार के कायवकाल में भी िृधद् हो सकती है।

यह सरकार और साधारण आस दौरान सरकार और साधारण न्यायालय कायवरत रहते


कानूनी न्यायालयों को धनलंधबत कर हैं।
देता है।

यह फकसी भी कारण से कानून एिं यह के िल तीन अधारों पर लगाया जा सकता है - युद्,


व्यिस्था में अए व्यिधान को समाि बाह्य अक्रमण ऄथिा सशस्त्र धिद्रोह।
करता है।

यह देश के कु छ धिधशष्ट धहस्सों में यह या तो पूरे देश में या आसके फकसी भी धहस्से में लगाया

ही लागू फकया जाता है। जा सकता है।

आसके सन्दभव में संधिधान में आसके सन्दभव में संधिधान में धिधशष्ट एिं धिस्तृत प्रािधान
कोइ धिधशष्ट प्रािधान नहीं फकया गया फकये गए हैं। यह स्पष्ट है।
है। यह ऄव्यि है।

219 www.visionias.in ©Vision IAS

Google it:- https://upscpdf.com


https://upscpdf.com << Download From >> https://upscpdf.com

5.25. ऄनु च्छे द 35: भाग 3 के ईपबं धों को प्रभािी करने के धलए धिधान

मूल पाठ

आस भाग के ईपबंधों को प्रभािी करने के धलए धिधान- आस संधिधान में फकसी बात के होते हए भी,-
(a) संसद (न की फकसी राज्य के धिधान-मंडल को) को यह शधि प्राि होगी फक िह-
(i) धजन धिषयों के धलए ऄनुच्छेद 16 के खंड (3), ऄनुच्छेद 32 के खंड (3), ऄनुच्छेद 33 और ऄनुच्छेद
34 के ऄधीन संसद धिधध द्वारा ईपबंध कर सके गी ईनमें से फकसी के धलए, और
(ii) ऐसे कायों के धलए, जो आस भाग के ऄधीन ऄपराध घोधषत फकए गए हैं, दंड धिधहत करने के धलए,
धिधध बनाए और संसद आस संधिधान के प्रारंभ के पश्चात् यथाशक्य शीर अ ऐसे कायों के धलए, जो
ईपखंड (iii) में धनर्ददष्ट हैं, दंड धिधहत करने के धलए धिधध बनाएगी;
(b) खंड (a) के ईपखंड (i) में धनर्ददष्ट धिषयों में से फकसी से संबंधधत या ईस खंड के ईपखंड (ii) में
धनर्ददष्ट फकसी कायव के धलए दंड का ईपबंध करने िाली कोइ प्रिृ्त धिधध, जो भारत के राज्यक्षेत्र में आस
संधिधान के प्रारंभ से ठीक पूिव प्रिृ्त थी, ईसके धनबंधनों के और ऄनुच्छेद 372 के ऄधीन ईसमें फकए
गए फकन्हीं ऄनुकूलनों और ईपांतरणों के ऄधीन रहते हए तब तक प्रिृ्त रहेगी जब तक ईसका संसद
द्वारा पररितवन या धनरसन या संशोधन नहीं कर फदया जाता है।
स्पष्टीकरण -आस ऄनुच्छेद में, ''प्रिृ्त धिधध'' पद का िही ऄथव है जो ऄनुच्छेद 372 में है।
धििरण
 आसका ईद्देश्य कु छ धिशेष मूल ऄधधकारों को प्रभािी बनाने एिं ईनके ईल्लंघन की दशा में दंधडत
करने हेतु संपूणव देश में कानूनों के संमरूपता हेतु संसद को शधि संपन्न करना है।
 आस प्रकार के धिधध धनमावण की शधि के िल संसद के पास है, न की राज्य धिधानमंडल के पास।

 कु छ धिशेष मूल ऄधधकारों को प्रभािी बनाने संबंधी संसद की शधि यथाः

o ऄनुच्छेद 16 - फकसी राज्य, कें द्र-शाधसत या स्थानीय प्राधधकरणों में रोजगार हेतु धनिास
संबंधी ऄधनिायवता।
o ऄनुच्छेद 32 - मूल ऄधधकारों के फक्रयान्ियन हेतु ईच्चतम या ईच्च न्यायालय के ऄधतररि
ऄन्य न्यायालयों को प्राधधकृ त करना।
o ऄनुच्छेद 33 - धिधभन्न सशस्त्र बलों, ऄधवसैधनक बलों, खुफफ़या एजेंधसयों के समुधचत
फक्रयाकलाप एिं ऄनुशासन को बनाए रखने हेतु मूल ऄधधकारों पर प्रधतबंध हेत।ु
o ऄनुच्छेद 34 - माशवल लॉ के कायावन्ियन के दौरान धनष्पाफदत कृ त्यों की क्षधतपूर्वत हेत।ु
 मूल ऄधधकारों के ईल्लंघन की फदशा में दंधडत करने हेतु संसद की शधि यथा :
o ऄनुच्छेद 17 - ऄस्पृश्यता की समाधि एिं छु अछू त के अचरण को दंधडत करने हेतु धिधध

धनमावण संबंधी शधि। ईदाहरण के धलए, नागररक ऄधधकार संरक्षण ऄधधधनयम, 1955 एिं

ऄनुसूधचत जाधत तथा ऄनुसूधचत जनजाधत (ऄत्याचार धनिारण) संशोधन ऄधधधनयम, 2015

धनर्वमत फकया गया है।

o ऄनुच्छेद 23 - मानिीय दुव्यवपार एिं बलात् श्रम को रोकने हेतु ऄधधधनयम। ईदाहरण के

धलए, ऄनैधतक व्यापार धनिारण ऄधधधनयम, 1956 एिं बंधधत श्रम पद्धत (ईत्सादन)

ऄधधधनयम 1976.

220 www.visionias.in ©Vision IAS

Google it:- https://upscpdf.com


https://upscpdf.com << Download From >> https://upscpdf.com

6. संप ध्त के ऄधधकार की प्रधस्थधत


ितवमान में संपध्त का ऄधधकार मौधलक ऄधधकार नहीं है। हालाँफक, संधिधान के प्रारमभ में यह ऄनुच्छेद
19 (1) (च) द्वारा गारंटीकृ त एक मौधलक ऄधधकार था। पुन: यह ऄनुच्छेद 31 का पूरक था, धजसने
संपध्त का राज्य द्वारा ऄधधग्रहण करने से सुरक्षा प्रदान की थी। आस ऄधधकार के कारण बहत से भ्रम
एिं कानूनी धििाद ईत्पन्न हए और संिैधाधनक संशोधन भी हए। ऄंततः 1978 में, 44 िें संधिधान
संशोधन ऄधधधनयम के द्वारा आन दोनों ऄनुच्छेदों को धनरधसत कर फदया गया और एक ऄन्य ऄनुच्छेद
300A को सधममधलत फकया गया धजसमें कहा गया है फक "फकसी व्यधि को ईसकी संपध्त से धिधध के
प्राधधकार से ही िंधचत फकया जाएगा,ऄन्यथा नहीं।" आसका तात्पयव है फक:
 आस प्रकार ऄब संपध्त का ऄधधकार मौधलक ऄधधकार नहीं है, बधल्क यह के िल एक संिैधाधनक
ऄधधकार है। ऄब कोइ भी नागररक आस ऄधधकार के ईल्लंघन के मामले में सिोच्च न्यायालय या
ईच्च न्यायालय में नहीं जा सकता है।
 आसके ऄधतररि यह व्यधियों को के िल मनमानी कायवकारी कारविाइ से बचाता है, न फक धिधायी
कारविाइ से।
 राज्य ऄधधग्रहण के मामले में फकसी भी मुअिजे का भुगतान करने के धलए संिैधाधनक रूप से
बाध्य नहीं है।
7. क्या मूल ऄधधकार अत्यं धतक हैं ?
मूल ऄधधकार फकसी व्यधि को अत्यंधतक शधियां नहीं प्रदान करते। ये युधियुि रूप से सीधमत
(restricted) ऄधधकार हैं। गोपालन िाद (1950) में ईच्चतम न्यायालय ने स्पष्ट कहा था फक अत्यंधतक
स्ितंत्रता जैसी कोइ भी संकल्पना ऄधस्तत्ि नहीं रखती क्योंफक ऐसी धस्थधत ऄराजकता की धस्थधत बना
सकती है। िहीं दूसरी ओर, यफद राज्य को अत्यंधतक ऄधधकार प्राि हो जाएँ तो तानाशाही का ईदय
होगा। मूल ऄधधकारों का ईद्देश्य धिधध के शासन की स्थापना है और आसीधलए व्यधि के ऄधधकारों एिं
सामाधजक ऄपेक्षाओं के मध्य संतुलन का होना ऄत्यधधक अिश्यक है। यही कारण है फक संधिधान
संसद को यह शधि प्रदान करता है फक िह व्यधि के मूल ऄधधकारों पर युधियुि एिं तकव संगत
प्रधतबन्ध लगा सके ।
युधियुि प्रधतबंधों के प्रमुख अधार धनम्नांफकत हैं:
 ऄनुच्छेद 19(2) में िर्वणत अधार।
 मधहलाओं और बच्चों सधहत ऄनुसूधचत जाधत, ऄनुसूधचत जनजाधत, ऄन्य धपछड़ा िगव और समाज
के ऄन्य कमजोर िगों की ईन्नधत।
 अम जनता, सािवजधनक व्यिस्था, शालीनता और नैधतकता के धहत में।
 भारत की संप्रभुता और ऄखंडता।
 राज्य की सुरक्षा।
 धिदेशी राज्यों के साथ धमत्रतापूणव संबंध।
8. मूल ऄधधकारों पर अपातकाल का प्रभाि
 ऄनुच्छेद 358 एिं 359 राष्ट्रीय अपातकाल के दौरान मूल ऄधधकार पर पड़ने िाले प्रभाि का
िणवन करते हैं। ऄनुच्छेद 358, ऄनुच्छेद 19 द्वारा प्रद्त मूल ऄधधकारों के धनलंबन से संबंधधत है,
जबफक ऄनुच्छेद 359 ऄन्य मूल ऄधधकारों के धनलंबन (ऄनुच्छेद 20 और 21 द्वारा प्रद्त मूल
ऄधधकारों के ऄधतररि) से संबंधधत है।
 जब कभी संधिधान के ऄनुच्छेद 352 के ऄंतगवत युद् ऄथिा बाह्य अक्रमण के कारण (ककतु सशस्त्र
धिद्रोह के अधार पर नहीं) अपातकाल की ईद्घोषणा की जाती है तो संधिधान के ऄनुच्छेद 19 में
ईल्लेधखत स्ितंत्रता संबधी ऄधधकार का फक्रयान्ियन धनलंधबत रहता है। ईपरोि अधार पर
अपातकाल की ईद्घोषणा के ईपरांत राष्ट्रपधत संधिधान के ऄनुच्छेद 359 के ऄंतगवत एक दूसरा
अदेश जारी कर ऄन्य मूल ऄधधकारों के फक्रयान्ियन को भी स्थधगत कर सकता है।

221 www.visionias.in ©Vision IAS

Google it:- https://upscpdf.com


https://upscpdf.com << Download From >> https://upscpdf.com

9. मूल ऄधधकारों की अलोचना


 यद्यधप ये मूल ऄधधकार कहे जाते हैं, परन्तु आन पर ऄसंख्य प्रधतबंध अरोधपत फकए गए हैं। आसके
ऄधतररि ‘युधियुिता’ में क्या शाधमल है, यह न्यायालयों की बदलती व्याख्याओं पर धनभवर
करता है।
 ये के िल राजनीधतक ऄधधकार प्रदान करते हैं। हालांफक राजनीधतक स्ितंत्रता तब तक व्यथव है, जब
तक फक सामाधजक और अर्वथक स्ितंत्रता न प्राि हो।
 ये ऄधधकार ऄपररितवनीय नहीं हैं। संसद द्वारा आसमें कटौती या आन्हें धसधमत फकया जा सकता है।
आनमें से ऄधधकांश राष्ट्रीय अपातकाल के दौरान धनलंधबत हो जाते हैं।
 मूल ऄधधकारों के ईल्लंघन के सन्दभव में, न्याधयक ईपचार जरटल और महंगा हैं, ऄधधक समय लेने
िाला और व्यिहार में ऄधधकांश जनसंख्या की पहँच से बाहर है।
10. मूल ऄधधकारों का महत्ि
 ईपयुवि अलोचनाओं के बािजूद, मूल ऄधधकार हमारे देश की ईदार लोकतांधत्रक ढांचे की
अधारधशला का धनमावण करते हैं। स्ितंत्रता के बाद के ऄनुभि से पता चलता है फक न के िल आसने
लोकतंत्र की जड़ों को मजबूत बनाने में मदद की है; ऄधपतु न्यायालयों की ईदार व्याख्या ने
व्यधिगत ऄधधकारों के दायरे का ऄत्यंत धिस्तार भी फकया है। ितवमान में ये कायवकारी धनरंकुशता
और धिधायी मनमानेपन के धिरुद् एक महत्िपूणव संरक्षण प्रदान करते हैं।

11. धिगत िषों में Vision IAS GS मेंस टे स्ट सीरीज में पूछे
गए प्रश्न (Previous Year Vision IAS GS Mains Test
Series Questions)
1. यह अिश्यक नहीं फक सभी को समान ईपचार प्राि हो, परन्तु सभी के साथ समान व्यिहार
होना चाधहए। आस कथन के अलोक में भारतीय संधिधान के ऄनुच्छेद 14 की व्याख्या
कीधजए।
दृधष्टकोण:
 धिधध के समक्ष समता तथा धिधधयों का समान संरक्षण के मध्य धिभेद की स्पष्ट समझ
होनी चाधहए।
 साथ ही धिधध धनमावण के समय संधिधान द्वारा फकस प्रकार के िगीकरण को ऄनुमधत दी
गयी है, ईसकी स्पष्ट समझ होनी चाधहए। ई्त र के पक्ष में, कु छ ईदाहरण आसे ऄधधक
साथवक/सुसंगत बनाएंगे।
 धिधध के समक्ष समता, धिधध का शासन एिं आसकी मह्त ा अफद पर कोइ चचाव नहीं
करनी है क्योंफक प्रश्न में धिशेष रूप से आसके ऄनुप्रयोगों के बारे में पूछा गया है।
ई्त रः
 ऄनुच्छेद 14 भारत के राज्य क्षेत्र में सभी व्यधियों को धिधध के समक्ष समता का
ऄधधकार प्रदान करता है ऄथावत् यह फकसी भी व्यधि को फकसी तरह के धिशेषाधधकार
की ऄनुपधस्थधत को दशावता है। आसका तात्पयव है फक यहाँ कोइ भी व्यधि कानून से उपर
नहीं है और सभी व्यधि चाहे िह फकसी भी पद या स्तर का हो साधारण कानून के
ऄधीन है।
 परन्तु धिधध के समक्ष समता के धसद्ांत में सभी के धलए धनरपेक्ष समता का धिचार
शाधमल नहीं है। आस ऄनु॰ में एक िाक्य शाधमल है- ‘धिधधयों का समान संरक्षण’ धजसका
तात्पयव है फक समान पररधस्थधतयों में सभी के साथ समान व्यिहार फकया जायेगा।

222 www.visionias.in ©Vision IAS

Google it:- https://upscpdf.com


https://upscpdf.com << Download From >> https://upscpdf.com

 ऄनुच्छेद 14 में जो धनषेध फकया गया है िो ‘िगव धिधान’ है न फक धिधान बनाने के ईद्देश्यों
के धलए िगीकरण। परन्तु यह िगीकरण ऐधच्छक नहीं होना चाधहए। यह तार्दकक होना
चाधहए तथा िैसे गुणों और ऄधभलक्षणों पर अधाररत होना चाधहए, धजनका धिधान के
ईद्देश्य से संबंध हो। ऄतः ऄनुच्छेद 14 यह नहीं कहता फक सभी व्यधियों पर समान कर लगे
बधल्क समान लक्षणों िाले व्यधियों पर समान मानकों पर अधाररत कर अरोधपत होना
चाधहए।
 िगीकरण, तार्दकक हो और मनमाना न हो आसके धलए दो शतें पूरी होना अिश्यक है।
प्रथम िगीकरण बोधगमय ऄंतर पर अधाररत होना चाधहये जो एक समूह को दूसरे से
पृथक करे और धद्वतीय यह फक यह ऄंतर ऄधधधनयम के ईद्देश्य से युधियुि रूप से
संबंधधत होना चाधहए।
 साथ ही समान संरक्षण की गारडटी, कानून धनमावण के साथ ही लागू नहीं होगी बधल्क
ईसके प्रशासन ि कायावन्ियन में भी लागू होगी। ऄतः कोइ प्रफक्रया जो साधारण कानून
से ऄलग ऄपनाइ जाये एक धिधशष्ट िगव के व्यधि के धलए तो यह भेदभाि तार्दकक
िगीकरण पर अधाररत होना चाधहये।

2. ऄनुच्छेद 21 के ऄंतगवत, मानिीय प्रधतष्ठा के साथ जीिन के ऄधधकार में सममान के साथ मृत्यु
का ऄधधकार भी सधममधलत है। आस मामले पर सिोच्च न्यायालय के धिधभन्न न्याधयक
घोषणाओं के अलोक में चचाव कीधजए। साथ ही, ''मेधडकल ट्रीटमेंट ऑफ टर्वमनली आल पेशट्ें स
धबल- 2016'' से संबद् धिधभन्न मुद्दों का अलोचनात्मक परीक्षण कीधजए।
दृधष्टकोण:
यह समझने की अिश्यकता है फक यहां धिशेष रूप से क्या पूछा जा रहा है, क्योंफक हाल के
महीनों में आच्छामृत्यु पर चचाव धिशेष रूप से सुर्वोयों में रही है।
 आच्छामृत्यु पर चचाव को भूधमका के रूप में प्रस्तुत कीधजए। संधिधान के ऄनुच्छेद 21 के
प्रािधानों का ईल्लेख कीधजए।
 हाल के कु छ ईदाहरणों के साथ आसके पक्ष और धिपक्ष में तकव दीधजए।
ई्त र:
ऄनुच्छेद 21 व्यधि को साथवक, संपूणव और सममानपूणव जीिन का ऄधधकार प्रदान करता है।
फकन्तु मरणासन्न रूप से रोगग्रस्त रोधगयों के मामले में आस ऄधधकार को ईधचत रूप से प्रयोग
नहीं फकया जा सकता। आस प्रकार, यह तकव फदया गया है फक गररमामय जीिन के ऄधधकार के
ऄनुरूप, जो व्यधि ऐसा नहीं कर सकता ईसे कम से कम गररमापूणव मृत्यु का ऄधधकार प्राप्त
होना ही चाधहए। आच्छामृत्यु या ऄधसस्टेड सुसाआड को 'रोगी को दुख से ईबारने ' के माध्यम के
रूप में बताते हए तकव फदया जाता रहा है। िषव 2011 में ऄरुणा शानबाग धनणवय में सिोच्च
न्यायालय ने धनधष्क्रय (पैधसि) आच्छामृत्यु की ऄनुमधत दी। यह ऄनुमधत प्रत्ये क मामले में
धिशेषज्ञों के पैनल से परामशव के अधार पर ईच्च न्यायालय की न्यायपीठ द्वारा ऄधनिायव
ऄनुमोदन से धनधावररत सुरक्षा ईपायों और धनष्पक्ष प्रफक्रया के ऄधीन थी। धनधष्क्रय आच्छामृत्यु
का संदभव जीिन को बनाए रखने के धलए अिश्यक ईपचार पर रोक लगाने या हटा लेने को
संदर्वभत करता है।
धजयान कौर बनाम पंजाब राज्य, 1996 के धनणवय के ऄनुसार ऄनुच्छेद 21 में 'जीिन' शब्द
का िास्तधिक ऄथव मानिीय गाररमामय जीिन है। यफद कोइ 'मृत्यु का ऄधधकार' का धिषय

223 www.visionias.in ©Vision IAS

Google it:- https://upscpdf.com


https://upscpdf.com << Download From >> https://upscpdf.com

ईत्पन्न होता है तो िह स्िाभाधिक रूप से ''जीिन के ऄधधकार'' ऄंसगत है, ठीक ईसी प्रकार
धजस प्रकार ''मृत्यु'' ''जीिन'' से ऄसंगत है।
भारतीय धिधध अयोग की 196िीं ररपोटव में धनधष्क्रय आच्छामृत्यु के पक्ष में धिचार व्यक्त फकए
गए हैं।
 आच्छामृत्यु के पक्ष में तकव :
o यह एक नागररक ऄधधकार है।
o यह धनजी स्िाय्त ता का प्रश्न है।
o यह सुधनधश्चत करना अिश्यक है फक कोइ भी व्यधि पीड़ादायक व्यथा से मृत्यु
प्राप्त न करे।
o आस प्रकार के कानून के प्रयोग को रोकने के धलए सुरक्षा फदशा-धनदेश फदए जा सकते हैं।
 आच्छामृत्यु के धिपक्ष में तकव :
o ऄधसस्टेड मृत्यु का प्रयोग ऄिैध रूप से व्यधिगत लाभ प्राप्त करने के धलए फकया
जा सकता है।
o आसे औषधीय देखभाल के सस्ते धिकल्प के रूप में प्रोत्साधहत फकए जाने की अशंका है।
o ऄस्पताल की देखभाल और ईधचत ईपचार नैधतक रूप से स्िीकायव समाधान प्रदान
करते हैं।
 आच्छामृत्यु एिं ऄधसस्टेड सुसाआड दोनों ही नीदरलैंड, बेधल्जयम एिं लक्जमबगव में िैध
हैं, जबफक कोलंधबया में आच्छामृत्यु िैध है।
''मेधडकल ट्रीटमेंट ऑफ टर्वमनली आल पेशट्ें स धबल 2016'' िस्तुतः ऄरुणा शानबाग धनणवय में
धनधावररत प्रारूप को संधहताबद् करने का प्रयास करता है। आससे संबंधधत कु छ धचन्ताएँ
धनमनधलधखत हैं:
 आसमें न्यायालय के ऄग्रणी धनदेशों के ईधचत रूप से कायावन्ियन हेतु पयावप्त सुरक्षा
ईपाय नहीं हैं।
 ईच्च न्यायालय से ऄनुमधत प्राि करने के प्रािधान पर भी प्रश्न खड़े फकए जा रहे हैं।
 रोधगयों के िगीकरण का न्याधयक जांच द्वारा परीक्षण संभि नहीं है।

3. सरकार, सािवजधनक लाभों की प्राधि के धलए जनता के समक्ष मौधलक ऄधधकारों के पररत्याग
की शतव नहीं रख सकती। हाल ही में अधार काडव से संबधं धत याधचका में ईठाए गए मुद्दों के
संदभव में आस कथन पर चचाव करें।
दृधष्टकोण :
यह प्रश्न भारत सरकार द्वारा अधार काडव के कायावन्ियन का सिोच्च न्यायालय द्वारा फकये गए
ऄिलोकन पर अधाररत है। आसधलए ई्त र में धनम्नधलधखत लबदु ऄिश्य होने चाधहए:
 अधार काडव के संबंध में सिोच्च न्यायालय के धपछले धनदेशों की संधक्षि चचाव कीधजए।
 आस मुद्दे से संबंधधत ितवमान धििाद का परीक्षण कीधजए।
 आसके ऄधतररि, अधार के लाभों का ईल्लेख कीधजए।
 धनष्कषव में समाधान प्रस्तुत कीधजए।
ई्त र:
कल्याणकारी योजनाओं के धलए अधार काडव के प्रयोग के सन्दभव में नागररकों के धनजता के
ऄधधकार तथा समृद् एिं धनधवनों के धलए समान रूप से ईनके संरक्षण पर देश में एक बहस
धछड़ी हइ है।
2013 में, सिोच्च न्यायालय ने यह धनदेश फदया था फक अधार नहीं होने के कारण फकसी
व्यधि को सुधिधा से िंधचत नहीं फकया जाना चाधहए। ितवमान सरकार ने सिोच्च न्यायालय
से ऄपने फै सले को िापस लेने का अग्रह फकया क्योंफक सरकार की मंशा धिधिध सेिाओं के
धलए अधार का प्रयोग करने की है।

224 www.visionias.in ©Vision IAS

Google it:- https://upscpdf.com


https://upscpdf.com << Download From >> https://upscpdf.com

सिोच्च न्यायालय ने कहा फक अधार का प्रयोग PDS, के रोसीन तथा LPG धितरण के
ऄधतररि ऄन्य फकसी भी ईद्देश्य के धलए नहीं फकया जा सकता तथा यह भी स्पष्ट फकया फक
आन सुधिधाओं का लाभ ईठाने के धलए भी अधार काडव ऄधनिायव नहीं होगा।
अधार से संबधं धत मुद्दे:
 सिोच्च न्यायालय के सभी धनणवयों की समीक्षा के पश्चात् आस धनष्कषव पर पहंचा जा
सकता है फक गोपनीयता का ऄधधकार जीिन तथा स्ितंत्रता के ऄधधकार का ऄधभन्न
पहलू है। आस सन्दभव में, सिोच्च न्यायालय ने कें द्र सरकार तथा UIDAI को रोकते हए
कहा फक िह अधार-धारक की धलधखत ऄनुमधत के धबना ईनसे संबंधधत जानकारी को
फकसी ऄन्य पक्ष से साझा नहीं कर सकते।
 सरकार के आस दलील को सिोच्च न्यायालय ने खाररज कर फदया फक धनधवनों को सधब्सडी
भुगतान तथा ऄन्य लाभों को प्राि करने के बदले ऄपनी गोपनीयता के ऄधधकार को
त्यागने हेतु तैयार रहना चाधहए।
 अधार के द्वारा ईठाए गए गोपनीयता के मुद्दों पर कोइ व्यापक क़ानून नहीं है।
 प्रत्येक सरकार का सबसे अधारभूत दाधयत्ि ऄपने नागररकों के ऄधधकार को संरक्षण
प्रदान करना है – चाहे िह ऄधधकार जीिन-यापन से जुड़ा हो तथा समान रूप से
स्ितंत्रता प्रदान करने िाली गोपनीयता का।
दूसरी तरफ, अधार में सािवजधनक सेिा धितरण व्यिस्था को पुनगवरठत करने की भारी क्षमता
है। आसमें बेहतर लक्ष्य-धनधावरण, त्रुरटयों को दूर करने अफद के रूप में लाभों का धिशाल समूह
समाधहत है। अधार सरकार द्वारा प्रस्तुत ढेर सारी महत्िाकांक्षी योजनाओं यथा डीजी-
लॉकर, इ-हस्ताक्षर अफद का मुख्य अधार है।
ऄतः पयावि तथा अिश्यक सुरक्षा के साथ नागररकों की गोपनीयता संबंधी लचता का
समाधान समय की मांग है। संसदीय ऄनुमधत तथा कानूनी समथवन के बगैर यह प्रफक्रया
िैधाधनक रूप से ऄसमथवनीय तथा ऄस्िीकायव है।

4. ऄधभव्यधि की स्ितंत्रता एक ऄधधकार है,परन्तु यह फकसी भी व्यधि की मानहाधन करने का


ऄधधकार नहीं देता है। सिोच्च न्यायालय के धिधभन्न धनणवयों के अलोक में आस कथन पर चचाव
कीधजए।
दृधष्टकोण:
 ऄधधकार के रूप में ऄधभव्यधि की स्ितंत्रता तथा ईस पर ईधचत धनबंधनों की चचाव
कीधजए।
 सिोच्च न्यायालय के धिधभन्न धनणवयों की सहायता से धसद् कीधजए फक मानहाधन का
ऄधधकार ऄधभव्यधि की स्ितंत्रता में सधममधलत नहीं है।
ई्त र:
ऄनुच्छेद 19(1)(a) के तहत भारतीय संधिधान द्वारा मूल ऄधधकार के रूप में ऄधभव्यधि की
स्ितंत्रता की गारंटी प्रदान की गइ है। आसका तात्पयव यह है फक प्रत्येक नागररक को स्ितंत्र
रूप से ऄपने दृधष्टकोण, धिचार, धिश्वास और ऄिधारणा व्यि करने का ऄधधकार है।
हालांफक, यह ऄधधकार धनरपेक्ष नहीं है। संधिधान (ऄनुच्छेद 19(2)) ने ऄधभव्यधि की
स्ितंत्रता के प्रयोग पर मानहाधन सधहत कु छ ईधचत धनबवन्धन अरोधपत फकए हैं।
मानहाधन, िैसी ऄपमानजनक सामग्री के प्रकाशन कायव को संदर्वभत करता है जो एक
सामान्य व्यधि के दृधष्टकोण से व्यधि या संस्था की प्रधतष्ठा को कम करती है। ऐसा मौधखक
या धलधखत शब्दों या दृश्य धनरूपण द्वारा फकया जा सकता है।

225 www.visionias.in ©Vision IAS

Google it:- https://upscpdf.com


https://upscpdf.com << Download From >> https://upscpdf.com

भारत में मानहाधन, दीिानी और फौजदारी (दांधडक), दोनों प्रकृ धत के ऄपराध के तहत अता
है। मानहाधन एक दांधडक ऄपराध (criminal offence) के रूप में भारतीय दंड संधहता की
धारा 499 के ऄंतगवत सूचीबद् है।
हाल ही में, सिोंच्च न्यायालय ने सुिमडयम स्िामी बनाम भारत संघ िाद में अपराधधक
मानहाधन की संिैधाधनकता को बनाए रखा। सिोच्च न्यायालय के ऄनुसार, स्ितंत्र िाक् एिं
ऄधभव्यधि के ऄधधकार का तात्पयव यह नहीं है फक एक नागररक दूसरे की मानहाधन कर
सकता है। आस धनणवय ने व्यधि की गररमा और प्रधतष्ठा के साथ जीिन के मूल ऄधधकार को
रेखांफकत फकया है।
 ऄनुच्छेद-21, गररमा के साथ जीिन के ऄधधकार, के ऄंतगवत प्रधतष्ठा का संरक्षण मूल
ऄधधकार होने के साथ ही एक मानि ऄधधकार भी है।
 व्यधि की गररमा और प्रधतष्ठा की रक्षा करने के धलए मानहाधन का ऄपराधीकरण
"ईधचत धनबवन्धन" है।
एक ऄन्य िाद, अर. राजगोपाल बनाम तधमलनाडु राज्य, धजसे सुधिफदत रूप से ऑटो शंकर
िाद के रूप में जाना जाता है, में सिोच्च न्यायालय का कहना था फक समाचार पत्र सहमधत
या प्राधधकार के धबना, ईस हद तक लोगों की जीिन की कहानी या अत्मकथा का प्रकाशन
कर सकते हैं, जहां तक िह सािवजधनक ऄधभलेख से प्रकट होती है। परन्तु यफद िे सािवजधनक
ऄधभलेख से परे जाते हैं तब आसे प्रकाशन में नाधमत ऄधधकाररयों की गोपनीयता पर हमला
और मानहाधन माना जाएगा, जो ऄस्िीकायव है।
आन धनणवयों के ऄनुसार, ऄधधकारों के संतुलन का धसद्ांत बताता है फक भाषण और
ऄधभव्यधि की स्ितंत्रता के ऄधधकार के साथ-साथ, दूसरों की स्ितंत्रता में हस्तक्षेप न करने
का, ऄन्योन्याश्रयी कतवव्य भी नागररकों पर लागू होता है, क्योंफक प्रत्येक व्यधि गररमा और
प्रधतष्ठा का हकदार है।

5. धिशेष ऄधधकार कोइ धिशेषाधधकार नहीं है, लेफकन आन्हें ऄल्पसंख्यकों को ऄपनी पहचान,
संस्कृ धत और परंपराओं को बनाए रखने के धलए प्रदान फकया गया है। ईदाहरण के साथ भारत
के संदभव में धिस्तृत व्याख्या कीधजए।
दृधष्टकोण:
 ऄल्पसंख्यकों को धिशेष ऄधधकार प्रदान फकये जाने की अिश्यकता पर चचाव कीधजए।
 भारत के संदभव में ऄल्पसंख्यकों को प्रद्त धिशेष ऄधधकारों पर चचाव कीधजए (जैसे
ऄल्पसंख्यकों के धलए संिैधाधनक सुरक्षा)।
 भारत में ऄल्पसंख्यकों के संरक्षण में योगदान देने िाली सरकारी नीधतयों / संस्थानों का
ईदाहरण दीधजए।
ईत्तर:
लोकतांधत्रक व्यिस्था में, सदैि बहसंख्यक िचवस्ि की प्रिृध्त होती है। धिधध के शासन पर
अधाररत राजव्यिस्था का तात्पयव, नागररकों के प्रत्येक समूह को, धिशेषकर ऄल्पसंख्यकों
को ऄपनी पहचान और संस्कृ धत बनाए रखने के संबंध में पयावि सुरक्षा प्रदान फकया जाना है।
ऄल्पसंख्यकों को प्रद्त धिशेष ऄधधकार यही सुधनधश्चत करते हैं। ऄल्पसंख्यकों को प्रद्त
ऄधधकारों का ऄथव ईन्हें कोइ राजनैधतक धिशेषाधधकार प्रदान करना नहीं है। आसकी पृष्ठभूधम
में यह धिचार धनधहत नहीं है फक ऄल्पसंख्यकों को जनसंख्या का धिशेषाधधकार प्राि िगव

226 www.visionias.in ©Vision IAS

Google it:- https://upscpdf.com


https://upscpdf.com << Download From >> https://upscpdf.com

माना जाए, ऄधपतु आसका ईद्देश्य ईनमें सुरक्षा की भािना ईत्पन्न करना है। संयुि राष्ट्र
घोषणापत्र में भी आन ऄधधकारों का िणवन फकया गया है। ऄतः ये ऄधधकार सािवभौधमक रूप
से स्िीकार फकए जाते हैं। आन ऄधधकारों का संबंध राष्ट्रीय या नृजातीय, धार्वमक और भाषाइ
ऄल्पसंख्यकों से संबंधधत व्यधियों के ऄधधकारों से है।
भारत के संधिधान के ऄंतगवत ऄल्पसंख्यकों के धलए धिधभन्न रक्षोपाय धनम्नधलधखत मूल
ऄधधकारों के माध्यम से प्रदान फकए गए हैं:
 ऄनुच्छेद 14 धिधध के समक्ष समता और धिधधयों का समान संरक्षण प्रदान करता है। आस
प्रकार ऄल्पसंख्यकों को बहसंख्यकों की तुलना में धिधधक रूप से हीन धस्थधत में नहीं
रखा जा सकता है।
 ऄनुच्छेद 15 और 16 कु छ अधारों (धमव, प्रजाधत, जाधत, ललग या जन्म स्थान) पर
भेदभाि का धनषेध करते हैं।
 ऄनुच्छेद 29 स्पष्ट रूप से धिधशष्ट भाषा, धलधप या संस्कृ धत िाले नागररकों के प्रत्येक
समूह को ईनका संरक्षण करने का ऄधधकार प्रदान करता है।
 ऄनुच्छेद 30 धार्वमक / भाषाइ ऄल्पसंख्यकों को शैधक्षक संस्थानों की स्थापना और
प्रशासन का ऄधधकार प्रदान करता है।
आसके ऄधतररि, धनम्नधलधखत ऄनुच्छेद धिशेष रूप से भाषाइ ऄल्पसंख्यकों से संबंधधत हैं:

 ऄनुच्छेद 347 में प्रद्त प्रािधानों के ऄनुसार, यफद फकसी राज्य की जनसंख्या के एक बड़े
धहस्से की आच्छा हो तो राष्ट्रपधत के धनदेश पर ईस भाषा को राज्य की अधधकाररक भाषा
के रूप में सधममधलत फकया जा सकता है।
 ऄनुच्छेद 350 के ऄनुसार, प्रत्येक व्यधि फकसी व्यथा के धनिारण के धलए संघ या राज्य

के फकसी ऄधधकारी या प्राधधकारी को, यथाधस्थधत, संघ में या राज्य में प्रयुि फकसी
भाषा में ऄभ्यािेदन देने का हकदार होगा।
 ऄनुच्छेद 350A में प्राथधमक स्तर पर मातृभाषा में धशक्षा के धलए सुधिधाओं का
प्रािधान फकया गया है।
 ऄनुच्छेद 350B में भाषाइ ऄल्पसंख्यकों के धलए धिशेष ऄधधकारी का प्रािधान फकया
गया है।
ऄल्पसंख्यक ऄधधकारों के संरक्षण के ईदाहरण
 राष्ट्रीय ऄल्पसंख्यक अयोग, ऄल्पसंख्यक शैक्षधणक संस्थानों के धलए राष्ट्रीय अयोग और

राष्ट्रीय ऄल्पसंख्यक धिकास धि्त धनगम (NMDFC) का प्रािधान फकया गया है।
 धशक्षा के प्राथधमक स्तर पर ऄल्पसंख्यकों को मातृभाषा में धशक्षा की ईपलब्धता।
 धत्रभाषा सूत्र का कायावन्ियन फकया जाना।
 ऄल्पसंख्यकों के कल्याण के धलए प्रधानमंत्री का नया 15 सूत्रीय कायवक्रम।

 नइ मंधजल, USTAAD, हमारी धरोहर, धजयो पारसी, ऄल्पसंख्यक छात्रों के धलए

मौलाना अजाद राष्ट्रीय फे लोधशप, नालंदा पररयोजना अफद जैसी धिकास संबंधी
योजनाएं।
 स्थानीय भाषाओं में सूचना का प्रचार-प्रसार फकया जाना।

227 www.visionias.in ©Vision IAS

Google it:- https://upscpdf.com


https://upscpdf.com << Download From >> https://upscpdf.com

ये प्रािधान ऄल्पसंख्यकों को कोइ धिशेषाधधकार नहीं प्रदान करते हैं। ये सुधनधश्चत करते हैं
फक स्त ाधारी पक्ष की धिचारधारा के कारण ईनकी प्रगधत ऄिरुद् न हो। ये ऄधधकार ईनकी
धिशेष धस्थधत के साथ-साथ लोकतंत्र की संभाधित चुनौधतयों की भी पहचान करते हैं।
समािेशी िृधद् और धिकास सुधनधश्चत करने के धलए आनका कायावन्ियन राज्य का संिैधाधनक
/ सांधिधधक ई्त रदाधयत्ि है।

6. जब तक अलोचनाएं "न्याय के प्रशासन" में बाधा न डालें, न्यायपाधलका के संबध


ं में
अलोचनाओं का स्िागत फकया जाना चाधहए। आस संदभव में चचाव कीधजए फक क्या ईच्चतर
न्यायपाधलका को दी गइ न्यायालय की ऄिमानना की शधि ऄनुच्छेद 19(1) (a) के तहत
प्रद्त स्ितंत्रता को सीधमत करती है और क्या आन दोनों में सामंजस्य स्थाधपत फकया जा
सकता है?
दृधष्टकोण:
 शुरुअत में संधक्षि रूप से कथन में धनधहत मुख्य धिषय को स्पष्ट कीधजये एिं ऄिमानना
की शधि और िाक् एिं ऄधभव्यधि की स्ितंत्रता के पक्ष में फदए गए तकों के साथ आसका
संबंध स्थाधपत कीधजए।
 न्यायपाधलका को प्रदान की गयी ऄिमानना शधि की अिश्यकता पर चचाव कीधजए।
 न्यायपाधलका को प्रद्त ऄिमानना शधि का भाषण की स्ितंत्रता पर क्या प्रभाि पड़ता
है ? चचाव कीधजए।
 आन दोनों के बीच कै से सामंजस्य स्थाधपत फकया जा सकता है? चचाव कीधजए।
ईत्तर:
न्याय के प्रशासन हेतु न्यायपाधलका को मजबूत रक्षोपायों की अिश्यकता होती है।
 भारत के संधिधान के ऄनुच्छेद 129 और 215 के प्रािधानों के ऄनुरूप क्रमशः सिोच्च
न्यायालय और ईच्च न्यायालयों को ऄपनी ऄिमानना के धलए लोगों को दंधडत करने की
शधि प्राि है।
न्यायालय की ऄिमानना ऄधधधनयम, 1971 न्यायपाधलका की ऄिमानना शधियों का िणवन
करता है:
 फकसी भी न्यायालय के प्राधधकार को सीधमत करने ऄथिा ईसकी प्रधतष्ठा को क्षधत
पहँचाने से रोकने के धलए।
 धनधावररत न्याधयक प्रफक्रया में फकसी भी हस्तक्षेप को रोकने के धलए।
 धिधधक प्राधधकरण के रूप में न्यायालय की छधि को सशि करने के धलए तथा यह
सुधनधश्चत करने के धलए फक कोइ भी कानून से उपर नहीं है।
 यह सुधनधश्चत करने के धलए फक कोइ भी ऄपनी स्ितंत्र आच्छानुसार न्यायालय के अदेशों
की ऄिहेलना नहीं कर सके ।
जहां ऄनुच्छेद 19 1(a) में भाषण और ऄधभव्यधि की स्ितंत्रता जैसे ऄधधकार प्रदान फकये
गए हैं। िहीं ऄनुच्छेद 19 (2) के ऄंतगवत प्रधतपाफदत प्रधतबंधो के संदभव में न्यायालय की
ऄिमानना शधि को महत्िपूणव अधार प्राि होता है।
अलोचकों का मानना है फक:
 न्यायपाधलका के प्राधधकार को सीधमत करना या प्रधतष्ठा को क्षधत पहँचाने के अधार पर
न्यायपाधलका ने धनयधमत रूप से ऄसहमत ऄधभव्यधियों को दंधडत करने के धलए ऄपनी
ऄिमानना शधियों का ईपयोग फकया है ।
 ऐसे भाषण और ऄधभव्यधियां जो ऄधनिायव रूप से न्याय के िास्तधिक प्रशासन में
बाधक नहीं रही हैं,ईन्हें भी न्यायपाधलका की प्रधतष्ठा को क्षधत पहँचने के अधार पर
दंधडत फकया गया है।

228 www.visionias.in ©Vision IAS

Google it:- https://upscpdf.com


https://upscpdf.com << Download From >> https://upscpdf.com

ऄनुच्छेद 19 (1)(a) के ऄंतगवत फदए गए ऄधधकार महत्िपूणव हैं क्योंफक िे:


 नागररकों को ऄपनी राय व्यि करने में समथव बनाते हैं जो कु शल सािवजधनक नीधतयों के
धलए अिश्यक है।
 गुणि्त ा पूणव जीिन सुधनधश्चत करने के धलए स्ियं में यह महत्िपूणव हैं। संधिधान के
ऄनुच्छेद 21 में भी ये ऄधधकार धनधहत हैं।
आस प्रकार, भाषण की स्ितंत्रता और न्यायालयों की ऄिमानना शधि के बीच सामंजस्य
स्थाधपत करना अिश्यक हो जाता है। धनम्नधलधखत तथ्यों को ध्यान में रखते हए आसे
सुधनधश्चत फकया जा सकता है:
 न्यायपाधलका ने स्ियं ही ऄिमानना की शधि के संयधमत प्रयोग के संबंध में फदशा-
धनदेश धनधावररत फकये हैं धजनके ऄनुसार स्ितंत्र अलोचना के ऄधधकार और
न्यायपाधलका की गररमा के बीच समन्िय स्थाधपत फकया जाना चाधहए, ईदाहरण के
धलए, मुल्गांिकर िाद 1978। साथ ही, राम दयाल मकव रहर बनाम मध्य प्रदेश 1978;
कन्साआंरटयस समूह बनाम मोहममद यूनुस 1987; P.N. डु डा बनाम P. धशि शंकर
1988; संजय नारायण, लहदुस्तान टाआमस बनाम ईच्च न्यायालय आलाहाबाद 2011 जैसे
प्रकरणों में की गयी रटप्पधणयां भी आस सन्दभव में महत्िपूणव हैं ।
 न्यायालय की ऄिमानना ऄधधधनयम, 1971 में 2006 में फकये गये संशोधन के ऄनुसार
" फकसी भी ऄिमानना की कायविाही में िैध बचाि हेतु में न्यायालय ‘ सत्य के अधार
पर औधचत्य की धसधद् (justification by truth )’ की ऄनुमधत दे सकता है यफद िह
संतुष्ट हो जाता है फक आस संबंध में बचाि पक्ष की ओर से फकया गया अग्रह प्रमाधणक
तथा सािवजधनक धहत में है ।"
ऄंतरावष्ट्रीय मानक और ऄन्य लोकतंत्रों के कानूनों से प्रेरणा ग्रहण की जा सकती है तथा आसके
माध्यम से ईपयुि मानदंडो का धनधावरण फकया जा सकता है। ईदाहरण के धलए, जमवनी,
फ्ांस, बेधल्जयम, अधस्ट्रया, आटली जैसे यूरोपीय लोकतंत्रों में न्यायालय की ऄिमानना का
ईल्लंघन करने पर सजा देने के धलए कोइ भी कानून नहीं है। U.K. में, न्यायालय की
ऄिमानना शधि ोत्म की जा चुकी है। संयुि राज्य ऄमेररका में, प्रेस और प्रकाशन के धिरूद्
ऄिमानना की शधि का ईपयोग तभी फकया जाता है, यफद लंधबत िाद के िास्ति में प्रभाधित
होने की अशंका हो।

7. ई्त रदाधयत्ि का धसद्ांत धिधध के शासन का एक ऄधनिायव ऄंग है। आस संदभव में,भारत में
न्यायेतर हत्या की घटनाओं के प्रधत सरकार के दृधष्टकोण एिं न्यायपाधलका की प्रधतफक्रया में
धिद्यमान कधमयों पर चचाव कीधजए।
दृधष्टकोणः
 फदए गए कथन के ऄथव की व्याख्या करते हए बताआए फक ई्त रदाधयत्ि और धिधध के
शासन के बीच क्या संबंध है।
 न्यायेतर हत्याओं के संदभव में , सरकार के दृधष्टकोण की कधमयों पर चचाव कीधजए ।
 धस्थधत को सुधारने के धलए न्यायपाधलका द्वारा फदए गए और ऄपनाए गए सुझािों पर
चचाव कीधजए।
ई्त रः
धिधध का शासन एक प्रशासधनक धसद्ांत है, धजसमें राज्य सधहत ऄन्य संस्थाएं सािवजधनक
रूप से प्रख्याधपत, समान रूप से लागू और स्ितंत्र रूप से धनणीत कानूनों के प्रधत ई्त रदायी
हैं। ई्त रदाधयत्ि का धसद्ांत धिधध के शासन का एक ऄधनिायव ऄंग है।
न्यायेतर हत्याएं
न्यायेतर हत्या, धिधध की मंजूरी के धबना, सरकारी ऄधधकाररयों द्वारा फकसी व्यधि की हत्या
है। भारत में पुधलस और सशस्त्र बलों द्वारा की गयी न्याधयक हत्याओं के अरोप तथा ईदाहरण
सामने अए हैं।

229 www.visionias.in ©Vision IAS

Google it:- https://upscpdf.com


https://upscpdf.com << Download From >> https://upscpdf.com

आससे भारत में न्यायेतर हत्याओं को धनयंधत्रत करने के समबन्ध में गंभीर लचता जताइ गइ है,
धजसमें सरकार के तरीकों में धनम्नधलधखत लापरिाही शाधमल हैः
 ऐसी धशकायतों की जांच के धलए एक स्ितंत्र धनकाय की ऄनुपधस्थधत के कारण न्यायेतर
हत्याओं की समुधचत रूप से जांच नहीं हो पाइ है।
 राष्ट्रीय मानिाधधकार अयोग (NHRC) द्वारा ईधल्लधखत फदशाधनदेशों को सरकार द्वारा
प्रायः पूणव रूप से कायावधन्ित न फकए जाने के कारण NHRC की सफलता सीधमत रह
गइ है।
 यूनाआटेड नेशंस कन्िेंशन ऄगेंस्ट टाचवर तथा आंटरनेशनल कन्िेंशन फॉर द प्रोटेक्शन ऑफ़
अल पसवन्स फ्ॉम एनफोस्डव धडसैधपयरेन्स (International Convention for the
Protection of All Persons from Enforced Disappearances) को ऄंगीकृ त
करने में सरकार की धिफलता।
 न्यायेतर हत्याओं के धशकार लोगों के पररिारों के धलए पयावि मुअिजा प्रणाली का
ऄभाि।
 सशस्त्र बलों में अंतररक ऄनुशासनात्मक सुनिाइ में दोषधसधद् की धनम्न दर और
पारदर्वशता की कमी। न्याधयक धिलंब के कारण भी दोषधसधद् कइ बार करठन हो जाती
है।
 अमडव फोसेज (स्पेशल पॉिसव) एक्ट [AFSPA] के कारण भी राष्ट्रीय धहत में सरकारी
ऄधधकाररयों के ई्त रदाधयत्ि कु छ कम हो जाते हैं ।
न्यायपाधलका की प्रधतफक्रया
 अर. एस. सोढी बनाम ई्त र प्रदेश सरकार, 1992: आस मामलें में सिोच्च न्यायालय ने
धनणवय फदया फक प्रत्येक पुधलस मुठभेड़ की जांच होनी चाधहए।
 2014 में सिोच्च न्यायालय ने न्यायेतर हत्याओं की जांच के धलए धिस्तृत फदशा-धनदेश
जारी फकए:
o CID या एक िररष्ठ पुधलस ऄधधकारी की देखरेख में एक ऄन्य पुधलस स्टेशन की
टीम द्वारा मुठभेड़ों की स्ितंत्र जांच हो।
o जांच ररपोटव को संबंधधत न्यायालय में ऄग्रेधषत फकया जाना चाधहए, ताफक
ऄदालती जांच कर ऄंधतम ररपोटव प्रस्तुत की जा सके ।
o घटना के तुरंत बाद संबंधधत ऄधधकाररयों को कोइ पदोन्नधत या तत्काल िीरता
पुरस्कार नहीं फदया जाएगा।
o यफद फकसी पुधलस ऄधधकाररयों के धखलाफ कोइ चाजवशीट दायर की जाती है, तो
ईस ऄधधकारी के धखलाफ ऄनुशासनात्मक कारव िाइ अरंभ करने और ईसे धनलंधबत
करने के साथ ईनकी सुनिाइ को भी शीर अ ही पूरा फकया जाना चाधहए।
o यफद ऄधधकारी ईच्चतम न्यायालय के धनदेशों का पालन करने में धिफल रहते हैं, तो
पीधड़तों के ररश्तेदार सत्र न्यायालय में भी जा सकते हैं।
हाल ही में, सिोच्च न्यायालय ने जनधहत याधचका के अधार पर मधणपुर में संफदग्ध न्यायेतर
हत्याओं के मामलों की CBI जांच का अदेश फदया था। आसी तरह, न्यायालय ने धपछले िषव
यह धनणवय फदया था फक सेना को धजन क्षेत्रों में AFSPA के ऄंतगवत धिशेष शधियां प्राि हैं,
ईन क्षेत्रों में भी ऄत्याचार की धशकायत पर सेना जांच से बच नहीं सकती है। न्यायालय ने
कें द्र को संबोधधत करते हए ईसे भी मानिाधधकार अयोग की लचताओं को ध्यान में रखकर
धस्थधत का समाधान करने के धनदेश फदए हैं। ऐसा करके न्यायालय ने आस बात को ही
दोहराया है फक ई्त रदाधयत्ि का धसद्ांत धिधध के शासन का एक ऄधनिायव ऄंग है।

230 www.visionias.in ©Vision IAS

Google it:- https://upscpdf.com


https://upscpdf.com << Download From >> https://upscpdf.com

12. धिगत िषों में संघ लोक सेिा अयोग (UPSC) द्वारा पू छे
गए प्रश्न (Past Year UPSC Questions)
1. In many democratic countries radio and television are not under the
control of the state. Do you think that the same policy should be adopted
in India? Mention briefly the points in favour of and against such a step.
(Not more than 200 words) (80/I/14/25)
कइ लोकतांधत्रक देशों में रेधडयो और टेलीधिजन राज्य के धनयंत्रणाधीन नहीं हैं। क्या अप
मानते हैं फक भारत में भी यही नीधत ऄपनाइ जानी चाधहए?आस प्रकार के कदम के पक्ष और
धिपक्ष में ऄपना धिचार संधक्षि रूप में प्रकट कीधजए।
2. The Press in India is free to publish any news and views except those,
which are objectionable from the point of view of the security of state,
friendly relations with Foreign States etc. What steps have been taken
recently by Government to prevent monopoly of the management of the
newspaper, to encourage the growth of small newspapers and to prevent
the exploitation of working journalists and other employees of Indian
newspapers? (Not more than 150 words) (81/I/6/25)
भारत में प्रेस फकसी भी समाचार और मत को प्रकाधशत करने के धलए स्ितंत्र है, धसिाय ईनके
जो राज्य की सुरक्षा, धिदेशी राज्यों के साथ मैत्रीपूणव संबंधों अफद के दृधष्टकोण से
अपध्त जनक हैं। हाल ही में सरकार ने समाचार पत्रों के प्रबंधन में एकाधधकार को रोकने,
छोटे समाचार पत्रों का धिकास प्रोत्साधहत करने और भारतीय समाचार पत्रों के कायवशील
पत्रकारों और ऄन्य कमवचाररयों का शोषण रोकने हेतु क्या कदम ईठाए हैं?
3. Bring out the significance of the Fundamental rights provided in the
Constitution of India. The right to acquire, hold and dispose of property
has ceased to be a fundamental right. Examine the purpose of the
change involved. (in about 150 words) (81/II/4a/20)
भारत के संधिधान में प्रद्त मूल ऄधधकारों का महत्ि बताआए। संपध्त ऄधधग्रहीत करने ,
धारण करने और धिक्रय का ऄधधकार, ऄब मूल ऄधधकार नहीं है। आस पररितवन के ऄंतर्वनधहत
ईद्देश्य का परीक्षण कीधजए। (लगभग 150 शब्दों में) (81/II/4a/20)
4. Why has there been reservation of seats for Scheduled Castes and
Tribes in the legislatures and in public services? Has the purpose been
achieved? Indicate recent developments. (in about 150 words)
(81/II/4b/20)
ऄनुसूधचत जाधतयों और जनजाधतयों के धलए धिधाधयकाओं और सािवजधनक सेिाओं में सीटों
का अरक्षण क्यों फकया गया है? क्या आनका ईद्देश्य फलीभूत हअ है? हाल के घटनाक्रम को
आंधगत करें।
5. Differentiate between Fundamental Rights and Directive Principles of
State Policy. Do you think that the latter have been adequately
implemented? Give reasons for your views (in about 150 words)
(82/II/5c/20)
मूल ऄधधकारों और राज्य की नीधत के धनदेशक तत्िों के बीच ऄंतर कीधजए। क्या अप मानते
हैं फक राज्य की नीधत के धनदेशक तत्िों को पयावि रूप से कायावधन्ित फकया गया है ? ऄपने
धिचार के समथवन में कारणों का ईल्लेख कीधजए।

231 www.visionias.in ©Vision IAS

Google it:- https://upscpdf.com


https://upscpdf.com << Download From >> https://upscpdf.com

6. Consider the recommendations of the Mandal Commission and offer


your comments, referring to the situations obtaining in the country. (in
about 150 words). (83/II/4c/20)
मंडल अयोग की ऄनुशंसाओं पर धिचार कीधजए और देश में धिद्यमान पररधस्थधतयों का
ईल्लेख करते हए रटप्पणी कीधजए।
7. What is meant by Habeas Corpus? What is the purpose of a writ of
Habeas Corpus? (83/II/8a(B)/2)
बंदी प्रत्यक्षीकरण से क्या अशय है? बंदी प्रत्यक्षीकरण याधचका का क्या ईद्देश्य है?
8. What is the present status of the Right to Property? (84/II/8a(B)/3)
संपध्त के ऄधधकार की ितवमान धस्थधत क्या है?
9. What is dealt with in Articles 25 of Indian Constitution? What was the
controversy about it recently? (Not more than 100 words) (84/I/9a/20)
भारतीय संधिधान के ऄनुच्छेद 25 में फकस संबंध में चचाव की गइ है? हाल ही में आसके धिषय
धििाद क्या था?
10. What are the main causes of anti-reservation stir in Gujarat? What are
the provisions in our Constitution regarding reservation? Do your
consider the policy of reservation justified? (About 200 words)
(85/I/11/35)
गुजरात में अरक्षण धिरोधी अंदोलन के पीछे मुख्य कारण क्या हैं? अरक्षण के संबंध में
हमारे संधिधान में क्या प्रािधान हैं? क्या अप अरक्षण की नीधत को ईधचत मानते हैं?
11. What to you understand by "preventive detention"? (86/II/8f(B)/3)
"धनिारक धनरोध" से अप क्या समझते हैं?
12. Discuss the importance of Article 32 of the Indian Constitution. (in about
150 words) (87/II/4c/20)
भारतीय संधिधान के ऄनुच्छेद 32 के महत्ि की धििेचना कीधजए।
13. Define the writ of Certiorari. (87/II/8a(B)/3)
ईत्प्रेषण ररट को पररभाधषत कीधजए।
14. What to you understand by 'positive discrimination'? (87/II/8b(B)/3)
'सकारात्मक धिभेद' से अप क्या समझते हैं?
15. Explain the concept of Minorities in the India Constitution and mention
the safeguards provided therein for their protection. (150 words)
(88/II/4b/20)
भारतीय संधिधान में ऄल्पसंख्यकों की संकल्पना की व्याख्या कीधजए और आसमें ईनके
संरक्षण हेतु प्रदत्त रक्षोपायों का ईल्लेख कीधजए।
16. What the "reasonable restrictions" mentioned in the Indian Constitution
accompanying the fundamental rights? (in about 150 words) (90/II/4d/20)
भारतीय संधिधान में मूल ऄधधकारों के साथ-साथ ईधल्लधखत "युधियुि धनबवन्धन" क्या हैं?

232 www.visionias.in ©Vision IAS

Google it:- https://upscpdf.com


https://upscpdf.com << Download From >> https://upscpdf.com

17. Explain the significance of Prasar Bharati Corporation in the context of


Modern mass media. (90/II/8a(B)/3)
अधुधनक जनसंचार माध्यमों के संदभव में प्रसार भारती धनगम का महत्ि स्पष्ट कीधजए।
18. Define writ of Mandamus. Explain its importance. (90/II/8e(B)/3)
परमादेश ररट को पररभाधषत कीधजए। आसका महत्ि स्पष्ट कीधजए। (90/II/8e(B)/3)
19. Discuss the secular nature of Indian polity and the position of minorities
in India. (in 150 words) (91/II/4c/20)
भारतीय राज्य व्यिस्था की धमवधनरपेक्ष प्रकृ धत और भारत में ऄल्पसंख्यकों की धस्थधत की
धििेचना कीधजए।
20. The writ of Mandamus will not be granted against certain persons. Who
are they? (92/II/4c/20)
परमादेश की ररट कु छ व्यधियों के धिरुद् नहीं स्िीकृ त होगी। िे कौन हैं?
21. Distinguish between preventive detention and punitive detention.
(93/II/8a(B)/3)
धनिारक धनरोध और दंडात्मक धनरोध के बीच क्या ऄंतर है।
22. When and why was the National Literacy Mission founded?
(93/II/8c(B)/3)
राष्ट्रीय साक्षरता धमशन कब और क्यों अरंभ फकया गया था?
23. What is meant by 'equal protection of law'? (93/II/8d(B)/3)
'धिधध का समान संरक्षण' से क्या ऄधभप्राय है?
24. What is the purpose of Article 24 of the Constitution of India?
(93/II/8f(B)/3)
भारतीय संधिधान के ऄनुच्छेद 24 के क्या ईद्देश्य हैं?
25. Difference between the 'due process of law' and 'the procedure
established by law' in the context of deprivation of personal liberty in
India. (94/II/8a(B)/3)
भारत में धनजी स्ितंत्रता के िंधचत होने के संदभव में 'धिधध की समयक प्रफक्रया' और 'धिधध
द्वारा स्थाधपत प्रफक्रया' के बीच क्या ऄंतर है।
26. Explain the meaning of ex post-facto legislation (94/II/8b(B)/3)
भूतलक्षी प्रभाि धिधध का ऄथव समझाआए।
27. Indicate the provisions of Indian Constitution relating to Secularism.
(94/II/8e(B)/3)
धमवधनरपेक्षता से संबंधधत भारतीय संधिधान के प्रािधानों पर प्रकाश डाधलए।
28. What are the constitutional rights of the citizens of India? What do you
think about the demand of the NRI's for dual citizenship? (150 words)
(95/II/4c/20)
भारत के नागररकों के क्या संिैधाधनक ऄधधकार हैं? NRI की दोहरी नागररकता की माँग के
संबंध में अपके क्या धिचार हैं?
29. What is the present status of the right to property as a Fundamental
Right? (95/II/8b(B)/3)
मूल ऄधधकार के रूप में संपध्त के ऄधधकार की ितवमान धस्थधत क्या है?

233 www.visionias.in ©Vision IAS

Google it:- https://upscpdf.com


https://upscpdf.com << Download From >> https://upscpdf.com

30. Why is Article 32 considered as the cornerstone of the Constitution?


(95/II/8c(B)/3)
ऄनुच्छेद 32 को संधिधान की अधारधशला क्यों माना जाता है?
31. The writ of Mandamus cannot be granted against certain persons. Who
are they? (96/II/8c(B)/3)
कु छ पदाधधकाररयों के धिरूद् ‘परमादेश’ जारी नहीं फकया जा सकता। ये पदाधधकारी कौन हैं?
32. What are the provisions regarding the protection of Linguistic minorities
in the Constitution? (in about 75 words) (97/I/3d/10)
संधिधान में भाषाइ ऄल्पसंख्यकों के संरक्षण के संबंध में क्या प्रािधान हैं?
33. What is Social Justice? How can reservation of seats for women in
Parliament contribute to the establishment of a socially just society in
India? (97/II/1b/40)
सामाधजक न्याय क्या है? संसद में मधहलाओं के धलए सीटों का अरक्षण फकस प्रकार भारत में
सामाधजक रूप से न्यायपूणव समाज की स्थापना में योगदान दे सकता है?
34. What, according to the Supreme Court, Constituted 'The Basic Features'
which it upheld in case known as
(i) Keshavanand Bharati v/s. State of Kerala (1973)
(ii) Minerva Mills v/s. Union of India (1990)? (in about 150 words)
(97/II/4c/20)
सिोच्च न्यायालय द्वारा 'अधारभूत धिशेषताओं’ में फकन्हें सधममधलत फकया गया हैं धजन्हें
धनम्न िादों में ऄिधाररत फकया गया था:
(i) के शिानंद भारती बनाम के रल राज्य (1973)
(ii) धमनिाव धमल्स बनाम भारत संघ (1990)?
35. What specific provisions exist in the Constitution of India about child
labour? (97/II/8e(B)/3)
बाल श्रम के संबंध में भारत के संधिधान में क्या धिधशष्ट प्रािधान धनधहत हैं?
36. What are the circumstances leading to the promulgation of Prasar Bharti
Ordinance in August 1998? (in about 50 words) (98/I/7a/6)
ऄगस्त,1998 में प्रसार भारती ऄध्यादेश जारी करने हेतु ईत्तरदायी धस्थधतयाँ क्या थीं?
37. State the amplitude of Article 21 of the Constitution. (98/II/8c(B)/3)
संधिधान के ऄनुच्छेद 21 का धिस्तार बताआए।
38. On what grounds does Article 15 of the Indian Constitution prohibit
discrimination? Indicate the way the concept of 'Special protection' has
qualified this prohibition, and contributed to social change. (in about 250
words) (99/II/1b/40)
भारतीय संधिधान के ऄनुच्छेद 15 द्वारा फकन अधारों पर भेदभाि धनधषद् है? आंधगत
कीधजए फक फकस प्रकार से 'धिशेष संरक्षण' के प्रत्यय ने आस धनधषद् भेदभाि को मयावफदत
फकया है और सामाधजक पररितवन को बढ़ािा फदया फदया है।

234 www.visionias.in ©Vision IAS

Google it:- https://upscpdf.com


https://upscpdf.com << Download From >> https://upscpdf.com

39. What is the status of the right to Property in the Indian Constitution? (in
about 25 words) (99/II/9e/3)
भारतीय संधिधान में संपध्त के ऄधधकार की धस्थधत क्या है?
40. Discuss the constitutional provisions regarding the rights of children. (in
about 150 words) (01/I/8c/15)
बालकों के ऄधधकारों के संबंध में संिैधाधनक प्रािधानों की धििेचना कीधजए।
41. Discuss how the Constitution of India provides equal rights. (in about 250
words) (04/I/7a/30)
चचाव कीधजए फक भारत का संधिधान फकस प्रकार समान ऄधधकार प्रदान करता है।
42. What is Habeas Corpus? (20 words) (04/I/9a/2)
बंदी प्रत्यक्षीकरण क्या होता है?
43. What is the special facility provided to the linguistic minorities under
Article 350 A? (04/I/9c/10)
ऄनुच्छेद 350A के ऄधीन भाषाइ ऄल्पसंख्यकों को कौन-सी धिशेष सुधिधाएँ प्रदान की गइ हैं?
44. Give your views on the right to freedom of religion as enshrined in the
Indian Constitution. Do they make India a secular State (250 words)
(05/I/7b/30)
भारतीय संधिधान में प्रधतष्ठाधपत धमव की स्ितंत्रता के ऄधधकार पर ऄपने धिचार प्रकट
कीधजए। क्या यह ऄधधकार भारत को एक धमवधनरपेक्ष राज्य बनाता है?
45. What are the constitutional limitations on the free movements of Indians
throughout the country? (150 words) (05/I/8a/15)
भारतीयों की समस्त देश में धनबावध अिाजाही पर कौन-सी संिैधाधनक सीमाएँ हैं?
46. What is the meant by 'double jeopardy'? (20 words) (05/I/9a/2)
‘दोहरे जोधखम ’ का क्या ऄथव है?
47. What is right to life and personal liberty? How have the courts expanded
its meaning in recent years ? (in 250 words) (06/I/6a/30)
जीिन और िैयधिक स्ितंत्रता का ऄधधकार क्या होता है ? हाल के िषों में न्यायालयों ने
आसके ऄथव का फकस प्रकार धिस्तार फकया है ?
48. Bring out the difference between the Fundamental Rights and the
Directive Principles of State Policy. Discuss some of the measures taken
by the Union and State Governments for the implementation of the
Directive Principles of State Policy. (250 words) (07/I/6b/30)
मूल ऄधधकारों और राज्य की नीधत के धनदेशक तत्िों के बीच ऄंतर स्पष्ट कीधजए। राज्य के
नीधत के धनदेशक तत्िों के कायावन्ियन के धलए संघ और राज्य सरकारों द्वारा ईठाए गए
ईपायों में से कु छ की चचाव कीधजए।
49. What is the importance of Right to Constitutional Remedies? (07/I/9e/2)
संिैधाधनक ईपचारों के ऄधधकार का क्या महत्ि है?

235 www.visionias.in ©Vision IAS

Google it:- https://upscpdf.com


https://upscpdf.com << Download From >> https://upscpdf.com

50. 'As we live in a plural society we need the greatest freedom to express
our opinions even if others find it offensive' – Do you agree? Discuss with
reference to some recent incidents in the Indian context. (09/I/9c/15)
चूंफक हम ऄनेकतािादी समाज में रहते हैं हमें ऄपने मतों को ऄधभव्यि करने की ऄधधकतम
स्ितंत्रता की अिश्यकता है, भले ही िे ऄन्यों को ऄधप्रय लगे ’ - क्या अप आस बात से सहमत
हैं? भारत के संदभव में हाल की कु छ घटनाओं का ईल्लेख करते हए आस पर चचाव कीधजए।
51. Discuss Section 66A of IT Act, with reference to its alleged violation of
Article 19 of the Constitution. (2013)
सूचना प्रौद्योधगकी ऄधधधनयम की धारा 66A की आससे कधथत संधिधान के ऄनुच्छेद 19 के
कधथत ईल्लंघन के संदभव में धििेचना कीधजए।
52. What do you understand by the concept “freedom of speech and
expression”? Does it cover hate speech also? Why do the films in India
stand on a slightly different plane from other forms of expression?
Discuss. (2014)
अप "िाक् एिं ऄधभव्यधि स्िातंत्र्य" की संकल्पना से क्या समझते हैं? क्या आसकी पररधध में
घृणा िाक् भी अता है? भारत में फफल्में ऄधभव्यधि के ऄन्य रूपों से तधनक धभन्न स्तर पर
क्यों हैं? चचाव कीधजए। (2014)
53. Khap Panchayats have been in the news for functioning as extra-
constitutional authorities, often delivering pronouncements amounting to
human rights violations. Discuss critically the actions taken by the
legislative, executive and the judiciary to set the things right in this
regard. (2015)
खाप पंचायतें संधिधनेतर प्राधधकरणों के तौर पर प्रकायव करने , ऄक्सर मानािधधकार
ईलंघनों की कोरट में अने िाले धनणवयों को देने के कारण खबरों में बन रही हैं। आस संबंध में
धस्थधत को ठीक करने के धलए धिधानमंडल, कायवपाधलका और न्यायपाधलका द्वारा की गइ
कारविाआयों पर समालोचनात्मक चचाव कीधजए।
54. Examine the scope of Fundamental Rights in the light of the latest
judgement of the Supreme Court on Right to Privacy. (2017)
धनजता के ऄधधकार पर ईच्चतम न्यायालय के निीनतम धनणवय के अलोक में , मौधलक
ऄधधकारों के धिस्तार का पररक्षण कीधजए।

Copyright © by Vision IAS


All rights are reserved. No part of this document may be reproduced, stored in a retrieval system or transmitted in
any form or by any means, electronic, mechanical, photocopying, recording or otherwise, without prior permission
of Vision IAS.

236 www.visionias.in ©Vision IAS

Google it:- https://upscpdf.com


https://upscpdf.com << Download From >> https://upscpdf.com

मूल कर्त्तव्य
विषय सूची
1. मूल कर्त्तव्य: एक विचार ____________________________________________________________________ 238

1.1. सरदार स्िर्त ससह सवमवि _______________________________________________________________ 238

1.2. मूल कर्त्तव्यों का संविप्त वििरर् ____________________________________________________________ 239


1.2.1. ऄनुच्छेद 51(A)(a): भारि के प्रत्येक नागररक का यह कर्त्तव्य होगा कक िह - संविधान का पालन करे और ईसके अदर्शों,
संस्थाओं, राष्ट्र ध्िज और राष्ट्रगान का अदर करे। ____________________________________________________ 239
1.2.2. ऄनुच्छेद 51(A)(b): भारि के प्रत्येक नागररक का यह कर्त्तव्य होगा कक िह-स्ििंत्रिा के वलए हमारे राष्ट्रीय अंदोलन को
प्रेररि करने िाले ईच्च अदर्शों को हृदय में संजोए रखे और ईनका पालन करे।_________________________________ 239

1.3. मूल कर्त्तव्यों को सवममवलि करने के ईद्देश्य _____________________________________________________ 242

1.4. मूल कर्त्तव्यों की विर्शेषिाएँ_______________________________________________________________ 243

1.5. मूल कर्त्तव्यों, नीवि वनदेर्शक ित्त्िों और मूल ऄवधकारों के मध्य समबन्ध___________________________________ 243

1.6. विवभन्न सवमवियाँ और न्यावयक वनितचन ______________________________________________________ 243

1.7. मूल कर्त्तव्य हेिु निीन संदभत ______________________________________________________________ 244

1.8. अलोचना __________________________________________________________________________ 244

1.9. प्रासंवगकिा ________________________________________________________________________ 245

1.10. मूल कर्त्तव्यों को प्रभािी बनाने हेिु कु छ सुझाि _________________________________________________ 245


2. विगि िषों में Vision IAS GS मेंस टेस्ट सीरीज में पूछे गए प्रश्न (Previous Year Vision IAS GS Mains Test
Series Questions) _______________________________________________________________________ 245
3. विगि िषों में संघ लोक सेिा अयोग (UPSC) द्वारा पूछे गए प्रश्न (Past Year UPSC Questions) _______________ 247

237

Google it:- https://upscpdf.com


https://upscpdf.com << Download From >> https://upscpdf.com

1. मूल कर्त्तव्य : एक विचार


 मूल भारिीय संविधान में नागररकों के कर्त्तव्यों से संबंवधि भाग को सवममवलि नहीं ककया गया
था। 42िें संविधान संर्शोधन ऄवधवनयम, 1976 द्वारा संविधान में एक नया भाग IV-A जोड़ा
गया। आसके द्वारा ऄनुच्छेद 51 के िहि ऄनुच्छेद 51(A) जोड़िे हुए 10 मूल कर्त्तव्यों को र्शावमल
ककया गया। 11िें मूल कर्त्तव्य को 86िें संविधान संर्शोधन ऄवधवनयम, 2002 द्वारा जोड़ा गया था।
आन्हें नागररकों के नैविक दावयत्िों के रूप में पररभावषि ककया गया है।
 आनकी प्रेरर्ा भूिपूित सोवियि संघ के संविधान से ग्रहर् की गयी है। मूल कर्त्तव्यों से संबंवधि
प्रािधानों को विश्व के ऄन्य प्रमुख लोकिांवत्रक देर्शों, जैसे- ऄमेररका, कनाडा, फ्ांस, जमतनी,
अस्रेवलया अकद में नहीं ऄपनाया गया है। हालाँकक, भारिीय संविधान में आन्हें ऄपनाए जाने के
कारर्ों में भारिीय सांस्कृ विक एिं भौगोवलक विविधिा और देर्श की एकिा एिं ऄखंडिा की
सुरिा के वलए सकिय नागररक सहभावगिा की अिश्यकिा के साथ ही युद्ध से ईभरी पररवस्थवियों
में भारिीय सुरिा व्यिस्था की सुदढ़ृ िा जैसे कारक प्रमुख रूप से ईर्त्रदायी थे।
 जहां वनदेर्शक ित्ि राज्य के वलए वनदेवर्शि हैं, िहीं मूल कर्त्तव्यों को नागररकों के वलए वनदेवर्शि
ककया गया है। मूल ऄवधकारों का लाभ ईठाने िाले नागररकों को संविधान के अदर्शों का सममान
करना चावहए और सद्भाि एिं भ्रािृत्ि की भािना को प्रोत्सावहि करना चावहए।
 मूल कर्त्तव्य गैर-न्यायोवचि हैं। आसका ऄथत है कक नागररकों को ईनका पालन करने के वलए बाध्य
नहीं ककया जा सकिा है। हालांकक, आनमें से कु छ प्रिितनीय कानून के भाग हैं। ईदाहरर् के वलए-
राष्ट्रीय गौरि ऄपमान वनिारर् ऄवधवनयम, 1971 आत्याकद।

1.1. सरदार स्िर्त ससह सवमवि

 देर्श में अपािकाल लागू ककए जाने के िुरंि पश्चाि् प्रधानमंत्री आंकदरा गांधी द्वारा सरदार स्िर्त
ससह सवमवि गरठि की गयी। आस सवमवि का ईद्देश्य ऄिीि के ऄनुभिों के अलोक में संविधान
संर्शोधन से संबंवधि प्रश्नों का ऄध्ययन करना और संर्शोधनों की वसफाररर्श करना था।
 सरदार स्िर्त ससह सवमवि की वसफाररर्शों के अधार पर ही 42िें संर्शोधन ऄवधवनयम,1976
पाररि ककया गया, वजसे "लघु संविधान" भी कहा जािा है। आसके िहि, विवभन्न ऄनुच्छेदों और
यहाँ िक कक ईद्देवर्शका में भी संर्शोधन ककया गया।
 सरदार स्िर्त ससह सवमवि द्वारा 8 मूल कर्त्तव्यों को जोड़े जाने और आनका ऄनुपालन न ककये जाने
पर दंड के प्रािधान की वसफाररर्श की गयी थी।

भारि के प्रत्येक नागररक का यह कर्त्तव्य होगा कक िह-


(a) संविधान का पालन करे और ईसके अदर्शों, संस्थाओं, राष्ट्र ध्िज और राष्ट्रगान का अदर करे ;
(b) स्ििंत्रिा के वलए हमारे राष्ट्रीय अंदोलन को प्रेररि करने िाले ईच्च अदर्शों को हृदय में संजोए
रखे और ईनका पालन करे;
(c) भारि की प्रभुिा, एकिा और ऄखंडिा की रिा करे और ईसे ऄिुण्र् रखे ;

238 www.visionias.in ©Vision IAS

Google it:- https://upscpdf.com


https://upscpdf.com << Download From >> https://upscpdf.com

(d) देर्श की रिा करे और अह्िान ककए जाने पर राष्ट्र की सेिा करे ;
(e) भारि के सभी लोगों में समरसिा और समान भ्रािृत्ि की भािना का वनमातर् करे जो धमत, भाषा
और प्रदेर्श या िगत पर अधाररि सभी भेदभाि से परे हो, ऐसी प्रथाओं का त्याग करे जो वियों के
सममान के विरं द्ध है;
(f) हमारी सामावसक संस्कृ वि की गौरिर्शाली परंपरा का महत्ि समझे और ईसका परररिर् करे ;
(g) प्राकृ विक पयातिरर् की, वजसके ऄंिगति िन, झील, नदी और िन्य जीि हैं, रिा करे और ईसका
संिधतन करे िथा प्रावर् मात्र के प्रवि दयाभाि रखे ;
(h) िैज्ञावनक दृविकोर्, मानििाद और ज्ञानाजतन िथा सुधार की भािना का विकास करे ;
(i) साितजवनक संपवर्त् को सुरविि रखे और सहसा से दूर रहे;
(j) व्यविगि और सामूवहक गविविवधयों के सभी िेत्रों में ईत्कषत की ओर बढ़ने का सिि प्रयास करे
वजससे राष्ट्र वनरंिर बढ़िे हुए प्रयत्न और ईपलवधध की नइ उँचाआयों को छू ले; और
(k) मािा-वपिा या संरिक है, छह िषत से चौदह िषत बीच की अयु के , यथावस्थवि, बच्चे या
प्रविपाल्य के वलए वर्शिा के ऄिसर प्रदान करेगा।

1.2. मू ल कर्त्त व्यों का सं विप्त वििरर्

1.2.1. ऄनु च्छे द 51(A)(a): भारि के प्रत्ये क नागररक का यह कर्त्त व्य होगा कक िह -
सं विधान का पालन करे और ईसके अदर्शों , सं स्थाओं, राष्ट्र ध्िज और राष्ट्रगान का अदर
करे ।

वििरर्
(i) संविधान की ईद्देवर्शका में वनम्नवलवखि अदर्शों का िर्तन ककया गया है वजनका पालन करने एिं
ऄपनाने की नागररकों से ऄपेिा की गइ है:
 सामावजक, अर्थथक और राजनैविक न्याय;
 विचार, ऄवभव्यवि, विश्वास, धमत और ईपासना की स्ििंत्रिा;
 प्रविष्ठा और ऄिसर की समिा; और
 सभी व्यवियों की गररमा और राष्ट्र की एकिा और ऄखंडिा सुवनवश्चि करने िाली बंधि
ु ा का
विकास करना।
(ii) संविधान की संस्थाओं के ऄंिगति, मुख्यिः कायतपावलका, विधावयका एिं न्यायपावलका सवममवलि
हैं।
(iii) यकद ककसी नागररक की समस्या की सुनिाइ आन संस्थाओं द्वारा नहीं ककया जािा है िो आसके वलए
ईसे ‘विद्रोह, हड़िाल एिं सहसा’ का मागत ऄपनाने के बजाय कानूनी एिं संिैधावनक मागत को ऄपनाना
चावहए िथा साथ ही, आन संस्थाओं का सममान भी करना चावहए।

1.2.2. ऄनु च्छे द 51(A)(b): भारि के प्रत्ये क नागररक का यह कर्त्त व्य होगा कक िह -
स्ििं त्र िा के वलए हमारे राष्ट्रीय अं दोलन को प्रे ररि करने िाले ईच्च अदर्शों को हृदय में
सं जोए रखे और ईनका पालन करे ।

वििरर्ः
(i) भारिीय राष्ट्रीय अंदोलन को प्रेररि करने िाले कु छ ईच्च अदर्शत वनम्नवलवखि हैं:
 विदेर्शी र्शासन से मुवि एिं एक ऐसी स्िर्शासन व्यिस्था की स्थापना करना जो ऐसे समाज का
वनमातर् कर सके जहाँ व्यवि, व्यवि का र्शोषर् न करे िथा जहाँ वनधतनिा, भुखमरी, ऄवर्शिा के
वलए कोइ स्थान न हो।

239 www.visionias.in ©Vision IAS

Google it:- https://upscpdf.com


https://upscpdf.com << Download From >> https://upscpdf.com

 ईपयुति ईद्देश्यों की पूर्थि िभी संभि है जब सभी नागररकों को ऄपने व्यवित्ि के विकास का पूर्त
ऄिसर वमल सके ।
 आस प्रकार के पूर्त विकास हेिु व्यवित्ि-वनमातर् करने िाली वर्शिा की अिश्यकिा होगी।
 आसके वलए, प्रत्येक व्यवि को, राष्ट्रवहि को व्यविगि वहिों से उपर रखना होगा।
 कु छ ऄन्य अदर्शों में समिापूर्त समाज का वनमातर्, ऄसहसा, भाइचारा, विश्व र्शांवि एिं स्ियं
ऄनुच्छेद 51(A) में ईवल्लवखि प्रािधान यथा मवहलाओं का सममान करना, भारि की सामावसक
संस्कृ वि की रिा करना, लोगों में िैज्ञावनक मनोिृवर्त् एिं मानििाद का विकास करना अकद
र्शावमल हैं।
ऄनुच्छेद 51(A)(c): भारि के प्रत्येक नागररक का यह कर्त्तव्य होगा कक िह-
भारि की प्रभुिा, एकिा और ऄखंडिा की रिा करे िथा ईसे ऄिुण्र् बनाए रखे।
वििरर्ः
(i) एक लोकिांवत्रक व्यिस्था में संप्रभुिा, जनिा में ही वनवहि होिी है। ऄिः नागररकों का यह परम
कर्त्तव्य है कक िे भारि की संप्रभुिा, एकिा एिं ऄखंडिा की ककसी भी वस्थवि में रिा करें।
(ii) व्यिहाररक रूप में, भारि की संप्रभुिा, एकिा एिं ऄखंडिा की रिा हेिु भारिीय दंड संवहिा
(IPC) और दंड का प्रािधान ककया गया है।
ऄनुच्छेद 51(A)(d): भारि के प्रत्येक नागररक का यह कर्त्तव्य होगा कक िह-
देर्श की रिा करे और अह्िान ककये जाने पर राष्ट्र की सेिा करे।
वििरर्
(i) अधुवनक राष्ट्र-राज्य की संकल्पना में यह वनवहि है कक प्रत्येक नागररक युद्ध या बाह्य अिमर् की
वस्थवि में ऄपने देर्श की रिा करने के कर्त्तव्य से बंधा है।
(ii) यहाँ ‘अह्िान ककये जाने’ का ऄथत िीनों सेनाओं के ऄविररि अम नागररकों से है वजनकी
अिश्यकिा ‘राष्ट्रीय सेिाओं’ के वलए हो सकिी है।
(iii) आसको ध्यान में रखिे हुए ऄनुच्छेद 23(2) में राज्य को यह ऄवधकार कदया गया है कक धमत,
मूलिंर्श, जावि, िगत या आनमें से ककसी भी अधार पर विभेद ककए बगैर िह नागररकों से ‘साितजवनक
प्रयोजनों की पूर्थि हेिु ‘ऄवनिायत सेिा’ ऄवधरोवपि कर सकिा है।
ऄनुच्छेद 51(A)(e): भारि के प्रत्येक नागररक का यह कर्त्तव्य होगा कक िह-
भारि के सभी लोगों में समरसिा और समान भ्रािृत्ि की भािना का वनमातर् करे जो धमत, भाषा और
प्रदेर्श या िगत पर अधाररि सभी भेदभाि से परे हो। ऐसी प्रथाओं का त्याग करे जो वियों के सममान के
विरद्ध हैं।
वििरर्ः
 यहाँ धमत, भाषा, प्रदेर्श या िगत अधाररि भेदभाि से परे होने का ऄथत आस विविधिा को सीवमि या
समाप्त करना नहीं है, बवल्क आनमें वनवहि सामूवहक एिं समान ित्िों के अधार पर समरूपिा की
भािना का लोगों में वनमातर् करना है।
 ईपयुति विवभन्न अधारों पर विविधिा के बािजूद सभी भारिीय एक संविधान, एक ध्िज एिं एक
नागररकिा के अधार पर अपस में जुड़े हुए हैं िथा यही हमारे भ्रािृत्ि की भािना के वनमातर् का
प्रमुख अधार है।
 आस ऄनुच्छेद का दूसरा भाग, लैंवगक अधार पर भेदभाि एिं पूिातग्रह का वनषेध करिा है। आसमें
मवहलाओं की गररमा के विरद्ध व्यिहार में लाइ जाने िाली ककसी भी प्रकार की प्रथाओं का विरोध
ककया गया है।
 मवहला की गररमा एिं सममान को व्यिहाररक रूप देने के वलए वनम्न प्रािधान ककये गए हैं-
o सरकार द्वारा पाररि सिी प्रथा (वनिारर्) ऄवधवनयम, 1987 एिं ईच्चिम न्यायालय द्वारा
िषत 1997 में कायतस्थल पर यौन ईत्पीड़न रोकने हेिु ‘विर्शाखा कदर्शा-वनदेर्श’ आस कदर्शा में
मील के पत्थर वसद्ध हुए हैं।

240 www.visionias.in ©Vision IAS

Google it:- https://upscpdf.com


https://upscpdf.com << Download From >> https://upscpdf.com

o ऄक्टूबर 2017 में ईच्चिम न्यायालय ने वनर्तय कदया कक 18 िषत से कम अयु की िी से


र्शारीररक संबंध बनाना (चाहे पवि द्वारा ही क्यों न हो) दुष्कमत की श्रेर्ी में र्शावमल है, चाहे
िह िी वििावहि हो या ऄवििावहि। यह िी के सममान की रिा हेिु एक सराहनीय एिं
प्रगविर्शील कदम है।
ऄनुच्छेद 51 (A)(f) : भारि के प्रत्येक नागररक का यह कर्त्तव्य होगा कक िह -
हमारी सामावसक संस्कृ वि की गौरिर्शाली परंपरा का महत्ि समझे और ईसका परररिर् करे।
वििरर्:
 हमारी सामावसक संस्कृ वि एिं गौरिर्शाली परंपरा के ऄंिगति वनम्नवलवखि र्शावमल हैं:
o पांच हजार िषों से भी ऄवधक पुरानी भारिीय सभ्यिा एिं संस्कृ वि।
o विश्व के समि प्रस्िुि मूर्थिकला, िास्िुकला, गवर्ि, विज्ञान, वचककत्सा, वचत्रकला से संबंवधि
भारिीयों का महत्िपूर्त योगदान।
o धरोहर के रूप में प्राचीन स्मारक, ककले, मंकदर, मवस्जद, चचत जैसे ऐविहावसक एिं
पुरािावत्त्िक महत्त्ि के स्थल।
o विवभन्न धमों यथा सहदू, बौद्ध, जैन, वसख अकद की जन्मस्थली होने का गौरि।
o सत्य, ऄसहसा एिं विश्व र्शांवि एिं बंधुत्ि के प्रवि हमारी दृढ़ वनष्ठा।
 ऄिः जो कु छ हमने ऄपने पूितजों से प्राप्त ककया है, ईसे ऄगली पीढ़ी िक ऄवधक सुरविि एिं समृद्ध
रूप में पहुँचाना हमारा कर्त्तव्य है।
 आसके वलए, नीवि-वनदेर्शक ित्त्िों के ऄनुच्छेद 49 के ऄंिगति राष्ट्रीय महत्त्ि या ऐविहावसक
ऄवभरवच िाले प्रत्येक संस्मारकों, स्थानों और िस्िुओं का संरिर् करना राज्य की बाध्यिा होगी।
 नागररक का बौवद्धक, र्शारीररक एिं अध्यावत्मक अकद सभी िेत्रों में विकास हमारी सामावसक
संस्कृ वि का ऄवभन्न ऄंग है। ऄिः ऄपने व्यवित्ि के विकास हेिु प्रयास करना, प्रत्येक नागररक का
कर्त्तव्य है।
 हमारी सामावसक संस्कृ वि सत्य, ऄसहसा जैसे नैविक मूल्यों पर विर्शेष बल देिी है।
 आसी सामावसक संस्कृ वि को परररविि करने हेिु भारि सरकार द्वारा 21 जून को ‘ऄंिराष्ट्रीय योग
कदिस’ मनाने का वनर्तय वलया गया, विश्व विरासि स्थल के रूप में स्थलों को संरिर् प्रदान ककया
गया िथा विवभन्न योजनाओं के माध्यम से समथतन प्रदान ककया गया है िाकक नागररक ऄपनी
संस्कृ वि की पांच हजार िषों से भी ऄवधक पुरानी गौरिर्शाली परंपरा के महत्त्ि को समझ सकें ।
ऄनुच्छेद 51(A)(g): भारि के प्रत्येक नागररक का यह कर्त्तव्य होगा कक िह-
प्राकृ विक पयातिरर् की, वजसके ऄंिगति िन, झील, नदी और िन्य जीि र्शावमल हैं, की रिा करे और
ईसका संिधतन करे िथा प्रावर् मात्र के प्रवि दयाभाि रखे।
वििरर्ः
 विगि कु छ िषों से वनरंिर बढ़ रहे पयातिरर् िरर्, प्रदूषर्, िनों की कटाइ, िैवश्वक िापन,
जलिायु पररिितन और िन्य जीिों के ऄिैध व्यापार एिं िस्करी ने समस्ि मानि प्रजावि के साथ-
साथ ऄन्य प्रावर्यों के समि ऄवस्ित्ि के समि संकट ईत्पन्न कर कदया है।
 ऄिः, यह प्रत्येक नागररक का कर्त्तव्य है कक िह ऄपने स्िर पर िनों की सुरिा करिे हुए
िृिारोपर्, नकदयों की सफाइ, जल संरिर्, साितजवनक स्थलों को स्िच्छ रखकर आस कदर्शा में
प्रयास करे।
 आसके ऄविररि, संविधान के नीवि-वनदेर्शक ित्त्िों के िहि ऄनुच्छेद 48(A) में यह ऄवभवनधातररि
ककया गया है कक, राज्य देर्श के पयातिरर् के संरिर् एिं संिधतन का और िन एिं िन्य जीिों की
रिा करने का प्रयास करेगा। वजसको व्यिहाररक रूप प्रदान करने हेिु राज्य ने पयातिरर् (संरिर्)
ऄवधवनयम, 1986; िन्य जीि ऄवधवनयम, 1972 अकद पाररि ककए हैं।
 हाल ही में, ईच्चिम न्यायालय ने कदल्ली में दीपािली के ऄिसर पर पटाखों की वबिी पर प्रविबंध
लगाकर आस कर्त्तव्य की ओर पुनः ध्यान अकर्थषि ककया है।

241 www.visionias.in ©Vision IAS

Google it:- https://upscpdf.com


https://upscpdf.com << Download From >> https://upscpdf.com

ऄनुच्छेद 51(A)(h): भारि के प्रत्येक नागररक का यह कर्त्तव्य होगा कक िह-


िैज्ञावनक दृविकोर्, मानििाद और ज्ञानाजतन िथा सुधार की भािना का विकास करे।
वििरर्:
 आसमें िेजी से बदलिी िैवश्वक पररवस्थवियों में मानि जीिन की गुर्िर्त्ा में िृवद्ध एिं संपूर्त
विकास हेिु प्रत्येक नागररक से मानििाद की भािना से ओि-प्रोि िैज्ञावनक दृविकोर् के विकास
की ऄपेिा की गइ है।
 िैज्ञावनक दृविकोर् का ऄथत है- सभी प्रकार के ऄंधविश्वासों से दूर रहना िथा ज्ञान की खोज एिं
िथ्यों के ऄनुसंधान के अधार पर व्यिवस्थि ज्ञान एिं ऄनुभि का विकास करना।
 ज्ञानाजतन एिं सुधार की भािना, सभी प्रकार के विकास हेिु पूितर्शित हैं।
ऄनुच्छेद 51(A)(i) : भारि के प्रत्येक नागररक का यह कर्त्तव्य होगा कक िह-
साितजवनक संपवर्त् को सुरविि रखे और सहसा से दूर रहे।
वििरर्ः
 विगि कु छ िषों में देखा गया है कक जब भी रैली, बंद, हड़िाल अकद होिे हैं िो आस दौरान
साितजवनक संपवर्त्यों यथा बस, रेन, भिनों अकद की िोड़फोड़ एिं अगजनी जैसी घटनाएँ प्रारंभ
हो जािी हैं जो कक आस मूल कर्त्तव्य का प्रत्यि ईल्लंघन है।
 आसे रोकने के वलए सिातवधक सर्शि ईपाय यही है कक विद्यालय के प्रारंवभक िषों में ही बच्चों में
आसके प्रवि जागरूकिा ईत्पन्न की जाए िाकक अने िाले िषों में साितजवनक संपवि की रिा करना
एिं सहसा से दूर रहना जैसे मनोभाि, ईनके चररत्र का वहस्सा बन जाए।
ऄनुच्छेद 51(A)(J): भारि के प्रत्येक नागररक का यह कर्त्तव्य होगा कक िह-
व्यविगि और सामूवहक गविविवधयों के सभी िेत्रों में ईत्कषत की ओर बढ़ने का सिि प्रयास करे वजससे
राष्ट्र वनरंिर बढ़िे हुए प्रयत्न और ईपलवधध की नइ उंचाआयों को छू ले।
वििरर्ः
 िितमान के ऄवि प्रविस्पधी दौर में जीिन के प्रत्येक िेत्र में ‘ईत्कृ ििा’ ही सभी प्रकार की सफलिा
एिं विकास का मानदंड बन गइ है, चाहे िह व्यविगि स्िर पर हो या सामूवहक स्िर पर।
 डॉ. दर्शरथी बनाम अंध्रप्रदेर्श िाद (1985) में न्यायालय ने कहा कक प्रत्येक नागररक का यह
कर्त्तव्य है कक व्यविगि िथा सामूवहक सभी स्िरों पर ईत्कृ ििा को प्राप्त करने का प्रयास करे िाकक
ईपलवधधयों एिं प्रगवि के नए मानदंडों को स्थावपि ककया जा सके ।
ऄनुच्छेद 51(A)(k): भारि के प्रत्येक नागररक का यह कर्त्तव्य होगा कक िह-
जो मािा-वपिा या संरिक हो िह, 6 से 14 िषत के बीच की अयु के , यथावस्थवि, ऄपने बच्चे ऄथिा
प्रविपाल्य को वर्शिा प्राप्त करने के ऄिसर प्रदान करे।
वििरर्ः
 आस मूल कर्त्तव्य को 86िें संविधान संर्शोधन, 2002 द्वारा ऄनुच्छेद 51(A) में जोड़ा गया।
 आसमें मािा-वपिा या ऄवभभािक को ऄपने बच्चों को वर्शविि करने का कर्त्तव्य सौंपा गया िाकक बच्चे
ऄपने व्यवित्ि का सिाांगीर् विकास कर सकें ।
 आसी संविधान संर्शोधन के द्वारा वर्शिा के ऄवधकार को ऄनुच्छेद 21(a) में मूल ऄवधकार के रूप में
िथा ऄनुच्छेद 45 में राज्य के दावयत्ि के रूप में सवममवलि ककया गया है।
मूल कर्त्तव्य, न के िल नागररकों पर बवल्क राज्य पर भी लागू होिे हैं।

1.3. मू ल कर्त्त व्यों को सवममवलि करने के ईद्दे श्य

42िें संविधान संर्शोधन ऄवधवनयम, 1976 द्वारा संविधान में मूल कर्त्तव्यों को वनम्नवलवखि ईद्देश्यों हेिु
सवममवलि ककया गया:
 नागररकों में देर्शभवि की भािना को बढ़ािा देने हेि;ु
 राष्ट्र को मजबूि करने िाली अचरर् संवहिा का पालन करने हेिु नागररकों की सहायिा करने हेिु ;
 देर्श की संप्रभुिा और ऄखंडिा की रिा करने हेिु ;

242 www.visionias.in ©Vision IAS

Google it:- https://upscpdf.com


https://upscpdf.com << Download From >> https://upscpdf.com

 राज्य के विविध कर्त्तव्यों के वनष्पादन में सहायिा करने हेि;ु


 सद्भािना के विचारों को बढ़ािा देने हेिु;
 राज्य के प्रवि नागररकों की प्रविबद्धिाओं को सुवनवश्चि करने हेि;ु
 साथ ही, व्याप्त ऄनुर्शासनहीनिा को समाप्त करने हेिु।

1.4. मू ल कर्त्त व्यों की विर्शे ष िाएँ

 ये नागररकों के नैविक दावयत्ि हैं;


 ये के िल नागररकों के वलए है न कक विदेवर्शयों के वलए;
 आनकी प्रकृ वि गैर-न्यायोवचि होिी है ऄथाति् ये न्यायालय में िाद योग्य नहीं हैं।

1.5. मू ल कर्त्त व्यों, नीवि वनदे र्श क ित्त्िों और मू ल ऄवधकारों के मध्य समबन्ध

 मूल कर्त्तव्य, नागररकों के ईर्त्रदावयत्ि हैं जबकक वनदेर्शक ित्त्ि , राज्य की नीवियों के प्रमुख अधार
हैं। हालाँकक, दोनों के ऄनुपालन संबंधी कोइ क़ानूनी बाध्यकाररिा वनधातररि नहीं की गयी है।
 मूल ऄवधकारों का ईपयोग करने िाले नागररकों को संविधान के अदर्शों का सममान करना चावहए
िथा सद्भाि और भाइचारे की भािना को बढ़ािा देना चावहए।
 ऄवधकार एिं कर्त्तव्य एक ही वसक्के के दो पहलू हैं। ित्कालीन प्रधानमंत्री श्रीमिी आंकदरा गाँधी द्वारा
कर्त्तव्यों के पि में िकत कदया गया था कक “मूल कर्त्तव्य, मूल ऄवधकारों को सीवमि नहीं करेंगे बवल्क
लोकिांवत्रक संिुलन स्थावपि करेंगे”। ईच्चिम न्यायालय द्वारा 1992 में व्यिस्था की गयी कक मूल
कर्त्तव्यों को कायातवन्िि करने के वलए वनर्थमि की गयी ककसी विवध को ऄनुच्छेद 14 एिं 19 के
ईल्लंघन के अधार पर ऄमान्य घोवषि नहीं ककया जायेगा।
 मूल कर्त्तव्य न्यायालय के माध्यम से प्रिितनीय नहीं हैं। आसका िात्पयत है कक नागररकों को आनका
पालन करने के वलए बाध्य नहीं ककया जा सकिा। िथावप, ईनमें से कु छ, प्रिितनीय कानून के भाग
हैं। ईदाहरर्ाथत– राष्ट्र गौरि ऄपमान वनिारर् ऄवधवनयम, 1971 (Prevention of Insults to
National Honor Act, 1971) अकद।

1.6. विवभन्न सवमवियाँ और न्यावयक वनित च न

 एम. सी. मेहिा बनाम भारि संघ िाद (1991) में ईच्चिम न्यायालय ने मूल कर्त्तव्यों से संबंवधि
वनम्न कदर्शावनदेर्श कदएः
o र्शैिवर्क संस्थाओं में सप्ताह में कम-से-कम एक घंटे के वलए पयातिरर् एिं िन्यजीिों के
संरिर् संबंधी प्रािधानों की वर्शिा प्रदान करने हेिु कें द्र सरकार व्यिस्था करेगी।
o आस ईद्देश्य की पूर्थि हेिु कें द्र सरकार, र्शैिवर्क संस्थाओं में वनःर्शुल्क पुस्िक वििरर् करेगी।
o आस ईद्देश्य की पूर्थि हेिु कें द्र सरकार, वर्शिकों के प्रवर्शिर् की व्यिस्था करेगी।
o कें द्र, राज्य एिं संघ-र्शावसि प्रदेर्शों की सरकारें ‘स्िच्छिा सप्ताह’ का अयोजन करेंगी।
o पयातिरर् की गुर्िर्त्ा में अ रही वगरािट के प्रवि लोगों को जागरूक करने हेिु ‘जागरूकिा
ऄवभयान’ चलाया जाएगा।
 1999 में नागररकों के मूल कर्त्तव्यों के वर्शिर् के विषय पर जवस्टस िमात सवमवि का गठन ककया
गया था। आसने विद्यालयों के पाठ्यिम और वर्शिकों के वर्शिा कायतिम िथा ईच्चिर एिं
व्यिसावयक वर्शिा के िेत्र में मूल कर्त्तव्यों को सवममवलि करने की वसफाररर्श की थी।
 िषत 2003 में ईच्चिम न्यायालय ने कें द्र को जवस्टस िमात सवमवि के सुझािों के ऄनुसार, नागररकों
द्वारा मूल कर्त्तव्यों के पालन हेिु कानून बनाने के वलए वनदेर्श कदया।
 भारि के पूित मुख्य न्यायाधीर्श, रंगनाथ वमश्रा ने भारि के मुख्य न्यायाधीर्श को एक पत्र वलखकर
ईच्चिम न्यायालय से मूल कर्त्तव्यों के विषय में ऄपने नागररकों को वर्शविि करने हेिु राज्य के वलए

243 www.visionias.in ©Vision IAS

Google it:- https://upscpdf.com


https://upscpdf.com << Download From >> https://upscpdf.com

अिश्यक वनदेर्श जारी करने का ऄनुरोध ककया था िाकक ऄवधकारों और कर्त्तव्यों के मध्य ईवचि
संिुलन स्थावपि ककया जा सके । आस पत्र को एक ररट यावचका के समान माना गया था।
 2002 में संविधान की कायतप्रर्ाली की समीिा करने के वलए राष्ट्रीय अयोग (National
Commission to Review the Working of the Constitution: NCRWC) की ररपोटत में
जवस्टस िमात सवमवि की ऄनुसंर्शाओं के कायातन्ियन की वसफाररर्श की गयी। आसमें ऄनु र्शंसा की
गयी कक प्रथम और सबसे महत्त्िपूर्त कदम वजसे संघ और राज्य सरकारों द्वारा ईठाए जाने की
अिश्यकिा है, िह है- लोगों को संिेदनर्शील बनाना और नागररकों के मध्य मूल कर्त्तव्यों के
प्रािधानों के ऄनुपालन हेिु जागरूकिा ईत्पन्न करना।

1.7. मू ल कर्त्त व्य हे िु निीन सं द भत

 1976 से ऄब िक मूल कर्त्तव्यों में वर्शिा प्रदान करने संबंधी मािा-वपिा (ऄवभभािकों) के दावयत्िों
को र्शावमल करने के ऄविररि कोइ बड़ा बदलाि नहीं हो पाया है। हाल के कु छ िषों में यह विषय
संविधानिेर्त्ाओं, प्रबुद्ध नागररकों एिं समाज के विवभन्न िगों के मध्य गंभीर सचिन और चचात का
विषय बना हुअ है।
 जबकक, आसी दौरान भारि की सामावजक-राजनीविक व्यिस्था में व्यापक पररिितन हुअ है। साथ
ही, ईदार न्यावयक व्याख्याओं के द्वारा मूल ऄवधकारों के दायरे का विस्िार हुअ है। आससे ऄवधकार
एिं कर्त्तव्य के मध्य ऄसंिुलन ईत्पन्न हो गया है। ऄिः आस पर ित्काल पुनर्थिचार करने की
अिश्यकिा है।
 ईच्चिम न्यायालय के िितमान न्यायाधीर्श कु ररयन जोसेफ ने ऄपने विवभन्न ििव्यों एिं लेखों के
द्वारा मूल कर्त्तव्यों में कु छ नये अयामों को जोड़ने पर बल कदया है जो वनम्न हैं:
(i) मिदान करने का कर्त्तव्य (Duty to vote)
(ii) कर ऄदायगी का कर्त्तव्य (Duty to pay tax)
(iii) दुघतटना पीवड़ि की मदद करने का कर्त्तव्य (Duty to help accident victim)
(iv) अस-पड़ोस को स्िच्छ रखने का कर्त्तव्य (Duty to keep the premise clean)
(v) ऄनुवचि कायों से स्ियं को एिं दूसरों को दूर रखने का नागररक कर्त्तव्य (Duty to prevent
civil wrong)
(vi) ऄन्याय के विरद्ध अिाज ईठाने का कर्त्तव्य (Duty to raise voice against injustice)
(vii) वहहसल धलोऄर की सुरिा का कर्त्तव्य (Duty to protect whistleblower)
(viii) सद्भािी वसविल सोसाआटी अंदोलन को समथतन देने का कर्त्तव्य (Duty to support bona
fide civil society movements)

1.8. अलोचना

 संविधान में प्रविष्ठावपि कर्त्तव्य, ककसी सुसंगि मूलभूि विषय-िस्िु को प्रदर्थर्शि नहीं करिे हैं। मूल
कर्त्तव्यों की गैर-प्रिितनीय प्रकृ वि के कारर् िथा आनके ऄवनवश्चि, ऄस्पि और ऄल्प व्यिहाररक
मूल्यों िाले एक नैविक विचार भर होने के कारर् आनकी अलोचना की गइ है। आनमें से ऄवधकांर्श
कर्त्तव्यों का ईल्लंघन पहले से ही विवभन्न कानूनों के िहि दंडनीय है। कु छ अलोचकों का मानना है
कक मूल ऄवधकारों के साथ-साथ मूल कर्त्तव्यों को न्यायोवचि दजात प्रदान करना ऄवधक ईपयुि
होिा क्योंकक ऄवधकार एिं कर्त्तव्य एक ही वसक्के के दो पहलू हैं।
 भारि के पूित महान्यायिादी, सी. के . दफ्िरी द्वारा मूल कर्त्तव्यों का विरोध ककया गया। ईन्होंने
िकत कदया कक ऄवधकांर्श नागररक कानूनों का पालन करिे है। ऄिः ईन्हें ईनके कर्त्तव्यों के बारे में
बिाने की अिश्यकिा नहीं हैं। “जब िक लोग खुर्श एिं संिुि होिे हैं िब िक िे स्ियं ऄपने
कर्त्तव्यों का पालन करिे रहिे हैं।”

244 www.visionias.in ©Vision IAS

Google it:- https://upscpdf.com


https://upscpdf.com << Download From >> https://upscpdf.com

1.9. प्रासं वगकिा

 कर्त्तव्य, नागररकों को ऄनुर्शासन एिं प्रविबद्धिा की समझ प्रदान करिे हैं। ऄवधकारों का ईपभोग
करने के साथ-साथ नागररकों के कु छ कर्त्तव्य भी होिे हैं। मूल कर्त्तव्यों से न्यायालयों में कानूनों की
संिैधावनक िैधिा की जांच करने में सहायिा प्राप्त हुइ है। आस प्रकार, ये कर्त्तव्य होने से पूित एक
महान वर्शिाप्रद मूल्य भी हैं। ये देर्श की एकिा एिं ऄखंडिा के साथ-साथ भूमण्डलीकृ ि एिं
भौविकिादी िैवश्वक व्यिस्था में गौरिर्शाली भारिीय सांस्कृ विक परंपरा, मानििाद जैसे मूल्यों
को भी सुरिा प्रदान करिे हैं। यह िितमान िैवश्वक समस्याओं जैसे पयातिरर्ीय िरर् अकद के
समाधान के वलए भी प्रासंवगक है।

1.10. मू ल कर्त्त व्यों को प्रभािी बनाने हे िु कु छ सु झाि

 मूल कर्त्तव्यों एिं संिैधावनक मूल्यों के प्रवि अम जनिा को जागरक करने के वलए राष्ट्रव्यापी
ऄवभयान चलाये जाने चावहए। मूल कर्त्तव्यों को प्रभािी एिं लोगों को जागरक बनाने हेिु जवस्टस
िमात सवमवि ने वनम्नवलवखि सुझाि कदए हैं-
o भारिीय संविधान की ईद्देवर्शका सवहि सभी मूल कर्त्तव्य को सरकारी प्रकार्शनों, कै लेंडर एिं
ऄन्य साितजवनक स्थानों पर प्रदर्थर्शि ककया जाना चावहए िाकक लोग जागरक हो सकें ।
o 3 जनिरी को 'मूल कर्त्तव्य कदिस' मनाया जाना चावहए।
o मूल कर्त्तव्यों को प्रभािी बनाने हेिु लोकपाल जैसी स्िायर्त् संस्थाओं का गठन ककया जाना
चावहए।
o मीवडया, रेवडयो, समाचार पत्रों अकद के माध्यम से लोगों को आन मूल कर्त्तव्यों के प्रवि
जागरक बनाया जाना चावहए।
o आनको प्रभािी रूप प्रदान करने में गैर -सरकारी संगठनों (NGO) की भूवमका ऄत्यंि
महत्त्िपूर्त हो सकिी है।
o आन्हें मूित रूप प्रदान करना िभी संभि होगा जब प्राथवमक, माध्यवमक एिं ईच्च र्शैिवर्क स्िर
पर सामूवहक पररचचात, भाषर्, िकत र्शॉप, वनबंध अकद के माध्यम से आन कर्त्तव्यों के प्रवि
विद्यार्थथयों को जीिन के प्रारंवभक चरर् में ही जागरक ककया जाए।

2. विगि िषों में Vision IAS GS मेंस टे स्ट सीरीज में पू छे


गए प्रश्न (Previous Year Vision IAS GS Mains Test
Series Questions)
1. जबकक एक जीिंि लोकिंत्र के ऄवस्ित्ि के वलए मूल ऄवधकार महत्िपूर्त हैं, िहीं मूल कर्त्तव्य
भी समान रूप से महत्िपूर्त हैं। मूल कर्त्तव्यों की गर्ना करिे हुए, आस कथन पर चचात
कीवजए।
दृविकोर्:
 मूल कर्त्तव्यों की संविप्त चचात कीवजए।
 संिेप में मूल ऄवधकार और मूल कर्त्तव्यों की िुलना कीवजए।
 एक जीिंि लोकिंत्र के वलए मूल कर्त्तव्यों के महत्ि पर चचात कीवजए।
ईर्त्र:
स्िर्त ससह सवमवि के सुझािों के अधार पर 42िें संविधान संर्शोधन ऄवधवनयम (1976) के
द्वारा संविधान में दस मूल कर्त्तव्यों को र्शावमल ककया गया।
ऄनुच्छेद “51(A)” के ऄनुसार, भारि के प्रत्येक नागररक का यह कर्त्तव्य होगा कक िह:-
संविधान का पालन करे और ईसके अदर्शों, संस्थाओं, राष्ट्रीय ध्िज और राष्ट्र गान का सममान
करे, स्ििंत्रिा के वलए हमारे राष्ट्रीय अंदोलन को प्रेररि करने िाले ईच्च अदर्शों को हृदय में

245 www.visionias.in ©Vision IAS

Google it:- https://upscpdf.com


https://upscpdf.com << Download From >> https://upscpdf.com

संजोए रखे और ईनका पालन करे , भारि की संप्रभुिा, एकिा और ऄखंडिा की रिा करे और
ईसे ऄिुण्र् रखें, देर्श की रिा करे और अह्िान करने पर राष्ट्र की सेिा करे , भारि के सभी
लोगों में समरसिा और समान भ्रािृत्ि की भािना का वनमातर् करे , हमारी सामावसक संस्कृ वि
की गौरिर्शाली परंपरा का महत्ि समझे और ईसका परररिर् करे , प्राकृ विक पयातिरर् की
रिा करे और ईसका संिधतन करें िथा प्रावर्मात्र के प्रवि दयाभाि रखे , िैज्ञावनक दृविकोर्,
मानििाद और ज्ञानाजतन िथा सुधार की भािना का विकास करे , साितजवनक संपवर्त् को
सुरविि रखे और सहसा से दूर रहे, व्यविगि और सामूवहक गविविवधयों के सभी िेत्र में
ईत्कषत की ओर बढ़ने का सिि प्रयास करे , बच्चों के वलए वर्शिा के ऄिसर ईपलधध कराए।
नागररकों के ऄवधकार और कर्त्तव्य ऄन्योन्याश्रयी और ऄविभाज्य हैं। मूल ऄवधकार
राजनीविक लोकिंत्र के अदर्शत को बढ़ािा देने और िानार्शाही िथा वनरंकुर्श र्शासन पर
वनयंत्रर् स्थावपि करने के साधन हैं। यह राज्य के हस्ििेप के विरद्ध स्ििंत्रिा और स्िाधीनिा
की रिा करिा है। पुनः यह राज्य की मनमानी र्शवियों पर ऄंकुर्श लगािा है और लोगों के
विकास के वलए कु छ बुवनयादी ऄविच्छेद्य ऄवधकारों को सुवनवश्चि करिा है। जबकक, मूल
कर्त्तव्य नागररकों को सचेि करिे हैं कक ऄपने ऄवधकारों का लाभ ईठािे हुए ऄपने देर्श, समाज
और साथी नागररकों के प्रवि जो कर्त्तव्य हैं, ईन्हें ईनके प्रवि भी जागरूक होना चावहए। आस
प्रकार ये कर्त्तव्य अधाररि दृविकोर् प्रस्िुि कर लोकिांवत्रक मूल्यों की वर्शिा वर्शिा प्रदान
करिे हैं।
ये राष्ट्र विरोधी और ऄसामावजक गविविवधयों के विरद्ध एक चेिािनी के रूप में िथा
नागररकों के वलए प्रेरर्ा स्रोि के रूप में कायत करिे हैं िथा ईनके मध्य ऄनुर्शासन और
प्रविबद्धिा की भािना को बढ़ािा देिे हैं। पुनः ये नागररकों में यह भािना ईत्पन्न करिे हैं कक
िे के िल मूकदर्शतक नहीं हैं ऄवपिु राष्ट्रीय लक्ष्यों की प्रावप्त में एक सकिय भागीदार भी हैं।
िषत 1992 में ईच्चिम न्यायालय ने यह वनर्तय कदया कक ककसी कानून की संिैधावनकिा की
दृवि से व्याख्या में यकद न्यायालय को ज्ञाि हो कक विचाराधीन क़ानून मूल कर्त्तव्य को
प्रभाविि करेगा िो ऄनुच्छेद 14 या ऄनुच्छेद 19 के संदभत में आसे िकत संगि माना जा सकिा है
और आस प्रकार ऐसी विवध को ऄसंिैधावनक होने से संरिर् प्रदान ककया जा सकिा है।
आस प्रकार, ये लोकिंत्र को बनाए रखने में मूल ऄवधकारों की िुलना में समान रूप से
महत्िपूर्त हैं। मूल ऄवधकारों की प्रविकियार्शील प्रकृ वि के विपरीि ये ऄंिःकरर् को ईद्वेवलि
और कियार्शील कर लोगों में एक सकिय मूल्य भािना का वनमातर् करिे हैं।

2. मूल कर्त्तव्य यद्यवप महत्िपूर्त हैं, िथावप आनकी कु छ सीमाएं हैं। परीिर् कीवजए।
दृविकोर्:
 ईर्त्र का पहला भाग मूल कर्त्तव्यों के पररचय के साथ ईनके महत्त्ि पर कें कद्रि होना
चावहए।
 दूसरे भाग ईन प्रमुख कवमयों को प्रस्िुि ककया जाना चावहए जो आनकी सितसममि
स्िीकायतिा में बाधक हैं।
ईर्त्र:
संविधान में मूल कर्त्तव्यों को 42िें संर्शोधन (1976) के द्वारा प्रविष्ठावपि ककया गया था।
आनका ईद्देश्य, भारि के प्रत्येक नागररक के वलए कर्त्तव्यों का एक समुच्चय प्रस्िुि करना था।
दर्शकों से आनके वनरंिर विद्यमानिा से आनके महत्ि और प्रासंवगकिा का पिा चलिा है। यह
नागररकों आस संबंध में सचेि करिे हैं कक ईन्हें ऄपने ऄवधकारों का लाभ ईठािे हुए ऄपने
कर्त्तव्यों का भी पालन करना चावहए।
मूल कर्त्तव्यों ने विवभन्न िैधावनक विकास को अधार प्रदान ककया है।
कइ बार न्यायपावलका ने भी वनर्तय हेिु आन मूल कर्त्तव्यों की विषय िस्िु का सहारा वलया है।
आसके ऄविररि, नागररकों के मध्य आन्हें प्राकृ विक न्याय के मागत के रूप में भी स्थावपि ककया
जािा है।

246 www.visionias.in ©Vision IAS

Google it:- https://upscpdf.com


https://upscpdf.com << Download From >> https://upscpdf.com

देर्श और समाज विरोधी गविविवधयों को मूल कर्त्तव्यों के अधार पर वनयंवत्रि ककया जा


सकिा है।
परन्िु कफर भी, कु छ ऐसे ऄिरोध विद्यमान हैं जो आनकी सितसममि स्िीकायतिा समि बाधक
बने हुए हैं। िे आस प्रकार हैं:
 प्राथवमक रूप से ये न्यावयक कारतिाइ से परे हैं, मूल ऄवधकारों की भांवि आनके ईल्लंघन
पर कोइ कानूनी कायतिाही नहीं की जा सकिी।
 ऐसा प्रिीि होिा है कक, मूल कर्त्तव्यों की यथाथत प्रकृ वि ईनकी ऄस्पि व्याख्या से धूवमल
हो गइ है, आसवलए आनका प्रभाि ऄवि न्यून है।
 आनके महत्त्ि के बारे में जागरूकिा ईत्पन्न करने हेिु कोइ ठोस प्रयास नहीं ककये गए हैं,
आसवलए नागररकों में आनके बारे में मूल ऄवधकारों के समान समझ नहीं है।
 संविधान के भाग IV के एक ईपांग मात्र होने के कारर्, मूल ऄवधकारों जैसा महत्त्ि, मूल
कर्त्तव्यों का नहीं रहा है।
मूल कर्त्तव्यों के िास्िविक महत्त्ि को कम कर देने िाले आन ऄिरोधों के बािजूद भी, भारि
जैसे विकासर्शील देर्श में नागररकों के ऄवधकारों और कर्त्तव्यों के मध्य सामंजस्य स्थावपि करने
हेिु संयुि प्रयास ककए जाने की अिश्किा है।

3. विगि िषों में संघ लोक सेिा अयोग (UPSC) द्वारा पूछे
गए प्रश्न (Past Year UPSC Questions)
1. भारिीय नागररकों के मूल कर्त्तव्यों पर संविप्त रटप्पर्ी वलवखए।
2. हमारे संविधान में सवममवलि मूल कर्त्तव्यों की प्रकृ वि को बिाएं।
3. मूल कर्त्तव्य और ईनके वनवहिाथत क्या हैं।
4. भारि के संविधान में ईवल्लवखि ककन्हीं चार मूल कर्त्तव्यों को वलवखए।
5. प्रमुख मूल कर्त्तव्यों की पहचान कीवजए।
6. 42िें संर्शोधन के बाद संविधान में सवममवलि मूल कर्त्तव्यों का िर्तनं कीवजए।

Copyright © by Vision IAS


All rights are reserved. No part of this document may be reproduced, stored in a retrieval system or transmitted
in any form or by any means, electronic, mechanical, photocopying, recording or otherwise, without prior
permission of Vision IAS.

247 www.visionias.in ©Vision IAS

Google it:- https://upscpdf.com


https://upscpdf.com << Download From >> https://upscpdf.com

Google it:- https://upscpdf.com

You might also like